0% found this document useful (0 votes)
416 views

Supplement

1. Supply and demand functions are used to model the relationship between quantity supplied/demanded and price of a good. The supply curve slopes up while the demand curve slopes down. Market price is where supply and demand are equal. Examples of functions are given. 2. Cost functions show the relationship between total cost of production and quantity produced. Total cost can be split into fixed and variable components. Average cost is total cost divided by quantity. 3. Revenue functions show the relationship between total revenue earned from sales and the price and quantity sold. Total revenue is price times quantity.

Uploaded by

nazmul islam
Copyright
© © All Rights Reserved
Available Formats
Download as PDF, TXT or read online on Scribd
0% found this document useful (0 votes)
416 views

Supplement

1. Supply and demand functions are used to model the relationship between quantity supplied/demanded and price of a good. The supply curve slopes up while the demand curve slopes down. Market price is where supply and demand are equal. Examples of functions are given. 2. Cost functions show the relationship between total cost of production and quantity produced. Total cost can be split into fixed and variable components. Average cost is total cost divided by quantity. 3. Revenue functions show the relationship between total revenue earned from sales and the price and quantity sold. Total revenue is price times quantity.

Uploaded by

nazmul islam
Copyright
© © All Rights Reserved
Available Formats
Download as PDF, TXT or read online on Scribd
You are on page 1/ 327

Supplement

BUSINESS MATIULMAIICS
(GUIDELINES LOR
OF a COM. (FIONS.)
Till 105 fliCd
Note : Problems and Theorems involving
not be done.
Concept of limits and continuity. Evaluatioll ot
1 CaLculus
limits, Simple application to Commerce such as I)epreeiati011, etc.
differentiation. Sum aiid
DetiiiitiOfl of d i fferentiation. Theorems Oil
difference of i0 1 1ti00s. Multiplication and Division of tunctions. i\
function raised to power of a function. Function ot a I unction.
lmI)IiCLL function (Derivation of one function with reSJ)eCt to
the formulae not necessary). (jeOilICtrlC
another junction (Derivation 01
and Economic interpretation ol derivatives.
Successive differentiation. Simple standard forms (without
Partial thWerentiation. DelinitiOn of hulerS
LEII3N [Z RULE).
Theorem (1st order). Total differentiation of implicit functions.
Maxima and Minima. Cases of one variable involving second and
higher order derivatives. Cases oh two variables involving one constraint.
Reducible to standard fors. m
Integration. Standard form s.
Method of substitution. Integration by parts. Use of partial fractions-
ill cases.
Definite integration for finding areas
Application of Calculus to Business and Economics
Knowledge of common forms of functions used in Business and
liconoinics and shapes of their curves like demand function, supply
iOU
function, cost function, revenue function, utility function, prod net
function with one or more factors of production (Tracing of curve not to
be done).
Mathematical interpretation of average, marginal and elasticitY
concepts. Derivation of their interrelations by using Calculus . Calc- u
obto niug of
lations of these values and from them (in simple forms)
original functions. Cross and Direct elasticities. Compound interest
and rate of growth.
Problems involving one or two variables of optiium level Of
production under monopoly. Simple cases of duopoly. Equihibfiu1
prices under Perfect Competition. Simple cases of inventory coptrol.
Consumers surplus and Producers surplus.
2. Matrices Definition. Different types of matrices. Algebra. of
Elementary row transfOi111atbol'
matrices. Transpose of a matrix.
including method of finding inverse.
Determinants. Properties of determinants. Calculation of values
and product of determinants tip to third order.
Adjoint of a matrix and method of finding inverse. Solution of
equation with the help of matrices and determinants.
Problems relating to Business and Economics based on solution of
equation and matrix multiplication. Leontief input and output model.

ACE 4
DUS]NLS MATtIEMArICS

tom ax
3. Linear Pr oy mming Gra
phical method. Problems rclating
rn isation and mm imisation involving two variables and in-
equalities of both types greater than and equal to and less than and equal
to. Cuses when no solution exist and multiple solution exist.
Shnpltx Meth od.
Concept of slack variable. Solution of problems
involving not more than three variables
in v olving ineoualitics Maxi
of t y pes less than and equalmisition
to.
problems
Dmility Miirnio Concept of
th problems involving ine qualities of type
greater
an and equal
niaxirnisatior to. Solving of them by reducing to a problem in

4 Probability
lity in irale Concept of probability. Calculation of probabi-
events A cases from definition. Independent and mutually exclusive
tJitiou rule for two or more mutually exclusive events.
of the t lir ii when i \ VO Form
events are not mutually exclusive.
plication ne ro ince peridep C events. Condit Multi-
ional probability
1 -' p tt
rel ating to Business iind Ryes' Theorem Definition and simple problems
and Commerce Situations.

SUGGESTED FURTHER R EADINGS (Fo) . flefrence)


1. 4 lr ,jf,j f Sul/,m, ;
Mathematics for Business and Social
Sciences John Wiley & Sons (Chapter III to XII).
2 R. B. ,tI/e,z Mathenmtici I
Mcmi!I A flal\'sis for Economist
St. Mrtnres Press - Chapter V to XTI. XIV to XV.
3. L. W T S !Ie Ifrd:
ltusincss Nit cm tics --Relevant Chapters.
4. Finite .Lui,frss AppIicwiu,i
i%IQ1IIC,,,(JII CS Wit/i
Kcni1u,y,
Schleifer, Snell a rid I hoinpsou. Prentice Hall of India-- FEE Book
Rekvant Chapter.

5, Line,r Programn ing :


N. Paul Loomba. MacGraw liii] Co.
SECTION A

Applications to Commerce
and IY'OIlOJJIiCS
FUNCTIONS
1. Supply Functions aixtl Deniard Functions. The StippltJ
function in economics is used to specify the UnlOillitS of a particular
commodit y that sellers have available to offer in the market at various
prices. The detnorid Jinc1iOn specifics the aniouuis of a particular
commodity that buyers are willing to purchase at various prices. It is
well known that an increase ill usuidly causes an increase in the
supply, but a decrease in demand ; on the other hand, a decrease Iti
prices brings about a decrcae in supply but an increase in demand.
The I nurkei price is defined as the price at which supply and demand are
equal.
Let x denote the quantity of commodity demanded and [3 its price.
x and p being variables we may write the demand function
x =f( p ) showing dependence of X On p or
p=g(x) showing dependence of p on x
(These are the explicit forms of the implicit demand function,
F(x, p)=OJ.
(o) The variables in the case of demand function, as ill the case
of other functions in economics, are hypothetical quantities and not actual
observable quantities. Changes in the values of parameters cause shifts
in the demand curve.
(b) The arguments given above apply to a supply function if x
stands for the variable supply.
(c) (i) The slope of a demand curve is negative, i.e,, it slopes down-
wards from left to right indicating that demand under normal circuni
stances expands as price is lower.
(ii) The slope of a supply curve is positive, i.e., supply curves
normaliy r i se from left to right.
(d) Examples of demand functions are
i) Qd = 53p, (ii) Qd , (III) Q - . 3p' +P4 65,
(lv) Q-5/p and soon,
Similarly the supply functions are
(1) Q,=3p-3, (ii) Q=2p . b-p, (lii) Q=3p-3, etc.
2. Cost functions. lix is the quantity produced of a certain
good by a firm at total cost C, we write the total cost function C-c(x)
explicitly. We may write this in the implicit form
g(C, X) =0
(a) It may be noted that the Cost (C) of producing so much good,s
can be analysed into two parts (f) fixed cost which is independent of x
BUSINESS MATHIMAl [CS

(with cctaio limits) and (ii) variable cost depending on X. Thus we


may hac cost function of the type
C(x) 200l- ax,
where a is a known coast lot.
(1,) Average cost of production or cost per unit is obtained by
dividing total cost by the quantity produced.

dC: C=/1C.

(C) Cost
curves are obtained froni the knowledge of production
functions Usually the cost curve is rising to the right as the cost of
prnductiofl generally increases with the output (x).
3 Total •veuuP FurcIiOfl. Revenue is the amount of money
derived from the sale of a product and depends upon the price of the
product and the quantity of the product that is actuall y sold. If Q4 is
the demand for the output of a firm at price p, then the total revenue (R)
collected by the firm is
R(x) -pQ j p.—

Thus the price p is also average revenue of the firm.


4 Profit Function. The revenue and Cost function lead to the
profit function of a firm, as the profit is the excess of revenue over the
cost of production. The profit fuiction of the firm is
P() = R(x')—C(x)
1etior FuTtetion. PrO(IUctiOfl of a firm cannot usually
5 p 1 . 9 satisfactorily
be expressed as a function of the single variable such as
capital for the simple reason that production necessarily implies the
of several economic factors such as capital, labour. etc.
coming togethe r
The production function is written as
P.:f(L, K)
are quantities of labour and capital respectively required to
where L K
produce P.
defined as
In FconomiC s the Co.DoUg1aS production function
P=c K z L3 , ctll.

is most generally used.


P-1(y)K O - 25 L0".9
OOK°"
denotes the satis raction obtained
6. Utility Function, If U (x, y)
by an individual when he buys quantities X and y of two commodities
(x. y). the function of two variables x and y is called
X and Y. then U
utility function or utility index of the individual.
U(x, y)= U0 . Since
(a) For a fixed value U= U0 . we get a curve
y) of the commodities X and Y which are on this
cnrnl)ifl t i oflS (,
give the same satisfaction to the individual, he would he in-
C1117 11
different to the particular combination (x, Y) that he buys. The curve is,
therefore, known as indifference curve.
APPLICATIONS TO COMMERCE AND ECONOMICS
(b) It may be noted that for different values of U0, we wilt get
different indifference curves.
(c) If U (x, y)--xy, then the indifference curves are hyperbolas
xy U0 , where U0 takes different values for different level of satisfaction
e.g., when
U—(x--3) (y+2)
U U
y------3-----2 and X= L2

7. Ovei-p il Cox;suuiption Function. If C is the total consump-


tion of the Comnmuility dependent on income Y and properity to consume
C the aggregate consumption function is indicated by

C=-a+cY
But since Y=C-1 S
S=rY_(a- cY)
This is the savings function of the community.
EQU I LI BR IUM
Equihbrium price or quantity can he found by equating demand
and supply functions or by calculating excess of demand over supaly as
shon below
Example I. Find equilibrium !rice and quantity, given the functions
Q- 2 -- 002 P
Q,=02+ 007 P
Solution. Take Qd= Q,
2-002 PO2-4 007 P
—002 P-007 Pr-2+0-2
->
Miter. Excess demand=—=Q,— Q
Excess Dernand=(2-002 P ) — (0'2+0'07 F)
=(2-02)—(002 P +007 P)18-009 P
Equating excess demand to zero, we have
P: - - -- =20
009
The equilibrium quantity is found by substituting the value of
quilihrium price in any of the given demand or supply functions.
Q 4 =2—o-02 P
With P20, Qd-2--(002 )< 20)=r2__4=1.6
Example 2. Find equilibrium price by the method of excess demand
given the functions
Q d =5 0 — ; Q,JQf?


BUSINESS MATHI3MATICS
ACO-S

Solution. Excess demand==Qj—Q.


i.e., Qi—Q'(50-- p)—(l0-1-p)
-=S0--- p-10— p =40-18[ p
Equating excess demand to zero, we have
181 p=40
40

E ' -im le 3. Find equilibrium price given

Solution. Let
-?--.= 8p=p2(p--2)
Se
Dividing both sides by p we get
8=p2 - -2P
p2--2p---8=0
p*4p+2p8r0
. p(p-4)+-2 ( p 4)--0 or (p-4)(p+2)O
p==4 or pr-2
Since puce cannot be a negative figure, p =4.
Example 4 Assume that for a closed economy, E_C+I4-G,
where E is total expenditure, C is expenditure on consumption goods, I is
expenditure on investment goods and G is Government spending. For
equilibrium, we must have FY, where Y is total Income received.
For a certain ecofloflly, It is given that C=15-1-0'9 Y, 1=20-1-005 Y,
and G=25.
Find the equilibrium values of Y, C and I. How will these change if
there is no Government spending 7
Solution. Here we are gwen that
E=C±I+G
and for equilibrium E=-- Y
From (I) and (2), we have
Y-=C+I+G
Substituting the given values of C, I and G in (3), we get
Y=r(I 5--09 Y)+(204 005 Y)---25=60+095 Y
Y(I-095)=60
60
=1200.

Now C=l5+09Yr15+jXI20Ol095
O


ACE-9
APPLICAT IONS ao COM I!RCI ANT) ECONOMICS

(6)

and 120+005Y201 l)() 1200=80.



The required equilibrium values are given by (4), (5)
and (6). f
there is no govern meat speuding then G —0 and the equilibrium equation
takes the form
.(7)
Y=c 1 I
Substituting the given values of C and I in (7), we find
Y (IS +09 Y) (20 005 Y)= 35+0 95 Y
Y(l -- 0'9)--35

: , = 7000 ... (8)


(Y05 5

Now C - 15 4 09 Y 15 1 700=645

1=20 I 005 Y=20+ x 70055. ... ( 10)


and to

The changed values of Y, C and I, if there is no government spend-


ing, are rc pectively given by (8), (9) and (to).

ELASTICITY
Elasticity of the function y=f(x) at the point x is defined as the
rate of "proportional change in .v or f(x) per unit proportional cli.ingc
in x"
Price l:L;ticity of Hupply is the relative change in supply in
response to a relative change in price. If now x stands for supply and
the supply function is written as x --g ( p ) the formula for elasticity of
77 -
supply retains the same form as that of
Ck
p. --i- , where x is supply function.
-
Since the slope of the supply curve is positi v e, 1 1, is also positive.
Price Elasiicty of Demand. The
average price elasticity of
demand is the proportionate change in quantity demanded to propor-
tionate change in price. Precisely if the demand changes from X to
x -f 8 x when the price changes from p to p 4- p then
X/x p 8x
Average price dasticity of demand - --- -
hp/p x .
The poloi ehsticftv of (Jemand. It is the elasticity of demand at a
particular price level say p, by definition it is the limiting value of
average price elasticity
Point elasticity of clemind at price 'p' is
(c'x) x . tJ' r)

1m m -- - =- Inn 1^x- - rip x


P P tp-o P
5JO
In general, the slope of demand curve is negative and hence Ili is
negative.

ACR-1Q
BUSINESS MAT11I3MATxc
p dx Marginalquantity demanded
x dp Average quantity damanded
(Usually we write 77 d in the form I which means that we only
consider the absolute value of )) d irrespective of its sign.)
The crucial value of 77, is I.
When I I >1, demand is elastic.
When I 11 ,i I <I, demand is inelastic
When 1 71, I =1, demand has unit elasticity.
Working rule for finding Ela sticity of demand
If x -=f( p) is the demand function, then
• .
(I) arg :al quantity demanded d
dp
(ii) Average quantity dcmandcd=-

(iii) I I

I llustration 1 . If the demand law is ,find the elasticity


Of demand in terms of x.
'Solution Tie elasticity of demand is defined as
p dx
—X---
x tip

Given: 10 -2

dp 20
-w (-2)(l)-
(x+l)
Substituting the values in (i), we get
10 1 C (x+1)2 ) ( X+1
'ld(x)iXX. —YO—
S

Illustration 2. Find the elasticity of demand for the demand functotn


where x is the demand of a good at a price p.
Ps
Solution. Marginal quantity demanded
dx 81

Average quantity demanded


x 27 1 27
____=7•
APPLICATIONS TO CONWERCE AND ECONOMICS ACE- Il

Hence ' = clasricitv of demand=


- xIp
(-8l/p)
27/pa
Illustration 3. Find '? d when P- 5, if the demand function is
x= 50 + p —pr whore x is the demand for the commodity at price P.

Marginal quantity demanded 'IP ) 1 +2p.

Averace quantity demanded -- ()(5o+p± p2)IP.

Marginal quantity demanded p(I+2P)


Average quantity demanded 150-lp-4-P2
li when p-=5 is given by
5(14-2x5)55 .
-= which i s < 1 shows that the demand is inelastic.

Income elasticity of dema!rtd. It is the elasticity of quantity


demanded in response to change in income. It is defined as
(IX

where x is te quantity demanded and y is the income per head ii the


relevant group of people.
If ti,> 1, the goods are Luxury.
0 the goods are Necessity of life.
and 1,<0, goods are Inferior.
Remark. Elasticities can also be expressed in terms of logarithms.
For example, let demand curve he
x=flp)
d I dx d I
then (log x)— . and (log

Fl dx d
- j—(logX)
d - d(logx)
dp = TPd (Iogp) d(logP)
dp
Example 5. Find the elasticity of demand w,r.t. price for the
following demand functions
(a) p= ya---bD , a anI b being constants.

(b) D=zr4.,
it
(C) D_—pe ; a, b and c are constants.
ACE-12
BUSINESS MATiiEMAfCS

77, I p dD
Solution.

(a)

£' (a—bD) x(-b)=_ 2a—bD)17

(ID 1 - 2(ab_D)l2
••
(Ip - (ilp/a'D) b

/a I
(ab -D)t'
x r 2(a—hD)'2
2
L--— b -
=(a-bD)

(b) D_- 8p - 312.

12p512

ldI —12p5'2

12 22 3 3

(c) D=p e'

p'1--J e-) -p e-?(P+)


di (b)

pa • e(P+C) —b
=p-'

x -L e(P+a) (a - bp)J
=

Example 6. Glen the demand function


20
Q=-7 , find the elasticity at point p3.

Solution. Q-1y=2O(P+ 11'


dQ
----20 / 2
dp ' 1 I
APPLICATIONS TO C0MMaRCI AND ECONOMICS ACV- 13

• p(p1) 20 p
I I
20 (p)2
when p =-3 is given by
3 3
=Y75

Example 7. ,l demand function is given by xp' =k, where n and k


e constants Calculate price elasticity of demand.
Solution. Here x=kp-"
dx
nkp
• .IP

Now
x dp

x (_nkp_?1)Hn
Hence the demand curve xp'k has elasticity equal to n at all
level of prices.
Eatnple S. Show that the elasticity of demand at all points on
Me curve xp =- 2 will /e numerically equal to one.
Solution. Here
dx
(I-),
dx
Now I -' . -
x
X(Y2)(

Hence the elasticity of demand at all points on the curve is one.


Example 9. Find the elasticities of detnind and supply at equilibrium
price for demand junction p= / 100 -x, and supply function x--2p--10,
where p is price and x is quantity. [Del/if Univ . B . corn. (lions.), 1992]
Solution, Equilibrium conditions are determined by equating
demand and supply laws.

4(100X !)=x+20x+ 100


X2
+4x-60=0
(X+I0)(x-6)_=0
X=6 or X=—]O
x=-- 10 is not admissible as quantity cannot be negative.
X=6
x+l0 6+10

ACE-14 BUSINESS W!AThEMAT1CS

p dx dx
7d and )1 R
x tip x dp
Calculation of 77 a.

100
(100—X2)-112 (-2x)=----_
dx 2
lO0x
- p dx 8 ( vi?_616
9
Calculation of
x=-2p— 10
• dx
2

• p (Ix 8
- x 2-- 8
-- 3
Marginal Revenue and Elasticity of Demand
We know
Total Revenue= Price x Quantity sold
or R=px

Average tvcnuc (AR) - -


R

dR dp
Marginal revenue (MR ) = d_ =p--x

x cip•

dx
p ( 1—,--_)[SinceVd=---__
P'- •

MRrrp ( 1—±-()

or MR=AR ( I _(!) or AR=MR.


d lid
It follows from this that when
(I) J ' = 1, TR remains constant with a fall in price
77d > 1, TR rises with a fall in price
0 0 I <1, TR faIIs with a fall in price.

APPLICATIONS FO COMMERCE AND ECONOMICS ACE-i5

Example 10. Verify the relations/zip


MR=p ( iL)
for the de mand Junction p =(12_x) 1 1 2 , Ox 12.
Solution. We have p=(12—x)10
dp
=W (12—x)/ (-1)

:{ _c! 1'L!. _2(l2_x)112

The total revenue is

(l2--x)/2

=(12_x)1I2Ii_ _2(l2—x)]
p(i _,;L ) [ From (1) and (2)]
Hence MR=p( i-----'
li d I
Example II. If AR and MR denote the average and marginal
revenue at any output, show that elasticity of (lemand is equal to
ARMR
Verify this law for the linear demand law p=a+bx.
Solution Total revenue : R=px
R dRdp
AR=— =p , whereas MR-- —p-I-
x fix x a-
AR
Now
R P_(P+xd) dx

('p
For p=a+bx,
Rpx_ax+bx
ACE- 16
ftUSlN?SS MATHEMATICS

So R (IR
,1R •-=a - hv and MR= __a+2bx
A (IX

- d (a+hX
I d I =
T dp x 1d,o
dx
(a -l.b I a-l-h (ai-/)x)
x h
,j R
AR—MR
EXERCISE (I)
I. What do you understand by market equilibrium ? State its uses.
Fxpla i n your answer graphically also.

Find the marketequilibrium of prices and quantities if the demand


laws for two commodities are
x5--pi q, y- lO—p+j
and the supply laws ire
x-5-4 . p+q i'=-2p 2q
where p and q represent the price per unit
respect ivclv of Commodities v and y

2. Explain what y
g ou understand by rnnrfet equilibrium Show
raphically Or otherwise that no price other than the equilibrium price
can last longer in the market.
Find the market equilibrium price and quantities if the demand laws
for two commodities are
q--27—x-3y
and supply laws are
Y = -- 6 1-2p—q,
)-= -- 3—p8q
where p and q represent the price per mut of commodities -v and Y
respectively.
3. Find the equilibrium prices and quantities for the two commo-
dity market models
X j i_ 2__p ..fq, X,jr=_2_q
Xi_3-_p_q, X4?__9fp±q
where p is price and q is quantity.
(Hint. At equilibrium, XJX.1 and .v 2 - x,J
4. (ci) Explain (i)
Denitnd function and Supply function. (ii)
Market equilibrium.
(b)
The price p of a certain commodity is partly constant and partly
varies as the reciprocal of the quantity demanded d.
is 5 -f-3p where a and 0 The supply functio n
are constants. The demand and supply

AC2tIl
APPLICATIONS 10 COMMERCIt A ECONOMICS

curves were drawii on the same graph taking the quantity nfl x-axis and
on Y-aXIS. ihe equilibrium point is (4, 6) arid at price 5 units,
the quantity demanded and the quantity supplied are 5 and 3 ii ruts
respectivel y . Determine the demand and supply tunction and find the
price when (0 the quantity demanded is S units and (ii) the qua ii lily
supplied is 10 units.

5. Explain what von understand by Demand and Supply tUilCtiOfl


and Market equilibrium.
The demand law of a commodity is p . ? n \ 1X Jr . If the price is
one unit, the demand is 100 units and if the demand is 16 unitS, the total
reveiiiie is 144 units. 1:iiid the constant ti and ii.
Hp -2 nnils what is the total revenue ?
l equilibrium.
6. F\plain the effect tit' taxation oi
lie &hentand law is 3P- 1 -2.v .27 and supply law is (;P--'-X:-9

(1) If the tax of per ulin IS iiiiposed, find the equilibrinni price
and quantit y and the total government revenue.
(li) It';I snhcrlv of I per unit is granted. find the new price and
cpia iitry rid
i total government expenditure.

7. Ex plain what you understand by demand and supply functions.


State their uses State reasons for the chance in demand and supply of
a eu a nudity.
W !c:i the price of sweets was Rs. 3 per kg. its tIeiid
12 0,ousiiwl ke. and when the price was Rs. 5 per kg. its demand WaS
tIiouiinti kg It' the demand function is P b.. find the vakics of
the constants a and b. \Vliat will he demand when the price is Rs. 7
per k,, . "• Which of thisc three prices of sweets will give more benefit ?
S. The demand curve and the supply curve of a commodity are
given 3p -. p2 and S Sp - I. Find the equilibrium price and
'lie q ia itt ity.
[Hint l:1r eqinhibrium, we have D S
19 3jip 2 .-.5p .. I
p24-Sp--20-0
(pi-10)(p -2)0, i.e.. p-2 and p 1
We reject the value i 10, since price cannot be negative. I ICflCC
equilibrium price is p s: 2 arid substituting it in the demand or supply
curve, we get
D=:S9.J
9. The demand functions of two commodities A and P are
0,4 = 10--pA - 2P B , D p s6 A Pa
and the corresponding supply functions ate
S 4=-3+p.4* p n, S8=-2+ps
where PA and p a denote the prices of A and B respectively. Find
ACE- 18
BUSINESS MATHEMATICS
(I) The equilibrium prices, and
(ii)
The equilibrium quantities exchanged in the market.
(Hint. For equilibrium,we have
and D8=S
10PA - 2p13_ 3+pA+p B and
2PA- 3p 5 — 13==o and
P4+2pa-80
Solving, we get the equilibriu
mprices as
P2 and p,=3
Sub
quantitiesstituting
are givenin by
demand function or supply function, the equilibrium

and
AO. The demand y for a co
mmodity when its price is x, is given
byy
; find the elasticity of demand when the price is 3 units.
11. Defineelasticity of
demand. Interpret 1
12. Define demand elasticity 'i
I nterpret the cases when 1>1, '7 for a given demand law and
=-1 and <I
If AR and MR be
the average and marginal revenue at any Output
show th,t 'i
AR1R at this output. Verify this relation for the
demand law
p. (1--.bx
13.
Define elasticity of a function, hence or otherwise explain in
p articular the elasticity of demand and supply.
If I
is the elasticity of 1(x), then find the elasticities of Xflx)
and

14. The supply of certain goods is given b


is price and a and b are positive co y X
nstants ( p
aQV p-b, when p
elastic i ty of supply e 1 >b), find an expression for
increases . Show that e, decreases as price and supply
arid becomes unity a the price=2b.
15. Express the
following demand laws elasticit ies of demand in terms of q for the
(a) P((j__bq)2
(b)p=-,/771
() P — r---- c
16. Determine the price elasticities of dcmancj for the following
(a) pqe", (h) p= -e-

APPLICATIONS 10 COMM[RCL AND ECONOMICS ACE-i9

1. b
(c) a=- qe (d) qb p, (e)

17. If the demand luoction is p-4-5x', for what value Of X, the


elastiey of demand will he unity 7

[Hint. p4--5x2

Differentiating w,r,t. p, we get


Adx I
1_— lOxX (/J)
Jp l0.

- p dx 4__52
xdp lOx2

Elasticity of demand will be unity if


2
4-5 =1 15 X2 or
IOX2 V15
18. If the demand curve is of the form
p=c2 e

where p is the price and x is the demand, prove that the elasticity of

Hence deduce the elasticity of demand for tlic curve


demand is

10 C

[hint. We have p a

l --ake. dx
dp
pd, ae k'

kx
For the curve p 1OP- 11 , we have

n=IQ, k-

The elasticity of demand foi the curve


i lOe '
is given by
2
I 17 I

19. J)eline elasticity of a function. Hence or otherwise explain


in particular the elasticity of demand and supply, State the uses of
elasticity in Economics.
AcI-2O
BUSINESS MATHEMA CS
If the (1Cm alli! Curve is given by
Pfd)
.-p- e-6(
show that the demand increases as the price decreases, becoming large as
the price approaches the value a/i,.
thilri a//p on the e hsticity of demand, Find the effect of any price greater

20. Compare the elasticities of the demand curves


C!
b
p—c —b and p I
at a price p a, b, c are
and p k th. r-.c 9 Positive x is the quantity demanded,

Toi.l, 'Iverage and Marginal Cost


Total cost (C)
is represented as a function of output x, i.e.,
C=f(x)
Remirlc.
Some books use the notation C=f(Q) where Cis the
cost and Q is the Output.

C
Average cost .=--- or 1(x)

The average cost (AC)


represents the cost per unit of production.
The term marginal cost represents the change in the total cost for
each additional unit of production. .1C
total cot 1UflCtO, i.e., is the first derivative of the

cost ('lfC)=. (IC


dX
Let us flow generalise the total cost function
Total cost (TC)-f(x)+/, (h is fixed cost)
From this total cost function,
as follow other cost functions can he derived

Average cost (AC)i.(.±_±b

Average variable cost (A VC)r-- -fix) )

Average fixed cost b ATC==AVC+AFC


(AFC)-=-_
+1'
Marginal cost dC x
(MC).

Relation bet w
een Average and Marginal Cost Curves

Although cost functions may assume many different shapes under


different circumstaiccs yet usually under natural economic limitations,
we assume average and marginal costs to have U-shapes. The relation
be tweenthem is established as follows

AI'PI.ICAIIONS TO COMMLRCu AND nCONUMLCS ACE-1-21

We know AC the slope is given by

(IC c
(C.' d.-k: I (dC C
(IX 'X) X2 Xdx X

(Mc'/IC)

Case 1. When average cost curve slopes downwards, i.e., when AC


is declining, i ts slope will be negative. In other words,
d C
-- )<o
(51'C— AC) <iO

MC<AC
Thus when AC curve slopes downwards MC curve will lie below
AC curve.
Cise II. When AC curve , reaches a minimum point, its slope be-
comes zero, IL'.,
(1 / C
-- 1=A)
dx \ x /

MC=AC
Thus SIC curve and AC curve intersect at the point of minimum
average cost.
Ce- Ill . When average cost curve rises upwards, its slope is positive.
In other words,

( ;>o Ic>Ac
(IX \ X I
Elitis when AC curve slopes upard, MC . curve will be above AC
Cu rye.

Example 12. The total cost Cfor output x Is given by


2 - 35
x +--
C. ==-
Fin! (i) Cost when output is 4 units,
(ii) Average cost of output of 10 units.
(iii) ,frgitiul cost when output is 3 units.

C— -2 x4 35
Solution. (I)

Acu-22 BUSINESS MATHEMATICS

C for 4 units==4- (4)+=2016 units

2 35 145
(ii) C for 10 units: (10)1 2416 Units

145 1 29
=-242 units

is.ic=L(4 x- _)=-_=o67
11 2
units
(MC is constant here
Ex4rflpe 13. The average cost function (AC) for a commodity is
given by

,IC—+5 - 36
-

in terms of the output x. Find the outputs for which AC is increasing and
the outputs for which AC is decreasing, with increasing output.
Also, find the total cost C and the marginal cost (MC) as function
of X.

Solution. Slope of AC=-(x+5+

AC is increasing if 1 -->0, i.e., if x'>36, or x>6


36
and decreasing it I72 <0, i.e., if x<6.

36x2-i-5J-36
Now AC=-x +5+—=
X x
Total cost (C)=x. AC=x 2 + 5x+36
dC
Marginal cost (MC)-=2X+5.

Example 14. The total cost function of firm is given by


C=004 q 3 _09 q 2 10 q+ 10
Find (a) Average cost (AC).
(h) Marginal cost (MC).
(C) Slope of AC.
(d) Slope of MC.
(e) Value of q at which average variable cost is minimum.

Solution. (a) AC=- 1 =004 q 2 -09 q+Io+-

(b) Marginal cost (MC)- 012 q 2 -1i q+10


APPLICATIONS TO COMMERCE AND ECONOMICS A03-23

(c) Slope of

q2-09 q+io±!-9.)

_(oo8_O9_-'--)

q ( q2-09
q
_L[(o.12 q2 -18 ± 10)
q

q—O9

- [MC—.AC]

(d) Slope ofq-18

(e) When A VC is minimum, the slope of A VC curve is zero, i.e.,

or f(OO4q?_0.9 q+I0)=0
dq (A VC) =O

008 q -09--=0 or q=1125


Example 15. Let the cost function of a fi rm be given by the
following equation

C=300x_1Ox 2 + x3 , where C stands for cost and xfor output.

Calculate (I) Output, at which marginal cost is minirnum


(ii) Output, at which average cost is minftnu,n
(iii)Output, at which average cost is equal to marginal cost.
[LC,W.A ., June 1991]
Solution. (i) C=300X_10x 2 +_}. x8

MC= dC 300-10 (2x)+ i-. 3x2


300-20x+x'
Differentiating wr.t. x and equating to zero, we have

dx —20+2x=Q

Aci-24 BUSINISS MATflI3MATICS

or x 10 is the CCsiI y condition IOU marginal Cost minimisation.


To get the sufficient condition, we have

I(MC)
it positive quantity which mcaiis that marginal cost is

minimum at X = 10.
C: 300x1OX-4 kx
(ii) Average Cost (AC)== T
300— tO

Now to find output at which avcrae cost is rniriimuiji, we liavc to


di1Tcrn tiale the ,IC and CqIJiI I ig it to zero.
• d(tC)00
2t , -o
dx

or
d2 (4j) d 2 2
A1s j- tO+- j X a positive quantity.
T ( )= -,

Second condition is atso satisi ed. Hence the output at whicb


AC is minimum is given by X: 15.

(iii) Now ACMC


300 i.Ls + 300 -- 20x 4-

1xi
lOx Of x 15

Hence for x 15, average cost is equal to marginal cost.

FxituipA p 16. The loft:l tiriibic cost of a monthly outplut x toils by

a firm producing a variable metal is B.c. 5x on/ the fixed cost

is Rs. 300 per month. Draw the overage cost curve whEn cost includes
(i) iarujble COSt i?ill, (11) all Co.cts. Fijuf the output for ,nirilmum average
cost in each case. [Delhi Univ. BA (lions) Econ. 1991
Solution. We have

TCIotal cost 3_ 3? + 5x + 300

and TVC Tot:i] variable cost x3— 3x? + 5x

(1) JV/icn cost includes variable cost only


TVC I
, VC-- Average variable cost f—3x + 5
APPLICATIONS TO CUMM1OCU AND FCONOMICS AC-25
It is a parabola with vertex at (15, 175) and the axis of the parabola is
= 15. The graph of the curve is shown in the figure below
d(AVC)
(IX
1v3
5__

2
(A VC) I
and
d.( 5-

d(1fV(--)
() 3.

or x:15
Hence average cost is minimum
when the output is IS tons.
(ii) H'/e,, cost Includes all costs

4,-,
n 'C
= Avcragc cost

\\
10 X 2 -3x f- 300

=-
x
The graph of the curve is shown in the
adjoining figure
d(AC) 1 300
dx 5
C (12
(AC)
and

dx 5
which gives x=191
If average cost is minimum
when the output is 191 toils.
Con3itions for Profit Mxniizatjon
We know that if y -J x) then for y to be maximum,

0 and
Now assuming that we are given the total cost function along with
the total revenue functionn----both in terms of output, i.e., given functions
are
Total cost function C=f(x)
Total revenue function : R (x)
Total profit:

ACE-26
uUSiNnSS MArFL\n11c3

For P to be maxi!nwn, the c o nditions are


First order CUi(Ii1iO/7

dP /R J( -
'IX (IV /
dR (IC
dx dx

Thus, the profit 1' is maximi'.ed at that quantity x for which marginal
revenue equals marginal cost.
It may be noted that AIR MC means that slope of
total revenue I'll net ion slope of total Cost function.
.''ooj order condron
jp diR d(
dx 2 (4x 2 dx <0
il 2 R dC
(.1V 2 dx2
(I (dR\ (I (dC
(Ix \dx / (IX "(IX
ZIP ± (AIR)< -L(MC)
(IC dx
Thus rate of change of AIR (slope of MR) should he less han the
rate of change of MC (I.e., slope of MC) at the profit maximising output
level.
We shall now discuss the problem of maximization of the Profits of
a firm under wirious market conditions
((') Perfect competition. Under perfect conipetitiot i , the Price
P is constant. The profit maximiration conditions obtained above, riz.,
AIR = MC, which is the condition for equilibrii.im of a firm and
d(AIR)
( i . e ., the rate of change of MR) should he less than C)
dx
dx
(le., the rate of change of MC) at the equilibrium output.
(b) Monopoly. Under monopoly, the n)onopoljst fixes the output
leaving price to be determined by demand conditions.
The profit maximization conditions, obtained above, apply to this
case also.
Example 17. Find the profit maximising Output given the fOIIOn'ing
revenue and cost functions
R(Q)=1000 Q-2Q2
C(Q) Q3-59Q2+1315Q-h-2000.
fDeThI Univ B.A. (lions.) Econ, 1991)
A pp ucArloN; TO COMMURC8 AND E(:oNoMcs ACL27
e I ul 1011. We have
P= Profit= R(Q)_ C(Q)
-(IOOOQ-2Q 2 )--(Q 3 -59Q 2 + I 351Qf2000)
---Q+57Q---315O--2OOO
First order CO,d it run
(//)

--3Q3fH4Q3i5

—3QH Il4Q-315=O

or Q 2 --38Q+ 1O4)

or (Q-3)(Q- 35)=O
Q=3 or Q-35
S'cuntl order condition
(12P
(/Q2<
(1 P
- —6Q+114

P
dQ2 Q=3 18+1 14=96>0

CJQ2 =-21() II4--96<0


Q-5
knce the profit ma irnising output is given by Q=35.
Example A radio manufacturer produces s ets per week at a
total cost of R.F. (x 2 -- 78x+2500). lie is a monopolist and the demand

function for his product is when the price Is Rs. p per set.

Show that maximum ri_I revenue (i.e., profit) is obtained when 29 sets are
produced per week. ?Vlrat is the monopoly price ?
Solution. Total cost (C)=x 2 +78X+ 2500
(IC
Mar g inal (MC)-rd_ =-2x+78
600-p
Demand function is x
Sx-600—p
p=600-.8x
Ac g -28 BUSINESS MATHEMATICS

Now total revenue for x sets is


R=p x x==(600-8x) x=600x-8x

Marginal revenue (MR).r_ : f (600x_8.vt)=600'_l6x ...()

Net revenue will be maximum at the level of output, where


MR=MC.
2x+78=600-- 16x
l8x522
522
X
18
Flucc in order to maximise his profit, the manufacturer should
manufacture 29 sets per week. Also the monopoly price is given by
p = 600--Sx 600-8 x 29Rs. 368.
Alker. We know Net revenue=Total revenue—Total cost
or V- px-- C=x (600-8x)—(x-j78x±2500)
dP
For maxima and minima ; =0 * 600--16x-2x---78=0
(IX

or x=29
Renark. Also examine whether second order condition is satis-
fied ftL output level.]
Exaiple 19. The total revenue function of a firm is given a
=2Jq _qz and its total cost function as q3_3q2_7q16, where

q Is the o ' ttpu. Find


(1) the output at which the total revenue is maximu,n, and
(U) the output at which the total cost is ,nlni,num
Solution. (1) R-2I q—q2
Differentiating w.r.t. q and equating to zero, we have
dR
-2l —2q=O
dq

or q - - 10-5 is the necessary condition for revenue maxi-


2
m is at ion
To get the sufficient condition, we have
—2, a negative quantity, which means the revenue is

maximum atp=l05.
APPLIC,VI IONS TO UOMM}RCFt AND ECONOMICS ACIt-29

(i/) C - q33q2 7q- 16

.Ddlercntiating w.r.t. 'j and equating to zero, we have


- dc --- 3q 2 ---32q - 7 - 0
(l(/ 3
q---6q--7=0
(q-7) (q-1-1)---o
q --7 or q --- -- I is the necessary condition for cost
maximisation or minimisation. j --1 is not admissible as output cannot
be negative.
To get the sit fficic ii t condition, we have
/- c:-
2q--6
(Iq.-
/2C
1
2 x 7-6 -2, a positive quantity which means that
[ j
cost is minimum at q==7.
EN—ample 20. The unit (J('otonJ function is x - (25-2p), where x
is the nnnrbc'r of wilts and p is the price. Let the average cost per wilt he
R. 40 . 1/ni
(a) the revenue Ji.'nction U in terms of price p.
(h) the cost function C,
(c) the prot Junction F,
(d) the price per unit that maxw:fzL'.c the profit function, and
(e) the tna.vi,nu,n proit.
Solution. (a) .R(.v) --- vp= j (25--2p) p--- (25p— 2p2).

(h) Q. 40x-40 (25-2p)= (25 —2p)

(c) P(x)= R(x)_.C(x)

-- (25p--2p)-- (25-2p)

25p 2p 2 1000 80p


3 - 3+3

= j [-2p+105p-1000]
(d) The derivative of l'(x) is

(-4p+ 105)

ACi- 30 BUSINESS MAJ1-EMATICS

Solving the equation P(.)- 0 we find that

p 2625

Using second derivative test, we have


P'(x)= ---- <0
Maximum prolit is found when j) -2625
(e) Maxinuini profit is
)2
2 105 - )_-I000.]=l2604
-4-
Example 21. The deminid function faced by a firm is p -500-0-2x
and its cost function is C 25 .k 1. 10 , 000 (p price, x - output and C.-=cost).
Find the ouijflit at Winch (lic profits of the firm are maximum. A iso find
the price it will charge.
Solution, Revenue, R(x)=p.x- x(500 --02x)--500x-- -02X2
Profit.- Revenue - Cost
P(X)r=R()_ C(x)=- 500x--02X2 ---- (25X+ 10,000)
r=475 v- 10,00-02x2
For maximum or minimum
/J)
—=-475-02x 2x475-.04x:0
dx
= 118750.
d P
Also
fx2

Hence the profit is maximum when the output (x)= 1 18750. At


this level, the price is given by
p='500-02x
=500-02 (118750)=26250.
Example 22. ABC Co. Ltd. is planning to market a new model of
shaving razor. Rather than set the selling price of the razor based only
on production cost estimates, management polls the retailers of the razors
to see how mao razors they would buy for various prices. From this
Survey it is determined that the unit detnwui function ((lie relationship
between the a/no ii,!! x each retailer would by and the price p he is willing
to pay) is
-1500 p+30,000
The--fixed costs to the compan y for product ion of the razors are found
to be Rs. 28,000 ann' the cost for material and labour to produce each razor
is estimated to be Rs. 8-00 per itriii J-"ji0t price should the company
charge retailers in order to obtain a tnaxfnum profit ?
APPLICATIONS to COMMERCE AND ECONOMICS Au-31
U mt on. Let x denote the number Of units produced, and C'
'' note tIiC cost (-)F proiliictiOii to the compsny, and let p denote
price per Unit (iii rupees).
Then tilecost C is given by CRs. 8x1-Rs. 28,000
and the unit demand is -v 1500
- p f- 30,00)

Substituting, we find thi:tt the cost function C(x) in terms of the


Price P per unit is
C(x) 8 (-1500 p 1-30,000) 1-28,000
C() - 12,000p -i-- 2,68,000.
The Iflolley derived from the sales of the shaving razors as t
function of the price p per unit is the product of the LflrlibCr sold by
the price per unit, i . e., tine revenue function R() is

R (- s ) --(---1 Sot) p 1-30,000) 1)


1500 p -1- 30,000 p.
1h ,, profit P to the company is merely the difference between
revenilte (money derived from sales) and total cost, i.e., tine profit
function is
P(x) .R(x)—C(X)
- 1500p-( 30,000 p) —( 12,000 p +2,68,000)
— 1 500 p' + 42,000 p— 2,68,000.
The derivative of P(x) is
-3000 p+42000,
Solving the equation, l''() =0, we find that
v- 14.

Using second derivative test, we have


l''L)= --30(R) <0
Maximum profit is found when p = 14.
The profit for this price is
— 1,500 (14 )1 + 4 2, 000 x 14 2,68,000
Rs. 26,000.
The number of units sold at this price p is
x=-(-1500)(14)-1-30,000 = 9,000.
Example 23. .4 company making a single product has manufactu r -
ingrind distribution divisions . Stock of finished goods are not he/of, all
production being to order.
The average net revenue per u'ii, allowing für quantity discounts,
is Rs, (/00 - 0-01 Qi where Q is the qucmxicy sold.

ACE-32 BUSINESS MATHPMATICS


The average variable costs per unit for the IWO divisions are
Manufacturing == Rs. 10 - 1-Rs. 0015 (2
Dis%rjbuj ion Rs. 2-1-Rs, 0001 Q.
i lie Jixed costs per annum are
Moitufacturing - Rs. 40,000
Distribution Rs. 20,000
You are required to calculate
(o the oplinit on annual production quantify to maxrvnise the profit
of the company
(b) the profit of the company at the level of activity in (a) above
() r e animal product ion quantify to maximise the manufacturing
dii Ision's profit if it has been instructed to transfer the product to the
distribution (lii isbn (it R.Y. 73 per unit.
d) the profit of the company, showing the results of the two divisions,
at the level of UC(jvjlV in (c) above.
'ut1(,n. (o) Profit Revenue Varjahje costs j:J costs
P:()(l()0__UO! Q)--Q10-f 0015 Q)-Q(2-{ 0001 (2)
(40,000 t-20,000)
P. .8 (2 -0'026 Q'--60,000.
I 'r maxIn1cat!on, we havr
-0052 Q 0 (2 - 1692 units.
- --

Also 0 052<0.

Profit max irnisation out put is whcn (2 :1692 units.


(b) When (2-- . 1692,
Profit-88 (1692)-0026 (1692) 2 60,000- Rs. 14,46154
(e) If the manufacturing division arc to transfer out at Rs. 73 per
Unit, we can express their profit as
Profit 73 x (Production quantity) -Manufacturing variable costs
- Manufacturing fixed costs
Prr73 Q—Q (10 1 0015 Q)-4,000
P=63 Q-0 ' 0 15 Q!_ 40,000,
For maxiruisation,
dP
003 C) O * Q = 2100 units.

Also —003<0,

APPLICATIONS Ti) COMthRCI3 AND ECONOMICS AcE-33

(d) (2oinpaiiy Profit (Part a)-88 Q--()'026 (22 --60,000.


if Q-2,100 ; Profit=- 88 (2, 100)— (Y026 (2,100)2
—60,000 Rs. 10,140.
Manufacturing Profit (part (7) -63 () 0'015 Q1-40,000
if (2 2100, Profit 63 (2,100) - 00! 5 (2,1 00) 2 _ 40,000
=Rs. 26,150.
Distribution profit - - Revenue —Ti ansfer cost arid Department Cost
.Q (100 00! (2) -1 7 3 (21 (2(230001 Q)-3-20,000
100 (2 001 (j — 73 (2-2 (2- o . 00l Q— 20,000
()011 Q2 Ll. 5 (2 20,000
if Q- 2,100 ; Profit --001! (2, 100)' 25 (2, 100)--20,000
-----Rs. 16,010.
This shows that there is a loss of Rs. 16,010 from the tiistrihution
unit.
Example
pIe 24. It is given that a cletna,rcl curve is ccmvc'x from below

(_-
> o) at all patius. Show that the marginal revenue curve is also

convex from be/ow either if- - is posi11 re or if __ is negative and is


dx clx
nutnerrcally less tlra,i If the ,/ema,r/ curve is' alwa y s concave

from below, does a similar property hold of marginal revenue curve ?


(Delhi Univ . /1 A. (/fomis) Economics 1991J
Solution. Let the demand curve be
p--J(x)
4
i
veand
412p
--------lye..
dx dx .c
Then, we have TR=_p . x

d(TR) - MR - p . 1 +x .

c/(i'1R) Jp dp dp
dx - +-
dp d2p
2. 1+x '
d 2 (%1 R) cl2p c12j:i -x
an

d2p dp
=3 +x

Ace- 34
BUSINESS MAT1 IF MAli CS

For AIR to be Convex from below

(f J) d1p
3.iX.-----'->()
(fyi
dx-
d1p
B
Rut _0 (given).

So for MR to be convex from below either


(1/)
or if
l3p
S negative then it should he numerically less
3 dip il2(MR)
than - so ihat --------t),
>
A ax'- dx-

1-or .ic l'niand cm ye will be negative, so for MR to be

dp d p L
Concave Iron-i below we should have either --- n egative or it

positive then it is numeric-ally less thn


3 (11P
a - -

Extimplc 25. The production Junction oJ a com;noduy is given by

Q =40E-312__

where Q is the total output and F is the wilts of input,


(1) Find the number of units of input required to give fliaxitnu,n Output.
(ii) Find th mi:i 'nlI,n value of marginal product.
(iii) Verify that when the average product is niuxinjurn it is equal to
marginal product

SoIut j 0xj, (I)


- 401 6P - 40+ 6F

(First order Condition)


For maximum or minimum
40+ 6F_F2=()
(F+1O)(F--4)=0
or F-4
F=-10 is not admissible as input cannot be negati\e.

6— 2 F (Second Or(li'r condition)

[ iF=4 62 (4)=-2<o.
Hence output is maximum when 4 units of input are used.

APPLICATIONS TO COM%1IR( P AND tCUNO?IICS

(ii) 1P.- 40 OF f?

d(MP)
For maximum or Ilunimum -. --- 0 21 )

1' 3.
(1 , !')'1 I')
A'so -2<0.

Hence maximum value of marginal product is when input is 3 units.


Value of marginal product-=40 4- 6 x 3 ­ 3 2 -=49 units.

(iii) Average product (A)')

40/' 31 it
- - 401 3F
For maximum or minimum
d(JP) 21
dF33

d(AP) 2
Also IE - -
<0.

Average product is maximum when F--fl

. 9
Average product ( hcn I' -- 45

-40 1 3(-)

Marginal product ( when Al' is maximum, i . e., F ­ 4'5

40+27-- -- 4675.

Example 26. The quantity cold q and the price p are related by
q = ae - b P
m are
The production cost is given by C(q) —1 -mq ; a, b, I and
positive constants. Find the optimal price which ,na-dUJISCS the profit ?
Solution. Profit P= Revenue--Cost pq —(1 -inq)
pae —( i -f- inae) =-ae (p--- in) —I ..

BUSINESS MATHEMATICS

Differentiating () wi .1. p., we get


dP
c/p _a/) e- (P--in)-1 Ue-'•

:OCbp (—hp+b,n I) (*)


() gives ae (-bp-bni 1)=O.

For any finite value of p , e_tP:1—O.


hp -4 bin + I ft
• I4-bin I
/3

Dilit r ( ) with respect to p, we get

e (p--in)—abe- 1 -abe

:ah C_bP

When p -- in,

a/re b (-
+111 )[
h( k - ' _) 2
=013 c--1' ( - I)
== —ab e-n -mi<O , since a, b> 0.

m maximises the profit P.


P ae- 1 (p - rn)--1

p=ae_b

e'm) l•

Example 27. A monopolist firm has 1/Jr following total cost and
demand functions
C=ax 2 +bx+c, p=(3—x.
J'/rat Is the profit maximising level of output when
(i) The Jirin is assume/ to fix the output
(ii) The firm is assumed to fix the price ?
S o lution, When flrn,fixes the output level:
Revenue (R)-=px x (p—ax) - x— x2

APPLICATIONS TO COMMElCU AND ICONOMICS

(14
MR -

Total Cost (C) =ax I -1 hx+c

MC - = 2ax -F b
1x
Now condition for profit maximising output level is
MR- Mc
i.e., 13-2zx=20x lb
[3—hrr2cix+2aX . -2X (a j a)
-b
x
2 (a - )
which iS the profit maximiSing level of OLltpUt.
(ii) When firm fixes the price : ti this case the total revenue and
cost are put in terms of price p.

Now p -f3—x -> x- p


a
R:.=p=p(±T)

( JR I
MR ( p. - - 2p) (*)

Also C=dX'i-bx+c

h
fp L c a
For profit maximisation A'fRr - MC

---(3-2p)-

2p 2a-
- -- — -H-
2ap 1
a ± --i- ---- -
ci. a

c3 4 2a -F ab 2pz ± 2ap 2p(a -F a)


a2
a-4-2a-f-ab
2(a)
ACE-38
1flJSNriss MATHEMATICS
Nov the de mt n d function is
p.
3 -4 2ci3 - - =
2 ( ( i)

a13. 2a3 1h 22aç3–$_b


2 (c-3- o ) 2 (cc +a)

±±_.
2 (x+(z) 2 (-i-f-cc)
which 9"s the scii level of output wh(u the firm assumed to fix the
output it-vel.
Exit mpl 29. -I 1Wiiop/ist 111lS (;)tUI Cost ficnctio,z C ax 2 -j.
and if 1JC11?cIlil 1OIY is p bx-f-c
•c.V, s/;occ' f/lit (/' tcutput for maximum revenue
is

Solut inrc - Total x ---


Net reenue Total revenue --Total cost
R ( -cc)---(cix2-f-h -f-c)
For 1 1 i:cxilnutn or rnnumurn
JR
- — 3cx-2ax---o—Q
lx
or x2-f-2ax—(3—b)=()

—2 cz f-' / 4a'-j . 4x 3x (13---b)


or
(kr.

- —a+\(277, (s—h)
3(
-/a2---3 (13-- b)—a
• •
3c
-a—
or ---- this value of x is
3
not admissible as output cannot he negative.

Also

243_(13---b)—a
When x=— ----, we have

ff ---2 (Va2+3 (13—b)—a)--20


APPLICATIO N S U) COMMhRCt ANt) 1C)NOMICS ACI1- 39

—2 s/a) 3 1) .
I fence the net ICVCT1UC is IflaXlIllIJfll svltcn the output IS given by

(- /)_/
x-
3x
in pie 29. file Wilt! CO1 /wict ion of a/ni/I iv
5x- 2$x 1(1
where C is 10(111 Cost Inn! .e is vulpat -i f(1 of f/Ic rate of Rs. 2 per li/lit
a! alit/mt is im/)OStfl/ and the .' roiliic€'r I/dill it tO Iii: co t . 1J the mirk Ct
ilen1vj /iiIlC(1o,l is
p -25!Q---5x,
where Rs, p is the price i.,nir eti oiir .f0w .fT/nd i / ic prftfit (naximtsunj ,(Ut-
['l/t and price.
I ( 4 I/li,rnIt'/wtc iI(J V 1990]
Solution. Foul revenue function, I R - - (23() - 5x) .v

After inipo5itlon ot 1 LX of Rs, 2 per flint,

Total cost In flcliOtl, 7C'r -- 5X 2 1- 2i.v -- I) -- 2

Now Profit, 1'- TR -- TC


..-
(2530xS--(S— 5x 30x f- 10)
For maxinhisation, WC hurl
('P
- (2530 l O x) —( x2 lOx 30):()
(1.V
=

X: -0
Out s -. ----50 is not :ulmi;sib!e as olilput can not be negative.
d2P
and
(IX
Profit maximising output is SC) 12nit.
Price WI/Cl) XS0 is p-=2530-5 x 50Y22SQ.
En)le 30. Suppose the demand (1,1(1 total cost functions of a
monopolist are p 20 -4.y (2/UI Tc= 4x -t-- 2 rL'spectil'h ishere p is price and
.\ is ql/(J?liiU. If the government i/?i[0 (Is tax at the rate of 20' of sales
i1€'tcrinnC (III 1(11(71 tax rClefl!ii' 117111 the ljovern,ncut ivill Ire able 11) collect.
I D('l/il (Jflj/ B. Co,,r ( 110/75 ), 1992]
SlJtlr1t1. WC ale givL'l that
p-20-4x and ['C-4x±2
Total Revcnuc== TR=rpX=2O -- 4x1

ACE-40 BUSINnSS MATHEMATICS

Tax-20 of TI? -- (20x-4x2)

Total new cost TC+Tax4x+2+ - (20x-4x2)

2+ x 12

Now, profit P Total Revenue ---Total new cost

:(20X j2) X28x+2)


(

v2fl22

dP 32 (I'P 32
--xf-l2 and
3x 5
dl' 32 12x5 15
=0 gives ---- x-l-12r=0 or "
v

will give maximum profit.

Also, x will yield the maximum tax

Tax when . !. given by

-[ 30x
15
6

Hence tiie government will he able to collect as tax revenue.

Example 31. Given the demand and cost functions


P=20-4x
C— 4x
(a) Find the optlrnuni quantity, price and the profit on this level.
(h) What will he the new equilibrium after a tax of Es. 050 is
Imposed'.'
(c) Determine the tax rate that will maxfrnfse tax revenue and deter-
mine that tax revenue
(d) Find the total tax revenue if in addition 10% sales lay is
a Iso impose/.
APPLICATIONS TO COMMERCE AND ECONOMICS ACE-41

Solution. (a) TR: 20X —4x, MR =2() —8 v


C--4, MC-=4
For optimum level, MR = MC
20-8-'C-=4, I.e., xrr2
p-12
(b) After Tax, C4x+05X
MC=45
At the optimum level MCMR
20-8x=45, i.e., when x-31/16==194
p=1225
(c) Tax revenue is maximum where
MR=MC (after tax 1)
20-8x=4+t x=(16—t)/8
New price after tax is
P-20-4( 2-- I2+-)

Thus, the increase in price is half of the tax imposed and profit
after tax is
Profit (P).- 2R—TC
=(20_4x2)_(4x+1x)=x(16_4x__1)

Substituting x= 1-—, we obtain

16—t i (16_t)2
Maximum profit=
r
161_12
Tax revenue

dl'
I will be maximum where - =-O and ------<0
a! (i!
16-2!
I-

!16-8
/\
Maximum tax-=1x=8 —)=8

(ci) With sales tax of 10% the net TR is


TR. 0Y90 (20--4x) x
MR=090 (20-8x)
At optimum level, MR=- MC
090 (20-8)=4 x-140/72--1'94-
ACE-42
BUSINESS MAT1II MATICS
Example 32. XYZ Company , as a result of past experience and
estimates for the future ,
has decided that the cost of product (on of thei
sold profuct F, an advanced process mac/tine , is
C - 7 :1064+5x+0-04 x,
where C=tota/ in cost '000 Rs.
Xr== quantity produced (and sold)
The marketing department has estimated that the price of the
product is related to the quail! ity produced and sold by iFe Pqua(1on
I'•-I57--3x,
where P== Price per unit in '000 Rs.
x=quan(ity so/cl
The government has proposed a tax of Rs 1,000 per unit on product
P hut it is not expected that this will have any effect on the
costs in-
curred in making F or on the demand price relations/tip Find
(a) the price and quantity that will nirlximise profit when there was
no tax
(b) the price and quantity that will 'naxi,nise profit if the proposed
tax is introduced
(c) how much of the tax t per unit is passed on to the customer
(d) the effect on the profit of the company if t was fixed at Rs. 4,000
per unit,
Solution. (a) Profit (Y)Rcvc 1 ue (R)—Cost (C)
Revenue (R)=Price ( P )xQuantity (x)
r(I573x) XlS7x-3x2
Cost P = 1064 4- 5x 1- 0'04 X2

Y=- I 57x-3x'-_-(1064+5x+o.04xt)
—304x2+J52x--1064
Differentiating Y W.r.f. x, we have
dy
dx 152— 6'08x= 0
X ---25 units
d2 }'
xi = -608-<o

Profit is maximum when 25 units are produced.


Now =157-3x25 in '000 Rs.
=rRs. 82,000 per unit
and Y= 152 x 25— 304x (25)2__ 1064
Rs. 836 in '000 Rs.
==Rs. 8,36,000.

ACE-43

APPLICAT I O NS TO COMMERCE AND ECONOMICS

(b) When a tax t) is jntoducd,


Y= R - C becomes
Y=(157 -- 3x)X - (1064+5x+004x2±t)
= I52x_304X 2 -1064 —t -
rentiatfl9 Y with respect to x and setting to zero, we have
dy =152 —608X---1=0
dx
152-1

dzy 152—'
when
dx
For imiximum profit,
Quantity (x) = 25- 6-. 0-
8 units
Substituting for x in the price equation, we have
F= t57-3 ( 25— )=82+

or
(c) The amount of tax passed on to the customer is
approximatelY 4934"o.
t4.
(d) When the tax per unit is Rs. 4,000, then
or 24 in whole units.

Now Y=152x-3 04x2_1064tX


=(152X 24)_304X(24)2_1064_4X24
=73696 in '000 Rs.
=Rs 736,960
8,36,000
The profit without tax in (a) above==R3.
Profit with tax of Rs. 4,000 =Rs. 7,36,960
Difference to profit = Rs. 99.040.
33. A monopolist's TC=aX+bX+C
total cost andis the
Example Is p - , where x and p denote the units of owput and
demand functio n and are positie constants. If the govern-
011(1 a. b, C.
price
mentres'peCtivelY
imposes tax at the rate oft per unit of output, show that time total tax
[Delhi Univ. B.Com . (Hans.). 1991
Is mna.ximU'fl when i=(—b)Q.
t per unit, the total cost
Solution. After the imposition of tax,
function, TC, is given by
TC__ax2+bx+C+x
Revenue function= R =px( — x) xX—ctX

ACI3-44
BUSINESS MATHEMATICS
Profit function P_(p _c(x $ ) . ( QX 2 fb V + c 1-tx)
For P to be maximum,
First order condition
dP
- =0, i.e., (3-. 2ax)—(2ax-1-b+t)O
dx

Second order condition


d2 J)
—i <0

dIP
—2- 2a= —2 (a + a)<O as a and a are positive constants.

Therefore, the level of output that maximises the profit is

2(c - a)
The total tax revenue for this level of output is
T=t1JPL't
2(a + a)
For T to be maximum,
First order conditloti
dT I.e., 13--b-2t
dx ' 2(°
or f=--- (f3-.b)
Second order condition;
d2T
ax <0
____2_

d2 7' I
d-' (+a) <0 as a and a are positive constants.
Hence the tax rate 1 that maximises the total tax revenue is

Example 34. There are Iwo duopolists manufacturing equal and


identical bicycles. The total Cost of an output of x bicycles
Xt
per month is
Rs. - 3.100 ) In each case When the price Is Rs. p per bicycle the
(
market demand is x- 75 -- 3p bicycle per month. Find the total equilibrium
Output per month.
Solution. Let x1 and x, denote the output per week
of the two
duopolists. Then x=x 1 -f-x,, is the total output.

APPLICATIONS ro COMMERCE AND ECONOMICS AC-45

Demand function
X (x-4-x)
— 25•-- -25-------- a—
- 3__ 3
Net revenue for the first firm
+
Tfj^ - R1 - - C1 25.Y1 —

28.x 1 2 x1x,,
(X 1 X0.11 — [^!-2
3 25
+ 3 v, 4- 100
j
75 3
For maximum net revenue,
x dxx2
75i
d.r1 3 • dx1 3

But the conjectural variation 0

(1)
75 3
Similarly, we can show, by considering the net revenue for the
second firm, that for maxirnu tn net revenue, with conjectural variation zero.

The equilibrium output of the two firms in duopoly are the simul-
taneous solutions of (1) and (2). They are
X 1,- 51150
511
f-
511-
i.e., X1=x2-20'37, approx.
Total output per week is 2(2037)--41 (approx.)

EXERCISE (II)
1. A man producing very fine earthenware lampstands found that
he could sell on an average of 4 stands per day at a price of Rs. 18 each.
When he increased his output to an average of 4'5 per day he could only
obtain Rs. 175 each, if he were to sell all his output.
Assume that he maintains no inventories, so that he sells all he
produces, and that the appropriate demand function is linear and is of
the form
x==a-4-hp
where a and i are Constants, x is the average number sold per da y and
p is the price. An accurate survey into his total daily production costs
produced the relationship
C_ x 2 — x+54
ACF.-46 I3USINESS MATHEMATICS

between the total production cost, C, and the average daily production x.
Required : (a) Determine the demand function giving the average
number sold per day, x, in terms of the price, P.
(b) Find an expression for the gross profit per day in terms of the
average number of stands produced and sold.
(c) Find the profit when 6 stands are produced and sold.
(d) What is the average number that must be produced and sold
for maximum profit?
[Hint. (a) Demand function :
When price p=Rs. 18, demand 4 per day on average
4:a+l8b (*)
When p=Rs. 175, dcmnand=45 per day on average
(*$)
45-=a+175b
Solving (1 and ($*), we get
a=22, b=-1
The demand function is x=22—p.
(b) Profit P= x(22—x)—( x 2 — x+54)
=-,3 x-1-22 x---54
(c) Gross profit when 6 stands are produced and sold is
=—X(6) 2 +22 x6-54=Rs. 27
(d) To maximise gross profit
dP j2p
—=0 and -j <0
dx dx
i.e., if x=7j then the maximum gross profit would be
-3 (7) 2 +22x 7— 54=Rs 3038 per day.]
2. Let the unit demand function be
x = ap -I-b
and the cost function be
c- ex--f
where x - sales (in units)
p = price (in rupees)
f= fixed cost (in rupees)
e=variable cost
b=demand when p=0
a=slope of unit demand function
APPLICATIONS TO COMMERCE AND ECONOMICS ACE-47

(a) Find the cost C as a function of P.


(1') Find the revenue function R(x).
(c) Find the profit function P(A).

3. (a) A man derives Rs. x from his business this year and
Rs. y next year. By alternative use of his resources he can very x and y
according to the following relationship,
X
y --l000--

What is the income this year if he plans for zero income next
12
year ?'' and - Y
Derive What is the shape of the businessman's
transformation Curve '?
(b) A sugar mill has total cost function given by
j2 }-5x+200)
where x tons of sugar are produced per week. LI the market price is
Rs. p per ton, what is the supply function of the firm ? What is the
average fixed cost ?
4. (a) A business produces an income of Rs. .v this year and
Rs. p next year, where these values can be varied according to the
{( dy
•y ) -
XZ ^
relation y— 100— Explain how i can be interpreted
250
as the marginal rate of return over cost. Show that the value of his
x— 125
marginal rate is when this year's income is Rs. X.
125
dip
(b) It is given that a demand curve is convex from below >0 )

at all points. Show that the marginal revenue curve is also convex from
/2,
below either if %--. is positive or if - is negative and numerically

3 d2
less than - . —i--. If the demand curve is always concave from below,
X(IX
does a similar property hold of the marginal revenue curve 7
S. Show that the demand curves

'° c and p=(a—bx)2


a
are each downward sloping and convex from below. Do the same
properties hold for the MR curves ? Show further that, for each of
ACE-48
BUSINESS MATHEMATICS
thc demand laws p=- va— bx and the demand and MR
curves
a, are downward sloping and concave from below. Assume that
b, c are positive.

(Hint A curve y.=-fix)


is downward sloping if < 0 ; and
convex Ironi below (or concave from below) if
'Iy (Iy
>0 (or -j— <0
6. (a) For a unit (fen)and function of
)J
p 24 -- 8x, where x is the
number
function of units in thousari(ls and p is the price in rupees, find the sales
If the average cost per unit is Rs. 8, find
(a) The profit function.
(b) The number of mi l ts that max mule the profit function.
(C)
Graph t lie cost and revenue functions.
[Flint P(x)R C(x)(24 8)x-- 8xJ
(b) If the to al cost function of a firm is

5x 2 1 30x+ W.
where C
is the total cost and x is the output, and price under perfect
Competition is given as 6. find for what value of x the profit will be
maximised. Exaudne hod first and Second ordei COfldjtjOflS.
. If the demnaiid function for a commodity is given by p 1 2e1'
"' here p is the price per unit and q is the number of units demanded.
D eter-ine the price and the quantity for hich the revenue is maximum.
int. Revenue function is given by
R-pq . l2qe1'
For R to be maxit-nurn
dR (/21?
-= 0 and <0.
dq
dR
12[qe (cili(_]14) e / I )1 l2e°/" (v)]
S. State the conditions for a m aximumprofit. Find the profit
maximising out-put level if p200— lOx and
AC=i0-f. .

. Suppose the W ill cost function is given by Ca-


bx4- cx 2 ,
wricre .r is the quant i ty of output produced. Show that the slope of the
average cost curve is 01C—AC), where MC= Marginal cost and
AC=Average cost.
•P PI.ICATIONS TO COMMERCE AND ECONOMICS ACI3-49

10. A firm produces an output of x tons of a certain product at a


total Variable Cost given by -- 4X 2 4--- lx, Find the output at
which he average cost is the least and the corresponding value of
the average cost.
11. A company notices that higher 531es of a particular item which it
produces are achieved by lowering the
price charged As a result
the total reve,e from the sales at first rises as the
number of units
sold increases, reaches a maximum and then falls off. This pattern
Of the total revenue is described by tile relation
Y 40.00,000 (x— 2000) 2 where .v is the total revenue and
X the
number of units sold.
(i) Find the number of units that maximizes total r(venue.
(ii) What is the amount of maximum revenue ?
(iii) What would be the total revenue if 2500 units are sold ?
FAns. (i) 2000, (ii) Rs. 40.00,000, (iii) Rs. 37.50,0001
12. If the est function is C(x)^4x f 9 and the revenue function is
9p x, where x is the number of units produced (iii
thousands) and R and C are nicasired in millions of rupees, find the
following
(ci) Marginal revenue.
(b) Marginal revenue at X - 5, .v 6.
(C) Mcirina1 cost.
(d) the fixed cost.
(e) The variable cost at x-- 5.
(f) The break-even point, that is, R() C(x).
(g) the profit function.
(/t) The most profitable output.
(i) The niaximum profit.
(j) The marginal revenue at the most profitable output.
(k) The revenue at the most profitable output.
(1) The variable cost at tile most profitable output.
13. Suppose the cost funct i on is given by C(X)_X2 -I-
5 and the price
function is p= 12— 2x, where p is the price
in rupees Alld x is the
number of units produced (in thousands). Answer the question
asked in Problem 12.
14. A Company has for x items produced the total cost
C and time total
revenue R given by equations R=3x and C=
1004 52, Find
how many items he produced to maximise the profit.
What is this
profit 7
15. A sofa-set manufacturer can manufacture x sofa sets per
week at a
total cost of Rs,
(_ x2 +3x+ 100
)_
flow many sets per week should

ACE-SO FIUSLNESS MATHEMATICS

he manufacture for maximum monopoly revenue when the demand


law of his product is X =l0\/25_p set per week. Also find the net
revenue with this output.
16. The cost function C() for producing x units of a commodity is
given by

C(X)=r -_ x- 5x2 +75x . 10

At what level of output the marginal cost (i.e., 2. ) attains its


dx
minimum ? What is the marginal cost at this level of production ?
[C.A. Intermediate, November, 19911
[Ans. 5, 50]
17. If q be the number of workers employed, the average cost of pro-
duction is given by

C_ 2(qJ4.j-+24

show that q=425 will make the expression minimum. In the


interest of the management will you then advise to employ four or
five workers ? Give reasons for your answer.
t!.C.W.A., June 19901
3
[Hint.
Hi C_2(q4)+24

dC —3
* /q2(q tic4)2+24

C will be minimum if a- 0

when 2(i _ 4) 2+ 24 ° - q=425

Since the function C is not defined at q=4, therefore, the value of q


must be 5.]
18 The following expressions define a firm's total revenue and total cost
functions:
Total revenue= 18x—x1+24
Total cost= x3-25x2+50
(a) Use calculus methods to find the optimum production level.
(b) State the firm's profits at the optimum production level.
(c) Using the same axes, sketch the graphs of the total revenue and
total cost curves, indicating the output at which profit is maximum.
[Ans. (a) 6, (b) 641

APPLICATIONS TO COMMERCE AND ECONOMICS ACE-51

19. A steel plant is capable of producing x tons per day of a low grade
steel and y tons per day of high grade steel, where If the fixed
market price of low grade steel is half of the high grade steel, show that
about 55 tons of low grade steel are produced per day for maximum total
revenue.
[Hint. Let p 1 be the price of low grade steel. Then 2p 1 is the price
of the high grade steel, p 1 is constant.
(40-5X\
Total revenue function, R2p 1 ,-j-----)±xp1
dR
Show that t) - X=10+2'5
dx

Further show that $<O for x= 10 2V5 and

->0forx==lO-I-2V5
I
20. Maximizing Profit . A tractor company can manufacture at
most 1000 heavy duty tractors per year. Furthermore, from past demand
data, the company knows that the number of heavy duty tractors it can
sell depends only on the price p of each unit. The company also knows
that the cost to produce the units is a function of the number x of units
sold. Assume that the price function is p=29,000-3x and the cost
function C2,000,000 '20,000x+5x 2 . How many units should be
produced to maximize profIts?
21. A manufacturer estimates that he can sell 500 articles per
week if his unit price is Rs. 2000, and that his weekly sales will rise by
50 units with each Rs. 050 reduction in price. The cost of producing
and selling x articles a week is C(x) 6200 i , 6 , 1 Ox + 00003x2. Find
(a) The price function.
(b) The level of weekly production for maximum profit.
(C) The price per article at the maximum level of production.
22. A trucking company has all engine overhaul cost of
Rs. 1000 and routine maintenance cost (in rupees) of C - -- 0 , 40X +10- 1 x2.
where x is the interval in kilometres between engine overhauls.
(a) Show that the total engine maintenance cost Rs. (per km) is
given by

04+ 10- Ix
(b) Find the rate of change of the total maintenance cost with
respect to the engine overhaul to interval
Wx-


ACE-52 BUSINESS MATHEMATICS

(c) Find the value of x at which the derivative in (L.) is equal to


zero.
(d) Evaluate and compare c for x==5,000 ; 10,000 ; 20,000 kms.

Ans
l0
(b) c/c 2 + 10, (c) 10000, (il) 029, 024, 029.
r
23. Given p==20--q
C==2-j--8q-}- q2
Find
(a q which maximizes profit and corresponding values of p(=Price)
R( total revenue) and Al ( = Profit).
(ô' q which maximizes sales (total revenue) and corresponding
values of p. R and Al.
(c) q which maximizes sales subject to the constraint M8 and
corresponding values of p and R.
24. A monopolist has the following demand and cost functions
P=30---q
C-- 160--8q
The Government levies a tax at the rate of 2 per unit sold. Find
profit rnaximi7ing price and quantitY after tax levy.
[Ans. p 10, p20)
25. A firm has the following functions
p=100--0'01 q
=50q+30,000
and a tax of 10 per unit is levied. What will be the profit maximizing
price and quantity before the tax and after the tax ? Which does the
monopolist find it better to increase the price by less than the increase
in tax ?
[An. Before tax q=2500, p75, Profit=32,500.
After tax q=2000, p=80, Profit= 10,000
A price higher than 80 will reduce profit below 10,0001
26. If the relevant position of the demand function is
P-100-0 , 01 q
when q is weekly production and p is weekly price and cost function is
c= SOq+ 30,000
(a) Find maximum profit, output, price and total profit.
(b) 11 suppose government decides to lev y a tax of Rs. 10 per unit
of product sold, what will happen to pride, quantity sold and total
profit ?
APPLICATIONS TO C0MMnRCB AND ECONOMICS ACi-53
27. (a) Given the demand function p(10_x) : and the cost function
C=-55x-8., find the flaXLinum profit. What would be the effect of an
imposition of a tax of Rs. 9 per unit quantity on price ?
[Azis. 54 ; Price increase= 15].
(I') Given the demand function Y r-=20_4x and the average cost
function Y=2, determine the profit maximising output of a monopolist
firm. What would be the impact of a tax of Rs. I per unit of output on
profit?
28. A monopolist has a total cost of output x given by ax" 4- bx±c
and the demand price for the output x is given by (3—ax. Find his
monopoly output, price and net revenue in equilibrium. How will these
change if a tax at Rs. I per unit of output is levied 7
[Ans. Before tax

Output= -------- Price 2af3-J- a13j/


2(-l- a) 2(- a) --

Net revenue= 3—
4(a-{a)
After tax:
bt t
Output = Price=
2(ot 2(cx

Net revenue= _t)2


4( oc i

29. A monopolist firm has the following revenue and cost functions
R=—Q'-f-3Q, (x, 3>0)
C=aQ+bQ-{-C, (a, b, c>0)
The government plans to levy an excise tax on its product and wishes
to maxirnise tax revenue T from this source. What is the desired tax rate I
(rupees per unit of output)?
30. (a) The demand and cost functions of a firm are given by
'1 = 10,000—
c= 59q+ 30,000.
and 100
where q---quantity demanded
p = price/un it
C-
--total cost,
Determine the optimum level of (j that the firm should sell.
(b) Assuming that the above firm has to pay a sales tax at the rate
of Rs. 10 per unit, find out the optimum sales.
ACE-54 BUSINESS MATHLMATICS

SOME APPLIED PROBLEMS


Example 35. Prove that a rectangle with sides x and y and a given
perimeter P has its area maximised if it is a square.
[Delhi Univ., B.A. (lions.) Economics 1991]
Solution. We have
P=2(x-l-y)

or VTPX

Px—x2

First orde r condition


dA
dx
c/A I
P— 2x
dx 2

clx 2

or x=-4- P

Second order condition


dtP

d2P
—2<0

x=4çP
and
Hence the rectangle has the maximum area if it is a square.
Example 36. A box with a square base is to be made from a Square
piece of cardboard 24 centimetres on a side by cutting out a square from
each corner and turning up the sides. Find the dimensions of the box that
yield maximum volume?
Solution. Let the volume of the box be denoted by V and the
d imensions of the side of the small square by X. Since the area of sheet
metal is fixed, the sides of the square can be changed and thus are treated
as variables, Let y denote the portion left after cutting the x's to make
the square, we have
Y.24-2x

APPLICATIONS TO COMMERCIE AND ECONOMICS ACE-55


Since thc height of the box is x and the area of the base of the box
is y, the volume V is given by
by V= V(x)= xy2
V(x)=(24— 2x) r-=4x3 96 2 ( 576x
To find the value of .v which maxjrnjscs V, we differentiate and find
the critical values, i. e.,
V'(v)r= 12x2-192x+576
I 2(x 2 — I 6X+48)== I 2(x— I 2)(x-4)
x= 12 is not admissible as in that case box cannot be formed.
x=4
Using second derivative test, we have
V"(x)=24x— 192
V "(4)=96— 192<0
Hence the dimension X-=4 maximises the volume and 4 X 16 x 16 are
the dimensions of the box.
Example 37. The rate of working of an engine is giien by the
4000
expression /0v-- , where v is the speed of the engine. Find the speed
at which the rate of working is the least.
Solution. We require to find the value of v for which the expression
4000is
lOv± a minimum.

Let

dH 4000
10 V2
(FV
dH
when v1 4OO, i.e., v=±20

v-20 is not admissible as speed cannot be negative.


vz=20.
d`2 1`1 8000
Akr
d2H
-dv->0
- when v=20

The rate of working, H, is a minimum when v=20.


Example 38. Afirni's annual sales are s units of a product which
the firm buys from a supplier. If the replenishment cost is Rs. r per
order holding cost Rs. It per unit per year, finel the economic order
quantity by using calculus, (Delhi Univ., B. C'otn, (lions), 1991J

AC-56 BUSINESS MATHEMATICS

Solution. Let x be the number of units ordered at any time. Then


the holding (storage) cost for x units is hx.

Number of orders ==

The total cost


lix- (-i-). r
Sr
or C=hx i

For C to he minimum,
First order c,Londix ion
sr
dc 0 ic. / — ---- =0
dx ' x2

I Sr
or X=AJ h

Second order condition


d'C
>0

dt C ..sr

Hence the economic order quantity is --. i.e., when


units are ordered at a time the cost is minimum.
Example 39, The production manager of a company plans to include
,180 square centimetres of actual printed ,natter In each page of a book
under production. Each page should have a 25 cm. wide margin along the
10/) and bottom and 2() cm. wide margin along the sides. What are the
tosi economical dirnenslor1s of each printed page.
Solution. Let x, ,v denote the length and breadth of he printed
matter in each page. Then
Arca of each pages AY= 180 .
Due to margin, the dimension of each page will be
x+2X 2X + 4 and y-j-2 < 25y+5
l.ct A he the area of each page then
A=(x+4) (y+5)=-xy+5x+4y+20

(*.)
APPLICATIONS TO COMMERCE AND ECONOMICS ACC-57

Differentiating (") w.r.t. x, we get


d4
—ç 720 —U
WT
x=144
x 12, discarding the negative value.
Using the Second Derivative Test,
dA 2 x 720
-= >0, when .v=12

x= 12 minimises A.
180
Substituting x-I2 ui (), we have

Hence the most economical dimensions are


Length =x+4=16 cms.
Breadth =y -f-5 = 2{) ems.
Exntple 40. Your company is planning to build a new factory.
The rectangular area required for manufacturing and office is /5,000
square metres. .....ar parking area to a depth of 50 metres is needed t
the from of the building, an access drive width of 15 metres is planned for
the side and a delivery/loading bay to a depth of 25 metres at the rear.
You are required to calculate the smallest total site the compalTy
should buy, to meet these requirements. Workings must be shown marAg
will be avtarcled for method used.
Solution. Let the length of the rectangular area required for
manufacturing and office be x and the width be y, then xy-151000
square metres.
x-i--50 4-25 is the length of the sides including the car park ano the
delivery/loading bay at the rear, I.e., x+75, and
y - 15 is the width of the site including the access drive. Thq area
of the whole site is then
A=(x+75(y+ 15)

v=(x4-75) (15000+15)
15,000
xy=15,000 .
I
5 000
=l5000+l5x+ 1 !_P_+ 1125

1A 1,1250Q0
Now --=0 15-- —_-=0
X72-
ax

ACE-58
I3USINESS MATHEMATICS

(12/1 2X 1,125,000
Also
(/12

Hence A is mjiijnuim when X- 273'86


15,000
Y-=5477

Hence t1e smallest total site the company should buy to meet its
requirement is

(27386+75) metres by ( 5477+ 15) metres.


F x i in plc 41. A metal box with a square top mid bottom of equal
is to conlam 1000 cc. The 'naterwi br the (op and bottom costs one
patsa per square cm and the material for the sides costs half paisa per
square cm. Find the least cost of the box.

Solution. As the base of the box is a square, the dimensions can


be taken as x, x, y. Then the volume is x 2 y.

x'Y= 1000 1000

Let C he the total cost.

The area of the top and bottorn=2x 2 sq. cm .


Cost for the top and bottom=1 >< 2X 1 == 2x2

Cost for the 4 sidesrr < 4x) , =- 2xy


Total cost, C-.=2x2+2xy (4.)
=2x 2 + 2x. 1000
X2

2 2000
(4Is)
tic =
x-
di
dC 2000
dx ewes 4x------..=O
x
4x 5 = 2000
500
X/55xi/5xI.5979S
4+ 2>( 2000>0 when x=795
2
(IX X
Thus x=795 minimises C.

ACF-59
APPLICATIONS TO COMMERCU AND ECONOMICS

Substituting x=-7'95 in (), we get

C.01 ,v -2 X (7 .95)2 + 37798 378 patse

A rectangular block with a square base has the total


Etrnple 42.
area of its surface equal to 150 square cms, and i/re sides of the base are
each x cm long. Prove that the volume of the block is j (75,v-- x 3 ) Cu.
cm., and hence Jurd tire volume of the block.

Solution. In order to obtain the maximum volume of the block


say V, we must first obtain m expression giving V in terms of one variable.
As is indicated in the question, Ny e will

first show that V= (75x--x3) where x is the


length of a side of the base.
We have V :Ir cu. cm., where It is the
height of the block.
10 obtain 11 in terms of x, we use the fact
that the surface area of the block is equal to 150
sq. cm . .
Surface area-- 2x'+ 150 I

x+2x/'7S
75_x2
2x

Hence V=X 11X2 2 - ._ = j (75x-- x 3 ) cu. ems.

1-Living obtained V in terms of one variable, we proceed to trii&t


maximum value in the usual way.
d 75 3x2
We have dx 2
d 75 3x2
----=0, when --.--=0
(IX 2 2
i . e., when v2 =2S, or x=-,-5.
In this case, the negative value of x has no meaning and we
discard it.
dV 6x
JT= - -- =-3x

(1 V
Whet x5, —i
(IX

d d2V
Hence, when x=5, - -r() and J2 is negative.
ACE-60 BhJtMP.sS MATHEMATICS

X=5 makes V a maximum.

.. Maximum value of V=-_ (375-125)125 cu. cm,

Example 43. A Wastepaper basket consists of an open circular


cylinder. If the volume of the basket is to be 200 cubic centimetres ; fi
the radius of Its base when the material used is a minimum nd

Solution. The mate iia. used in making the basket depends on the
surface area of the basket.

Hence we require to find the radius of the base when the surface
area is a rajilimum

We must first of all obtain an expression giving the surface area


(say sq. cia) iii terms of the radius of the base (say r).
The total surface area S==gr2 rh sq. em, where 'h' is the height
of the cylinder.
To obtain S in terms Of r alone, hi roust be obtained in terms of
r.
'This is done by using the fact that the volume of the basket is equal to
200 cu. cms.
We have, volurnc=2OOr2/z
100
/.
r2
Hence S=r'4-2rh

or 2
S=,tr8-f21r . .
7;r2
==r'
r
(IS 4
dr
(iS 400
dr 0 when 2r---..0

I.e., when r8= 200


—. or r.= iö

(/S 800
Also ----2n +

When r -' /200 dS .


dr
is positive, i.e., S is a minimum.

Hence, the amount of material used will be a minimum when

r= 3 —=3'99 cm.

APPLICATIONS TO COMMERCE AND ECONOMICS ACE-61

Eatuple 44. ABC Co. Lid, wishes to produce a cylindrical


container with a capacity of 20 cubic feet. The top and bottom of the
Container are to be mw/c of a material that costs Rs. 6 per square Jbot,
while the side of the container is made of material costing Rs. 3 per square
foot. Find the dimensions that will minimise the total cost of the
container.
S olution. Let denote the height of the container and r the
Ii
radius, then the total area of the bottom and the top is 2 i r 2 and the area
of the lateral surface of the container is 2trh.

Ar
Top

Laiernn
Bottom surface

The total cost C of manufacturing the container is


C=-(Rs. 6)(2itr 2 )+(R. 3)(2rh)
- - 12gr3-I-6tr/i
Since volume of the cylinder is fixed at 20 cubic feet, i.e.,
V=20=ir 2 h zlo Ii
rrr2
Substituting ($) in (, we get
C=12itr+6rr 20
-
ir2

=l2itr2+

To find the value of r that gives minimum cost, we differentiate C


w.r,t. r Thus
dC 120
dr r2
24rr8-12O

The critical values obey C'(r)=O


24tr3-120=0


AcI3-62 ]BUSINESS MATHEMATICS

5__

Using the Second Derivative test, we have

C"(r)= 24it

and C" ()=24+2.!>o

Thus for rl'l7 feet, the cost is a relative minimum. The corres-
ponding height of the cylindrical container is
2 20
Ii ==0
-
=465 feet
rtr2 n[5/itj-13

These are the dimensions that will minimise the cost of the
material.

EXERCISE (111)

I. An open tank with a square bottom to contain 4000 CC, of water


is to be constructed Find the dimensions of the tank so that the
surface area may be the least.
[Ans. Base dimensions 20 cm - IIeiIit 10 'n'
2. A rectangular box with no top is to be made from a rectangular
piece of metal with dimensions 32 cm by 60 cm by cutting equal
sized squares from tin' corners, then turning up the Sides. What
should be the side of the squares cut off if the box is to have maxi-
mum volume ? (Ans. 5 cmnsj
3. A company has scrap pieces of metal sheeting left over at the cod
of its production line. The company has no other use for the scrap
and it can manufacture new boxes on present underutilised machi-
ner'. The market is willing to pay Re. 050 per cubic centimetre of
such bOXCS, so the company wishes to maxiniisc the volume that can
be made by cutting equal squares out of the corners of the scrap
pieces that measure 4 cm x 10 cm. The cost of manufacturing and
selling the boxes is Rs. 3-00 per box. The production department
states that the metal costs Re. 010 per square cm. What is the
volume of the largest box that can be made from the scrap 2 Should
the company produce the box ?
[Ans. 1624 c.c. nearly. The company should produce the box]
4. A box with square top and bottom is to be made to contain 500
Cubic ems. Material for top and bottom costs Rs. 4 per square cm
and the material for the side costs Rs. 2 per square cm. What is
the cost of the least expensive box that can be made ?

F

APPLICATIONS TO COMM1RCB AND ECONOMICS A03-63

[Hint. Volume of the box, X y=500


Cost for the top and bottom= Rs. 4x2x 2 =Rs. 8x2
Cost for the 4 lateral sides= Rs. 2 x 4xy=Rs. 8xy
C= 8x 2 +8xy ...(•*)
500
From (), we get Y =- —i-

Substituting in (), we have


C=8x 2 .8x . =8x2 400_0•]

5. A box with a rectangular bottom and no top is to be made from a


rectangular piece of material 30 ems. long and 16 cms, wide by cutting
equal sized square corners, thenturning up the sides. What should be the
dimensions of the squares if the box is to have maximum volume ?
[Hint. Let x cm. be the side of each square cut off from a corner.
Then the dimensions of the box made are
30-2x, 16-2x and x
V=(30-2x)(I 6-2x)x
=4x 3 -92x 480
dv
3x-46x+120=U

x=12 and 10-


3
x = 12 is not admissible.
(dV
r(24X_ 184) <0
\dx 1013 x1O/3
Hence in order to have maximum volume, the side of tL square cut
off at a corner should be cms. ]

6. One side of a rectangular enclosure is formed by a hedge the


total length of fencing available for the other three sides is 200 yd. Obtain
an expression for the area of the enclosure, A sq. yd., in terms of its
lengths x yd , and hence deduce the maximum area of the enclosure.

[Ans. A=100--,5000 sq.yd.]

7. If the volume of a circular cylindrical block is equal to 800 Cu,

ems., prove that the total surface area is equal to 2x 2 + sq. cms.
where x ems is the radius of the base. Hence obtain the value of x which
makes the surface area a minimum.
[Ans. x = 503 cms.
1
ACE-64
USNLSS MF}[EMATtCS
8. A closed rectangular box is
thickness, the length of made of sheet metal of negligible
the box being twice its width. lithe box has a
capacity of 243 en. ems., show
that its surface area is e q u al to 4x2+729
J-le p c obtain the dIrneri (
-)s of the box of least surface area.
[Ans. 9, 9/2, 61
9. A rectangular sheet of metal is 8
squares of side x (m. are cut from each metre by 3 metre. Equal
folded up o form an of the corners and the whole is
open rectangular tray of depth x ems. Find the
volume of the tray in teims of x, and its maximum volume.
[Ans. V 4x ( 4 O0x)(l5O_) en. ems.,
ma y . volume7 cu. fll.
10. A long Strip oF metal 60 cms. wide
baSe and two is to be bent to form the
sides, of a shute of rectangular cross-section.
Of the base so that the area of the Find tire width
maxrmutn. rectangular cross-section shall be a
[An. 30 ems.]
I. An open rectangular box is to he made out of cardboard and
to hae it vo!Ijfl)c of 288 c.
ems. The length of the ho is to be twice
the width. I f the width is x ems., show that the area of the cardboard
requ ired is 2xx+ sq. eros, and find the vahe of X for i1k area to be
a niinifllum
-. [Arts. 6]
12, A rcclangular box is to have a volume oF 100
lengil is to be twice its breadth. .c in. and its
Find an expression foT'th square of
the length of a (lIagor11 ol the box in terms of the
x headth in Find
abo the minimum porhIc length of this (Iiaona1. (Find the minimum
of the square ot' the length of the diagonal um

[ Ans. 5x+^ 0 ; \' n.


1
13. A close d cylindrical can
cm . Find, in terms f the maxi is to have a surface area of 150 sq.
O v, inu ill volume of the can.
[Axis, 250 C.c.]
14.
An open cylindrical call
cm. hod, in terms of i to have a surface area of 147r sq.
t, the maximum volume of the Can.
[Arts. 343i cc.]
15. A skeleton of a box is to be
formed from three metres of wire.
If the length of the box is to be twice its width,
so that its volume shall he as large as possible, find in ems. its dimensions
fAns 12, 6, 9 cms.]
16. A closed box is to have a volume of 225 c. ems, and the length
of tire base is to be I times the width. Find the d
incnsiQns for the
minimum surface area.
[Arts. *, 5, 6 ems.]

APPLICATIONS TO COMMERCE AND ECONOMICS ACE-65

17. A closed cylindrical can is to have a certain given surface area.


Show that the maximum volume is obtained when the height of the can
is equal to its diameter.
[Ilint. S2rr2±2nTh (fixed)
h (S-2r2)

Vr 2 h=r 2 x (Sr2r3)
2rr 2
(S-6r2)

------i) S=6rr2 or
Jr
or 2rIi=4tr2 or h==2r
d2 V
--i.
ar
Volume is maximum hen height of the can is equal to its
diameter.)
IS. An open tank with a square base and vertic.nl sides is to he
constructed of sheet metal so as to hold a given quantity of water. Show
that the cost of the material will be least when the depth is half of the
width.
19. A manager of a printing firm plans to include 20() square
centimetres of actual printed matter Li each page of a hook under
production. Each page should have a 25 cm. margin along the top
and bottom and 2 0 cm. wide margin along the sides. What are the
most economical dimensions of each printed page 7
20. A printer plans on having 50 square inches of printed matter
per page and is required to allow for margins of 1 inch on each side
and 2 inches on the top and bottom. What are the most economical
dimensions for each page if the cost per page depends on the area of the
page.
21. The total cost C of sampling information is given by
C=a 1 n± - , where a 1 is the unit cost of sampling an item, a2 is the
cost of a unit error in estimation and n is the size of the sample. Find
the number of items to be sampled that minimises the total sampiiflg
cost.
22. There are 60 newly built apartments. At a rental of Rs. 45
per month all apartments will be occupied. But one apartment is likely
to remain vacant for each Rs. 150 increase in rent. Also an occupied
apartment requires Rs. 600 more per month than a vacant one for
maintenance and service. Find the relationship between the profit and
the number of unoccupied apartments. What is the number of vacant
apartments for which the profit is maximum 7 What is the maximum
profit ? [Ans. P=2340+5lx-15x3, 17, Rs. 2773501

ACL-66
BUSINESS MATHEMATICS
23.
A farmer wishes to enclose 12,000 sq. metres of land in a
rectangular plot and then divide it
into two plots with a fence parallel to
one of the sides. What are the d iinensjo,
of the
require the leasta nionit of fence ? rectangular plot that
F1 rnplc 45. S/tow t hal the rwe
cotnflw/, ' with respect to y is equal to of ciwuge of m arginal till/il)'
of
the rate of cliwige of Marginal
utility o / p
tilt/i res/'eel to x, where utilityf net/on is given by
UJx2y2_fy2
Sol u tiozi.

a (3v ?y 2 1 y).3),2 (X2)+


07V (Y2)6y1x
ax

(3. 2y 2 y 2 )32 a
(Y 2)+2y-- 6xy+2y
I6yX,f,.6x?y+2y

J r—(L). (6y2x)=6 (y I 2xv.


u- a
(6x-yH2y)
ox
a. (6x2y) (2y)
r- X

Now } is the marginal utility of X.


L, will be the rate of change of marginal utility of
K ivy!. ),
Similarly f
W.r,t. x will be the rate of change of marginal utility of Y
Hence /rf,,
E xample 46. F/nd the ratio of the marginal
when theoil/it)' Jiu:ct ion i U (x + cJ)P 111i//lies Jr two goods
( y + b)'. S/row that 1/ic same result
Is obtained when the ziflhtty Junction is taken as
Urp log (x f ")+ q log (Y+ b).
So1utj0
' = ( x- J- a ) (Y-1-I))
oil
— p(x-f a)'(y b) and — all q(xa)r(y I
ax

• au au p
a.v ay xa
For utility function U-p Jog ( x
±a)+q log (i'
all
an d° —
• all a ll p q

ACE-67
APPLICATIONS TO COMMERCE AND ECONOMICS

Marginal Products
(L) and
If the output (Q) of a firm is a function of two tiiputs labour
capital (K), SU1)pOSC
Q=f(L, K)
Then, it often becomes necessary to take dectsLons regardtnt; changes
in the inputs with regard to their separate contributiOnS to the enhiancelncflt
in the rate of output. The partial derivatives, in this case, are known as
Marginal producti v i t y (or product) of labour

and Marginal productivity (or product) of capital•


aK

Example 47. The production function of a firm is given by


Q=4Li 1 1)1 4 , L>O wid K>O.
Find the marginal p!OduCHVJtiL'S of Labour (F) wid Capital (K). is it
true that the marginal productivity of labour decreases through positive
values as 1. increases ? Does a similar statement regarding K hold ?

Also, show that L *K =Q


(iPL) is
Solution. The marginal productivity of labour
4(3/4) L 0 '' K 1 / 4 3K' 'I/J)l

and the marginal productivity of capital (MPK) is


(1/4) L 3 R K/ =
aK
Since L >0 and K>O
>0 and decreases as L increases

and -- 0 and decreases as K increases.


3fr1JI L34
Further L K— =L )+K (-)
==3L 311 A)l'-j-L3t J\]jI
=4L 3 i 4 K' J1 Q.
of a firtn he given by
Example 48. Let the production functuot
Q=81-K_1-2_K.
Fiad the !iIPL and MPK. Show th.it

F ----K --2Q
aL 6K
(AIPL) is
Solution. The marginal productivity of labour
-8K--2L
BF-
Ace-68
BUSINESS MATHEMATICS
and the marginal Productivity of capital (MPK) is
aQ
-8L-2K
There fore
L +K L(8K2L)1 K(8L2K)
--I6LK_7L22g2
=2(8LK_ I-2--.K)
2 Q.
F , a in p1 e 49
(liven the 1r0dtcti0 function P =
where L represents labour and K capital find L2 2KL + 2K2,
labour L 2 and K=- 3. m arginal physical product of
Sj 0 ,., P= I.-2KL+2K2
ap
2L--2K- R24K
when L=-2 and K-3, we have
ap
2x2-23 -2
aQ

Example 50. G
iven the production function
I' 4KL --2K2_ [2, fInd the 'flaxilflu,, l
P with the COnS/Pal,,: + K= 10.
So1ut0 Sjc K-3-L 10;
K=10_L
Now, P can be e
of K=i0.J xpresse(f as a function of L by su
bstitutitig the value
P-4 (IO—L) [ -2 (10L)?_L2
80 L— 7L2 — 2 00
For P to be maximum we have
dP
;;7----() and dII'
dp
80-14L or L4O/7
and (iSP

Hence rnaxiinj 1
P is given by
40 40 2 200
- —7 () —200=7-
=2857
Exanp1e 51. Given the
C obb-Douglas production function
=io L'K
APPLICArIONS ro COMMERCE AND ECONOMICS ACE- 69
Find the Output levels Jor
(a) K is fixed at 100 and I. rises 5, 10 , 15.
(b) L isfixecf at 100 and K rises 5, 10, 15.
(C) With L is 10 and K is 15.

SoIuto. (a) With K fixed at 100, the function becomes


P=- to L 1 too"
10 L'/ 100
=100 L1
with L 5, 10 and IS the production levels will he 74767, 1778'28 and
2951-98.
(b) With L fixed at 100, the function becomes
P=lo x l00 • K
=r 31622 K
With K5, 10 and IS, the production levels will be 70711.
99991
and 122386.
(c) With L-_ 10 and K 15
P= lOx 101-25 X 15- 1 :68872.

homogeneous Funtfou
If u --f (x, y) be a JuncHi)u of two variables, then this /thi
ct/ui: is
said to be a Ito mogeneou$ Jimct Ion of degree n (or of order a) ft/ic /llow.
fng relations/up holds
f(tx, f)) f(x, y) ; 1>0.
Remark. A Function is said to be linear homogeneous function,
if the following relationship holds
f(tx, ty) t f(x , y)
Example 52. Let q be the quantIty , p be ,"rice andy
Show that the demand function Shown as be income

q-f(p, where k is a constant , homogeneous of


degree zero.

Solution. Here f(p, •y)—


kp
p
f(tp, t))- ty to± f(p, y)

• . The demand function is homogeneous of degree 7ero.


Example 53. Let
Q=IOL-0-1 1) -- 15K_02K 2 + 2KL
be the production function of a commodity with Q standi
for labour and Kfor capital.
ng for output , L
AC-70 BUSINESS MATHEMATICS

(a) Calculate the n1ar'u1aI products of the two inputs When 10 units
each of labour and cap Ital are used.
(b) ,fsswning that 10 units of capital are being used, indicate the
upper limit for use of labour which a rational producer will never exceed.

Solution. (a) - 04-0+15 , ( K)—O2_(K12L&(K)


=21-04K+ 15
Now substituting 1. -: 10, and K= 10, we get
Marginal product 2x10-0*4x 10+ 15=20-4-1-15-3i

l0.(L) - 0l(L2) 00i2K (L)

---2K-0'2L+ 10
NOW substituting K-- 10, and L 10, we get

Ma rginal product for Labour


=2x 10-0*2x 10+10=28.
(5) N ow, the upper I mit for use of labour which a rational produces'
will never exceed, where II) units of capital are being used, can he obtained
by using the following condition

>f)
'L
cl )K= 10 - -

I 2K--0'2L+101
JK=iu -
>0

2x 10-02xL+100
30
--
02 1-

L150
Hence, the upper limit for the use of labour input will be 150 units.
Example 54. Show that the production function
,f(l, k)=2Vlk
(where )c, / and k are the units of output, labour and capital respectively)
gives constant return to scale and diminishing returns to Inputs.
iDe/hi Univ., B. Corn. (Hans.), 1992]
Solution. The given production function
x=j'(l, k)=2-//k
is homogeneous function of degree one. Replace l by il and k by k in x,
we have
x=f(l, k)24,/i.Xk-2i/lk
APPLICA [IONS TO COMMERCE AND ECONOMICS
ACEt-71
Hence the given function is a homogeneous function of degree onc.
So, the function gives constant returns to scale.

Now MP1=

and _-(.'tfP1) - I
e nec the function gives the diminishing return to labour.

Al Pk

(1Pk)± <

icilce the Function gives the dimiuishiig return to capital.


The muncton gives the dirninishin returns to inputs.
Lulers 'I'lleoi'eni
luler hi ,],()wll that if Z- f
decree n, then
(v 1 , x) is a 1101110C11CQUS Function of

al a/
ax, tIZ

Exa niple 55. The Cn


as a whole is 'ivc h bb_Dozi7j,c pro luction finicijo,, for (I?,' economy

Q=a La K
where 0, a, P are Constants such that
Show that
(a) Q is linear hon oge,,eo,,s fu,idllon of L and K
(b) Prove that
L '- + Ka- - Q

S0ij 0 , ( a) Let
Q=f(L, K)a L" A,
Then f(t L, £ K)a (1 L) a (1 K)
=/ (a L K )
t' f(L, K).
Hence Q = f( L K) is a linear homogeneous function of L and K.
(b) -_-r La1 KO and
---=°t3 La K'.

Hence L -+Q-=a.a L K --a$ L K

=aL a K (a±3)=rQ


ACI-72 BUSINESS MATHEMATICS

since f3= 1. [Fkre, we have verified Eul(;r's Theorem for the Cobb-
Douglas (linear homogeneous) production function]
Example 56. Verify Eukr's Theorem for
ria.cbx2y-3-cxy 2 4. dy.
Solution Here the given function is homogeneous and of the third
degree in x and y. It is required to prove that
ZL
x — +)' --3u
13 X ay
Now

(ax +bxy i- CXy +-(1Y3)


---

=$_ (ox) --- (bxi') - (cxp) (dy3)


BX 3X 6x ax

=3(j52+2byX+Cy2

(Here )' is constaflt)

and (a -- hxy I cx y' j dy)

(ax) - 0.
- (bxy) F . -- ( CXy) + ( dY3)

==hx2+2cxy+3dy2
(Here X is constant)
Multiplying (I) by X and (2) by y, we have
a" =3ax a
42hx iy+ cxy2

y. hx2y + 2cxy2 + 3dy3


ay
Adding, we get
u
au +Y —r--3 - 3bxy43GXyz.F3dy3
OX ax
3(ax 3 + bx 2y +Cxy 2 +dy 8 ) =- 3u

Example 57. Verify Euler's theorem for U xt' log

Solution. Here the given function is of the nth degree, the degree
of y/x being zero. It is required to prove that
u a"
x–– +y j;=flU.
ax a
Since u=X Jog


APPLICATIONS TO CO4MERC:E AND ECONOMICS ACE-73

log - )=X't --(iog 2)+!og f. (x')


TV/X) ax
(-f)+iog .nxn-1.

X" - (-)+nx-'.
X2 log--- (l-Ierc y is constant)
y

= —x--1+nx"-'. lo b --

log L"1 ---(log _-_


a)' ay \ xl ny'.
n (y\ x I x
X .(2)
(y/x) x ' x J y X y

Multiply (1) by x and (2) by y, we get


au y
X - = - x" + rix" log -
ax

y ay

Adding, we get
U
X—
i1 +y — =nx log 2-rr flU.
ax x
Example 58. Define the degree of homogeneity and stale Euler's
theorem.
If the supply function x=f (p1 , p2 ,..., p,,,) is homogeneous' of
degree n, show that the sum of the partial price elasticities of supply equals
n (x denotes the quanifly supplied of a particular commodity and p1 . p2,...,
p,, are the prices of the different commodities.
[Delhi Univ. B. Corn. (Mons), 19911
Solution. If u==f (x, y) be a function of two variables, then this
function is said to be a homogeneous function of degree n if the following
relationship holds
f((x, ty)t=t"f (x, Y); 1>0.
If Z=f (x, y) is a homogeneous function of degree n, then
Z aZ
x----+ y - —=rZ.
ax ay
The partial elasticity of supply x w.r.t. p1.
pt ax
= -(I1 2... m)
x - apt
Sum of partial elasticities of supply
n
pt ax
X
1-1

A-74 BUSINISS MATHEMATICS

=1 pi.
1 -api
1-1
=-i-(p1.-a+p2.

flX
[By Euler's Theorem]
= n.
Example 59. A production function is given by
q=4L' I3 C'13
where L=labour, C==rcapital
(a) Find the behaviour of the marginal product of each factor.
(b) What Is the nature of returns to scale?
(c) What is the reward of labour and capital if each factor is paid a
price equal to its marginal product ?
Solution. (a) We have q=4L2/C113
5.(4L 2 I3Cl I 8 )=4C1 /3
-_ L2/3J-
aL
C'/ L'i

Marginal product of labour=-- C" L-'l

Rate of change of Marginal product of labour


i(!_' (!_ c'i L-'' 3 '\---,( _ ) L" '"I
aL 'L ) a L '., 3 3' 3
—8 L- 4 1 3 C' 13

hich shows that as L increases, Marginal product of labour decreases.

Again {;- (4L2/3 C'/3)_ 4- C--' '3 13

Marginal product of capitai= 4- C213 Dl'.

Rate of change of Marginal product of capital


-__2-(2_'_. - --- ( i L2 IS
13 L28 C-5
aCaC)aC 3 C-"3 ) 9
which again shows that as C increases Marginal product of capital decreases
(b) Let q=f(L, C)=4L'13 C''
ACE-75
lPPL1CATWNS TO COMMERCE AND ECONOMICS
1 1/3 C/3
f(IL, :C)=4 (tL) 13 (tC)113=412 13 L 13
=t [4L2 13 cl Ii=tf(L, C)
.. The production function is homogeneous of degree one, which
shows that the reward to all the factors is exactly equal to total product.
[Remark. If the production function Is homogeneous of degree
greater than one, we shall have a case of increasing returns and if it is of
degree less than or equal to one, this shows we have a case of diminishing
returns to scale.)
(c) 11 each factor is paid a reward equal to its marginal products
then we have
L f- C =L (--
8C
hIn L13)-l-C (4-2' c- L21)

C'i L 2 13 -__-- C'I L/=-4C' JI=_q.

Example 60. The product ion fI4flct ion is x=Aa h, where


Show that there are decreasing returns 10 scale and deduce that the total
product is greater than a tunes the marginal product of Labour plus b tune
the marginal product of capital. What economic interpretation can you
give for this ?
Solution. Here x=A a b P and ci+<l
tIP= Marginal product of labour
Ox h
--A c a '
aa

a LX =il a a h =X
aa
MPr==Marginal product of capital
ax
=A a
ab

IX
h-=3x.
ab

Thus a— +b

aa 2b
I lence there are decreasing returns to scale.
Example 61. Find the first and second order total differentials of
the function
Z=rftx, y)-7y log (1+x)
[Delhi Univ., B.Cm. (Hans), 19921

AcE76
BUSINESS MATHEMATICS
Solution. We have
Z7y log (jx)

dZ4 + . dy
ay
aZ 7y
d az(I+
7 log x)
x 14x anay

dZ-_ dx+7 Jog (1 +x) dy


l+x
d
=7-+1og(1+x)dy
1
111Z=-!-_(dZ). dx+-P_. ( dZ). dy

( 7ydx ldy I 7dx


. dX+<
( (1-f- x)2 1-f-x (l+x ) ay
14dxdy lydzx

E xample 62.
Given linear homogeneous production functjo,
ZALa KO P1 ,
show that L, K, P, stand for factor quantities and A is a Constant,

(1) the swn of


IS unity. the production elasticities with respect to tile factors

(ii) the sum of


marginal products of factors each multiplied by its
respective quantity equals the total output.
(iii) Zn (I) and (ii) above, consider how these results change
if the
given production function is not linear homogeneous but homogeneous of
degree n.
Solution. (i) ZALP KO pr

K P P1

L aZ_ AL" KP p1
KO p1
Similarly
K Z p z
- p and -

we getAdding the above production elasticities with respect to the factors,


L aZ K Z P az
• +- • --- .
VP ++vrl.
(. For linear homogeneous production function +(3+y=l)

APPLICATIONS TO COMMERCE AND ECONOMICS ACE-77

OZ aZ aZ
(ii) L y f-K
- — -f-P-yp (cc ++v) Z=Z

Hence sum of marginal products of factors multiplied by L, K and


P is equal to total output Z.
(iii) In each case, the production elasticity will be multiplied by TJ
and also Z will he multiplied by n•
Marginal Demand Functions and Partial Elasticities of Demand
Let the demand functions for two related commodities x and x
with the respective prices p and p2 be
=f (P11 p) and x2 =g( p 1 , p2)
X1

Then the partial derivatives of x1 and x are known as the (partial)


marginal deinand fi,ncuio,, of x i and x2 , respectively.
In particular,
the (Partial) marginal demand of x 1 w.r.t. p 1 is

the (Partial) marginal demand of x 1 w.r.t. p 2 is ax )

the (Partial) marginal demand of x 9 w.r.t. p 1 is


DPI
and the (Partial) marginal demand of 1 2 w.r.t. p, is

For the usual demand functions


iI'p, is constant, x increases (decreases) as p 1 decreases (increases)
and if p 1 is constant, x 2 increases (decrc3ses) as p 2 decreases (increases)
and hence, and are negative for all economically relevant
values (positive or zero) of p 1 and p2. Further,
ax2
if and are both negative for a given ( p p ) then a
decrease in either price corresponds to an increase in both demands ; and
the commodities X and A'2 are said to be complementary.
On the other hand.
if .-L and - are both positive for given (p 1 , p2 ), then a
ap2

decrease in either price corresponds to an increase in one demand and a


decrease in the other and the commodities x 1 and x 2 are said to be
Competitive.

If then x and x3 are neither complementary nor


---- <0,
ap2 ap1
competitive.

ACE-78 BUSINESS MATHEMATICS I
The partial elasticity of demand is the ratio of the proportional
change in quantity demanded of one commodity (say, x 1 ) to the pro-
portional changes in price of one commodity (p 1 , or p 2), with the price
of the other commodity (p 2 or p) held constant. Thus,
the partial elasticity of demand x 1 w.r.t. price p, with p—'constant is

(log x1)
aP1
_. (log p1)
Ox1

the partial elasticity of demand x 1 w.r.t. price p 2 with p, =constant is

(log xi)
— P 2 — ax1
'712/12
ap, -- (log p2)
ax1
the partial elasticity of demand x 2 w.r.t. price p 1, with p 2 =constant is
a - (log x2)
—p 1 ax,
2j/21
(log p1) =
ax 2
and the partial elasticity of demand x 2 w,r.t, price p 2, with p s=constaflt is
----(log x2)
- —p ax2
222 x 2 p2 _i (log p2)

Example 63. For the following pair of demand functions for two
commodities X1 and X2 , determine the four partial marginal demands, the
nature of relationship (Complementary, Competitive or neither) between X1
and X2 and the four partial elasticities of demand
4 /6
x1 =— and x2=—.
p12p2

Solution. Partial marginal demands:


3x 8
pl p3J)
ax, —32
<,
p, j7 3p2 p1 2P2
ax, —16 <
and
Op, —.
Hence X and A', are complementary commodities.
Partial elasticities of demands:
I I = (-2), I I = (-1), I =(— 1), I 1 = (— 2).

APPLICATIONS TO COMMERCØ AND ECONOMICS ACB-79


Example 64. The following are the demand functions for two
commodities X 1 and X1
- j -1.7
X1—P 0.8
Pa
x2=pI0.5 P2- 0-2

Determine whether the two commodities are complements or substitutes In


some sense. [Delhi Un iv., B.A. (lions.) Economics 19911
Solutio n. We have
x1=p1'lp268 and

—P7 p 1 - 3 ' P208

-=08 p(17 P2 - '.t

=05 p1-0.5 p1-.0-2


ap1

=-02 p 105 p21•'


02
Since ----and are both positive, the two commodities are
ap2 5p1
substitutes in sonic sense.
Example 65. The demand functions of two commodities X 1 and X
are x1 =p1 1 p20.6 and x 2 = p 1 05 p 2 - respectively , where x 1 and x2 are the
quantities Iernande1 of X anj X2 respectively and
p 1 and p 2 are their
respective prices . Find the four partial elasticities of demand and determine
whether the commodities are competitive or complementary.
[Delhi Univ. B. Corn. (Hans.), 1991]
Solution.

The partial elasticity of demand of x 1 w.r.t. prices p,=-—p, ax


-:L1 - - ---

6z(_l4)p14p2o6..p4

The partial elasticity of demand x 1 w.r.t. price p1


-. x1
x1 p11.4p20. x(06) P1 4 p°'=-06
The partial elasticity of demand x 2 w.r.t. price p1
_Li.- —P'
- x(05) p 1 -0.5 p"—_o 5
x2 p
3 1 P3 0" p2-1-2
The partial elasticity of demand x 2 w.r.t. price p,
- CE!-- - x(— P2)p10 P2 12= P2
- xl p1 p108p2-i.2

ACE-80 BUSINESS MATHEMATIC S

Since both-s-. and


a p 2 ap1 are negative, the commodities X1 and X,
are said to be complementary.
Example 66. Find the elasticity of substitution (a) for the production
function Q.f(1, k)=[akb+(1_.a) t b J_u/ b by using the formula

f.lfk d(kJl)
k I id(f, fl)
(Delhi Univ. B. C'onz. (lIons), 19911
Solution. We have
lib
Q=-. [ ak+(l_a)l-b]
or Qak+(l_a)J_b (1)
Differentiating (I) partially w.r.t. k, we get

ak

Q ok'b—1
fk

- Q (l.a)l--!
Similarly, -b-1
---

ak1.Q+t)
Now &=-()=-(

=k__ i (b^ J)Qb


al

ak- b- 1 (b+1)Qb (1—a)l-'


= Qb_1

aQ aQ
.1. I fk d(k I)
kj ak a/
d(f,If ^a'Q
aliJc
ak b-i (1 __a)16_1 Q-l'-1
X
XQk.b_1(b+ I) Qb(I -

FTI
Example 67. The demand (D) of passenger automobiles is given
by D= 090 11 p -07 where i is the income and p is the price per car.
Find the (1) income elasticity of demand and (ii) price elasticity of demand.
Solution. The income elasticity of demand is given by
laD
' 1
APPLICATIONS TO COMMERCE AND ECONOMICS ACS-8I

r (09x 0'90 x 1•1Io•p-°1


JI.J-G.?

Price elasticity of demand is given by


p DD
1r
ap

0901''p°7 x 090 x (-07)I''P-11

Example 68. The demand function for a commodity 'X' is given by


x=300_051)4.002p0+005y
where x is the quantity demanded of 'X', p the price of X, pa the price of a
related commodity and y is the constant income. Compute
(1) The price elasticity of demand for X,
(ii) The income elasticity of demand for X • , and
(iii) Cross elasticity of demand for X, w.r.t. p0
when P=I2, p o =IO and Y=200.
Solution, (1) Price elasticity of demand for X is given by
77,
p =_
3 x-
X 3p,
PX
x {05 x (-2p3}
300— 05p 2± O02Po-f O'05y
[from ...()]
2

3000'5p 2 -4 002Po HJ'OSy


When p . ,=12, p0 = 10 and y-=200,
I —144 1 144
300-72 02± 10
(ii) The income elasticity of demand for X is given by
ax
Y
x8y
0 05y
from
3-0r5p, 2 ±002p1,f 0 05y
When p,=l2,p=10 and y=200, as in part (I)

•q 2382 =004
(iii) Cross-elasticity of demand for X w.r.t, Pa is given by
p0 ax
x apo

ACE-82
BUSINESS MATHEMATICS
(positive sign is taken since, from () we see that p0 and x change in the
same direction).

• • • OO2p0
'?o from *
Atp'12,p0 o and y =200, we get
002 x 10

Maxima and Minima for Function of two Variables


It is beyond the scope of this book to obtain the general conditions.
We shall merely state a set of sufficient conditions, which are applicable
to a large number of problems.
For a function Z=f(x, y), if at the point (x 1 , y1)
(j) =,
3X ay

and (ii) 3 ' 7 Z 52 Z 2


()()(—)>o axay
then Z is maximum or minimum according as

is negative or positive at (x 1 , y1).


Example 69. The joint cast function for two commodities is
+ 2xx2l_3x22
The prices are 81' for x) and 12( for x,) per unit Find the
fit and the total cost.
"
maxirnupn pro-
Solution. Total revenue 8x, + 12x2
Total cost rX1+2xix2f3x22
Total profit: P=TR_TC
=(8x 1 + I 2x2) - ( x1 i + 2x1 x2 + 3x22)
OP

and ax,
—I2--2x1--6x2
The condition (1) gives
8-2X 1 -2x2 O and
S olving these simultaneous linear equations in x
1 and x2, we get
x L-3 and xr=l
P can have a maximum value at (3, 1).
a'P
Now —r=_2, a2P
x1 2 ax2' —==-6 and =-2
ax1ax2
APPLICATIONS TO COMMERCE AND ECONOMICS ACE-83

/_
• ( alp ) ( 32P 0
2 ) x2)
The condition (ii) is satisfied at (3, 0.
a21'
Also -- —2 is negative
axis
P has a maximum at (3, 1) and the maximum profit
=(8x3-l-12x 1)--(9 •-6+3)=18
EXERCISE (IV)
1. Find - -- for
ax' y
(1) =x2y 2 -- x5 +y 6 , ( ii) uXy+xy 3 , (iii) u==+y3-l- 3axy,
(iv) u=log (x 2 +Y2)513 (v) u.- ii / 2x-4-y2.
[Ans. (i) 2xy 2 -l--5 X4 , 2xy 2 -r-6Y 5 , ( ii) 3x 2y4-y 2 , x+3xy2,
(iii) 2x + 3ay, 3 y2 -j-- 3 ax, (iv) lox/3(x" -l- y2 ), I Oy/3(X 2 +Y')
(v) - 2x(2x2 + y2 )- 2
, — y(2x 2 +y2)3 (2]
2. Find the first order and second order partial derivatives of the
following functions
(1) ti=x2 -5Xy-4-y 2 , ( ii) U= x2e'.
[Ans. (I) f=2x— 5y, f=2, f, = -5,
f-5x42y, f=2,J.=-5.1
3. Find the first order and second order partial derivatives of the
following functions
(1) u=x-5xy+y2, (ii) u=e u=x2e'

(iii) u=-e (iv) u=x2e',


a'u 2u
4. Verify that when U—'±Y
au
5. If u=rlog [x+ \/ X 1 +y t ), au + Y - =1.
prove that x —
ay
6. If u_rax 2 +2bXy1 . by Z , show that
au azi
x — Y --=2u.
ax ay
7. A utility function is given by
u = 2q 2 q 2 -4-- 3q1q.2.
Show that the rate of change of marginal utility of commodity q1
w. r. t. q2 is equal to the rate of change of marginal utility of q 2 w. r. t.
q1.
8. (a) If show that
au au
X +Y
ax

ACE-84
BUSINESS MATULtMATLCS

(b) If OX 2 +b 2 y 2 ==c2u, show that


u a?bl
ax2 ay c2u
9. Verify that
a 2zzaXaY lu
±2xy ---+ y2 — a2z 2u, where uxl+2xy_.y2.
10. ' U xa±y_3jxy2 verify that
a 2" a2" a2u
+2xyf-y2 -. r6U

If z=(ax lby)-', find the value ofa2z a'z

12. For the production function zcix y' , show that


az az
(:) x ----- - 1-y — 4-)z and
i)x ay
(u) x ---±2xy a2z --l-y 2
axay
13. Suppose there is a production function of the type
z-= e' 2y + 3y2)
where Z is the product and x and y are different factors of production,
find the marginal products of x and Y.
14. If q3L 2C2_ 2L 2 0, where L
Capital. Find Average and C are inputs Labour and
If input be fixed, wh.tt is the value of input Lproduct of labour (L).
C product and Marginal
for which AP be maxi-
mum ? Does the maximum of Marginal Product Curve reach at lower
level of labour ?
[flint. Lq_?_ (3 L20_2L 2C8) - ( 3L3C2)__ (2L2C3)
3C2. 3L 2 2C3.2L 9C2 L2
--- 4cL.
Marginal product of labour (MP)9C2L2_4L
Also Average product of Labour (AP)=Tq 3L3C2_ 2L2C3
=
=3L1C2...2LC8
Now for maximum value of AP,
/VJPA P
or 9C2L2. 40L3L2C22LC3
6C)L2_2C2L0, 6C2L2=20L LL.
3
Again Marginal product of labour (MP) is maximum when slope of
MP curve is zero, 1e., when - (Mp)=O.

APPLICATIONS TO COMMERCI3 AND ECONOMICS ACE-85

- 9C' (L2)_4C3 j- (L)=O


OL
2
L=C

15. The demand function is q=3y+2y 2 -6x 2 --- 5x-4,


where x>O, y>O, q is quantity demanded, Y is income, X is the price.
Find what is the slope of the demand curve ? Is the commodity
normal or inferior ? Is the reaction of demand to price is independent of
the level of income ?
[hint. ax
Lq
ax
(3y+2),3-6.2--5x)=----- I2x-4-20x

aq
Slope of Demand curve —I 2x+2OX.

If fy => O, then the commodit y will be normal.


DY
ay iY (3y+2y2-6x54)=3+4y.

Since y is positive, f>O


The commodity is normal commodity and is not inferior.
Now the reaction of demand to price is independent of the level of
income iff=f=o.

Now 3+4y and - —12x+20x


ax

Now (3+4y)=0
axay ax ay ax
and j=JL -)=-(_ l2x-l-20x)r0.
The reaction of demand to price is independent of the level
of income.]
16. The following are two linear homogeneous production func-
tions where X, 1.., K represent output, labour and capital respectively.
Show that in each case, L times the marginal product of labour plus K
times the marginal product of capital equals total product.
(0 X=4L K, (11) X=aL-fbK.
Find what is the sum of the partial elasticities in each case.
17. If a' men are employed in planting 'b' acres with timber,
the amount of timber cut after '1' years is x=J (a, 5, 1). What meaning
can he attached to
x ax
—ann—?
a' b a'

ACE-86 BUSINESS MATHEMATICS

The production of a particular commodity was estimated as


X= L 064 Ko36, where X is the production of that commodity,
L is labour and K is capital.
Determine the marginal productivities for L15 and K11
units.
18. Q= 101 L05 K° 2 , where Q=output, L= Labour, K= Capital.

Prove that L --KQ.


1. K
1. The following is a linear homogeneous production function
where X, L, K represent output, labour, and capital respectively
X= ,/ aL2+2hLK±bK1.
Show that L times the marginal product of labour and K time
the marginal product of capital equals total product
20. (a) For the linear homogeneous production function
X=AL K1
where X, L and K denote output, Labour and Capital respectively ; show
that the average and marginal products of each factor L and K are
functions of the relative amounts of L and K used.
(b) If the production function is given by XrAL K 1 , show that
there are increasing, decreasing or constant returns to scale as
+>l, <l or1.
(c) For the production function X=ALD K P where X , L and K
represent Output, Labour ana Capital respectively, show that-
(i) a and 3 represent the labour share and capital share of the
output respectively.
(ii) a and 3 aire also the elasticities of output with respect to labour
and capital respectively.
K aX L aX
[Hint. (ii) Calculate - . - . . I
21. The production function is P=-AK` L5 where +3<l. Show
that there are decreasing returns to scale. Deduce that total product
is greater than total income distributed between K and L, when income
is dis'fributed according to each factors marginal productivity. What
will be the economic interpretation of the residual ?
22. Explain what you mean by production function. State the
factors which are generally involved in it- State the mathematical form of
Cobb-Douglas production function, interpret its constants and describe
the method to fit it to the production data.
For the production function
x=A aP bt
APPLICATIONS TO COMMERCE AND ECONOMICS
ACE-87
show that the average products and the marginal products are functions
of the ratio of the factors used.

23. A production function is given as x4 a b where


-i-13> I,
and factor quantities are a and b for labour and capital respectively.
Show that there is increasing returns to scale and deduce that the total
Product is greater than a time the marginal product of labour plus
b
times the marginal product of capital.
24. For the production function

Q=AkL-1(I_) A - 9,
1- ]/
where A>O, O<<1 and -/O are constants, find the marginal products
of labour (L) and capital (K). Further, if
aQ Q
1.

^
'LlaQic
is the elasticity of' Substitution, show that =
is a constant.
25. If Uf(x 1x2,......-) is the (total) utility
(index) function in
terms of the amounts x 1 , x 2 ..,,, , consumed of the
n respective goods
(commodities) A'1 , X......, X,, then the marginal utility
of the goods X1,
is defined to be
au
, at a point (x 1 , X,.., X),
Find
(i) The marginal utilities with respect to two commodities
X0 , when x =j and x2 X, and
-=2 units of the two commodities are consumed,
if the utility (index) function of A' 1 and A'2 is given by
U= (xi +3) (x2+5).
(ii) The ratio of the marginal utility of the good A' 1
utility of the good A'2 , if the utility function of the goods the
to marginal
A'1 and A'3 is
given by

(a) U=,
- 11
(b) U=1og, (ax1+bx2+c./Tc)
[Ans. (i) Marginal utilities
(q)
\ x 1 (I, 2) =7;(- =4
I(1 2)
(ii) In (a) as well as in (b)
(U/ (U \ /xix.+cx2
k' ) / TT)2b \/ . 1 CX
26. If Xrf(p ,, p, M) is a homogeneous demand function of degree
zero, where P. and P. are prices of two commodities x and y, and Al' is
the money income ; then prove that the sum of the partial elasticities is
equal to zero.

ACE-88 BUSINESS MATHEMATICS

27. The supply function for


1 2
x =ft p , p . ps,..., p,.)
(where p1 , p2 , p, •. , p,, are the prices of several goods) is a homogeneous
function of degree n. Prove that the sum of partial elasticities of x must
total fl. [Delhi Univ., B.A. (Econ. Hens.) 1990 (N.S.)J
28. A manufacturer finds that his costs are given by the function
Q=a -I-8b. Under the assumption that he keeps his cost fixed at the value
of Q= 50 and that his production function is defined by
U==32ab-7a 2 - 102
prove that his maximum production is U- 500.
29. For the linear homogeneous production function:
2Iiab---Aa2—Bht
= Ca±Db
where H, A, /1, C and D are positive constants, and a and b denote
Labour and Capital respectively show that the average and marginal
products of the factors depend only oil ratio of the factors.
30. If x 3 and p 1 are demand and price of tea and x 2 and p., are
demand and price of coffee, and the demand functions are given by
x 1 -- I I -1-3 J-' 0.5
x2=plo•3 p o•S 5_ J
show that the two commodities are competitive. Also find four partial
elasticities of demand.

[Hint. r=0.5l.I p ° 5> 0 an( ia —03 -0.7


p1

Since both >0 and the two commodities, viz., tea


ap, , ap1
and coffee, are competitive.
Partial Elasticitiess
p1 x1
(i) [ (— 13) p23 ]=1'3
X1 •p 1 - p14p
P2 3XI p2
(ii) + — l•3 ,, +, ,----'(o-5)P,,
=1W)
, -
(Note that here positive sign is taken since from (*), x 1 and p2 move
in the same direction).
(ii) >< - [ o " S -0.7 i'n -0.5
X /'
ap1 p ° .3 po5
(Here also note the positive sign)

I p 10.3 (-05) p15 I = o 5


(iv) —b- BX 2— p2
X2 pi P1
,, 0.5
0.3 i-i .

. pc-89
APPLICATION S TO COMMERCE AND ECONOMICS

31 Determine the partial elasticities and nature of commodities for


the demand functions,
P.-
1 =I'iP"O'sx p
32. When are two goods X 1 and X said to be (a) CoapeitIYC,
(b) Complementary ih demand ?
Examine the relation between X i and X 2 in the case of the follow
ing demand functions
(a) x1=a1—a 12, f a 2 1,2
a-+ a2 --
a 2.4

(h) x1=

x, — +a21 p1

(c) xp1°'' ehIPt±1)


Xp2°" e(aPl+G1)

33. The demand functions for two commodities X 1 and K, iii terms
of their respective prices p and p, are given by

_G1 and .2_p.0' eh1P1
where a1, 0 2, h 1 , h 2 and c 1 , C. are constants.
Find the four partial marginal demand functions and show that
The 'direct' pr ice- elasticities (viz.,ap, -' and axt are independent of
ap, )
the prices ; while the 'cross' price-elasticities ( viz., _L.i and __!)are
at' 2 at'1
determined in sign, by the constants b 1 and b2.
34. Show that x, =a, - is an example of a demand law for good
Pi
X i in competition with good x 2 and that is a corresponding
P, P,
law where x 1 and x 2 are complementary.
35. The cost functions for two duopolists producing a common
good are respectively by
c 1 =5x 1 and c2=5x2!.

The demand function for the good is given by


p= 100-0'5x
where xx1+x2.
Assuming the duc'polists take independent decisions regarding out-
puts (1 e., there is no conjectural variation), find their equilibrium
outputs and profits.
(Hint. The profit functions of duopolists are
P 1 =px 1 —c1 and P,=-px2—c2
ACE-90 BUSINESS MA7HEMATICS

respectively. Equilibrium outputs are determined by the condition that


P1 and P2 are maximum.]
(Ans. Equilibrium outputs : x 1 ==3980/43 and x2 =210/43 ; equili-
brium profits: P1 4628 and P2l3O.]
36. A monopolist firm produces chocolates of two types X 1 and
1. The constant average cost of X1 and X. are respectively, Rs. 250
and Rs. 300 per kg. For price of pi and p, the demands for A'1 and A'2
are respectively, given by
x 1 =5(p 2 —p 1 ) and x2=32-f-5p1--10p2.
Find the levels at which prices will be fixed for A' 1 and A'2 for
maximum joint monopoly profit.
Also find the prices of A'1 and X2 fixed by two independent mono-
polists.
[Ans. For a single monopolist, levels of prices Rs. p 1 445 and
p 2 470 for two independent monopolists, levels of prices is
Rs. p1 32 and p 2 n39.]
37. A monopolist produces amounts x 1 and x 2 of two goods A' 1 and
A'2 at a total cost 7r=x 12 +2x 1x2 .f 3x 22 . The demands for the two goods
in the market are
p1 r=36-3x 1 and /)2=40-5x2
where p 1 and p2 are the prices charged. Determine the quantities and
prices which maximise the profit. Find also the value of the maximum
profit.
38. The demand for a good x is represented by the demand relation
p=41 (x). The production of the good is shared between two duopolist
firms selling at the same price p. The first duopolist produces an output
x2 at a total cost of -1==F1()c1) and the second doupolist produces an out-
put x2 at a total cost 01 i 2 =F2(x2). Find the equations which determine
the output of the two duopolists. (Assume zero conjectural variation.)
APPLICATIONS OF INTEGRATION
To find the cost function when marginal cost is given:
We know that if the total cost function, say C, is given then the
marginal cost function is the first derivative of the total cost function.
It follows, therefore, that the total cost function is the integral of the
marginal cost function.
If C represent the total cost of producing an output x, then the
marginal cost is given by
dC
MC=--

C=(MC)dx±k
The constant of integration k can be evaluated if the fixed cost (i.e., the
cost when x='O) is given.

PPLlCATIONS TO COMMERCE AND ECONOMICS ACE-9I

Further the average cost AC can be obtained from the relation

Example 70. The marginal cost function of a product is given by


==
4c_ioo—ioq+o'i q$,
where q is the output. Obtain the total and the average cost function of
the firm under the assumption that ifs fixed cost is Rs. 500.
dC
Solution. = tOO - lOq+Oi q2 = MC
dq
Integrating both sides w.r.t. q, we have
C==f(lOO_10q4-01q2)dq

=100__10.±40.1 .

Now the fixed cost is 500, i.e., when q.-0, C=500.


k=500.
Hence total cost function is
C=loOq-5q 2 -4 +500

Average cost function is


C q2 500
AC=—=l00-5q±--+--

Example 71. The marginal cost function of manufacturing x shoes


is 6+I0x-6x. The total cost of producing a pair of shoes is Rs, 12. Find
the total and average cost function.

Solution. MC=6+10X-6x

C=4(6+10X_6X2)dX

-=6x4-10 .

where k is the constant of integration.


Now C=12, when x=2.
12==6(2)+IOX ----6x ---+k
k=12-12-20+16=-4
The total cost function is
C=6X+ 5x'-2x3-4

ACE-92
BUSINESS MATHE)4%11C

Further the average cost function AC is given by


C 4
-.
X x
Example 72. The marginal cost function ofaJirn is given by
MC=3000 e°-j--50,
when x is quantity produced. If fixed cost Is Rs. 80,000, find the total cost
function of the firm, (Delhi Univ., B. Corn. (lions) 1990]
Solution, The total cost function ot the firm is given by
TCJ(MC)dx .{ k, where k is constant of integration.

TC1(3000e03+50)dx±k

=3000. 4—±5ox+k

= 1 0000e°'3+50x+k
When =0, TC=80000, therefore, we have
8000010000+k
k=7000
TC.= 10000e°"+50x--70000
Example 73. Assume that the marginal cost in bk/is of rupees is
given by
MC=4+5x1+4
where x is the quantity produced. Find the total cost of production when
x=2, Iffixed cost is Rs. 6 !akhs, [Delhi Univ., B. Corn. (Hons,), 1992]
Solution, We have

MC=4+5x2 + 3 e

TC4MCdX =J(4+5x1 42. e" )dx

=4x+------ e--fk,
where k is constant of integration.
We are given that when x=0, TC6
6=_-+k krr-15

TC=4x- -i-- - e

PPLICAT1ONS TO COMMERCE AND ECONOMICS ACE-93

iC(atx=2)=4x2+

=8 -
=Rs. 28 63 lakhs.
[Lct y -- e2 , ., log y=-2 log e=-2x04343
1 1314
y-r flfl log ( 1.1314)=01 3531
To find the total revenue function and the demand function
when the marginal revenue function is given.
TI R is the total revenue when the output is x, then the marginal
revenue MR is given by
dR
MR
dx
Hence if the marginal revenue MR is given, then the total revenue
R is the indefinite integral of MR with respect to x i.e.,
(MR)dx+k

The constant of integration k can he evaluated from the fact that


the total revenue R is zero when the output X is zero.
Further, since the demand function can be easily obtained as
R
X
is gtvefl
Example 74. If the marginal revenue function for output
integration. Also
by R,-_ (x2)25"' the total revenue function by
deduce the demand function.
Solution. Total revenue function is given by
RB dx
2)2 I
dx+5 dx
1(x+2r
±5x+k
( x±2)
Since total revenue is zero at x=. 0, we get
k==3

R=3-

ACE-94 BUSINESS MATHEMATICS

Also we know R=p>x


6
R

x
3
6 +5
x x(x-2)
-. 3x+6-6
6-6 +
x(x-p2)

x+2

Hence • p= +5 is the required demand function.


EampJe 75. If the marginal revenue function is

MR=Tb) 4

show that p=•b --- -C


-x
Li the demand law.
ab d
Solution. MR (b) c_•d(R)

Integrating both sides wr.t. x, we have


f( ab I
dx-f-k,
where k is a constant of integration.
=ab (x- b)- 1 dx-c dx+k
J J
=ab (' _-ab -cx+k •.(*)
Now when x=o, total revenue=O.
ab

0. k=-a.
Hence the total revenue function is given by
- ab
TR b -CX--a=px
-ab a
b)
x(x - x
-ab--ax+ab -ax
- x(x_b° x(x - b)
APPLICATIONS TO COMMERCE AND ECONOMICS ACE-95

—a a
x—b _C==b__
C

is the required demand law.


To find the consumption function when the marginal pro-
pensity to consume (MPC) is given.
If P is the consumption when the dispQsable income of a person is
X, the marginal propensity to consume (MPG) is given by

MPC
dx
Hence if MPG is given, the consumption P is given by the indetinite
integral of U PC with respect to x, i.e.,
P(MPC)dx-fk

The constant of integration k, can be evaluated if the value of P Is


known for some x.
Eiam pie 76. If the marginal propensity to save (MPS) IS
15 02x- 2 , when x is the income. Find the consumption function, given
that the consumption is 4-8 when income is ten.
Solution. Now "derivative of consumption function w.r.t. output
represents marginal propensity to consume".

MPS = 15-F02x2=
dx
x-2+1
P= (l 5+02x 2 ) (1X I 5x+02( 2 )+k

=1 5x—

Now P=4g when x= 10


48=15x10----+k
k=—l0l8
Hence the consumption function is
P=15x— —1018
Maximum Profits
Suppose we are required to find the maximum profits of a firm when
only the marginal cost and the marginal revenue functions are given. Then
our problem is, how to compute maximum profits? By equating marginal
cost to marginal revenue, we can find the output that maximises total
profits. To calculate total profits at this output, we have
dP dR dC
dx - dx dx
ACE-96 US)NESS MAThEMATiCS

where P. R, C, x represents the total profit, total revenue, total cost and
output respectively.
Integrating, we have
P ^ i dR dx_j_ . dx+k=R—C+k

here the constant of integration, k, can be found from the additional


information given.
Remark. It may be noted that profit is maimiscd when
marginal revenue equals marginal cost, given the assumption of pure
competition Total profit is the integral of marginal revenue minus
marginal cost from zero quantity to quantity for which profit is niaxiinised.
Example 77. The marginal cost of production of a firm is
given as
C'(q)r 5 -1- Ol3q
Further, the marginal revenue is
R'(q)=18 -
Also it is given that C'(0)= Rs. 120. Compute the total profits.
Solution. Since profit is maximum, where,
marginal cost=-marginal revenue
i.e., C'(q)- R'(q)
5+0l3=l8

('P
Also _zrJ R'(q)dq_JC'(q) dq

Now S R'(q)=J 18 c1qr 18q--k1,


where k 1 is an arbitrary constant.
Put k 1 -0, as under pure competition, total revenue = output x price.
R(q)= I R'(q)dq 18g.
Also SC'(q)dq=J(5+0l 3q) dq
C(q)=5q+013
where k2 is an arbitrary constant.
From tb,e additional information C(0).-= 120, we have
C(0)=5(0)-+ O 3(0)-f-k 2 = 120
k 2 = 120, ... C(q)=5q005q$20.
Now P(q)_—R(q)_C(q
=18q5q_0065q?_ 120
13 q -0'065q 1 — 120
APPLICATIONS TO COMMERCE AND ECONOMICS ACE-97

Total profit, when q— 100 is


P(1 0O)- 13— I00_0065(l00) 2 - 120
= I300--60-_l20Rs. 530.
Example 78. The ABC Co. Ltd. has approximated the marginal
revenue function for one of its products by MR= 20 - 2x The marginal
cost function is approximated by MC----81 --l6x+x2.
Determine the profit. maximizing output and the total projit at the
optimal output.
Solution. Solving for profit-maximizing output, set MR equal to
MC, i.e.,
MR=-MC
20x_2x2=81_6X+x
8136x_-3x=()
x2—l2x-i-270
(x-3)(x-9)r-0
x=3,9.
The second derivative of MR - MC is the second derivative of total
profit. The sign of P"(x) indicates whether x is a relative maximum
or relative minimum.
d(MR— A fC)
=36-6x

P(3)-36 -6(3)=-- 18.


P(9)=36 —6(9)=-18.
Therefore, at .v=9, profit is maximum.

Total profit= J 81+36x3 d= (_81x+18x1—XI)

: [_8I(9)- 18(9)1=0
which indicates no profit. A negative sign would signify a loss.
Example 79 XYZ Co. Ltd. suffers a loss of Rs. 12150 if one of
its special product does not sell. Marginal revenue is approximated by
MR —30—ox and marginal cost by MC= —24+3x.
Determine the total profit function, the break-even points, and the
total profit between break-even points.
Solution. Solving for total profit, first determine marginal
profit.
MP=MB—MC
=(30-6x)—(---24+ 3x)
=54-9x
Total profit function
= ,f MP dx

ACE-98
BUSINESS MATHEMATICS

=f(54x-9x) dx

=54x?+k

Since a loss of Rs. 12150 occurs when there are no sale3, k must
equal —12150. Consequently, total profit function equals
(
P X) r -l2l'5O+54_ 9 -X
Solving for break-even points, set P(x) 0
0 r -121'50l54x - x'
(X-3)(x_9)=0
x=3,9.
Integrating the profit function between break-even points will give
total profit between break-even points.

TPJ(—l2l50+54x_ 9 )dx

=[(-121 • 50x+54 ._ - x )

=[ —12150(9)-l--54 (9)'

121•50(3)+54_--(3)3]
=Rs. 4536.
Ezample 80. The price elasticity of a demand curve x = AP) is of
the form (a —bp) where a and b are given constants. Find the demand
curve.
Solution. We are given
p dx
_-
x WP
a—bp dx
( )dp+ _==O
a dx rQ
( -_b)dp+
Integrating, we get
(alogp—bp)+log x=logc
log (p ebP)+ log x= log C
xp° e'==c
xcp-a eb?,
where log c is the constant of integration.
APPLICATIONS TO COMMERCE AND ECONOMICS AC-99

Example 81. Derive the demand function which has the unit price
elasticity of demand throughout. (Delhi Univ., B. corn. (Hons.) 19911
Solution. Since the elasticity of demand is unity throughout,
we have
p dx
I
dx dp
or
x p
Integrating both sides, we have

where k is the constant of integration.


or logxr-=logpk
or log x+ log p=k
or px=ek=c
pX=c is the required demand function,
Consumer's Surplus
Suppose the price p a consumer is willing to pay for a quantity X
of a particular commodity is governed by the demand curve
p D(x).
In general, the function D(x) is a decreasing function, since, as the
price of a commodity increases, the quantity the consumer is willing to
buy declines.
Further, suppose the price p that a producer is willing to charge
for a quantity x of a particular commodity is governed by the supply
curve
p=S(x).
In general, the function S(x) is an increasing function since, as the
price p of a commodity increases, the more the producer is willing to
supply the commodity.
The point of intersection of the demand curve and the supply
curve is called the equilibrium point E.
ACE-100 BUSINESS MATHEMATICS

If the coordinates of £ are (x0 , p) then p 0 , the market price, is the


price a consumer is willing to pay for and a producer is willing to sell
for. a quantity x 0 , the demand level, of the commodity. The total
revenue of the producer at a market price p 0 ; and a demand level X0 is
p 0 xo (the price per unit times the number of units) which can be inter-
preted geometrically as the area of rectangle OAEB.
In a free market economy, there are times when some consumers
would be willing to pay more for a commodity than the market price p0
that they actually do pay. The benefit of this to consumers, i.e., the
difference between what consumers actually paid and what they were
willing to pa', is called consumer's surplus (CS). Thus

CS" (Total area under the demand curve O (x) from x-O to XX0)

{the area of the rectangle OAF-B)


XO

D(x) d -- < p0

In other words, consumer's surplus is the amount which a consumer


is willing to pay for a commodity rather than go without it, minus what
he would have to pay actually for it at the market price.
Remarks. 1. Under pure competition, the price p 0 is determined
by equating the demand and supply functions, and from this relation the
demand x0 is calculated.

I. Under monopoly, the price Pa is determined by equating MR


and MC functions. From this price value Po, we obtain the correspcnding
value of x 0 and then the consumer's surplus is calculated in the usual way.

Surplus
In a free market economy, there are also times when some
producers would be willing to sell at a price below the market price P0
APPL I CATIONS TO COMMERCE AND ECONOMICS ACII - 101

that the consumer actually pays. The benefit of this to the producer,
I.e., the dtlrcrct'ce between the revenue producers actually reeevC and
N

what they have been willing to receive, is known as producer's


surplu (PS).
PS (Area of the rectangle OAETh—(Area below the supply
function from 0 to x0}

X0 x p0 S(x) dx

Example 82. The de,nand law for a commodity is


P=20—D_D0
Find the consumer's surplus when the demand is 3.
Soluiio. Here
Also when the demand D,= 3, the price
P0 20-0)— (W 8

Consumer's surplusJf(D) dD—p0D0

(20—D—D^) c/D—(8x 3)
^j

4 20 D-_1_2d
('2 (11
=r 20x3____ _1_24=4:5

Example 83. I f supply curve is '- and the quantify


sold in market is 6 Units, find the producer's surplus.
ACE-IO2 BUSINESS MATHEMATICS

Solution. Now X 0 =6 p0: / 10+6 =h4


x0=6 and p0=4 (•.• p_4 is meaningless).
Hence producers' surplus

6x 4—VI0+Xdx

—24— (I04x)3
3/2
0

=24___[ (16)3/2_(10)321=2'42

Example 84. Determine consumer surplus and producer surplus


under pure competition for the demand function p=36_xz and supply
2
function p=o -.-- -, where p is the price and x is quantity.
[Delhi Univ. B. Corn, (lions.),
Solution. Under pure competition, market equilibrium conditions
can be obtained by equating the demand and supply.
5X2

36--x2=6+-- or

x30<4=r24 - x=2V"6X0
or

p,=36-24= 12

Consumer's Surplus= D(x) dx—p0x0

(36—x2) dx-2 i/6x12

2V6

= 36x_._ —24 '6

=72v' 6 —16-,/T —24V 6 =32V -6 -


XG

Producer's Surplus S(x) dx


APPLICATIONS TO COMMERCE AND ECONOMICS ACE-103

2V6
X
=2V6X12— 64- )dx
J(
0
2/ 6
=24 V6 - 6x--1

=24\/6 —l2/6 —4i/6 =8\/6


Example 85. Find the consumer surplus and producer surplus
under pure competition for demand function p=-- 7----2 and supply function

P= 4- (x+3), where p is price and x is quantity.


[Delhi Univ. B. Corn (Hons)., 1992]
Solution. Under pure competition, market equilibrium conditions
can be obtained by equating the demand and supply.

—2=--- (x+3)

or 16-4(x+l)=(x--l-3)(x+1)
or 16-4x-4=X2 +4x4-3
or
or (x+9)(x—l)=O
or x=-9
x=-9 is inadmissible as quantity cannot be negative.

When x=], p=---(x+3)=--- (143)_-2.


xo
Consumer surplus D(x) dx—p00
0

J( .1_ 2 )dx_i x 2
0

log (x+ I)-2x] —2


BUSINESS MATF1E'1ATCS
104
=8 log 2-2-2=8 log 2-4.
Xa

Producer surplus S(x)


dx

=1 x 2 (v+3) dx

=2— [(+3x)]

2--
T3)
I

7
I

1
2

Example 86. The (lemand and supply function under perfect


compeluion are y=16—x 2 and y=2x 2 4-4 respectively, Find the market
price, consumer's surplus and producer's surplus.
Solution. Demand function : y=16 (*)
(*)
Supply function y=2x2+4
Subtracting (1) from (2) wc have
0=12-3x2
x = 2=x0
When x=2,
y- 16—(2) 1 I 2=y0
Thus when the quantity demanded or supplied is 2 units, the price
is 12 units.
Consumers' surplus

(16 _x 2) dx-2 12

2
=[ 16X----1--24

=32-- -. —24= =533


APPLICATIONS TO COMMERCE AND ECONOMICS ACE- 105

Producers' Sw plus

=2x12- (2x2+4)dx

—24 ["^ ' + Zlx ] = 24


LT +8]=T
—"'

1067
LxarnpIe 87. Demand and supply functions are D(x)=(l2-2V
and S(x)=56 -4x respectively. Determine CS under monopoly (so as to
maximnise the profit) and the supply function is identified with the marginal
cost Junction.
Solution. Total revenue -TR=x x D(x)
=(144-48x -1- 4x2)x
1 44x 48? + 40
MR_r.144_96x.4I2X2
Since the supply price is identiti(,d with MC we have
MC= 56+ 4x
In order to find Cs under monopoly, i.e., to maximise profit, we
must htve
MR=MC
144-96x+12v2=56+4x
12x 2 -- lOOx+88=0
3x2--25x-f-22=0
22
xI=X0 or X-

When x0 - 1, D( 9 )=p=(1 2-2)= 100

Cs_-_f (144_48X+4X2 )dx—IX 100

144x--48 +4 . _]_ioo

144-24+_100_=units.

22 ( 4'1' 64
Again when x ; PD —
= 12— --) =--
BUSINESS MATHEMATICS
ACE- 106
22/3
22 64
CS= (144_4X+4x2)dx

2211
x t x3 22 64 19360 -
K4x- 48. ---44 -s- < ----- -- units.
-

Exa"tp1e 8R. Wizen the price of pocket calculators averaged


.Rs. 400, ABC Co. I41, sold 20 every month. When the price dropped to
an average of Rs. 100, 120 were sold every month by the same company.
When the price was Rs. 400, 200 calculators were available per week for
sale. When the price reached Rs. 100, only 50 remained. Determine con-
sumers' and producers' surplus.
Solution. The demand and supply functions are obtained as
follows
L)(q)- 400 100--400
D (q)-
(1--20 120-20
D(q)=460-3q
S(_0
S (q) 50-200
q--200
S(q)=2q
At equilibrium, D(q)=S(q)
460-3q-=-2q
q=92-'q0
With q0=92 ; p0 = 184

C.S. j (460— 3q) 1q—(92X 184)

= 460q - — 16928

=460x92— .x(92)2 -- 16928= 12696

P.S.92x 184— 2q dq

= 16929— [ qt] 8464


APPLICATIONS TO COMMERCE AND ECONOMICS ACE-107

Example 89. Let p be the price of rice, q the quantity of rice, and
5, the amount of fertiliser used in rice production. Using data fm India
for 1949-- 1964 (Timmer and Pate!), we find for the per capita - demand
function for rice p =0964-6?73q
and for the supply function
q-0 063+0036 S
(1) Find the equilibrium in the rice market if 5=05
(ii) Find the consumer's surplus.
Solution. The dcniaiid function for rice is
J) 0 , 904 6773q
The supply function is
q=-0063 10036 S
For equilibrium, quantity demanded = quantity supplied.
From the two equations, we have on eliminating q)
p0964-6i73 (0063+0036 5)
For 5=0-5, 1)=0 , 964 - 6773 (0 , 063+0 , 0-36x 5)
=0964 --6773 (0-063+0-018]1=0-41 5=p,
and c -0063-I-0'036 == 0063 +0Ol 8- 0081 =q0
are the equilibrium price and quantity exchanged.
0081
(b) The required consumer's surplus =- p dq - p0 q9

0081
(0964--6'773 q) dq-041 5 x 0081

0081
q 6773q2
=[o'964 —0'033615
0

=0964x0081—-7 -- (081)'-0'033615=002225

The Learning Curve

In certain industrial operations such as assembling of television


sets, cars, home appliances, operating printing presses, workers learn
from experience so that the direct labour input per unit of product
steadily declines The rate of reduction in direct labour requirements
is described by a curve calked Learning curve. Ihe general form of the
function is usually taken as
IX)z4x
ACE- 108 BUS1NSS MATHEMATICS
where f(x) is the number of hours of direct labour required to produce
the xth unit, - la<O and A>0. The choice of x" ,with --l0,
guarantees that, as the number of x units produced increases, the direct
labour input decreases.

The learning curve can be used as a predictor to determine the


number of production hours for future work, once it has been determined

Yt

for a gross production process. From a given learning curve the total
number
'1J. is of labour hours required to produce units numbered 'a' through

N: f() dxr AX a dx

Example 90. ABC Co. Li5! tnanufacrur p s air. conditioners on an


assembly line From experience it was determined the first 100 air con-
ditioners required 1400 labour hours - For each subsequen t 100 air
conditioners (1 unit), less labour hours were required according to the
learning curve
Jtx)= 1400 x°
Where f(x) is the rate of labour hours required to assemble the xih unit
(each wilt being 100 air-conditioners). This Curve was determined after 100
units had been manufactured. If the company is in the process of bidding
for a large contract involving 20,000 additional air-conditioners or 200
aj/di!fo,ial units, find the man power required to complete the job.
Solution. The labour hours required to assemble the additional
200 units can be estimated by evaluating
300 3C0
N=J fix) dx= 1400 X-° • dx
100
J
IO
30{)
I 1400 x°•'
= i 07
100
APPLICATIONS TO COMMERCE AND ECONOMICS ACE-1 09

=2000[ (300)°—(100)
I
=2000[ y—z 1 , say

Let y=(300)°7
log y 07 log 300 07 x 2477 I 173397
y=Antitog (173397)=5420
Also let z - (100)0"
log z ==07(log I00)=07x2=14
z=Antilog 14=2512
Substituting the values in (), we have
1V=2000(5420 —25'12)r=58,160
Hence the company can hid estimating the total labour hours
needed as 58,160.
Example 91. After producing 35 units, the production manager
of a company determines that its production facility, is following a learning
curve of the form
AX) 1000.X-0.5
where fix) is the rate of labour hours required to aice,nbie the xlii Unit.
How mony total labour hours should i/wv estimate are required to produce
an additional 25 units.

Soiutiin. NJ 1000 x o• dv

2000 x1212000 ( 601-35'/ )

-2000 (7746-5916)=3660 hours


Rate of Sales
When the rate of sales of a product is a known function of x, say f(x)
where x is a time measure, The total sales of this product over a time period
Tis

/Zx) dx
j
Example 92. Suppose the late of sales of a new product is given by
f(x)=200_90 e-

ACE- 110 BUSINESS MATHEMATICS

where x is the number of days the product is on the market. Find the total
sales during the first 4 days.

Solution. The total saies=J fi x) (IX

1 (200-90 e) dx= 1200x1 90 e I

s 800+90 e 4 -90710+9O e-4


•710-f 90(0'018)=71 162 units.
Fxairiple 93. Assume that in 1990 the annual world use of natural
gas was 50 trillion cubic feet The annual consumption of gas is increasing
at a rate of 3% compounded continoiisly how long will it take to use
all available gas, If it is known that in 1990 there were 2200 trillion
vhic feet of proven reserves ? ,f.csiiine that no new discoveries are made.
[Delhi Univ., B. Corn. (Hons.) 1991]
Solution. We are given that

50. e°°'dt22OO

________I
50 . 003 =2200
0
5000
3 (e''_ )=2200

e°°3 '== ____

003t log e_]og 232


(003 t)(04343) (Q3655)
01655
03x04343281
year

Exaitp1e 94, A firm has the current sales of Rs 50,000 per month.
The firm wants to embark o pt a certain advertising '
campaign that will
Increase the sales by 2 01.
per month (compounded continuously over the
period of the campaign which is 12 months
. Find the total increase in
sales as a result of the campaign. Use calculus.
[Del/If Univ., B. Corn . (lIons.) 19901
APPLICATIONS TO COfMRCE AND ECONOMICS ACE-1 It

Solution. Total increase in sales is given by

-50000e° O2'(jf - 50000x 12


1
12
coot'
-50000 . —50000x 12

=25,00,000(e o it_i) -50,000x 12


Let y=e° 24

log y024 log e-=0 24x 04343 =01042


or y antilog.(01042) = 1272
Total increase in sales is given by
25,00,000 (1 '272-- 1)-- 50J)C0 x 12
6,80,000 - 6,00000 80,000.
Example 95. A company whose annual sales are currently Rs.
5,00,000 has been experiencing sales increase of 20 % per year. Assuming
this rote of growth continues, what will the annual sales be in five years.
Solution. If A is the annual sales in five years, then

A 5,00,000 e°' 2' cit

5,00,000
[e-i
02
-= 5,00,000(8591 39)-. Rs, 4295695
Amount of an Annuity
The amount of an annuity is the sum of all payments made, plus all
interest accumulated.
If an annuity consists of equal annual payments P in which an
interest rate of r°/, per annum is compounded continuously, the, amount
A of the annuity after N payment is

AJPe" d,


ACE-1 12 BUSINESS MATHEMATICS

Example 96. XYZ hank pays 10% per annum compounded


continuously, if a person places R& 10,000 In a sa y ings account each year,
how much will be in the account after 5 years?
Solution. Here P=10,000; N=5 and r=010. The amount A
after 5 years is

10,000 e° '°' dt

5
10,000
1 e 0 10, - '.9.(e5_- 1)
= 010 - 010
0

[06488]==Rs. 64880
011
Example 97. A bank pays interest at the rare of 6% per annum
compounded continuously. Find how much should be deposited in the
bank each year in order to accumulate Rs. 6.000 in 3 years
[Delhi Univ. , B. Corn. (lions.) ; 19921
Solutj, Let Rs. A be deposited each year. Then, we have

6000=,l . e °' di

3
re° 06 ' '1 A
LJ
6(eols_e0)
0

--------te°'8---
1)
006 '
6 000X0'06=A(e O ' i8_ I)

A 6000x006 360
= J-1
Let
log y==O'18 log e=018x0434300782
y=antilog (00782)==1198)
360 360
• A 1 818 18
1198-1 0198
EXERCISES
I. If MC of firm is given by
C(q)==250,


APPLICATIONS TO COMMERCI3 AND ECONOMICS ACE- 113

find total cost if C(0) 100. Also find average cost. What will be
the marginal, average and total cost for q=6() units ?
2. Let the marginal cost function of a firm be 100-- lOx-f 0 . 1 x2,
where x is the output. Obtain the total cost function of the finn under
the assumption that its fixed cost is Rs. 500.
[Hint. MC= 100— 10x+01x2
TC =1(100—lOx .-I-01 x2 ) dx

lOOX_5X2+lk

Fixed cost is 500

TC=l00x_5x 2 + + 500]

3. The marginal cost of production is found to he


MC=' 2000-40x 1-3x 2
where x is the number of units produced. The fixed cost of production
is Rs. 18,000. Find the cost function.
If the manufacturer fixes the Price per unit at Rs. 6800,
(I) Find the revenue function.
(ii) Find the profit function.
(iii) Find the sales volume that yields maximum profit ?
(iv) What is the profit at this sales volume ?
[Flint. C(x)=r1(2000— 40x+3x 2) dx- 2000x--20x2 +x 3 -l- k
C(0)= 18,000 4 C(x)=x— 20x 2 -f 2000 X + 18,0003
4. A company determines that the marginal cost of producing
x units of a particular commodity during a one-day operation is
MC-16x--1591, where the production cost is in rupees. The selling
price of commodity is fixed at Rs. 9 per unit and the fixed cost is
Rs. 1800 per day.
(a) Find the cost function.
(b) Find the revenue function.
(c) Find the profit function.
(d) What is the maximum profit that can be obtained in a one-day
operation ?
[Hint. (a) C(x)=J (MC) dx=-J(16x-159I) dx=8x— 1591x+k
C(0)= 1800 C()=8x2— 1591x 4 1800
(b) R(x)9x
(C) P(x)=- R(v)— C(x) - 8x 2 + 1600x— 1800
(d) P '(X) r—_16X+1600=0 x=100
The maximum profit that can be obtained in one day is
P (lOO) c= __8(100) 2 + I ,60,00— I ,800'= Rs. 78,200.1
ACI3-1 14 BUSINESS MATHEMATICS

3
S. If the marginal cost function is given by
=3q±4 and
fixed cost is 2, find the average cost for 4 units of output. [Ans. 8/71
6. Find the total revenue functions and the demand functions
corresponding to the following marginal revenue functions.
(i) MR=9-4x', (ii) MR=7-4x—x2;

(iii) MR==-6---------
(q2)? 5.

[Ans. (i) Rr=7x --- 2.V —,2 AR=7 --- 2x ---

7.The marginal revenue function of a commodity for output q is


dR 1q -
given by , where R stands for total revenue. What is
the demand function ? [Ans. p=q121

S. If the marginal revenue of output q is given by the equation


dR
where R is total revenue. Find the total revenue function
= --q,
dq
and hence deduce the demand function.
j2
[Ans. B =q-- andq]

ab
9 If the marginal revenue function is MR_ - 1- - show

that is the demand law.


xtb
r C b (IR
t, MR=
Hint, C
- ( (x--b) dx

Nab ab
----c dXr=-------C-fk
J((X$-b)' x+b
where k is the constant of integration. Now .R=O when x=O.
—ab
— + k =O .. k==a.
oh ax
T+--b + xx+b CX

R a
x x+h
10. If the marginal revenue and the marginal cost for an output x
of a commodity are given as
MR=5_4x±3x2 and MC=3+2x

APPLICATIONS TO COMMERCE AND IC0NOM(CS ACe- I 15

and if the fixed Cost is zero find the profit function and the profit when
the output is xr4.
(Ans. Profit function =2x 3x2 + x3 ; 24]
11. Additional earnings obtained by purchasing a new machine is
approximated by R(x) 50x -x2. The annual maintenance costs foi the
machine are C(X) = 4x'. how many years should the machine be
maintained, assuming no salvage value ? What are the total net ciriiings
for that period 2 Costs are in Rs. 100 units and x is in. years.
[Ans. 5, Rs. 1251
12. If the marginal cost function is MC=x 2 - 16x+20 and margi-
nal revenue function is MR=20 2x, determine the profit-maximizing
output and the corresponding total profit. Cost is in units of Rs. 1000
and x is in units of Output.
13. The marginal propensity to consume out of income for the
economy as a whole is given as 4. It is known that when income is zero,
consumption equals Rs. 12 billion. Find the function relating aggregate
consumption to national income. Find aggregate saving as function
of income.
[Ari. C= Yj- 12, S- Y_ 12.]
14. In an economy, the marginal propensity to consume of domes-
tically produced goods is given by
dC dM
and marginal propensity to import is

where C, M and Y stand for consumption, imports and income respec-


tively. What will be the equation for aggregate expenditure of the
economy ? Also give economic interpretation of the constant of
integration.
[Ans. E=K-i-08 Y, where E is aggregate expenditure of the eco-
nomy and K represents autonomous expenditure.]
15. Determine the consumer's and the producer's surplus, given
the demand function D(x)= 25 - 5x-i- (x/4) and supply function
S(x) 5x + (x 2 /4). Assume a monoply situation,
[Ans. 13'02, I 825.]
16. Under pure competition for a commodity, the demand and
supply laws are

and p =_- (xf3) respectively.

Determine the consumer's surplus and the producer's surplus.

[Ans Cs (2)dx2x181og2._2_2 ]
ACI- 116 BUSINESS ?,MTL11MA TICS

17. Find the consumer's surplus (at equilibrium price) if the demand
25 p
function is D= and supply function is p -= 5+ D.

18. Find consumer's surplus and producer's surplus defined by the


demand curve D(x) •20—. 5x and supply curve S(x)4-f-8
Sketch also the appropriate graphs.
413 413
[Hint CS=j (20-5x) dx--P.x--, PS = ±._J(4xi8) dx]

19. The quantity sold and the corresponding price under monopoly
are determined by the demand law p=16--xl and by the CM=6+x in
such a way as to maximise the profit. Determine corresponding CS.
In the above question, if demand law is p=45—x 2 and
determine C.S.

20. Assume that the demand and average cost curves of steel are
p =234— 134x

and AC= —0834 085 x,


x is the quantity of steel demanded or produced.
Show that consumer's surplus under monopoly and perfect competi-
tion is 0351 and 0*129 respectively.
Show also that C.S. would have been cqual to 2 , 043 if steel were
a free good.
21.Find the consumer's surplus if the demand curve is
D(x)=50— 0025x2
and it is known that the market quantity is 20 units.

[Hint. cs=J (500025 x) dx— 40 x 201

22. A business organisation made an analysis of production which


shovs that with the present equipment and workers, the production is
10,000 units per day. It is estimated that the rate of change of produc-
tion P with respect to the change in the number of additional workers
X is
dP
=2003X1l
dx
What is the production (expressed in units per day) with 25 addi-
tional workers 7
A PELICA I IONS TO COMMFRCI ND ECONOMICS ACE- 117

Hint. x detiotes the change in the number of workers. Wheit


there is no change in their number, x.0. When 25 additional workers
lie taken. x 25.
dP
- 2O3x".
dx
Integrating both sides with respect to X • w. get
S (/1'.J (200—.3x';') (Ix
3x3

2i-[k (*)
Using the condition that when x==0, P 10,000, () becomes
10,000---200,<o--o k
10,000
Hence l'-=200x--2 -I 10,000
When x 25, J' 200:' 23 -2(25)'4- 10,000= 14,750.)
23. Fhc production manager of an CICL tron cs coinp.lnv obtai::cd
the following functioti
f(x)= 13564x°°
whereJ (x) is the rate of labour hours required to assemble the h unit
of a product. The function is based on the experience for asenibling
the lust 50 units of the product. The Company was asked to but on a
new order of 100 additional units. Find the total labour hours required
for assembling the 100 units. [An s. 31,460)
24. The purchase price of a car is Rs. 15,000. The rate of cost
for the replir of the car is given by the function
C=60() (I --e-')
where I represents the years of use since purchase and C denotes the cost.
Find the cumulative repair cost at theend of 5 years. Also find approxi-
mate)v the time in yeats at which the cumulative repair cost equals the
original cost of the car.
25. If Rs. 500 is deposited cacti year in a saving account paying
55 per annum compounded continuously, how much is in the account
after 4 years ?

Hint 500 °' 11=9090(eo02236.]


[

26. What is the present value of Rs. 1200 per year at 7 for five
years ? How does this compare with Rs. 100 per month ? (Assume
continuous discounting). (Ans. Rs. 506249, saine)
27. A small daLi-processing company is planning to acquire addi-
tional components for its main Computer. Estimated maintenance COStS
for each unit are C(x) = 3x 2 . Anticipated savings from each added mind

ACE-1 18 uLsu':rss MKIHEMA TICS


are approximated by S(x)2x2+ 16. C(x) is in Rs. 1000 units; 5(x) is
in units of Rs. 10,000 ; and x is the number of units added. How many
units should be added and \hhat are the resulting earnings 7
28. The anticipated idditiona1 sales from a newspaper advertise-
ment campaign are approximated by R(., )=16088 e°°", where R(x) is
extra daily sales in rupees and x is in days. Research has found that
10 days is the maximum period of return for an advertisement If
the advertisement cost is Rs. 11 '99, what is the exnected additional
income at the end of the first day 7 At the end of the fifth day ? At the
end of the tenth day ?
9. Pareto's hypothesis concerning income distribution is given
by the equation y=- A - Ib+1 A and b being positive constants, where y
represents the number of persons with an income of Rs. x and is a conti-
nuous frquencv distribution of persons according to their levels of
income.
Find (i) the number of laconic recipients between income levels p
and q and (ii) their average income
A f 1 1 b
b L ph q1-I --b pb_qb

30. Suppose a law of income distribution states that

0 t /(

where X is income level, U and a are constants and y is a cumulative


frequency of income recipients. Find the number of people falling into
the income bracket (x 1 , x2).

IAns.----[
1b+l_X2b+1 11
31. If the investment flow is given by L— 5(1/4 and the capital
stock at t0 is K0 , find the time path of capital K and also find the
capital formation in the t th period.
[Ans. 41 -f-k0 , 4 3 / 4

32. Obtain the demand function for a commodity for which


elasticity of demand is constant '' throughout.
/)dX
[Hint. - =. dp
- dx -
xdp x p
__ f dx di)
J x j p
—log x=,,. log p +log k. log p +log k
Hence xp 0C
=c.]
APPLiCATLOtS TO COMMERCL AND LCoOMlCS A CE- I 19

APPLICATIONS TO MATIUCFS

Ex al "Ple 98. Mr V LV <1 so!e trader. ni r/ruJaciurin ,ç' tables and


chairs. Each table requires 5 hours of labour and 6 unItS of material A
chair requires 3 labour hours and 3 units of material. if Mr X plans to
produce 10 tables and lS chairs in the ,,et week how may hours will lie
need to work and how much material will he require ?

Solution. The labour requirement is (10 x 5) + (IS < 3)-- 95 hours


The material requirement is (10 x 6'4 (IS 3)= 105 units.
The matrix solution would be
Tables Chairs Labour Materials Labour Materials
(10 I s< () =(95 jos)12

It may be noted that

( 5 6 / 10 1 MO
I 3 is ) 7
is incorrect as labour hours are being added to units of material.
Example 99. A firm produces di fferent pump Units , (- 0(11 0/ which
requires some components shown be/ow in a tabular form
Pump housing Impeller 13o11.v Couplings Inlets Armoured
Hose
Type A 1 / 5 4 2 8 in
Type B 1 1 7 3 2 4 ,n
Type C 1 1 3 5 2 3 tn.
The firm receives an order for 8 Type-A pump units , 4 Type- B units
and 2 Type-C units . Using tire notion of Mairi- multiplication, ohiin die
matrix whose elements may represent the quantifies of each item required
to make up the order.
Solution. The specifications of the different pump units with their
components can be represented by the following matrix,

11 I 1 where each column represents the type of the pump and


each row represents the different components required.
111 The firm has received order for 8 type .4. 4 type B.
2 type C units. This can he represented by the matrix,
I 5 7 31

12221 II 4 II
I I
8 4 3 j 6x3 L 3 J31

ACE-120 BUSINESS MATHEMATICS

Therefore the matrix multiplication of these two matrices gives


1 1 1 1 ) I 1x8±1x411x3 _1
I 1 I
1 1 1 1x8f-1x4+1x3 I
181 I
I 5 7 3 I
II 5<8+7x4-I-3x3
4 I
I 4 3 5 1 I 1 4x8-j-'3X4+5x3 I
I I3 jI
I 2 2 2 f I 2X8+2x4+2x3
L S 4 3 J L 8XI4x4f-3x3 i
The Ivst clement of matrix (= 14) gives the number of component
for housing, the second ( 14) gives that of impeller and so on.
Example 160. The following matrix gives the number of units of
three products (P, Q and R) that can be ircesed per hour on Three
machines (,1, ii and C)
A B C
P ('10 12 15)
() I 13 11 20
R 16 18 14
Determine by using matrix algebra , how many units of each product con he
producer!. if the hoiir available on machines A, ii and C are 54, 46 and 48
respectively. [Delhi Unlv, B. Corn. (lions.), 19921
Solut LOl).
4 1? C
J' o 12 1 F 54 1
15
Units of productcQ 13 11 20 I I 46 I B
RI: -- 16 18 14 i L 48 J C
540-f-5521 720 1
702+506+960
L 864+828+672
1 1812 1 °
2168 IQ
2364 J B
1812, 2168 and 2364 units of product P, Q and Rare produced
respectively.
Example 101. The following matrix gh'es the proportionate mix of
constitUe ' rH used for three fertilisers
Constituent
A B C D
1 05 0 05 0
Fertiliser 2 02 03 0 05
3 02 02 01 05
APPLICATIONS TO COMMERCE AND ECONOMICS ACE-l21

(1) If sales are 1000 tins (of one kilogram) per week, 20/ being
fertillser 1, 30% being fersillser 2, and 50% fertiliser 3; how much of each
constituent Is used?
(II) If the cost of each constituent is 50 palse, 60 poise, 75 poise and
100 paise per 100 grams, respectively, how much does a one kilogram tii
of each fertiliser cost ?
(Ill) What is the total cost per week?
Express the calculations and answers In matrix form.
Solotion, (I) The sales of fertilisers per week can be expressed as
the following matrix
1000(0-2 03 05)=(200 300 500)

Thus
05 0 0'5 0

(200 300 500) f 02 03 0 O'5


02 02 01 05
=(260 190 150 400)
Requirements of constituents are
A: 260, B: 190, C:150, D. 400
(II) Costs of each constituent are 50 p, 60 p, 75 p, and 100 p per 100
grams, I.e., 500 p, 600p, 750 p and 1000 p per 1,000 grams (one kilogram)
of each constituent, respectively.
Thus
500
/05 0 0'5 0 /625
\ 600!
I 02 0. 3 0 0'5 Ix =1 780
1 750
\ 02 02 01 0'S / \ 795
1000
Costs per 1 kg tin of fertilizer are
1: Rs. 6'25, 2 : Rs. 710, 3 : Rs. 7'95.
(1(i) The total cost of fertiliser if 1,000 one-kilogram tins are needed
per week may be calculated by either
/ 625
(200 300 500) ( 780 )=(7.s6500)
795 /

ACE- 122

BU51NSS MATHEMATICS

500

or by (260 190 150 400) 600=(7,56,500)


750
1000
Hence, totalst per week is Rs. 7,565.
Example (A1jJ The total cost of nianufacturjn5 three types of motor
car is given by :/-folhwing table
Labour Materials Sub-Contracted
(lirs) (units) work (units)
Car A 40 100 50
Car B 80 150 80
Car C 100 250 100
Labour costs Rs. 20 per hour, units of material cost Rs. 5 each and units
sub-contracted work cost Rs. 10 per unit. Find the total cost of of
'nanufac
luring 3,000; 2,000 and 1,000 vehicles of type A, B and C respectively.
(Express the cost as a triple product of a three element row matrix
a 3 < 3 matrix and a three element column matrix and perform the ,nu/jj_
plicatlon according to the same rules you used for 2x 2 matrices)
Solution. Let matrix P
sub -contracted work for three typesrepresent labour hours, material used and
of cars A, B, C respectively,
r 40 10050
P==j 80 ISO 80
L 100 250 100
Further let matrix Q represent labout cost per unit, material cost
and cost of sub-contracted work
r 20
Q= S
L 10
The cost of each car A, B, C is now given by the column matrix
11800')
PQ== I 3150 I

L425oJ
Let the number of cars A. B, C to be manufactured in that order
be represented by the row matrix
R==[3000 2000 1000]
Hence the total cost of m anufacturing three cars A, B and C
by the matrix is given

APPLICATIONS TO COMMERCE AND ECONOMICS ACE- 123

11800
PQR=z 131501 x[3000 2000 1000]

4250j
- [1,59,50,000]
F:xat-npte 103. A manufacturer produces three products : F, Q and
B which he sells in two markets. Annual sales volumes are indicated as
follows:
Markets Products
P Q R
I 10,000 2,000 18,000
II 6,000 20,000 8,000
If unit sale prices of P, Q and R are Rs. 25Q, 125 and 1-50_
respectively, find the total revenue in each market with the help of MhJffL
Algebra.
If the unit costs of the above 3 commodities are Rs. 1 80, 120 arid
080 respectively, find his gross profits. -
Solutioii. Total revenue in each market is obtained from the
matrix product
110000 60001

[250 1'25 150]x 2000 20000 =[54500 520001

L18000 8000J
6000
1-10000
Total cost r =tI80 1-20 080) x 1 2000 20000

L18000 8000]
=[34800 412001
Profits from market A-:54500-34800'19700
Profits from market B=52000-41200 10800
Example 104. In a certain city there are 25 colleges and 100 schools.
Each school and college has 5 peons. 2 clerks and I cashier. Each college
in addition has I accountant and I head-clerk. The monthly salary of each
of them is as follows
Peon—Rs. 300 ; Clerk—Rs. 500 Cashier—Rs. 600 ; Accountant—
Rs. 700 ; and Head-clerk— Rs, 800.
Using matrix no:aiion,find
(a) the total number of posts of each kind in schools and colleges
token together.
(b) the total monthly salary bill of each, school and college separately,
and

ACE-124
BUSINESS MATHEMATICS
(c) the total monthly salary bill of all the schools and colleges taken
together.
Solution. (a) Consider the row matrix of order I x2
A=(25 100]
This represents the number of colleges and schools in that order.
5 2 1 1 1 ]
Let
2
L 5 1 0 0
where columns represent number of peons, clerks, cashier, accountant,
head-clerk while rows represents colleges and schools in that order.
Then
rs 2 1 1 1
AB=[25 lOOJxI
lx 2L S 2 1 0 0
2x5
=-f625 250 125 25 100]
1x5
where first element represents total number of peons, second represents
total number of clerks, third represents total number of cashiers, fourth
represents total number of accountants and fifth represents total number of
head-clerks.
(b) Let the column matrix

30 r
1 oo

600
700

L8
represent monthly salary of peon, clerk, cashier, accountant and head-
clerk in that order. Then
(300
I I
I 500 I
r s 2 1 1 i
ftCrr
I x I 600 I
L 5 2 1 0 0J I
2x5 I 700 I

[800 3
1500+1000+600+700+800
[ 1[ 4600
1 500+1000+600+0 +0 3100
2x1 2-<l
APPLICATIONS TO COMMERCE AND ECONOMICS ACt- 12

Thus. total monthly salary bill of each college is Rs. 4600 and of each
school is Rs. 3 100.
(c) The total monthly salary bill of all schools and colleges taken
together is
4600
A ( BC) = [ 25 100] Xr
x2 [ 3100
2x I
r= ,15,000±3,l0,000]
=-[4,25,0001.
Example 105. The allocation of service department costs to produc-
tion departments and other service departments is one area where nafr1X
algebra may be used.
Consider the following data
Service departments Production department
Maintenance Electricity Marching Assembly
Manhours of
maintenance tune - 3,000 16,000 1,000
Units of electri-
city consumed 20,000 - 1,30,000 50,000
Department costs
before any alloca-
tion of service
departments Rs. 50,000 Rs. 4,000 Rs. 1,40,000 Rs. 2,06,000
You are required to
(i) Calculate the total Costs to be allocated to the production depart-
ments using matrix algebra (Formulate the problem and show all workings)
(ii) Show the allocation to the production departments, using matrix
methods.
Solution. (i) Let X be the total cost of the maintenance department
(i.e., including an allocation of electricity costs).
Let Y be the total cost of electricity (i.e., including an allocation of
maintenance costs).
Proportion of maintenance time consumed by electricity department
is
3000 3000
3000+16000+100020000_0 15
i.e., 15% of the maintenance deptt. costs should be allocated to the
electricity department.
Y==4000+015 X

Ace-126 BUSINESS MAT}IEMArICS


Likewise, the proportion of total electricity consumption used by
the maintenance department is
20000 20000
I
so that 10 0% of the electricity cost should be allocated to the maintenance
department.
X=50000+0'I '
From (') and (), we get
—0'15X F Y=4000
X01Y500OO
--015 l\ ( X ) == ( 54000
i —011 \ Y 0000
/ )^( X --015
1
1 )- 4000
Y 1 —01 \50,000
1 01 1 4000
0985( 1 0.15)x(5000{))
( 51,168
1I,675 )
Rence X-=Rs. 51,168 and Y=Rg. 11,675.
(ii) The proportions of maintenance and electricity consumed by
the production departments are
Maintenance Electricity
16,000 1,30,000
Machine ----.= 08
2,00,000 =065
1,000 . 50,000
Assembly 2,00,0000 25
20,000 =0 05
Accordingly the allocations of maintenance costs to the production
department is
(01 O'65\ ( X
I\0•05 025)
(01 0-65)
0Q5 0-25) 11,675
r.e., Rs. 48,523 to machining and Rs. 5,477 to assembly, a total of
Rs. 54,000.

APPCAT1ONS TO COMMERCE AND ECONOMICS kC13-127

Example 106. A, B mid C has Rs. 480, Rs. 760 and Rs. 710 respec-
tively. They utilised the amounts 10 purchase three types of shares of
prices x, y and z respectively. A purchases 2 shares of price r, 5 of price
y and 3 of price z. B purchases 4 shares of price x, 3 of price y aid 6 of
price z, C purchases I share of price x, 4 of price y and 10 of price z. Find
X, y and z.
Solution. We obtain the following set of simultaneous linear
equations
2x+5y-3Z 480
4x+ 3y+ 6z== 760
x-4y4- 10z=710

The above system of equations in the matrix notation is


12 5 31 (x 148O
I I! I
4 3 61 X y = 7601
I II I
LI 4 10J LZJ L 710
1 X i 12 5 3) •-1 1 480
I I 1 I
y !=H4 3 6X I 7601
I I 1 I
LZJ L 4 loJ L71 0J
1 2 5 3

Now A -I; where I A 4 3 6-1I9

—38 +21
1+6
and Ad] A = —34 +17 0 (Try yourself

L+13 - 3 —14J
From ('), we get
I - 1-1-6 —33 4-211 f 4801
I II
'' I--3 4 +17
yI=—------ 0 lxi 7601

L z J L+13 —3 —14J L 710J

1 r 6x480-38x760-I--21X710
—34x480+17X760-F0 x710
119
L 13x480-3 x760-14x710
r —11090 1 110901119
-1 —3400 == 34001119
119 5980/119
L —5980 J

ACE-123
RUS!NJsS MATHEMATICS

Hence
11090 3400 5980
Z_1•
x= 119'
Example 107. To control a certain crop disease it is necessary to
use 8 units of chemical 4, 14 units of chemical B and 13 units of chemical
C. One barrel of spray P contains one unit of A, 2 units of
Of C. One barrel of spray Q contains 2 units of A, 3 units of13 and 3 units
of C. One barrel of spray R contains one unit B and 2 units
of A, 2 units of B and 2 units
O f C. flow many barrels of each type of spray should be used to control
the disease 7
Solution. To grasp the situation easily, let us tabulate the data
as follows
Spray Requirement in
chemicals
P Q
A 1 2 1 8
Chemical B 2 3 2 14
3 2 2 13
Quantity in C
each spray x y z
Let x barrels of spray P, y barrels of spray Q and z
R be used to control the disease. Then barrels of spray
x+2y--z =8
2x+3y+2z14
3x+2y+2J3
Writing the equations in the matrix form, we get
[1 2 11rx1r8
2 3 2 xj y 14
L3 2 2J LzJ [i3
rxl ri 2 11- 1 r 8
I -' I I 2 3 2 Ix 14
LzJ L3 2 2J L13
Now r i 2 1 1-1 r +2 —2 +1 1
2 3 2 +2 —1 0 (Try yourself)
L3 2 2J L-s +4—h
r x 1r 2 —2 -l-I1r81
Y '=1 +2 —1 0 x 14 kI 2
L z - L —5 +4 —1 [ 13 J [ 3
X=1,y2 and z=3
Hence I barrel of the spray P.
ray R should be used to control 2 barrels spray Q and 3 barrels of
the disease.

APPLICATIONS TO COMMeRCIR AND ECONOMICS AC1-129


Example 108. The XYZ Bakery Ltd. produces three basic pastry
mixes A, B and C. In the past the mix of ingredients has been as shown
fri the following matrix
Flour Fu Sugar
A 5 1 1
Type B 65 25 0'5
C 4-5 3 2
(All quantities in kilogra,n weight)
Due to changes in consumer tastes it has been decided to change the
mixes using the following amendment matrix
Flour Fat Sugar
A 0 +1 0
7ype B —0-5 +05
C +05 0 0
Using matrix algebra you are required to calculate
(I) the matrix for the new mix:
(ii) the production requirements to meet an order for 50 units of
type A, 30 units of type jj and 20 units of type C of the new mix;
(iii) the amount of each type that must be made to totally use up 3700
kgs, of flour, 1700 kgs of fat and 800 kgs of sugar that are at present In
the stores.
Solution. (i)
The new mix is given by the addition of the original
mix matrix and the amendment matrix.
5 1 1\ / 0 -f-I 0\ /5 2 1
65 25 05 —05 +05 +05 6 3 1
=
(4 5 3 2 ./ o 0/* 5 3 2
Therefore, the answer to part (i) is
Flour Fat Sugar
Type 5 2 1
Type 6 3 1
Type 5 3 2
(Ii) To determine the production requirements it is necessary to
multiply the order vector by the new mix matrix,
5 2 1
(50 30 20) 6 3 1 =-(530 250 120)
5 3 2

ACE- 130
BUSINESS MATHEMATICS
•. 530 kgs of flour, 250 kgs of fat, 120 kgs of sugar
(Iii) 5X,-6K2+5X3700
2X 1 + 3X2 4-3X3 1700
X1 +X21-2X3=_800
5 6 5 X1 / 3700
/ '
2 3 31 A'2 1700

1 2/ \ 800
AK=-B

/X1 \ / 5 6
A'2 I"- 2 5\-1 /3700
3 3 x( 000
\X,/ i 1 2/ '\ 800
On s i mplification, we get
X1 = 400 X.4 =200 and Xz= 100.
Example 109. A mixture is to be made of three foods A, B,
The three foods A, B, C contain nutrients P, C.
Q, R as shown in the tabular
column. Flow to forma mixture which will have 8 gms of F, 5 gm
of Q, and 7 grns of R,
gqis per kg of
Food Nutrient P
Nutrient Q Nutrient R
A 1 2 5
B 3 / 0
C 4 2 2
Solu tion. Let x kgs of food A, y kgs of food B, and z kgs of food C
he chosen to make up Uie mixture.
Then we have the equations,
x+3y+4z-8
2x+ Y+2z=5
5x+2z7
Expressing these equations as a single matrix equation, we have
1 3 4)X( x \/8
2 1y s
0 2z/\7
3 4 )=( 8 Apply
/1
or [0 —5 - 6)x( yx -11 R+(-2)R1
0 —15 —18 z _J3 / R3+(-5)R1
APPLICATIONS TO COMMFRCC AND P.CONOMICS A(,E-131
1 3 4\ 8 Apply
y 1
/ X

or 1 0 5 6 x I It 1) R,

0 0 0) 1
0 R3-l-(-3)R2

Therefore, we have
x -1- 3y+ 4z - S • .
5 Y-1- 6z=- • (4*)

Let zfl-a. From (**), 5y-6a=I I, i.e.,

Substituting in (*) x + 3 + 4a 5

5x-l-3(11 --6a) F204O


7--2a
2a or
5x=7--2(z x

7-2n 11 -6a
The solution I S x = ---, y ------- , z =a.

As 'a' changes, we can get any number of solutions and thus there
are any number of mixtures. Sinc ' x, y, z take non-negative values zO,
I.e., a > 0.
Considering the value of x, we have

0, i.e., 7 -2a 0. i.e., 7 2a, i.e., a J1)

Considering the value of y,

0, I.e., It —6a 0, i.e., I I 6a, I.e., a ...(1I)

The restriction (II) covers the restriction (1)

Therefore, we have 0 a

When a==I,xl,yr=1 and z=I.


Example 110. ABC company has two service departments, S1
and S,and four production departments, P1, P1. P. and P4.
Overhead Is allocated to the production departments for inclusion
in the stock valuation. The analysis of benefits received by each depart-
ment during the last quarter and the overhead expense Incurred by each
department were

AcE- 132 BUSINESS MATHMA11C5

Service Percentages to be allocated to departemnts


Department

S1 S2 P, P, P3 P4
S1 0 20 30 25 13 10
S2 30 0 10 35 20 3

Direct overhead 20 40 25 30 20 It)


F-Vpenje Rs '000
I J
You are required to
(i) express the total overhead of the service deparimenis in the form
of simultaneous equations
(ii) express these equations in a matrix form
(iii) determine the iotal overhead to be allocated from each of S 1 and
S2 to the production department.
Soludon. (I) Let
S 1 = tal overhead of service department S1
S,-= total overhead of service department S,
Then S=20,000+03 S2
32 -- 10,000+02 S1
Written as sjmujtaneoiy equations, this becomes
S1 -03 S,=20,000
—02 Si-I- S2==40,000
(ii) In matrix form, the equations are written as
E A S
/ 20,000\ / 1 —03) X ( S1
1=1
\ 40,000 1 \ - 02 1 S2
S A-' E
S1 ( I —03 \' / 20,000
=1 lxi
\ S2 / \ — 02 1 / \. 40,000
(Iii) By the normal rules for finding the inverse of a 2 > 2 matrix, this
equals
S1 1 03 )X( 20,000 34,043
( (
S2 ) tY94 02 1 40,000 46,808

APPLICATIONS TO COMMI RCH AND ECONOMICS ACE-133


The allocation of overhead from S and S1 becomes
Pt
P
( 1 )x(0'3 025 015 01)=
F'
P4
10,2!)
/
,51E
(34,043)X(03 025 015 01)
5,106
3,404
P1

and (S.)x(0'1 035 02 005)=


P3

P4
4.681
16,383
46,83) (010 , 35 02 025)=
9,367.

2,340
The final allocation becomes

Department Total P, P3 P3 PA

Rs. Rs. Rs. Rs, Rs,



S1 27,234 10,213 8.511 5,106 3.404

S. 32,766 4,681 16.383 9,362 2,340

Total 60,000 14,894 24.894 14,468 5 , 744

LEONTIEF INPUT-OUTPUT MODEL


The Leontief input-output model in ec
onomics (named after
Wassily Leontref, a recipient of the Noble prize in Economics in 1973)
may be characterised as a description of an economy in Which input
equals output, or in other words, consumption equals production, I.e.,
the model assumes that whatever is produced is always Consumed
ACE-134 BUSINESS MATHEMATICS

Input-output models are of two types : closed, in which the entire


productio:i is consumed by those participating in the produ':tion ; and
open in which some of the production is consumed by those who produce
it and the rest of the production is consumed by external bodies. In
the closed model we seek the income of each participant in the s)stern.
In the open model, we seek the amount of production needed to achieve
a forecasted demand when the amount of production needed to achieve
a current demand is known.
Consider an economy consisting of n industries where each industry
produces only one type of product (output). There is all
of industries in the sense that one must use other products to
operate. Also the production of the finished product must meet the
final demand as well as the demand of the other industries.
Our problem is to determine the production of each of the
industries if the final demand changes, assuming that the structure of
the economy does not changc. The data is tabulated in the following
input-output transaction table
To (user) Final Total
1 2 3 ... n demand output

I .x11 X X1 ('I


2 . X22 'T23
X,
x2


X X J ,, (/3
3 X31 x31
A.,

n X.1 Xn2 X .. d
x,.,, X.
j, i.e., it represents
where X, 1 is t he output of industry I sold to industry
the rupee value of the product of industry I used by industry j.
Now
represents the rupee value of the total output of industry I.
x,1 Rupee value of the product of industry I used by industry j.
Xj Rupee value of the total product of industry I.
=Rupee value of the out-put of industry I that industryj must
purchase to produce one rupee worth of its own product.
=a,1 (say)
In other words,
X, i —_ aaj Xj amounts to saying that sales of industry ito industry
j,
j are a constant proportion a of the output of industry j.
=Rupee value of the product of industry I used by industry].

A PPLICATrONS TO COMMFRCE AND ECONOMICS


ACE-135
Now we introduce the matrix of input cOefficit5.

Oil a 12 ... a1
I I x
a 21 a, 2 ... a2 where aIlkL

etc.
a 1 a,. J
Replacing each X, 1 by a, ,, X,,
in the table, we get the set of simul-
taneous linear equations

a 11 X1-f-a1, A' + ... + a 1, X+d1-X1


a 21 X,+ a22 X2 f • ..• .-f 1 5 Xfl + 11,
=
a,1 Xi+ a,,2 X, 4- X,.+drX
which may be written in the matrix form as
X=AX+D
X—AX=D
(I — a 11 ) X1 —a 12 X—a 1 Y3—_—a

— a 21 X1 —(1_a 22 ) X2 —a25 X.__ —a Xd2


— a. , X1 —a, 2 X2—aX_. ...(I a) X,,=d
In the matrix notation this may be written as
/1—a11 —a1,
J—a55
12
/ X1 \ / d1
1122 —as,, X2
( ( d2
—a,,. • . 1—a,,/ X"
(1—A)X=D
4.

where I is the matrix of input coefficient, while X and D


of output and final demand of each industry. are tho vectors

'Leontief w hile developing the input-output analysis made the


of direct proportionality between the output and the I ndividual inputsa ssumption
industry. of the
Ac13-136 BUSINESS MATHEMATICS

Example iii. Given the following Transaction matrix, find the


input-output coefficient:

\ Purchasing
sector 4grc1ure Industry Final
demand

Producing \
sector
_--
Agriculture 300 600 100

Industry 400 1200 400

Lmer3fl 200

Find also total output as well as total input.


Solution. Total output for Agriculture is
300+ 600+ 100=1000 and
for industry 400+1200+400=2000
Similarly total input for Agriculture is
300+ 400+300=1000 and

for industry 600+1200+200=2000


The above transaction can be put in the following way

\ prchas1ng
1
sector Industry Final Total
Agriculture
demand output
Producing \.
sector input

300 600 100 1000


Agriculture

400200 400 — 2000


Industry

300 200 0 500


Conun1er
-
1000 2000 500 3500
Total input

APPLICATIONS TO COM.MIIRCO AND ECONOMICS ACO-137


Now to lind out input-output coellicicuts
A coefficient is obtained by industry's input by total output. It is
an indication of the number of any industry's Output needed to produce
one unit of another industry's output.
Therefore, coefficient of input-output can be obtained as follows
300 -- 600 - —0 30

1000 —0 0 2000
40() -- 126P
0
1000 --040 ' 2000 - 06
300 030 - 200 00
1000 , 2000
which can be represented as follows

"N.Purchning
\,sect,r
'N output Agr(cultu r Industry
'N
Producing \
sector input \

Agriculture 030 030

Industry 0-40 0O

Consumer 0 30 010

Exariiple 112. Suppose the Interrelationship between the production


of Iwo industries R and S In a given year ic -
Current Consumer

R S Demand Total output

P 14 6 8 28

S 7 18 11 36
If the forecast demand in two years is
r 20 ]
D2=j
L 3°
What should be total output X be ?
Solution. Step 1. To obtain the input-output matrix, we deter-
mine how much of each of the two products R and S is required to
produce one unit of R. For example, to obtain 28 units of R requires
the use of 14 units of I? and 7 units of S (the entries in column one).
Forming the ratios, e find that to produce 1 unit of R requires
AC-138
BUSINESS MATHBMATIc,s
14128== of R, 7/28 of S. If we want, say X1 units of R, we will
require j X1 units of 1?, j X1 units S.
Continuing in this way, we can construct the input-output matrix as
follows
R S
R1
li
It may be noted that column 1 represents the amounts
R, S required
for one unit of R, Column 2 represents the amounts of R, S
required for
one units of S. For example, the entry in row 1, column 2 represents
the amount of S needed to produce one unit of S.
As a result of placing the entries this way, if

XH
rxi
L x j
represents the total output required to obtain a given demand, the
product AX represents the amounts of R and S
Co nsumption Here the total output is required for internal
28 r
L 36
The correctness of the values in A may be verified by noting that
[1 X [ 281 r 20

] 36 J 25
r 20 1
where
25 J
I represents the internal needs of R and S.
L
If the demand vector is
181
Do=I
L 11 J
then for production to equal co nsumption, we must have
Internal needs+Consumer demand =Total output
(*)
In terms of the input-output matrix A, the total Output X, and the
demand Vector D0, (4) becomes
AX+D0X
Again, the correctness of this result may be verified since for the
demand vector D 0 , we know the output is
1 28 ]
XH
L 36

APPLICATIONS TO COMMERCE AND ECONOMICS ACE-139


To find the total output X, required to achieve a future demand
1 20

L 30
we need to solve for X j
AX+D2X
Siniplifyi ng, Nve have
(I - - X—D
Solving for X, we have
X; -(I --A)-' D2.

-, 1--i r201

J L30
24 1 r20
5 30
2 4 1 15 1 1 72
5 [ 20 YL 96
Hence the total :utput of B and S for the forecast D 2 is
X1 72, X=96.

Ex.inple 113. Given the following transaction matrix, find the gross
output to meet the final demand of 200 units of Agriculture and 800 units
of Industry.
Producing Purchasing Sector Final
Sector Agriculture Industry Demand

Agriculture 300 600 100


Industry 400 1200 400

Solution.
Producing Purchasing sector Final Total
sector 4gr icu It ure Industry Demand Output
Agriculture 300 600 100 1000
Industry 400 1200 400 2000
The input-output coefficients can be obtained as follows
300 3 600 3
0iL 1606 To' fOO6 10
4 00 2 1200 3

c13-140
BUSINU.SS MA1ATC

The technology matrix is


/3 3
(10
3
5

—5
5
(- -5 - )
Il---AI =Th-x----(----)x(--)=
/2 3
(I_A)-1=(5 10
4 2 7
10
Now X=r-(I_--A)-' D
/2 3\
(X =(I—A)-' D 25 ( 5 10 (100
4 t 2 7) \400
10/
25 160'\ ' 10
00
4 320 )r=ç 2000)
which verifies the given data.
The new demand vector is D_1 200
800
Then

X(1—AY' D? ( 200
)x(

X1 ' 25 (320 ) ( 2000
' X, 4 640 4000
Hence the Agriculture and Industry sector must produce 2000 and
4000 Units to meet the final demand.

EXERCISES
I. The prices of 3 commodities A, B and C
Rs. 6 and Rs. 10 respectively. Customer X buys 8 Unitsin a shop are Rs.
of A, 7 units of
B and 6 units of C. Customer Y buys
6 units of A, 7 units of B and 9

APPLICATIONS TO COMSWR03 AND eCONOMICS Ac-141

units of C. Show in matrix notation, the prices of the conn11odities7


quantities bought and the amount spent.
2. Two types of food, I and 2 have a vitamin content in units per
kg given by the following table
Vitainn A V(t,n1, B
Poodl 3 7
Food 2 9
Express the vitamin content of 5 kg of food I and 6 kg of food 2
as a matrix product and evaluate it. If food 1 costs 30 paise per kg and
food 2 costs 35 paise per kg, express the cost of 5 kg, 6 kg of foods 1, 2
respectively as a matrix product and evaluate it.

9), i.e., 27 units of vitamin A and


I Hint. (5 6) (
89 units of vitamin 11.
(5 6) ( )(36o), i.e., the cost is Rs. 360.)

3. A motor corporation has two types of factories each producing


buses and trucks. The weekly production figures at each type of factory
are as follows
Factory A Factory B
Buses 20 30
Trucks 40 10
rue corporation has 5 factories A and 7 factories B. Buses and
trucks sell at Rs. 50,000 and Rs. 40,000 respectively. Express in matrix
form and hence evaluate
(i) The total weekly production of buses and trucks.
(ii) The total market 'value of vehicles produced each week.
30 ( 5 " (310'
[Ans. (1) (20 )27oP i.e., 310 buses, 270 trucks
40 10) 7
30)\
(ii) (50000 40000). 20
10 / ( )
=(50000 40000) . ()=(2,63,00,000),
i.e., the total weekly value =Rs. 2,63,00,000]
4. In a certain coal mine, the amounts of Grade 1 and Grade 2
coal (in tonnes) obtained per shift from each of two teams, A and B are
given by the following table
Grade I Grade 2
Team A 4,000 2,000
Team B 1,000 3,000

Ac13- 142 BUSINESS MATHEMATICS

Team A has worked 5 shifts per week and team B has worked 4 shifts
per week. Grade 1 coal sells at Rs. 9 per tonne and Grade 2 coal sells
at Rs. 8 per tonne. Find
(1) the total amount of coal mined each week,
(Ii) the market value of the coal mined each shift,
(III) the market value of the coal mined each week.
[A us. (i) (24,000 ; 22,000) tons of Grade 1 and Grade 2 respectively.
/ 52,000 \ / 4,000 2,000 \/ 9
(ii) ). (iii) (5 4)( II
\ 33,000 / \. 1,000 3,000 A 8
5. A builder develops a site by building 9 houses and 6 bungalows.
On the average one house requires 16,000 units of materials and 2,000
hours of labour ; one bungalow requires 50,000 units of materials and
4,800 hours of labour. Labour costs Rs, 5 per hour and each unit of
material costs, oil average Rs. 10. Express in matrix form and hence
evaluate
(i) The total materials and labour used in completing the site.
(ii) The cost of building a house and a bungalow.

Ans. (1)
I (9 6)
/
(iii) The total cost of developing the site.
16,000
\ 50,000
2,000
4,800 )
/ 16,000 2000) ( 10
(ii)
50,000 4,800 5)
/ 16,000 2,000\ / 10 \1
(Iii) (9 6)
50,000 4,800) 5 )j
6. Two television companies, TV, and TV, both televise documen-
tary programmes and variety programmes. TV, has two transmitting
s tations and T V,
has three transmitting stations. All stations transmit
different programmes. On an average the TV, stations broadcast 1 hour
of do cumentary and 3 hours of variety programmes each day, whereas
each TV2 station broadcasts 2 hours of documentary and 14 hours of
'ariety programmes each day. The transmission of documentary and
v ariety programmes costs approximately Rs. 50 and
Rs. 200 per hour
resp ectively. Express in matrix form and hence evaluate
(1) The daily cost of transmission from each TV, and each
TV,
station.
(ii) The total number of hours daily which are devoted to docu-
mentary and to variety programmes by both corporations.

APPLICATIONS TO COMMflRCC AND ECONOMICS ACE-143

(Iii) The total daily cost of transmission incurred by both corpo-


rations.
r (1 3)X( 50 \/650
Aiis. (I) (2
t oo / \. 400
i.e., Rs. 650, Rs. 400 per day respectively for each TV, 7'V2 station.
/1 3 \
(ii) (2 3) ( 1=(8 1OD
\2 UI
i.e., 8 hours documentary and 10 hours variety.
/ 1 3\ / 50\ /650\
(iii) Rs. (2 3) ) ' Rs. (2 3) - Rs, 2,500
\2 l. / \ 200/ \400/

7. A firm produces five qualities of its product which needs the


following materials
Quality Materials needed
m a M4
A l 6 6 10 8
A, 3 4 12 6
A 5 4 5 15 8
A 4 2 2 12 5
A 5 3 2 10

If the firm has to produce, respectively, 3, 22, 20, 12 and 7 units of


the five qualities find the amounts of different materials required by
writing their requirements as a row vector.
(Ans. (169, 194, 658, 324)J
8. A publishing house has two branches. In each branch, there
are three offices. In each office, there are 6 peons, 8 clerks and 10 typists.
In one office ofa branch, 12 salesmen are also working. In each office
of other branch 4 head-clerks are also working. Using matrix notation
find (i) the total number of posts of each kind in all the offices taken
together in each branch, (ii) the total number of posts of each kind in all
the offices taken together from both branches.
A1 A2 A3

9. A=I 4 6 /14 6 8\ 10 14
1! 8 10 12,B=10 12 14 ,C 18 32 26
==
III 16 18! 18 20/ \3o 34 38

ACE-144
BU3)NFS3 MATHEMATICS
Matrix A shows the stock of 3 types
shops A 1 , A 2 , Aa. Matrix B of items i 11, 111 in three
shows the number of items delivered to
three Shops at the beginning of a week. Matrix
C shows the number of
items sold during that week. Using matrix algebra, find
(i) the number of items im
mediately after the delivery,
(ii) the number of items at the end of the week.
10. The following matrix gives the v
in con veniently chosen units itamin content of rood items,
Vitamin: ' A B C D
Food! s i o o
Food II 3 0 2 '1
Food III - '1 •2 5
If we eat 5 units of food I,
Ill, how much of each types 10 Units of food I!, and 8 units of food
of vitamin we have consumed7 If we pay
only for the vitamin Content of each food, paying 10 paise, 20 paise, 25
pa 'se, 50 paise respectively for Units of the four vitamins, how much
does a unit of each type of food costs 7 Com
food eaten. pute the total cost of the

1 15
[Ans. (63 3 . 3 36 50); 13 ; Rs. 469]
33 j
11. A m anufacturing unit produces three types of products A,
B, C.
The following matrix shows the sale of products in two different Cities.
B C
1200 900 600
(
900 600 300
If cost price of each product A, B, C
3000 respectively and selling price Rs. 1500, is Rs. 1000, Rs. 2000, Rs.
Rs.
tively, find the total profits using matrix algebra only. 3000, Rs. 4000 respec-
It The production of a book involves several steps first it
must be set in type, then it must be printed and finally it must be
supplied with covers and bound. Suppose that type setter charges Rs.
6 per hour, paper costsJ paisa per sheet, that the printer charges Ii
paise for each minute that his press runs, that the cover costs 28 paise,
and
a pu the binder charges 15 paise to bind each book. Suppose now that
blishers wishes to print a book that requires 300 hours of work by
the ty pesetter, 220 sheets of paper per book and five minutes
time per book. of press
APPLICATIONS TO COMMERCE AND ECONOMICS ACE-145
(1) Using matrix multiplication, find the cost of publishing one
copy of a hook.
(It) Using matrix addition and multiplication find the cost o
printing a first edition run of 5000 copies.
(iii) Assuming that the type plates from the first edition are used
again, find the cost of printing a second edition of 5000 copies.
(Ans. (i) Rs. 180153, (ii) Rs. 9450, (iii) Rs. 7650]
13. One unit of commodity A is produced by combining I unit
of land, 2 units of labour and 5 units of capital. One unit of 11 is produc-
ed by 2 units of land, 3 units of labour and I unit of capital. One unit
of commodity C results if we use 3 Units of land, I unit of labour and 2
units of capital. Assume that the prices are P 27, Pb l6 and Pr _19.
Find the rent R, wage W and rate of interest I. (Use matrix method).
14. To control a certain crop disease it is necessary to use 7 units
of chemical A, 10 units of chemical B, and 6 units of chemical
C. One
barrel of spray P contains 1, 4, 2 units of the chemicals, one barrel of
spray Q contains 3, 2, 2 units and one barrel of Spra y R contains 4. 3, 2
units of these chemicals respectively. flow much of each type of spray
he used to control the disease ?
[Ans. 14 barrels of spray P, j
barrel of spray Q and one barrel
of spray RI
15. A certain company gets the automobile chassis and then builds
3 types of bodies, v(2., luxury coaches, ordinary passenger bus and lorries.
For a luxury coach 5 supervisors and 20 skilled labourers, for a passenger
bus 3 and 12, for a lorry 2 and 11 of these categories, are required for a
(lay's work. If 50 supervisors and 260 skilled labourers are available
how many coaches, buses and lorries could be built 7
16. A firm manufactures 3 products P, Q, R using 20 machines
of type L, 12 machines of type M and 15 machines of typ
e N. If the
machinery time requirements are given in the following table, find the
production quantity of each product during a 40-hour week.
Alec/tines
Product L M N
P 3hr. 2hr. 4hr.
Q 2hr. 1 hr. 2hr.
R 4hr. 3hr. I hr.
[Ans. 16 units of P, 232 units of Q and 72 units of product R.]
17. In a market survey three commodities A, B and C were con-
siderecl. In finding out the index number some fixed eights were
assigned
o to three varieties in each of the c ommodities. The table below
Pr vides the information regarding the consumption of three commodities
according to three varieties and also the total weight received by the
commodity

Ad! . 146 BUSINESS MAThEMATICS

Commodity Variety Total Weight


1 11 111

A 1 2 3 11
B 2 4 5 21
C 3 5 6 27
Find the weights assigned to the three varieties by using Matrix-
inverse method, given that the weights assigned to a commodity are equal
to the sum of the weights of the various varieties multiplied by the
corresponding consumption, [Ans. 2 3, I]
18. The monthly expenditure in an office for three months is
given below according to the type of staff employed
Total
Month No. of Employees monthly salary
Clerks Typists Peons (Rs)
April 4 2 3 4,900
May 3 3 2 4,500
June 4 3 4 5,800
Assuming that the salary in all the three months of different cate-
gories of stall did not vary, calculate the salary for each type of staff
Per mensum using the matrix method. [An. 700, 600, 3001
9. The following table shows the fixed cost (F) and the variable
cost (V) of producing 1 unit ofXand I unit of Y

Product
X y

5 8 (Rs. '000)
Cost F J
4 12

When x units of X and y units of I are produced, the total fixed


cost is Rs. 6,40,000 and total variable cost is Rs. 8,20,00 0. Express this
information as a matrix equation and hence find the quantities of X and
Yproduced. [Ans. x=40, Y=55]
20. A salesman has the following record of sales during three
months for three items A, B and C which have different rates of
commission.

APPLICATIONS TO COMMIRCO AND ECONOMICS ACE- 147

Months Sales of Units Total Commission


drawn (in Rs.)
A B C
January 90 100 20 800
February 130 50 40 900
March 60 100 30 850
Find out the rates of commission on items A, B and C.
[Ans. Rs. 2,4 and Ill
21. (a) We consider buying three kinds of food. Food I has one
unit of vitamin A, three units of vitamin 13 and four units of vitamin C.
Food II has two, three and five units respectively. Food 1(1 has three
units each of vitamin A and vitamin C and none of vitamin B. We need
to have 11 units of vitamin A, 9 of B and 20 of C. Find all possible
amounts of the three foods that will provide precisely these amounts of
the vitamins.
(b) One unit of food I contains 100 units of vitamins, 60 units of
minerals and 80 calories. One unit of food 11 contains 150 units Of vita-
mins, 60 units of minerals and 180 calories. One unit of food 111 contains
90 units of vitamins, 40 units of minerals and 100 calories. Diet require-
ment for a patient is 1100 units of vitamins, 500 units of minerals and
1200 calories. Find out either by matrix method or by determinants
method how many units of each food be mixed to form the diet which
would meet the requirements exactly.
22. An automobile manufacturer uses three different types of
trucks T1 1 and T,,, to transport the number of station wagons, full size
and intermediate size cars as shown in the following matrix
Station Full-size Intermediate-size
Wagons Cars Cars
'1L 2 6 9
Trucks '2 7 12
TaL 6 6 8
Using the inverse of the matrix, determine the number of trucks of
each type required to supply 58 station wagons, 75 full-size, and 62
intermediate-size cars to a dealer in city A.
If a dealer in city B orders 46 station wagons, 60 full-size and 64
intermediate-size cars, how many trucks of each type does the factory
need to make this delivery.

[Aus. A'
[ i - _!
City A; Station wagons 2; full8ize cars 3 ; Intermediate cars 4
City B: I P, 5; ,, 3; ,, ,, 21

ACE- 148 I3USINJ3SS MATHEMATIC

23. For the following input-output table, calculate the technolog'


matrix and also write the balance equation for the two sectors
Sector A B Final demand
A 50 150 200
/1 100 100 75
24. Suppose the interrelationships hetseen the production of two
industries P and Q in a given year is
Curreni Consumer
P Q Demand iota! output
P 30 '00 60 130
Q 20 40 10 70
If the forecast demand in two years is
1 o
L 40
what should the total output X he ?
25. The following table gives the input-output coefficients for a
two-sector economy Consisting of agriculture and manufacturing industry.
!npu'-ou1p4( Coefficient

I\ mnpu
\.,
I Al

LL
I

The final demands for the two industries are 300 and 100 units
respectively. Find the gross outputs of the two industries.
j
If the input coefficients for the labour for two industries are respec-
tively 05 and 06, find the total units of labour required.
26. Consider an oversimplified two sector economy in which
there are two industries, each producing a single commodity. The
production of Re. one worth of the first industry's Product requires
material worth of 30 paisa of the first industry and 20 paisa of the second
industry. The production of the second industry's product worth Re.
one requires 10 paisa and 30 paisa material of the first and second
industries respectively. Determine the output levels of each industry
necessary to meet the open sector demand of Rs. 12 million and Rs. 5
million worth of goods of the first and second industries respectively.
[Ans. 20, 10]

APL'UCArIONs TO COSIMJIRCU AND IiCONOMICS

27. In aii economy there auc two industries A and ii and the
following table gives the supply mid dcinauid position of these ill iuiilhon
rupees

User Final 'I'otal

A II Demand Output

Producer A 15 10 10 35

/3 20 30 15 65

Determine the total output if the final demand changes to 12 for


and 18 for B. [Ai1. 42, 78]
28. In an economy of three industries A, B, C the data is gi-p'"
below (in millions of rupees of products).
User Haul Total
A B G deniwj if output
/1 80 100 100 40 320
Producer 13 80 200 60 60 400
C 80 100 100 20 300
Determine the output if the Final demand changes to
(/) 10 for A, 40 for II, 20 for C.
(ii) 60 for A, 40 for 13, 60 for C.
{A,;s. (1) 179 : 13, 24522, 189'1 3 (ii) 41739, 45565, 41739J
29. Suppose that the final demands for steel, coal and electricity
in an economy consisting only of these three Sectors are Rs. 10
crores, Rs. 5 crorcs and 6 crores respectively. It is given that a
Rupee worth of steel requires 21) paise, 41) paisc and 10 paise worth
of steel, coal and electricity respectively as inputs, a Rupee worth of
coal requires 30 paise, 10 pamse and 30 paise worth of steel, coal and
electricity respectively as inputs and that a Rupee worth of electricity
requires 20 paise worth of steel, coal and electricity each as inputs
respectively. How much of steel, coal and electricity should be produced
to satisfy both final and intermediate demands ?
[Hint. Matrix of input-output coefficients is
r 020 040 010
A= 030 010 030
- 0*20 020 020
30. A pharmaceutical company produces three products X, Y
and Z which are partially used in the manufacture of these products.
However, none of the products is In its Own manufacture The
quantities of the outputs of each product which are used as inputs in the
manufacture of one unit of each of the other products are

BUSINESS MATHEMATICS
ACE- 150
Input
X Y Z

X o 03 0•4

Output Y 02 0 03

z 0-i 0-5 0

The production targets for each product are Rs. 1,50,000 for X,
Rs. 2,00,000 for Y and Rs. 100,000 for Z, these being the amounts of
the three products which are to reach the final consumer. Use input-
output analysis to determine how much of each of the products should be
produced.
31. From the following mat rix , find out the final output goals of
each industry assuming that consumer output targets are Rs. 80 million
in steel, Rs. 30 million in coal and Rs. 0 million iii railway transport
Steel Coal Railway transport
Steel 03 0,2 02
Coal 02 01 05
Railway transport 0 . 2 0.4 02
Labour 03 0.3 0_I
What would be the labour requirements in final output of three
industries ?
-i 07 ---02 —0-2
uint. .'. [1 —A)= —02 -+09 05
1 —02 —04 +08
Substituting in K=[I—A] D, we get
r X1 1 r —02 01 —02 —0'2 jl r 80
XH X HI 019 —05 xj 30
%X9 J L —0'2 —04 0'8 I 50
After inverting the matrix, we get the required result.]
32. D Limited produces three products, x, y and z oil different
types of machine installed in three departments A, 11 and C. The
departmental monthly capacity is limited to
Department Machine hours
A 1,800
B 2,100
C 1,300
The machines are purpose built and each type can perform specia-
lised task only.

APPLICATIONS TO COMMERCE AND ECONOMICS Ac13-151

The three products are proposed in all three departments but take
varying amounts of time in each as follows
Departments
Products A B C
Hours per unit
X 2 6 1
Y 2 1 3
z 3 2 2
The production controller has been instructed to obtain the fullest
possible utilisation of all machines.
Calculate the number of units of products X, y and z to he produced
in order to fill the capacity of all three departments for the month.
[Aii, x r 200, y= 100, z400J
33. The prices of the three commodities K, Y and Z are x, y and z
per unit respectively. A purchases 4 units of Z and sells 3 units of X and
5 units of Y. B purchases 3 units of Y and sells 2 units of K and I unit of
Z. C purchases 1 unit of K and sells 4 units of Y and 6 units of Z. En the
process A, B, C earn Rs. 6,000, Rs. 5,000 and Rs. 13,000 respcctivtly. Using
matrices, find the prices per unit of the three commo dities.(Note thaE
selling the units is positive earnings and buying the units is negative
earnings).
[Hint. The above data can he written in the form of simultaneous
equations as
3x+ 5y-4z= 6,000
2x-3y + r=x 5,000
—+0+6z= 13,000
and the equations can be written in the matrix form as
/ 3 5 4)X( X )==( 6,000
2 —3 1 y5,000
U 4 6 Z 13,000

AX=B :). X=4-B


/ X\ / 3 5 —4 )-1 / 6.000
y )=j2— 3 1 x( 5,000
z / \—1 4 6 \ 13,000
—22 —46 - 7 \/ 6,000 \ / 3,00o
=----H —13 14 —Il 5,00
151 1,000
- 7 —19 /\ 13,000 \ 2,000 1
Hence x=-3,000 ; y= 1,000 and z2,0001
SECTION B

Linear Programming
"LP is only one aspect of what has been called a systems
approach to management where all programmes are
designed and evaluated in terms of their ultimate effects
in the realisatioLt of business objectives."
N. Paul Loornba
INTRO DUCT ION N
The central theme of economic theory and management science is
o optimnise the use of scarce resources which include machine, man-
)()Wt, money, warehouse space or raw material. There are several
theorettcal to; to accomplish this purpose in both the sciences. But
such tools are not adequate for treating a complex economic problem
with evcraL alternatives each with its own restrictions and limitations.
It is for tackling such problemi that the use of linear programming has
been found to he most useful. The technique was first invented by
the Russian Mathematician L. V. Kantorovich and developed later by
George B. Daiitzii, the Simplex method is particularly associated with
his name.
MEANiNG
Linear programming is a method or technique of determining an
optimum programme of inter-dependent activities in view of available
resources. In other words, it is a technique of allocating limited resources
in an optimum manner SO 1S to satisfy the laws of supply and demand
for the firm's products. fn general, Linear Programming is a mathe-
matical technique for dctcrmniiiiig the optimal allocation of resources
and obtaining a particular objective (i.e., cost minimization or inversely
profit maximization when there are alternative uses of the resources
Land, Labour, Capital, Materials, Machines, etc.
pr, ' grriflfliflg is just another word for "planning" and refers to the
process of determining a particular plan of action from amongst several
alternati''S. The word linear stands for indicating that all relationships
involved in a particular problem are of degree one.
APPLICATIONS
The use of L1 is made in regard to the problems of allocation,
assignment, transportation etc. But the most important of these is that
of allocation of scarce resources on which we shill concentrate, Some
allocation problems are as follows
i. Devising of a production schedule that could satisfy future
demtnds (seasonal or otherwise) for the firm's product and at the same
time minimise production (including inventory) costs.
2. Chocc of investment from a variety of shares and debentures
so as to maximfliSe return On investment.
3. Allocation of a limited publicity budget on various heads in
order to maximise its effectiveness -
4. ScicctiO of the product-mix to make the best use of machines,
man hours with a view to maxmmise profits
LINEAR PROGRAMMING

5. Selecting the advertising mix mat will maximise the benefit


subject to the total advertising budget, Linear Programming can he
effectively applied.
6. Determine the distribution system to minimise transport costs
from several warehouses to various market places.
Three Typical Probleint. Three problems have become elas3ical
illustrations in linear programming.
A. The Diet Problem
It is the problem of deciding how much of 'ii' different foods to
include iii a diet, given the cost of each food, and the particular cornht
nation of nutrient each food contains. The object is to minimise the cost
of diet such that it contains a certain minimum amount of each nutrient.
B. Optimal Product Lines Problen
1-low much of 'n' different products a firm should produce and sell,
when each product requires a particular combination of labour, machine
time and warehouse space per. unit of output and where there are fixed
limits on the amounts of labour, machine time and warehouse space
available ?
C. Transportation Problem
It is a problem of determining a shipping schedule for a commodity,
say, steel or oil, from each of a number of plants (or oil-fields) at
dificrent locations to each of a number of markets (or refineries) at
ditlerent locations in such a way as to minimise the total shipping Cost
subject to the constraints that (I) the demand at each market (refinery)
will be satisfied, and (2) the supply at the plant (oil field) will not he
exceeded
General Linear Programming Problem
Let Z be a linear function defined by
i) Z=c1x1-4-c2x+ ... +cx
where C ' S are constants.
(ii) Let (a,) he inn constants and let (b) be a set of in constants
such that
(1 "X1 -1- a12 X 2 +... - -f- a 1 ,x, ( - )) b1
- {- a2 x + .. . -- a2.v, (, =, ;;a ) 112

a,,, 1 x 1 4 a,,, 2x2 + (. fl---. )) b,.


and finally let
(1(1) X, > 0 x ,>O, -.
The problem of detcrmming the values of x 1 . x.....- x which
makes L a minimum (or maximum) ) and which satisfies (ii) and (ni) is
called the General Linear Programming Problem.
LP-3
usiss MATHEMATICS
(al Objective fun ction.The linear funci ion
Z_—c1x1c2x,, 4-c1x,
which is to be minimized (or maximized) is called the
Of the general L, P. P. Objective function
(b) Const raints. The inequalities (ii) arc called the
of the General L.P.P. constraints
(c) Non-negative restricijons.
The set of inequalities (iii) is
usually known as the set of non-negative restrictions
of the General L PP.
(d) So l ution V alues of unknowns x 1 , x 2
constraints of a General L.P.P. is called a soiw ion ,..,x, which satisfy the
to the General L.P P.
(e) Feasible Solution.
Any solution to a General L.P.P. which
satisfies the fl on-ncçatjve restrictions
solu piü,, to the General L PP. of the problem, called feasible
(f) Opt I in ii Sl ni ion. Any feasible
(minimizes or 1111aximj7es the objective fiitiosd ii tion which optimizes
called an Optitnuin solution to the general L.P.P. i i of a General L.P.P. is
Example I. A
a certaiti un o r manufacturing firm has discontinued production of
olitable produce line, and this has' created considerable excess
production Capacity .
capaci Maiiagenie,it is considering to devote this excess
ty to produce one or more of three products 1, 2 and 3. The available
excess capacity on the machines which might 1i;"it output, is summarised in
the following table

iWochj,,e typ e AvotiriHe excess Capacity


(In machine hrurs per week)

Milling mac/line 250


Lathe 150
Grinder

The number of mnachinehours requires for each unit of the respective


product Ic give,, below

COP,7citY Requirement
(in nzachin . h,,j per unit)

Machine Type Product 1 Product 2 Product 3


-J
Milling mochir,' 8 2 3

Lathe 4j3 0

Grinder 2 1 0 I
LEAR PROGRAMMING
LP 4

The per unit contribution would be Rs. 20, Rs. 6 and Rs. 8 respec-
tively for products 1, 2 and 3. Formulate the problem mathematical!)'.
Solution. Step 1. Let the number of units of the products 1,2
and 3 manufactured be designated by x 1 , x and x 3 respectively.
Step 2. Since it is not possible to manufacture any negative
quantities, it is quite obvious that in the present situation feasible alter
naties are sets of values of x 1 , x 2 , x 3 satisfying x 1 )0, x 2 0 1
x30.
Step 3 The objective here is to maximize the profits which is
given by the linear function
(maximize) Z=r20X14-6X2+8X3

Step 4. Next ' express in words the influencing factors or


constraints (or restrictions) which occur generally because of the cons-
traints on availability (resources) or requirements (demands) Here in
order to produce X L units of product 1, x 2 units of product 2 and X units
of poduct 3, the total time needed on Milling machine, Lathe, and
Grinder are given by
8x 1 +2X,,+3x 3 , 4x 1 +3X 2 and 2X1+X3
Since the manufacturer does not have more than 250 hours available
on Milling machine, 150 hours available on the Lathe and 50 hours
available on the Grinder, we must have

8x 1 *2 x 2 +3 X 3 <250
4x 1 f3x 2 <150

2x 1 +x <50
Hence the manufacturing firm problem can be put in the following
mathematical form
Determine three real numbers x, x 2 and x 8 such that
8x 1 +2x 2 4-3x <250
<150
2x 1 +x 3 e.50
Xu, X 21 X 3 >0
and for which the expression (objective function)
Z r 2OX j 46 X2 4 8x5
may be maximum.

ample 2. Produc t ion Problem. A company iS


Manufacturing two products A
and B. The manufacturing times required to
make them, the profit and capacity ova liable at each work centre are given
by the following table:
LP-5
BUSINESS MATHEMATICS
Work
\Centre
Matching Fabrication Assembly f'roJii per unit
(in Rs I
L!'roduct \

10(1

Total
Capacity 720 hours ISOt) hours 900 hours

/-orrnuIce the L. 1'. 'node!

Solution. Step I. The key decision to be made is to determine


the nu mhcr Of Units of product .1 and B
to he produced by the coin pariy.
Step 11.
Let .v he the number of units of product A and x, the
number of units OfProduct B
which the company decides to Produce.
St . p III
'111C total profit that the riianijfacturer gets after selling
the two products A and 1? is given by

Z-O +uo .v
Se}) I V. No w,
the total Ur11[)
611 order to produce these two products .4 and B,
of hours required at rnatchiiig centre is given by
+ 2x,
The total number of hours required at fabrication centre is
5x 1 +4X,
and the
total number of hours required at assembly centre is given by
3x 1 + x
Since the matching centre is not available for 1110cC
.fa brication Centre than 720 hours,
centre is available only for 1800 hours and assembly centre is
a 'ail," bl e only for 900 hours we have

X, -4-2x2 sç 720

5Xi+4x21800
3x -fx79Oo
Step V. Also. Since it is
not possible for the manufacturer to
Produce negative tiumb of the products it
have is obvious that we imist also

x and xs0

LINEAR PROGRAMMING LP-6

Step VI. The above allocation problem of the manufacturer can


he mathematically expressed as follows

Find two real numbers, x 1 and x 7 such that


x k -4-2x2 720
5x 1 44x2 s 1800
3x+X2 < 900
x , x, ?- 0
and for which the expression (objective Function)
Z=80x-I 100x
may be maximum (greatest)

Exatple 3 A com;)Ofly produces three products P. Q and R from


three raw materials 1 8 and C. One unit of product 1' requires 2 units
of A and 3 units of P. One unit of product Q requires 2 units of 11 and 5
units of C and one writ of product R requires 3 units of A, 2 units of B
and 4 wills of C. The compane has 8 units of material JO writs of
material 1? on] 15 units of material C available to it l r ofit.c per writ of
products P. Q and R are Rs. 3, Rs. 5 and Rs. 4 respectively.
Formulate the problem ,nathe,na:ical!y.

Type of raw material Profit per unit


Decision Product
variables A B C (Rs.>

P 2 3 - 3
- 2 5 5
Q
R 3 2 4 4
x3

8 10 15
Units of material
available : maximum maximum inaximutn -----
------- -- ----- P
x 1 =number of units of Product
x2 =number of units of Product Q
x5 =number of units of Product R
-
rhe given problem is formulated as the LPP as follows
Maximize Z= 3x 1 + 5X 2 -j- 4x
Subject to the constraints
' +3x38
3 1 +2 2 +2x 3 10
5X2+4x3
X1 , X 2 x > 0.
wishes to ensure certain
Example 4. A diet conscious housewife
C for the family. The
minimum
minimum intake of vitamins A B and
LP-7 BUSINESS MATHEMATICS

daily (quantity) needs of the vitamins A, B, Cfor the family are respectively
30, 20 and 16 units. For the supply of these minituwn vUa,nfn require..
nlents, the housewife relies on two fresh foods, The firs: one provides 7, 5,
2 units of the three vitamins per gram respectively and the second one
provides 2, 4, 8 units of the same three vitamins per gram of the foodstuff
respec:iyely. The first foodstuff costs Rs. 3 per grain and the second Rs. 2
per gram. The problem Is how many grams of each foodstuff should the
housewife buy everyday to keep her food bill as low as possible ?
Formulate the underlying L.P. problem.
Solution. Step 1. By designating the number of units of foods
Xnd Y by x 1 and x 2 respectively, the data of the given problem can be
summarized as below
Decision Food Content of vitamins Cost per unit
variables type

A B C (Rs)
xI P 7 5 2 3
X 2 Q 2 4 8 2

Minimum vitamins 30 20 16
required
x- number of Units of food P
x=- number of units of food Q
Step 2. Here the objective is to minimize the cost and, therefore,
the objective function is
Z=3x1 + 2x,
As the minimum required amounts of vitamins A, B and C are 30,
20 and 16 respectively, the constraints of the problem are
7x1 +2x 1 )30; 5 x 2 +4Xs>20; 2x,+ 8x,,>,
Thus the given LP problem is:

Minimize:
Z =3x1 +2Xe
Subject to the constraints:
7x 1 +2x2 >30
5x 1 +4x 2 > 20
2x1 +8x,> 16
x1, x1 > 0

LINEAR PROGRAMMING

Example 5. A city hospital has the following minimal daily


recuiremews for nurses
Minimal Number of
Period Clock Time (24 hour day) Nurses Required
1 6A.1W. 10 A.M. 2
2 JO A.M. . 2 P.M. 7
3 2 P.M. - 6 P.M. 15
4 6 P.M. - 10 P.M. 8
S 10 P.M. - 2 A.M. 20
6 2A.M. - 6A.M. 6
Nurses report to the hospital w the beginning of each period and
work for S COflSCCU(JYC hours. The hospital wants to determine the
minimal number of nurses to be employed so that there will be sj/icient
number of nurses available for each period. Formulate this as a Linear
Programming Problem by setting up appropriate constraints and objective
function Do not solve.
Solution. Let x 1 , x 2 , x 3 , x 4 , x 5 and X be the number of nurses
commencing duty at 6 A.M., 10 A.M....., 10 P.M., 2 A.M. respectively.
(I) Requirement Constraints, Between 10 A.M. and 2 P.M.. the
nurses who start work at 6 A.M. (x 1 ) as well as those who start work t
10 A.M. (x 2 ) will be available. Since the requirement of nurses during
this intcrvat is 7,
x1+x2>7
Similarly X., + x) IS

x4x5>20
X5 +X6>,6
X e + X j >2
X 1, x 2 , x, X4 . X5 , x6)0

Objective Function: To minimise


z=x1+X2+X+Xg+xI+x6
EXERCISES
1. A small manufacturing firm produces two types of gadgets,
A and B, which are first processed in the foundry, then sent to the
machine shop for finishing. The number of man-hours of labour
required in each shop for the production of each unit of A and of B,
and the number of wan-hours the firm has available per week are as
follows
Foundry Machine Shop
Gadget 10 5
Gadget B 6 4
Firm's capacity per week 1000 600
LP-9
BUSINESS MATHEMAI1cS

The profit oil sale of A is Rs, 30 per unit as compared with


Rs. 20 per unit of B.
The problem is to determine the weekly production of gadgets
A and B, so that total profit is maximized.
[hint. Determine two unknown variables X 1 and x 2 , such that
l0x-f- 6 r
(1) 1000 (Foundry constraint)
(it) 5XL 4. 4x 600 (Machine shop constraint)
x x2 >0 (non-negativity constraint)
and 1,)r which the ex pression (objective function)
/ = 30 x, -f 20 x2
may be a m aximum (greatest).]
2. 1 he ABC' Electric Appliance Company produces two products
refrigerators and ranges. Production takes place in two separate
departments. Refrigerators are produced in Department I and ranges
are produced in department II. The company's two products
are
produced and sold on a weekly basis. The weekl y production cannot
exceed 25 refrigerators iii Department I and 35 ranges ill
II, because of the liniited available facilities
in these two departments.
The company regularl y em p loys a total of 60 workers
Ia the two
departments. A refrigerator requires 2 rnn-scks of labc'u:, while a
range requires one man week of labour. A refriee.rator contr i butes, a
Pro fit of Rs. 60 and a range. a profit of Rs 40.
The problem is to determine the weekly production of refrigerators
and ranges so that total
contribution is maxirnised.
Formulate thc above problem as a linear programming problem
Ans Maxinii Z=60 x +40 x, subjci to the Constraints
2 X 1 +a 2 60 x, 25;x 35 x,, x 2 >0, wheje Y , and x be the
riumbec- of units of refrigerators and ranges respectively,]
3. Three products are processed through three ditiercot operations.
The time (in minutes) required per unit 1 each product, the daily
capacity of the operations (in minutes per day) and the profit per unit
sold for each product (in rupees) are as follows

Tittle i^er unit (rnizluies)


Operation .
Operation Capacity
Produ ct .t Product II Product
(rniiriies doy)
M

4 3 43
2 I 0 4 46
3 3 6 2 I 42

Profit unit
(I(s) 2 2 3
LINEAR PROGRAMMING i.p-lt)

The zero times indicate the product does not require the given
operation. It is assumed that all units produced are sold. Moreover,
the given profits per unit are net values that result after all pertinent
expenses are deducted. The problem is to determine the optimum daily
production for three products that maxiinitcs the profit.
Formulate the above production plan n i rig problem in a linear
programming format.
[Hint. Find the real numbers x 1 , x.,, x 2 as to maximize
Z=2x 1 12x.2 1-3x,
subject to the constraints
3r 1 I
5x 1 ±4x<46
3x 3 + 6x2±2X342
with restrictions
X, x ,
.v301
4 I Vitamins A and B are found in food F arid /. One unit of
food contains 20 units of vitamin 4 arid 31) units of ",, -ita!Iun B. One
unit of food F. contains Of) units of vitamin ,l aid 411 units of vitamin
B I unit of each of foods F and I, cost Rs 3 and Rs. 4 rcsncctrVCly.
'Ihe inininiurn daily requirement (for it) of vuainmS A and I! is
SO 111) : ts arid 10 1 ) Units resct jvc h, A'.'arnilaL' that an%111171 11 in excesS of
daily ;iiininiurn requiremenk ci vitamins A and I? is not hartnt'ul. find
OUt the opti ii urn mixture of foods 1", and F-, at the ruin mom co,t Which
mccl ,, the daily minimum requirements of vitamins .4 and I?
Formulate the above problem as it programming probleni
;r fl int. Find two real numbers x and r, s ucli that
204 6();'8()
30x -i- 40y 100
X , )'.()

and for which the expression (objective tune tiun)


z =3-j-4y
may be a minimum (least)]
5. A feed mixing company purchases and mixes one or more of
the three types of grain, each containing different amounts of four
nutritional elements ; the data is given below

I One unit weigh; of _"M 1010!


Ifem over __ requirenlenf
plannihg horion
I
Grain I Grain 2 Grain 3

Nutrinonal ingredient .4

4 6 I r 125


Nutritional ingredient 1? 0 2 24

Nutritional ingredient C 80
>

Cost per unit weight (Rs.) 25 15 18 Minimize

LP- I BUS1NES MATHEMATICS

The production manager specifies that any feed mix for his live-
stock meet at least minimal nutritional requirements, and he seeks the
least costly among all such mixes. Suppose his planning horizon is a
two-week period, i. e., he purchases enough to fill his needs for two weeks.
Formulate the above problem as a linear programming problem.
fAns. Find three real numbers x 1 , x., x, so as to minimise
Z=25x 1 + 15x2 + 18;
subject to the constraints
2x1 -f- 4x., -f 6; 125
2x-t-5x324
5x1+x,+3x.>80
and X'- 'X 2, x30I
6. Lhe XYZ Company Ltd. manufactures two products A and B.
These products are processed on the same machine. It takes 20 minutes
to process ne unit of product A and 15 minutes for each unit of product
B and machine operates for a maximum of 80 hours in a week. Product
A rcquiics 3 kg and product B, 2 kg of the raw material per Unit, the
supply of which is 1200 kg per week. Market constraint on product B is
known to be 1500 units every week.
If tho product A costs Rs. 10 per unit and can be sold at a price
of Rs. 15, product B costs Rs. 15 per unit and can he sold in the market
at a unit price of Rs. 22 ; the problem is to find out the number of units
of A and B that should be produced per week in order to maximize the
profit potentially
Formulate this problem in the standard linear programming format.
Do not solve it.
7. A firm manufactures 3 products A, B and C. the profits are
Rs. 6, Rs. 4 and Rs. 8 respectively. The firm has 2 machines and below
is the required processing time (in minutes) for each machine on each
product
Machine Products
A B C
X 8 6 10
Y 4 4 8
Machine X and Y have 4,000 and 5,000 machine minutes respectively.
The firm must manufacture 200 A's, 400 B's and 100 C's but no more
than 300 A's.
Set up a L.P. problem to maximise profit. Do not solve it.
[Hint. Find the real numbers x1 , x2 and X 3 so as to maximize
Z = 6x1 +4x,+ 8;
LINEAR PROGRAMMING

subject to the constraints


8x4 (x 2 -3- I0x34,000
4x 1 + 4x2 + 8; 5,000
with restrictions
200x1' 300
r24Q0
xa 100.]
8. The manager of a company, which supplies office furniture, has
asked you to prepare a profit maximizing schedule for their production of
desks. This particular company sells a basic line of four desks. (Type A,
Type B, Type C and Type D) to local distributors at the prices given below.
Costs of producing each type are also given
Desk Selling Price Production cose
Type (In Rupees) (In Rupees)
A 28 21
13 35 30
C 52 39
1) 72 54
For short-run scheduling, labour must be considered a fixed quantity
and desks production is a two-step process, requiring labour for carpentry
and finishing operations. Labour is not transferable between operations.
6,000 hours and 4,000 hours can be used in carpentry and finishing
respectively. The labour hours required for each desk are given below
Desk flours of flours of
Type Carpentry Finishing
A 4

B 9 1
C 7 3
D 10 40
Formulate this as a Linear Programming problem.
9. A media specialist has to decide on the allocation of advertise-
meat in three media vehicles. Let x 1 be the number of messages carried
in the z-th media, i== 1, 2, 3.
The unit costs of a message in the 3 media
are Rs. 1000, Rs. 750 and Rs. 500. The total budget available is Rs. 20.000
for the campaign period of a year. The first medium is a monthly
magazine and it is desired to advertise not more than one insertion in one
issue. At least six messages should appear in the second medium. The
n umber of messages in the third medium should strictly lie between 4 and

BUSINESS MATHLMATICS

8. The expected effective audience for unit message in the media vehicles
is shown below
J/ehkle !ir peeled effective audience
80,000
2 60,000
3 45,000
Build the linear programming model to maximise the total effective
audience.
[Ans. maximize Z= 80.tjOOx 1 + 60,000x0 -f 45,000x3
subjects to
10004 750y 2 ±500x20,000 (budget)

X,, x2. x301.


10. The manager of AC Oil Co. wishes to find the optimal mix of
two possible blending processes. For process I. an input of 1 unit of
crude oil 4 and three units of crude oil B produces an output of 5 units of
gasoline X and two units of gasoline Y. For process 2, an input of 4 units
Of crude oil A and 2 units of crude oil B produces an output of 3 units of
gasoline X and 8 units of gasoline Y. Let x 1 and x he the number of
UflitS the company decides to use of process I and process 2. respectively.
The maximum amount of crude oil A available is 100 units and that of
crude oil B is 150 units. Sales commitments require that at least 200
units of gasoline X and 75 units of gasoline Y are produced. The unit
profits of process I and process 2 are p and P2 respectively. Formulate
the blending problem as a linear programming model.
An. Maximise Z=p 1 x 1 4p x2
subject to
x1-+4x2100)
> Availability
3x 1 +2x2 150 I
5x14 3x2 )20Q '1
Demand
2x1+8x275 j .
X 1 , x0J
GRAPHIC METHOD

Summary Procedure for the Graphic Method


Step I. Formulate the appropriate LPP.
Step 2. Construct the graph for the problem as follows
'Treat each inequality as though it were an equality and for each
equation arbitrarily select two sets of points. Plot each set of points and
Connect them with appropriate line'.
LINEAR PROGRAMMING

Step 3. Identify the feasible i.e, that Space which satisfies


all the constraints simultaneously. For less than or equal to' and 'less
than' constraints this is generally the region below these lines. For
.greater than or equal to' or greater than' constraints, this is generally the
region which lies above the lines.

Step By choosing a Convenient profit (cost) figure draw an


isoprotit (isocost) line so that it hails within the shaded area.
Step S. Move this isoprofit (isocoSt) line parallel to itself and
izirthcr (closer) from (to) the origin until an optimum Solution iS
determined.
Eampte 6. .1 factory manufactures u'wo articles A and /1 f'o
raanujac(ure the 0, tic/c A, a certain machine has to be worked for 15 hours
and in addition a craJtsniaa has to work for 2 hours, TO tnantfocture the
(1rtt'de B, the niactiuit' has to be worked for 25 hours and in addition //U?
craftsman has to itork Jar 15 hours. In 0 Ueek tire Jactory can rivail o
80 hours of me chine time and 70 hours of craftsman's time. The profit on
each article 4 iv Rs, S and that on each article 9 is 1/s 4. if all the er(rcis
prr'c/nced carp be SO/d away, laid how 1)1011)-' of each k ira! c/iou!,] be rrtIuct'd
to earn the tfl(7XiPfltilfl projit jit'i' week.
Iornuulate the hi,iew' programming problem.
Solution. Step 1.
I)ATA SUMMARY CHAR I'

I), ciS ion Article 1/ours on I'roft per 107t


IJt1t(JI 1 (5 ,'ti'a/rpue (.rnJfsman
15 2 Rs. 5,00
.v2 /3 2 Ii Rs. 400
Hours tivailable 80 70
,per 'Ytt" I ItliiXiflhlJfll rn,uxuinum

- - number of units of article A


uunihc'r of units of a rt j cle B

1 hus the given problem is formulated as a L.P.P. as follows


Maximize Z. 5-x, -14
... V)
subject to the constraints
15 x 1 +2'5 x,,8Q
2x1j-1'5x270 ...()
x1.x2O
Step 11. Construct the graph. Next we construct the graph by
drawing horizontal and vertical axes which are represented by the
X,-axis and x-axis in lie Cartes:an X,OX Plane- S iec lili\' print
which satisfies the conditions x, o and x 0 lies in the first quadcaru
cnly our search for the desired pair (x,. X) is restricted to the poinic oh the
first quadrant only,


LP- 1 5 BUSINESS MATHEMATICS

Now the inequalities are graphed taking them as equalities, e.g..


the first constraint l5x1 ±25X2 80 will be graphed as l5x1+25x=80,
and the second constraint 2x 1 ±15x2 70 as 2x1 -I-15x2 =70 and the
third constraint x, x 2 > 0, merely restricts the solution to non-negative
values.

Further, since the functions to be graphed are linear we need plot


only two points per constraint.
Thus to graph each constraint,
we arbitrarily assign a value
F (0.32) to x and determine the correspon-
ding value of x 2 . The procedure
FeosbIe oreo is then repeated for another pair
for nwch.ne of values for the same constraint.

Thus for the first constraint we


have two such points as P(0, 32)
and Q(533 0), which upon
joining represents
0 0
it6Ct0 l'5x1-j--2'5x2=.80.
Fig. 1.


If xi 0 55,3

MM

Similarly, by considering the set of points satisfying x 1 >0, x2>O


and the second constraint 2x 1 + 15x2 70, we obtain the shaded area of
Fig. 2 as shown below

If xi 35 0 I 2.157O 1
40 LIJ

3O f-ras,bie area
for craftsman

Step lU. Identify the feasible


region. The feasible region, i.e.,
solution space, is the area of the
graph which contains all pairs of
values that satisfy all the constraints.
In other words, feasible region Article A

Fig. 2
LINEAR PROGRAMMING LP-16
will be bounded by the two axes, and the two lines 1 , 5x 1 + 25x 80,
2x ,+ 1 5x. - 70, and will be the common area which falls to the kit Of
these constraint equations as both
the constraints are of the 'less than
equal to' type.
Step IV. Locate the solution
Points, The shaded area OPTS
represents the Set of all feasible
solutions The four corners of the
polygon are 0 (O, 0), P=- (0, 32)
Tr (20, 20) and t
.S'-.-.(35, 0).

.
live funcon Dantz's
guarantees that the optimal solution
to an L. P.P. occurs at one or
theobjec.
0P14
Aritc'e A
a ,

more of the Corner points, we Fig. 3


evalwiic the objective function at each of these four points as folio

Corner point Objective function Value


(x i , x2 ) Z==5x1 - 4x2

0 -- (0, 0) 5x0f4x0 Z(0)= 0

P(o, 32) 5 xO-l-4x 32 Z(P)- 123

T= (20, 20) 5x20+4x2O Z(T)= 180
S=(35, 0) 5 x 35+ 4x 0 Z(S)== 175

Now the optimal Solution is that corner point for which the objec-
tive function has the largest value. Thus the optimal solution to the
present problem occurs at the point T= (20, 20), i e , x, 20. x -=20
with the objective function value of Rs. 180.

Hence to maximize profit the company should manufacture


20 units of article A arid 20 units of article B per week.

ExmpIe 7. 4 company produces two articles X and Y. There


are two departments through which the articles are processed viz., assembly
and fini5hing. The potential capacity of the assemhly departrneni is 60
hours a week and that of the finishing departmen t is 48 hoursrt a week.
Production of one unit of A' requires 4 hours in ussemb!y and 2 hours
in finishing. Each of the unit Y requires 2 hours in assembly and 4 hours
in finishing. If pro 'it is Rs 8 for each unit of A' and Rs 6 for each
unit of Y fin d 0111 the number of units of X and Y to be produced each
week to give maximum profit.


LP-17 BUSINESS MATHEMATICS

Solution.

Time required for producfng


one Unit
Total hours
Products
available
x

Assembly Department 4 2 60

Finishing Department 2 4 48

Ptoflt per unit R. S Rs. 6

Z=8X-i 6Y
Objective function
Subject to constraints 4?(+2Y60
2K+4Y48
Non-negativity requirement XI, Yo.
Plot the constraints in a graph given below. X is shown on the horizontal
axis and Y is shown on the vertical axis. Consider the constraint
4X-+-2Y60. When production of X is 0, then Y-- 30. Plot the point
(0, 30) in the graph.
Again when production of }' is 0, then X= 15. Plot the point
(15, 0) in the graph. Joining these two points, the resulting straight
line BC is such that area ABC of the graph represents the ineuality
4X+2Y60 as long as X and Y are both greater than 0.
Similarly plotting the constraint 2X— 4Y48, i.e., joining E(0, 12)
and F(24. 0). The area AEF contains all possible combinations which
will satisfy the restriction of the finishing department.
LINEAR PROGRAMMING I.t'I

Therefore the best combination of .( and Y which must not exceed


the available time in either assembly or finishing should fall with in
the areas .1 JJC and ,fEI'. 'I he area which does not excec(l either of the
two constraints of the assembly and fluishing departments is the thadcd
area AEDC.
Now observing front graph, the point which yields the greatest
profit is the point L) (12, 6).
Point Total p r ofi t (applying oLjectIve function)
Rs. 8X-4-Rs. 61
4(0, 0) 0
C(15, ) Rs. 8(15) IRs. 6(0) 7 Rs. 120
1)(12, 6) Rs. 8(12) I Rs. 6(6)- Rs. 132
E(0, 12) Rs. 8(0)-I Rs, 6(12) -= Rs. 72
1 his may also he obtained algebraically by solving
4X-I-21'60 and 2X-f4Y=48 or 8X+4Y=l20, 2X+ 4Y48
BY subtraction 6X-=72 X== 12 and Y6
Applying it to the objective function Z 8X-1-6Y, the maxiniuni
P rofit equals to Rs. 8(12) 3- Rs. 5(6) Rs. 132. Thus 12 units of X and 6
units of Y give it maximum profit of Rs. 132.
Rernai-1. If there is a third constraint as shortage of labour
which restricts the production of Y to a maximum of 4 unit.s per eck.
then Y is less than or equal to 4 units per week and X and Y
are non-
negative.
Now plot the Constraint in the graph given below and draw it
straight line parallel to the hori,onitah axis. The feasible alternative
wIl he somewhere in tIo shaded area A IIGC. The point which yields
the greatest profit is loun(l out by testing tine four corners of the shaded

Asse-bly

. , r.rç .-ln

0 L 1Z'\isi 2
1. 6 n? 16 2) 24 28 32 4
Pr.t

area. This is the point (1 13, 4). Therefore the optimum production
per week is 13 units of X and 4 units of Y and the maximum profit
max 1= Rs. 8(13)-I Rs. 6(4)= Rs. 128.
LP- 19 J3USINsS MATIMAT

Example 8. Solve the following linear programming problem


graphically:
Maximise Z=4x6y subject to constraints x4-y =5, x > 2 , y<4,
X. y>O [Delhi Univ. J3.Coin. (lions) ; 1992)
Yii Solution. Clearly each point (.v, y)
satisfying the conditions x0, y > 0 must
lie in the first quadrant only. Also since
x- 1'y=5, x)2 and y4, the desired point
1.4)
lies somewhere on the line CB. The co-
(i ordinates of C= (2, 3) and II (5, 0). The
values of the objective function Z at these
Points are
Z(C)=4X2+6X3.26
ZB)4x5+6X0::20
Since the niaxiniutn value of Z occurs
at the point C(2, 3). Thus to maximise
Fig. 4 /, x=2 and y=3.

EXERCISES
I. (a) Describe the graphic method of solving a ]nicar program-
rniiig problem.
(h) S ' l y e the following prohem by graphic method and for that
show
(I) Objective function (Ii) Set of feasible solutions
(iii) Optimum solution (li) Extreme points
Maximize Zr= 3x 1 j . 4
subject to the constraints
4x 1 -.f-2x, 80
2x1 - 5x1 < 180
x1 , x2 > 0
2. It is required to maximise Z2x1 +5x2 subject to x 1± x 2 24,
3x 1+ x 21, x1 +x2 < 9, x ) 0, y > 0. Show graphically how to arrive
at the solution and find the maximum value of Z.
3. A dealer wishes to purchase a numbet o f fans and sewing
machines. He has only Rs. 5760 to invcst and has space for at most
20 items. A fan Costs him Rs. 360 and a sewing machine Rs. 240. His
expectation is that he can sell a fan at a profit of Rs. 22 and sewing
machine at a profit of Rs. 18. Assuming that he can sell all the items
that he can buy, how should he invest his money in order to maximize
his profit ?
[Hint. Maximize Z=-r22xi+18x
5.!. Xi+ 20
LINEAR PROGRAMMING

r.-20
360X 1 -j-240x 576-()

X11 x, > 01
fAns. x 1 =8, '2 12 ; wax. Z:Rs. 3921
4. A manufacturer produces tubes and bulbs. It takes 1 hour of
work on machine M and 3 hours of work on machine N to producc one
package of bulbs while it takes 3 hours of work on machine M and 1 hour
of work on machine N to produce a package of tubes. He earns a profit
of Rs. 1250 per package of bulbs and Rs. 5 per package of tubes.
How many packages of each should he produced each day so as to
maximize his profit if he operates. the machines for at most 12 hours
a day.
[flint. Maxim i ze Z -= 1250 x 1 -+ 5x,
$(.x -3.v, 12
3x 1 -1x 2 ' 12
X 1 , x2 >
[Ans... x., 3 ; max. Z--- Rs. 52501
5. A dealer deals in only two items, c y cles and scooters. He has
Rs. 50,000 to invest and a space to store at most 60 pieces. One scooter
costs him Rs. 2500 and a cycle costs him Rs. 500. 1-Ic can sell a scooter
at a profit of Rs. Sot) and a cycle at a profit of Rs. 150. Assuming
that he call all the items that lie buys, how should he invest his
money in order that he may maximize his profit ?
[Hint. Maximize Z.r50Oi 1 -j- 150X2
S . I. 60
2500x 1 +500x 2 50,000
X1, x, > 01
[Ans. x 1 =10, x 2 =50, Max. Z=12,5001
6. A firm makes two types of furniture chairs and tables. The
contribution for each product as calculated by the accounting depart-
ment is Rs. 20 per chair and Rs. 30 per table. l3oth products are
processed on three machines M 1 , M2 and M. The time required in hours
by each product and total time available in hours per week on each
machine are as follows
Machine Chair Table Available Time
M 1 3 3 36
M1 5 2 51)
2 6 60
flow should the manufacturer schedule his production in order to
maximize contribution ? (Use graphic method only.)
[Ans. X 1 =-3, x., -- 9, Max. Z=330]
7. Food X contains 6 Units of vitamin A per gram and 7 if of
vitamin B per gram and costs 12 paise per grain. Food Y contains 8
r-21 BUSINESS MATHEMATICS
units of vitamin A per grain and 12 units of vitamin 13 and costs 20 paise
per gram. The daily minimum requirements of vitamin A and vitamin B
are 100 units and 120 units respectively. Find the minimum cost of
product mix using graphic method.
(HLn(. Minimize Z l2x 1 20x2
subject to the constraints
6x1-l-(x1>too
7x1 -- 12x2 120
_v l . x10

An, . x 1 15, x= ; minimum Z- 205]

8. A farm is engaged hi breeding pigs. The pigs are fed on


various products grown on the farm. Iii view of the need to ensure cer-
tain nutrient constituents, it is necessar y to buy two products (call them
A and B) in addition. The contents of the various products, per UfliL
in nutrient constituents (e . g ., vitamins, proteins etc.) is given in the
following table
Nutrients Nutrient Cc)flten( Minimum amount
ill product of nutrient

A 1.1
M 36 6 10
12
10 1(10
The last ciii inn of (lie above tahk: gives the iiUiirflUm amounts of
flUtr;cut Coflstitue.i(s If LW
xvilich
tf must he gi . en 10 the pigs. 11
the products A nd 11 cost R. 2( and Rs. 40 per unit rcsicctivelv, how
flIUCIL each I,I (II(SC
IS lI)! illflij y ecj !
two products should he hoocht so that the total cost

[Hhi. Find real riumhrrs x. and .v, So as to ii iimnc the objective


function
/. 20x 1 -f 41
subject to the constraints
36x f (t l0
3x 1 .4 ]2x2>6
20x-4 l0x1OQ
x+x2u
Thc farm should purchase 4 i l product A w'd 2 units of product
B in order to maintain a minimum Cost of Rs. 100.1
9. A scrap metal dealer has icceived an order from a customer for
at least 2,090 kilograms of scrap metal. The customer requires that at least
1,000 kilograms of the shipment of metal must be high quality copper that
can be melted down and used to produce copper ttibmgs Furthcrmoic,
LINIAR PROGftAMMU'OTj

the customer will not accept delivery of the order IF it containu more than
175 kilograms of metal that he deems unfit for commercial use, I &, metal
that contains an excessive amount of impuritica and cannot be melted
down and refined profItably.
The dealer can purchase scrap metal from two different suppliers lit
unlimited quantities with the following percentages (by weight) of high
quality copper and unfit scrap.
Supplier A Supplier B
Copi:cr 25% 75%
Unfit scrap 5% 10%
t A and
The Costs per kilogram of metal pure! e'd fron
supplier B are Re. I and Rs. 4 respectively. 'rhe problem is to determine
the optimum quantities of metal for the dealer to purchase from each of
the two suppliers.
[Ithit. Our problem is to find the real numbers x 1 and x 2 so as to
nun mule
Z=.x1+4x
subject to the constraints x 1 2,000

1000

x i x
175
20 to
X" x2 >0
The dealer should purchase 2,500 kilograms of scrap metal from
supplier .4 and 500 kilograms of scrap metal from supplier B ',it to
maintain a minimum cost of Rs. 4,500.]
10 A cold drinks company has two bottling plants, located at two
different places. Each plant produces three different drinks A, B and C-
The capacities of the two plants, in number of bottles per day are as
follows
Product A Product B Product C
Plant 1 3000 1000 2000
Plant If 1000 1000 6000
A market survey indicates that during any particular month there
will be a demand of 24,000 bottles of A, 16,000 bottles of B, and 48,000
bottles of C. The operating costs, per day, of running plants I and it
are respectively 600 monetary units and 400 monetary units. How many
days should the company run each plant during the month so that the
BUSINUS MATHEMATICS

production cost is minimised while still meeting the market demand ? (Use
graphic method).
[Hint. Minimise cost 600X1-4-400x2
st. 3000x1 4-I 000x 24,000
I 000; + 1 000v 2 16,000
2000x 1 -1-6000x2 ) 48,000
0 1 x 2 ) 0.1
II. The manager of an oil relincry wants to decide on the optimal
W ix o Iwu possible blending processes I and 2 of which the inputs and
Outputs per production run are as follows
Input (Units) Output (Units)
Process Crude A Crude 13 Gasoline X Gasoline Y
5 3 5 8
2 4 5 4 4
• Fhc ulaxilutjjn amounts available of Crudes A and I? are 200 units and
ISO unts respectively. At least 100 units of Gasoline X and SO units of
Y arc requLLed. The profit per production run from processes 1 and 2
arc Rs. 300 and Rs. 40() respectively. Formulate the above as Linear
programming problem and solve it by graphical method
[Ans. Maximize 7= 300x 1 + 400x2
s. t. 5x1 +4x2200
3x 1 j 5x,,5;150
5x1 -f 4x 2 100
8x1 +4x, ,>-80
x?0, .v0.1
SIMPLEX METHOD
In most of the linear programming problems, we have more than
two variables and, therefore, it cannot be conveniently solved by a graphic
method. A procedure known as 'Simplex Met/r od' can he used to find the
Optimal solution. The method is in fact an algorithm or a set of instructions
which seeks to examine corner point in a methodical manner until the
best solution ensuring highest profit or the lowest cost under given
C onstraints is obtained. Fortunately, computer programme is available
for dealing with problems involving several variables but to understand its
m echanics we shall confine to a few variables only.
Slack and Surplus Variables The formulation of a linear
prog ramming problem for simplex method requires introduction of slack
or surplus variable to convert a linear inequality into linear equality.

LINEAR PROGRAMMING it-P-24
(i) Let the constraint of LP problem be 2X1-f-3X210
Then the rR)I-negative variable S 1 which satisfies
2x 1 3x9 -lS1 - 10
is called a slack variable.
(ii) lithe constraint of a LP problem is 4xj-5x>25
Then the non-negative variable S. which satisfies
4.x 1 -1- 5X.2 —S==25
is called a surplus variable.
The variible .S is called slack variable, because
Slack --= Requirement—Production
The variabie SE is called surplus variable, because
Surp1us=Production -Requirement
rhese slack or surplus variables introduced in an appropriate manner
to linear cmctraints expressed generally as inequalities get represented in
the objective function so that the number of variables in objective function
has correspondence with those in the constraints but they do not contribute
anything to the objective function and their cocflicicnts in the objective
function are only zero.
Illustration.
Problem Maximise protit==7x1+5x2
Subjct to 2X1+lx10
4x1-l-3x224
X 1 0, x20
The inequali t ies expressing constraints are converted into, equalities by
adding slack variable to each inequality as follows
2x1 + lx2+S1=l0
4x1 +3x2+S2=24

Now, the objective function is being transformed to accommodate


slack variables with zero coefficients as follows
Maximise profit ==7x 1 + 5x, +0S, +02
But, since all equations must have equal number of variables that is
made possible by Incorporating the slack variables of other equations
with a zero coefficient as follows
2v 1 + Ix1+IS2+0S2=10
4x 1 +3X 2 +0S1 + 1S1=24

BUSINBSS MATHEMATICS

A model of simplex tableau to present these is given hereunder


Simplex Tableau

Co efficients af Programme Available


Progr,nrne t t t 0 0 0 Objective variable
variables quantities coefjicients
variables of
In objective -
variables x1 x 1 . S S S3
ob/rcttye ()bjc;it
funciiorrvariable row

o
o S,-- -1 •
* *
* . 00
Si (i 0
o S S 0

t
Structural Identity
coefficients, matrix
matrix
S ummary Procedure for the Simplex Method
( M ax j mjz; 01 Case)

The various steps involved in the simplex method may be inmed


up Z's follows
1. Formulate the problem and the objective function.
2 Develop equations from the inequalities by add iitg slack
variables.
3. J)evelop the initial simplex tableau including the ittit al (trivial
SOhtIjOi

4. Obtain the 2-11 and G1 —Z 1 (index row) for this solution.


5. Choose the highest positive number in the index row.
6. The highest positive number determines the key column.
7. Divide the numbers in quantity column by corresponding
numbers in key column,
8. Select the least positive ratio of these quotients.
9. The row containing the least positive ratio is the key row.
10. The key number is at the ir,tersction of the key column and
key row.
II. Divide every figure in the key row by the key number.

@ Data on total available capacities.


Data on cotPije t ts of variables in the objective function.
Data of Coefficients of structural constraints.
LINEAR PROGRAMMING 1.r-26

12. The quotient of the key row divided b y the key number is the
main row in the next table. Tile formula is
Key row number
Main row
Key number
13. All other numbers for the next table are dcrici the
arm ul a
ICorresponding Corresponding
Derived Selected I number in ke y < numhcr in key
number number -. row column
Key number j
14. Repeat steps 5 to 13 until no positive numbers exist in the
index row. When no positive numbers exist in the index rew, an
cpimum solution has been obtained.
Remarks . Sfmplijiccrt on of Calcu?aiion,c
It is possible to sim plify the calculation process by fohl ing a
few rules
1. Any variable lit tie variable column will have a 1 where thr
row of that variable intersects with the column of that variable, and nh
other figures in the column of that variable Nil] he zero.
2. If there is a zero in the key column, then the row in v.hieh that
?Cfu appears will remain unchanged in the subsequent matrix.
3. II there is a zero in the key row, then the column in which tInt
zero appears will remain unchanged in the subsequent matrix.
B y observing the above three rules, the number of items for which
derived numbers are to he cnculatcd will be greatly reduced. \Vlre a
simplex solution has to be worked by hand methods, the saving III tulle
and effort is significant. When computers are used, it is desir able 10
allow the normal procedure to he followed.
2. Rules for Ties
In choosing the key column and key row, whenever there is a tie
¶etween two numbers the follow ing rules may he adopted
1. Select the column farthest to the left, whenever there is a tie
between two numbers in the index row.
2. Select the ratio (8) nearest to the top whenever there is a tic
between two ratios in a matrix.
Illustration. A factory can manufacture 2 products X 1 and X2.
Each product is manufactured by a (wastage process which involves
machines / and Ii and the time required is as follows
Machine Product
X 1 x2
I 2hr. 1 1r,
Ii 3 hr. 2 hr.
DUSNaSs MATHnMATICS
Available hours on mach Inc I is 10 hours and mach Inc It is 16 hours.
The contribution for product X is ]?s. / per unit widJir X. is R g . 3 per unit
What should he the M a nufacturing policy for the factory ?

SoltI 0 . Step I. Formulation of the LP problem.

Maximise (Profit) Z=4X1 -t- 3X2


Subject to 2X1 -l-X.<10 (constraint on Machine 1)
3X1 - 16 (constraint on Machine II)
X >0. X>(J

Step 11. Develop Equations from the Inequalities . The first


step in the Simplex Method is to convert the inequalities (or restrictions)
into equalities. This is (]one by adding what are known as slack variables
(slack variables in economic terminology represent unused capacity but
the contribution associated with them is zero). After adding the slack
v ariable. all the above express iOOS
can he written as
2X1 + Y' I - S' =10
3\J 2X
.(2)
4k--3V -0.S=Maxouie / (3)
I Iei e the slack variables S 1 and
mach l ;ic5 I and 11 rcspcctivclv, 2 represent the idle hours on

Step II!. Designing the Initial Programme. Set the basic


variables equal to zero in winch case the slack variables assume the full
value of the resources available and the contribution at this stage is
flnniinu in.

A first feasible solution is thus

X, =- X2 = , S t = 10, S2 = 16

The profit contribution resulting from this programme can be


determined by substituting the values of the different variables in the
objective function. Thus

Profit contribution = 4(0) + 3(0) + 0(10) I 0(16) 0.


Step IV. Develop Initial Simplex Tableau. We can now set
out this whole problem in what is known as a Simplex Tableau.
The
simplex tableau also known as simplex matrix is a table consisting of
rows and columns of figures We illustrate below the form of simplex
tableau and explain its various parts
LINEAR PROG\SIM1NG

TABLE 1. PARTS OF INITIAL SIMPLEX TABLEAU


(.' Coluititi i.e.. pr(ii rer will)
I I'i,(luct'inl nc colt.iinii)
I CootanI column (i.e., quaniitie3 of product in usc MIX)
I I Variable columns

C, I'roduct R. 4 Rs 3 R 0 Rs. 0 * C'J row


- Quani ii y -5 ariahe mV
175 X1 X .51 S
0 Rows ilusir-
- 10
ing COiil 1110
RS, I) S- 3 2 0 I equations
(toc I lie it' it s


- --- _Y__ - - -- --

Body alau rix ld'n1ity


Coni. iSling of Man ix
cu . c0iclents consisting of
of real pin- co-ellicients
duct variables of lack,
Variables.
((i) C1 row or n/n/cc/ire row. Oil top row o the tableau known
as (. 'j row or objcctie row, we insert the coefficients iii the ob1eetivc
equation
c/i) [lie identit y matrix is formed by the slack variables and
consists of a diagonal or l's and U's. It may be noted that the identity
should tie, ci has e negative numbers.
((') Ihe ii.i,/y matrix consists of all restrictions and equationS and
includes the coefficient of all variables not in the identity. The numbers
in the body can be zero, positive or negative.
(d) The quantity co/unto represents the list of constants of the
equations. [very number in the quantity column (excluding index
row) must he zero or positive. This condition is true from the time of
setting the matrix iiiit ii its solution stage.
(1') The product mix CO/i//flfl in the initial programme is a list of the
variables in the identity. (It may be noted that the row headed by S1
and the column headed by S 1 cross in the identity where the 1 occurs.
The same is true for Sn also). it may be noted that the product-mix
column shows the variables in the solution. The variables in the first
solution are S and S. (the slack variables representing unucd capacity).
In the quantity column, we find the quantities of the variables that ale
in the solution
10 hours available on Machine I
S-= 16 hours available on Machine 11.
As the variables K1 and .V 2 do not appear in the product-nds, they
are equal to zero.
f)
C, or object ice column. The C, or objective .oluinn at the left
end shows the profit per unit for the variables S and S. For example,
the zero appearing to the left of the S 1 row means that profit per unit
i. p - 29 BUSINESS MATIlLMAi1C

of S is zero. Likewise, the zero to the left of S2 row means that profit
per unit of S, is zero. The initial simplex tableau will now appear as
follows

Pro?uct' Rs. 4 Rs. 3 Rs. 0 Rs. 0


C, Quantity
mix T__ 2 i 2

Rs. 0 S i 10 2 1 1 0

Rs.0 S2 16 3 2 0 1

( e) 7, row. The 7 1 is the C, for a row times the coefficient for that
row Within the tableau, sum ned by column. In other words, to arrive
at the 7, value for a particular column, we first multiply each coefficient
in that column by the C', against that coefficient and then add up the
products So obtained. The four values of 7, under the columns of
variables X1 , .1'.,, 1 and S are likewise computed as follows
7, for column X Rs. 0(2)--' Rs. 0(3) Rs, 0
7, for column .Y, Rs. 0(I) + Rs. 0(2) Rs. 0
7, for column S1 Rc. 0(1)1 Rs. 0(0): Rs. 0
/, for column .S 1 _ Rs. 0(0) I Rs. 0(I) Rs. I)

The above values of 7, represent the amounts by which profit


would he redured if I unit of an y of the variables ( X,, S1 , S) were
added to the rni.
(h) C,—Z, (Injex) or ;\e! Jialuajiop row, C,-7 1 represents the net
profit that v. ill Occur from introducing one unit of a variable to the
production schedule or solution For example, if I Unit of V 1 adds
Rs. 4 of profit to the solution and if its introduction causes no loss, then
CIT /I for .\' = Rs. 4. The net profit per unit (i.e., C,--Z1) of each
variable is calculated as shown hclow

I'rojit per PrnJit lost Net profit


V(lrtables unit per unit per unit
(C) (Z,) (C, -7,)

.vi 4 0 4

3 0 3
S3 0 0 0
S2 1) 0 0
1.1
LINEAR 111106RA5I1I(i

lABLE Z INITIAL SIMF1EX FAI31.EAU


COM1LElE1) -

4 3 I)

t'rod',tt (liutly -
CI fflI
X .t, 'c -

0 Ii) 2 I I)

- :-•• -- --- -: -

C1 Z1 (Ind row) 4 3 1) (1

Remark. Ely cxaniiniisi the numbers in the ((-', -- <,) r ow of hihie


2, e hrid t 114 C total profit can be increased 1)y Rs. 4 for each unit 1
added to the mix or by Rs. 3 for each unit of \ added to the inix.
Thus a po,itive niinsher iii Iii: (C /,) row indicates that profits call be
improved by that amount per unit of Y j added. On the other hand, a
negative isu III hec in the C, - /,) row would indicat tue anioUrit IY
%NhIch profits would decrease if one u sit of the variable head I n,g that
column were added to the solution. I l':itec the Optimuiru SolLitiorl is
row.
reached \Vheii 110 p()SitiC riuiIl)CrS are there in C1 -/,
Step V. Developing Improved Solutions. After the initial
simplex tableau is set up, thut: next Step is to determine if the improve-
ment is possible. l'hie Computational procedure is as follows
(a) Chrosinc; i/ic cnrcrine variable. We choose the va IRINC CO b.3
added to the first selutiouu v fuels contributes the highest probt Cf Unit.
This is done by ideiitik ing the column (and hence the variable) which
offers the largest positive number in the (C, - t,) row. As will he seen
from 'Fable 3, bringino in X 1 v. ill add Rs. 4 per unit to profit. The
column is the O/)litIIiD)1 colrinuii, also commonly known as Pivot Column or
Ke y Co/twin, Ely definition, the optimum colu mu is that column which
has the largest positive value in the C, - row, or iii other words, the
column v. bose product will contribute the highest Profit per Unit. Ins-
pection of key or pivot column indicates thhtt the variable .Y1 should be
added to tile product mix replacing one of the variables present in the
mix. The variable X is, thus, the catering variable.
(I) Choosing 1/ic departing variable. Since we have chosen a
variable to enter the solution mix we have to decide which variable is to
be replaced. This is done in the following manner.
First, divide each number in the quantity column (also kno vu as
constant column), i.e., it.) and 16 by the corresponding numbers in the
key column.
Second, select the row with the smallest non-negative ratio as the
row to he replaced.

flftj555 MATHEMATICS

Here the ratios would be


row : 10 hours/2 hrs, per unit-5 units of X1
row 16 hours/3 hrs. per unit-- 5j units of X2

As the S row has the smallest positive ratio, it is called the replaced
'ow, or the pivot row or key row, This row will be replaced in the next
solution by 5 Units of X, i. e ., the variable S. (unused time) will be
replaced by 5 units of X1 in the next solution.
The number at the intersection of key row and key column is
referred to as the pivot or key number which is 2 in the present case.

TABLE 3. INITIAL SIMPLEX TABLEAU, KEY ROW,


KEY NUMBER, KEY COLUMN
('I I'n-duct Rs. 4 Rs 3 Rs. 0 it 0
,m.v Quantity X X, S1 52 -
4 Key number
Rs, 0 5, O 2 1 1 0
st Replaced row
Rs. 0 !i 3 2 0 1 or Key row
Zj Rs. U R. 0 Rs. 0 Rs 0 Rs. 0
Cj— ;7j 4 3 0 0 -- -
O1ii aim Column or
Key Column
Step VI. Develophig Second Simplex Tableau. Ilavitig chosen
the optimum solution and the replaced row, a second simplex tableau
can be developed, providing an improved solution.
(a) Compute new values for the key row. For this we have to simply
divide each number in the key row by key number, The key row now
bccornea

C) P,oduci.n,Ix Quantity XL

X. s, s,


4 XL 1 5 0

It may be noted that in the product mix, S1 has Oeeu replaced by


X1 and the corresponding C1 value also has been replaced (4 for 0).
(h) Compute new values (derived numbers) for each remaining rows
To complete the second tableau, we compute new values for the remain-
ing rows. All remaining rows of the variables in the tableau arc calculated
using the following formula
( New ( Elements in' Intersection Correspondiiig ele-
X
row ) ' the old row I [(element of old row ments in replacing
row
Using this formula, we get the new S 2 row as follows:

LINEAR PRJ. INC. 1,1--32

Element in Intersectional - Corresponding - New S row


Old S. row eletflCflt of element in
S. row key row -- -
-
(1) (2) (3) (J)-(2)\ (3)

- 16 3 5 16-3x 5_1
3 3 1 3--3x1()
2 3 2--3x=
(J 3
1 3 0 3 0.=1

Thus, the new S, row will be


(1, 0, , I)
An alternative formula is as follows
Derived Number=- Selected number
/ Corresponding Corresponding
x
number in key row number in key column
- Ky number
The computations will be as under
2x3 3 1
10x3 1 •3 0 2
2 2

---
- •i-
2 ' 2
(c) Conpuiing Z 1 and C1 - 7 1 rows. Now, we shall compute the Zj
and ('-Z rows (the profit opportunities) according to the methods
discussed earlier.
TABLE 4. SECOND SIMPLEX TABLEAU

Cj- 4 3 0 0

Product 'nix Qua,itiiy XL X1 S S2

4 Xi 5 0
- S1 - 1 0 —3!2 1row

Zj 20 4 2 2 0

0 1 —2 0 Index
- -- -- ----- _row
$
------ Key Column
Keynumber

BUSINESS MATHEMATICS

-! he computation of Ihe /, row of the second tableau is as follow;


/j (i.e., total pcolit) for quantity colitrnii
('I X 5) i-(Ox 1)=-20
/j for X co lumn (4 X 1) - (Ox 0) 4
/, br K2 column (4 x ) - (0 X ) 2
Z for S column (4 x ) - { Ox ( -- )} = 2
Z for S column (4. 0) 4(0 x 1) ­ 0

The computation of the (C, -- Z) row of the second tableau is as


follows

Variib!es Profit per Plojif lost jVet profit


14/lit per unit (t) per unit (C 1 - -
4 1 0
3 2 1
s t 0 2 --2
0 0 1)

Step VII. [he presence of a positive number in the K 2 column


of the C, - /, row of the second tableau shows that J)OBitiVC improve-
ment is possible. Ileilce the process ucd to develop the second solution
mu3t now he repeated to obtain it third solution. Accordingly, we find
that
(i) The variable ,V 2 will enter tile solution by virtue of C 1 -Z = 1
being the largest and only positive number in that row. This means that
for CVCi y unit of X., that we produce, the objective function will increase
by Re. I.
(fr) rh opt i iiinm colu inn or key column is X column.
(C) ihe replaced row is 2 row also known as key row or pivot
row. This is found by (f) dividing 5 and I in the quantity column by
their corresponding numbers in the key column, i.e, J and J respectively,
(ii) choosing the row with the smaller ratio as the key row.
((J) Intersectional element of .l' row is A, arid the intersectional
element of S, row is also . This will he the pivot number or the key
number.
(e) The key row is replaced by dividing every number in it by the
key number, i . e , the key row now becomes

C1 Proc/u, t ,tci Quantity X I X 2 S1 S,

2 1 - 32

LINEAR PROGRAMMING LP-34


f) The new values Intersectional
of the X1 row (tnird tableau) are
Correspon(/ing \
/
Element in clement of x element of = New X 1 row
\. old X 1 rov / \ X row new X2 row
5 x 2) - 4
-- x 0) =
- ( x 1) 0
- ( )< —3) = 2
I) - 2)
(We can compute the new X, row through the alternative formula
as well.)
TABLE S. THIRD SIMPLEX TABLEAU

Ci 4 3 0 0

Frothier mi-i Qua#itiry X1 X. St S

4 4 1 0 r —j .-Fey row
3 2 o I —3 2

----- -.-------- __ ______


22 4 3 —1 2
I
Cj - o 0 1 —2 A Index ru

t ____Key column
Step VIII. Once again, we find that all the values of this row
are not zero or negative, therefore, we have to proceed a little further.
However, the key row s key column as well as the ke y number have been
indicated in the third simplex tableau.
Step IX. By repeating what has been done earlier we arrive at the
final tableau IV.
TABLE 6. FOURTH SIMPLEX TABLEAU

Ci 4 3 0 0

Product mix Qiinriy X1 A'2 S S1

St 2 0 I —1
3 X28 1 0

24 3 0
Z'

CI—?) - 0 0 -
LP-35 BUSINESS MATHEMATICS

As there are no positive values in Cj —Z1 row, no further improve


ment is possible, and the optimum solution has now been obtained This
Solution is X, = 0, X2 = 8, S 1 = 2
The Z total, i.e., Rs. 24, represents the profit obtained under the
Optimum solution.
Example 9. An electronics firm is undecided as to the most p r ofit
-able,nixfortspduc.The nowmaufctrednsio,
resistors and carbon tubes with a profit (per 100 units) of Rs. 10, Rs. 6 and
.Rs. 4 respectively. To produce a shipment of transistors containing 100
units requires 1 hour of engineering, 10 hour.s of direct labour and 2 hours
of administration service. To produce 100 resistors are required 1 hour,
4 hours and 2 hours of engineering, direct labour and administration time
respectively . To produce one shipment of the tubes (100 units) requires I
hour of engineering , 5 hours of direct labour and 6 hours of administration.
There are 100 hours of engineering services available, 600 hours of direct
labour and 300 hours of ad,ninist ration What is the most profitable mix 2
Solution. For the sake of convenience, we tabulate the data in
the following manner

Products
Available
hours
Transistors Resistors Carbon Tubes

Engineering I I I 100

L'bour 1 10 4 5 600
Administration 300

Profit per 100 units I Ks. 10 1 Rs 6 I Ks. 4

Objective Function : Maximise Profit


Z=10x1+6x2-I-4x3
Subject to the constraints:
X1+X2+X3'100
I OX, +4x 2 -f5x3 600 .. (3)
2x1 ±2.v2 +6x3 300 (4)
X 1 , x2 , X3 ,-
>
The first step in the Simplex Method is to convert the inequalities
(or restrictions) into equalities. This is done by adding a slack variable
(unused capacity of the department). After adding the slack variables,
all the expressions (1) to (4) can be written as
X 1 + x, + x3 + S 1 100
l OX, ±4X2+5X3+Sr600
2x 1 1- 2x2 4 6x + S,-3OO
lOx1 +6x.1 ±4 x3±OSr1- OS + O-S, = Maximise Z
The simplex method always begins with a zero solution. i.e., it starts
at the point of no production whatsoever. This enables the steps in the

LINEAR PROGRAMMING LP-36

solution to determine the appropriate quantity of each item to produce,


subject to the objective function and the restrictions.
In other words, if X 1 . X., and x3 are not produced, then the unused
capacity of the three departments as given by S, and S3 will be 100,
00 and 300 hours respectively. The solution at the first programme is
given by the quantity column and the product mix column. The solution
at this point is S,=100, S 2 600, S3=300.
SIMPLEX TABLEAU I

CJ 10 6 4 0 00 Rciio
Product Quantity Column
Mix K1 X2 X3 S1 58 S8

0 100
0 60
I

Z i 1 0 0 0 0 0 0

Cj - Zj 1 10 6 4 0 0 0

SIMPLEX TABLEAU II


0 S1 40 0 6/10 5/10 1 -I/JO 0 67
f
10 X1 60 1 4110 5/10 0 1/10 0 150

0 S, 180 0 12110 5 0 -2/10 I 150

Z, hG 4 5 0 1 01
Cj -Zj 0 2t -1 0 -1 0
J
SIMPLEX TABLEAU III

5/6 10/6
6 400/6 0 1 -1/6 0
16 -213 I/O 0
JO 100/3 1 0

0 S, 100 0 0 4 -2 (1 0

Zj JO 6 20/3 10/3 2 3 0

0 0 -20/3 -I0/3 -2/3 0


LP-37
BUSINESS MATHeMA1 IcS

4 00100
Hence the most profitable mix is resistor and transistors.

The maximum profit is 400-I 1000 733 3
Example io. Vitamins A, B and C are found in foods F1 and F2.
One unit of F1 contains I Mg of A, 100 ing of B and 1(1 nig of C. One unit
Of F contains I mg of A, 10 mg oJjJ and 100 tng of
requirements of ,t, Li and C are / C. The sninilnum daily
tng, 50 ing and 10 mg respectively. The
Cost per unit of l', and f are Re. I and Rs. 150 respectively. You are
required to (i) Joririulate the above as a linear programming problem tniiumis-
Rig the cost per day, (ii) write the dual
of the problem and (ii) solve the
diiul by using simplex method and read there from the answer to the primal.
[Delhi Un i v. , B. Corn. (lions.), 1992]
Solt0 (i) Let x units of l' and x 2 units of F2 be purchased.
Primal: Minimise (cost per day) : Z=x 1 -- 15, subject to
x 1 -t-x 2 1, 100x1 ± l0x5O. 10x 1 + l0Ox>lO
x)O, x20.
(ii) Dual Let p, q. and r be the dual variables. Then we have
Minimise C==p+ 50q-- lOr subject to
,p,q,rO.
(iii) Solution to Dual : Introducing slack variables s and s, the dual
may be written as under
Maximise C =p+50q lOr40.s 1 +0.s2 subject to
p f 100q j- lOr
p -f-10q---l0O r -f Os f-S=
p. q. r S1 , S ' > 0

4 Variables Basic Ratio


Variables p q r S1
1 I 100 10 1 0
10 100 0 1

- CJ -z - 1 -50 t 10 0 0
50 q r+-ç 1 rt r'r 0
0 99*1
4 505 4 0
C—Z1 3 0 5 0
10
q V5 T+T I 0 th
ci__Z/ ir
- 0 0
I no
10 r 0 —I I
z, 1 100 10 0 0
0-50 0
Answer to primal = 1. x2 0 and total cost =
LINEAR PROGRAMMING LP-38
EXERCISES
I. Why is the simplex method a better technique than the graphi-
cal approach for most real cases ?
2. Give outlines of 'Simplex Method' in Linear programming.
3. (a) A manufcuirer produces two items X,and X9 . X1 needs
2 hours oil A and 2 hours on machine B. ''2 needs 3 hours on
machine A and 1 hour on machine B. If machine A can run for a
maximum of 12 hours per day and P. li hours per day and profits from X1
and X2 are Rs 4 and Rs. 5 pc tcm respectively, find by simplex
method how many items per day be produced to have maximum profit.
Give the interpretation for the values of 'indicators' corresponding to
slack variables in the final iteration
(h) A manufacturer produces bic y cles and scooters. each of
which must he processed through two machines A and B. Machine A baR
a maximum of 120 hours available and machine B has a maximum or 180
hours available. Manufacturing a bicycle requires 6 hours ill A
and 3 hours in machine B. Manufacturing a scooter requires 4 hours
in machine A and 10 hours in machine B. If profits are Rs. 45 for a bi-
cycle and Rs 55 for a scooter. determine the number of bicycles and the
number of scooters that should he manufactured in order to maximize
the profit.
4 A novelty manufacturer makes two types of emblems, A and B.
flc uses three departments : preparation, cutting and packaging. Each
department is used for both types of emblems. Processing rates are
Type A Type B
(min/pc (min/pc
Preparation 4 3
Cutting 8
Packaging 6 3
The profitper unit is Rs. 2 and Ps. 3 for t y pe A and type B res-
pectively. TI I ,00 minutes are available in each of the departments,
determine the optimal production schedule. Use Simplex Method.
5. A firm makes two types of furniture: chairs and tables. Profits
are Rs. 20 per chair and Rs. 30 per table. Both products are processed
on three machines M1 , M2 , and M3 . The time required for each product
in hours and total time available in hours on each machine are as
follows
Machine Chair Table Available Time
M l 3 3 36
5 2 50
2 6 60
(a) Formulate the above as a linear programming problem
to
maximise the profit; (b) Write its dual: and (c) Solve the primal by
simplex method. [Ans. 3, 9; Rs. 330.1

LP-39 BUSINESS MATHEMATICS

6. A manufacturing company contemplates to produce two addi-


tional products, called A and B, which can be marketed at prevailing
price ,- in any reasonable quantities without difficulty. It is known that
Product A requires 10 and 5 man-hours per unit in the foundry and the
machine departments respectively ; and that product B requires only 6
and 4. However, the profit margin of A is Rs. 30 per unit as compared
with Rs 20 per unit of ii. In the week immediately ahead, it IS esti-
mated that there will be 1000 and 600 man-hours available in the
foundry and the machine departments respectively. How much of '
and B should be produced in order to most profitably utilize the excess
capacities ?
7. A company makes three products X, Y, Z out of three matarials
F1 , P2 and P3 . The three products use units of three materials according
to the following table
Materials
I

X 1 2 3
Products Y 2 1 1
Z3 2±

The unit prolit contributions from the three products are



Product : X Y Z
Profit Contribution (in Rs.)
3 4 5
and availabilities of the three materials are
Material P1 P2 P.
Amount available (in units) :
10 12 15
I he problem is to determine the product mix which will maximize
the total profit.
[Hint. SIMPLEX MATRIX V
C-+ Product Quantity 3 4 5 0 0 .0
Mix X y S i SZ 53
0 S2 0 0 4/5 —h'S 1 —3/5
3 x 4 I 0 —1/5 —1/5 0 2/5
4 Y 3 0 1 8/5 0
315 —1/5
ci—zi 0 0 —4/5 —9/5 0 - 2/5
The optimal solution of the primal problem is to produce 3 units of
product X, 4 units of product Y and no units of prcduct Z which gives a
ma'umum profit of Rs. 24.1
R. A manufacturer of leather belts makes three types of belts
B and C which are processed on three machines M. M 2 and M3 . A,
Belt

LP-40
LINEAR PROGRAMMING
Belt B
A requires 2 hours oil M 1 and 3 hours on machine M 2 .
requires 3 hours on machine M 1 , 7. hours on machine M2 and 2 hours
and 4 hours
on iiiaelune M 3 and Belt C requires 5 hours on machine M 2
on machine M, There are 8 hours of time per day available oil
Al 1 , 10 hours of time per day available on machine M 2 , and 15 bouts of
time per day available oil M 3 . The profit gained from belt A is
Rs. 3'00 per unit, from Belt B is Rs. 500 per unit, from belt C is Rs. 4-00
per unit. What should be the dail y production of each type of belts so
that the profit is maximum.
(Hint. Maximize
z3x 1 + 5x2-j-4X3
Subject to the constraints
2x,1 -f3x2 '8
2x 2 + 5 X 10
3x1+2X2F-4X315
x 1 , x 21 x)0.
Using simplex method, we get
89 5062 765
1 =-1-, x 2 =-1 - , x= and max. Z=--1
X
9. Explain the nature and significance of L.P.
A farmer has 1.000 acres of land on which he can grow corn, wheat
or soyaheans. Each acre of corn costs Rs. tOO for preparation, requires
7 man-days of work and yields a profit of Rs. 30. An acre of wheat
costs Rs. 120 to prepare, requires 10 man days of work and yields a
profit of Rs. 40. An acre of soyabeans costs Rs. 70 to prepare, requires
8 man-days of work and yields a profit of Rs. 20. If the farmer has Rs.
1,00,000 for preparation and can count on 80,000 man-days work, how
many acres should be allocated to each crop to maximise the total
profit 7
[Ans. Corn 250, wheat 625, soyabeans 0, Profit Rs. 32,500,1
10. A small-scale industrialist produces four types of machine
components tif 1 , M2 , M 3 and M 4 made of steel and brass. The amounts
of steel and brass required for each component and the number of man-
weeks of labour required to manufacture and assttnble I unit of each
component are as follows
M2 M Availability
Steel 6 5 3 2 100 kg.
Brass 3 4 9 2 75 kg.
Man-weeks 1 2 1 2 20

The industrialist's profit on each unit of M1 , Mp M3 and M'4 is


respectively Rs. 6, Rs. 4, Rs. 7 and Rs. 5.
How many of each should he produce to optimize his profit and
bow much is his profit ? (Note that the values given are the average

LP-41
BUSINESS MAThEMATICS
values per week. If a uracional Value appears in the answer, it should be
Interpreted as an average value)
[Ans. M, : 14 ; M2 : 0; M3 : 10/3 ; M4
0
Profit ; Rs. 13 -- per week]
DUALITY IN LINEAR PROGRAMMING
Associated with every linear-programming problem is a related
dual linear- progra in ming problem. The originally formulated problem,
in relation to the dual problem, is known as the primal linear program-
ming problem. If the objective in the primal problem is niuim,za(:on of
some functjo, then the objective in the dual problem is rnininiiuit ion of
a related (but different) function. Conversely, a primal minimization
problem has a related dual maximization problem. The concept of
duality is niOrc effectivity demonstrated in the following illustration
Primal Dual
Maximize : Z 3v 1 +5x4. Minimize Z'=8y1-4-l0y+l5y
Subject to Subject to
2x1 +3x8 2y1 -f- 3Y3>3
2x 2 + 5x 3 10 3P1 -4-- 2y -I- 2y 3 >5
3xi-4-2X+4x15 5y2+4y3>4
X i >0. X'>'0' X3>0
Y1>0,y2>0,y3)0
PRiMAl.

xt Re/tion Cons:ang

yl 2 3 -1< 8
DUAL - 2 5
I
y 3 3 2 I '5

Relatton
mm Z'

Con stants 3 5 4 max Z
It will be seen that
I. Primal, here, involves maxi- Dual involves minimizatjo
mization.
2.- In primal, we write objective
In dual, we write objective function
function as Z. as Z•
3. In primal, the variables are
Dual has a new set of variables, i.e.,
X1 , x 2 and xa. y1, y2 and y3.
LINEAR PROGRAMMING i.p-42
4. Prima] has three varia blcs, The dual, three
therefore. has
iz V 1 , X2 and x3. Consf aints.
5. The primal has three cons- The dual, therefore, has three
train is. variables, oiz . , y 1 . v. and .v.
6. In primal's objective func- In dual. 3, 5 and 4 become cons-
tion. 3, 5 arjd 4 arc the tants of constraints on the right hand
coctficiet5. Side.
7. In primal, the coefficients In dual, each C011.111111 takes the
of constraints, columnwise, position row-wise as under
are
2 3 2 3
- 2 5 3 2 2
3 2 4 -. 5 4
8. In primal, the signs of In dual, the signs of the constraints
constraints are less than or are just the reverse, i.e.. greater than
equal to or equal to.
9. The non negativity cons- The non- negativity constrinis are
Iraints are as man y as the as many as the variables in the dual,
varia hk's in the primal, i.e. , 3.
i.e , 3,
10. The signs in the non-ncga ti- The signs in the non-negativity
vity Constraints are greater constraints do not change and remain
than or equal to. the same.
Concli,sjon
The foregoing examples make it clear that the transformation of
a givea primal problem involves the following considerations
I If the primal involves maximization, the dual involves mini-
mization, and vice versa,
2. A new set of variables appears in the dual,
3. Ignoring the number of non negativity constraints, if there
are n variables and in inequalities in the primal, in the dual, there will
in
be and n inequalities.
4. The coefficients in the primal's objective function are put as
dual's constraint constants, and vice versa.
5. Of the primal's constraint inequalities, the coefficients column-
wise (from top to bottom) are positioned in the dual's constraint
inequalities row-wise (from left to right), and vice-versa,
6. IF the primal's constraints involve signs, the dual's constraints
involve ) signs, and vice versa,
7. The signs in the non-negativity constraints are > both in the
primal and the dual.
Example 9. Food F1 contains 6 units of vitamin A, 7 units of vita-
min B and 8 units of vitamin C. It costs Rs. 10 per unit. Food F 2 contains 7

L'-43 I3USH'1€SS MATHEMATICS

units of vitamin .4, 6 UflhtS of vitamin B and 10 units of vitamin C. It


costs Rs. 12 per unit. Food F 3 contains 8 units of vitamin A. 9 units of
vitamin /3 and 6 units of vitamin C. It costs Rs. 15 per unit. The daily
minimum requirement of vitamins A, B and C are 100 units, 120 units and
150 units respectively. Write the dual of the above linear programming
problem, solve the duai From the optimal solution of the dual, find the
optimum solution of the prirno! problem.
Solution. 'The given problem, i.e., the primal problem, stated in
an appropriate mathematical form is as follows
Minimize C (Total cost) = I Ox 1 + I 2x 2 + I 5x
Subject to the constraints
6x1-l-7x2+8x.1>I00
7x1+6x1-I-I0x)I20
8x 1 +9X 2 4-6x 3 150
x1 0 1 x 1> 0 1 x3>O

where.
- Number of units of Food F1

xn==Number of units of Food F2

-= Number of units of Food F3

The dual to the above problem is


Maximise 7= 100y 1 -1- 120y 2 + I 50y
Subject to the constraints
6;' -I- 7y, 4-

7y 1 -f-6y2-f9y3l2
8y 1 -l-lOy.+6y3 15
y1 s0, y 2 )0, y3I)
Solution of Dual Probletn
For solving the Dual problem, we convert the inequalities by adding
slack variables S 1 . S2 and S3.
Maximize 7 = 100 y 1 4- 120) 2 + I 50y3 -4- O.S + 0.S n 1­ 0-S3
6;' -i- 7y 4- 8y 3 ± S 1 = 10
7y 1 + 6y, -f 9y 3 + S 2 == 12
8y 1 +10y2+6y3+S3=15
Y1, y2 , y, S1 , S.., S3>0
As usual, if we make an initial decision of no production, this
decision summarized in tabular form ill be as follows

LP-44
LINEAR PROGRAMMING
SIMPLEX MATRIX I

Cf Product Quint it y 100 120 150 0 0 0

YI Y2 Y3 S1 S1 S


U Si 10 6 7 8 1 0 0 - hey
Cottilnn
o 12 7 6 9 0 I 0
o s 15

8 tO 6 () 0


Cf - Zj 100 120 ISO 0 0 0

t
L ----Key Row
In simplex matrix I, e find that key column is corresponding to
the variable y and key row is corresponding to the variable S 1 We
now proceed to Simplex Matrix II.
SIMPLEX MATRIX II

I 1 1 100 120 1.50 0 0 0


ci-.. I -___ ------- ___
I'roduci Quantity Yi Yz )3 S1 Si S
Mix

150 54 3/4 7/8 1 I/S 0 0


0 ,ç 3/4 114 —15/8 0 —9/8 1 0
0 1512 7/2 19/4 0 — 3/4 0 I

--T J—Zj 1-25/2 - 45/4 —0 150/8 0 0

The optimal solution to the dual problem gives a maximum value


or for the objective function.
Interpreting Primal-Dual Optimal Solutions
Once the dual problem has been formulated and solved, there,
remains the vital step of correctly interpreting the optimal solution to
the primal. The solution values for the primal can be read directly
from the optimal solution of the dual. This can be, described in the
following steps
Step I. Locate the slack variables in the dual programme. These
correspond to the primal basic variables in the optimal solution.
Step 2. Read the values of the index numbers in the index row
corresponding to the columns of the slack variables. This directly gives
the optimal values of the basic primal variables.

Lip - 45 BUSINESS MATHEMATICS

Step 3 The optima) values of the objective function for the


problems are the same.
150. 70
- ; x=x9 = 0 and Mm. Cost
Remark. It may be noted that if the primal problem involves
lesser number of variables than the number of restrictions (constraints),
the computational procedure can he considerably reduced by converting
it into dual and then solving it This offers advantage in number of
applications.
EXERCISES
1. Solve the following primal graphically. Write down its dual
and solve this also graphically.
Maximize Z=x1+5x2
subject to the constraints
5x + 6x230
3x1 + 2x2 12
x1 , x2>0
[Ans. x 1 r=O. x-=5. Z,,,,.251
2. Some town, a small township of 15,000 people. requires, on the
average, 3.00,000 litres of water daily. The city is supplied from a central
waterworks where the water is purified by such conventional methods as
filtration and chlorination In addition, two different chemical com-
pounds (i) softening chemical and (ii) health chemical, are needed for
softening the water and for health purposes. The waterworks plans to
purchase two popular brands that contain these chemicals. One unit of
ARC Corporation's products give S kilogram of softening chemical and 3
kilogram of health chemical. One unit 01 XYZ chcmicil company's
product contains 4 kilogram ard 9 kilogram per unit, respectively.
To maintain the water at a minimum level of softness and to meet
a minimum in health protection, experts have decided that 150 and 100
kilogram of the two chemicals that make up each product must be
added to water daily. If costs per unit. for ABC corporation's and
PQR chemical company's products are. Rs. 8 and Rs. 10 respectively,
what is the optimum quantity of each product that should be used to
meet the minimum level of softness and a minimum health standard ?
Write also the dual to the above linear programming problem and
solve it.
[Hint. The relevant data may be tabulated as below:

Chemical
I ABC Brand XYZ I Deily Requirement

(1) Softening 8 4 150


(it }iealmb 1 3 9 1 tOO

Cost/unit of c y ch brand 1 8 10

LINIAR p Ro(;RAMMR4C; u'-46


1'rirui1 Minimise (cost), Z=8x 1 F lOx2
Subject to the constraints
8x 1 +4x_ 150
2x 1 -l9x, l0()
and x1>O, X, -- 0.
Minimum cost -= Rs. 185.
Dual: Maxitnise, Z= I 50y1 -I bOy2
Subject to the constraints
8 y1 -F 2y 2 sç 8
4y1+9y2b0
.y 1 , y>01
3 A diet for a sick i)trsoi must Contain at least 4000 units of
vitamins, 50 units of minerals and 1400 of calories. Two foods A and B
are available at a cost of Rs. 4/- and Rs. 3/- per unit respectively. If
one unit of A contairts 200 units of vitamins, I unit of mineral and 40
calories and one unit of food B Contaiils 100 units of vitamins, 2 units
of minerals and 40 calories, Iuid b y simplex method what combj!iatuJn
of foods be used to have least cost.
[Hint Primal Minimize Z-4 1 -l--3x
Subject to the constraints
200x1 + 100x 4000
x1--2x25O
l0x 1 +40j2 1400
x 1 , x20.
Dual : Maximize Z*=400y15y2 1400y3
Subject to the constraints
200y 1 1-y2+40y4
100y+2y0-j--40y33
Y 1 1 Y2 1 YaO
x 1 =5, x2 30, Z,, 1 RS. 110]
4 A company makes three products, K, Y, Z out of three
materials P 1 , P2 and P2 . The three products use units of three materials
according to the following table
Pt P2 P3
X I 2 3
Y 2 1
Z 5 2 1
The unit profit contribution of the three products are Rs. 3, Rs. 4
and Rs. 5 respectively Availabilities of the materials are 10, 12 and 15
units respectively. The problem is to determine the product mix that
will maximise the total profit Solve the primal problem, write its dual
and give the economic interpretation.
SECTION C

Probability
INTRODUCTION
Two types of phenomena have usually been observed in nature and
in everyday life. These are
(1) deterministic. and
(ii) probabilistic.
In the first type, the hypotheses are stated exactly and no 'chance
elements' are involved subsequently during the analy sis of the pheno-
menon. Consequently, in such a case predictions of complete reliability
can be made, e g,, if we are given that a train is running at a uniform
speed of sixty kilometres per hour, then we can predict with cent per cent
surety that it will cover one hundred twenty kilometres after two hours,
assuming. of course. that it never stopped during these two hours. Most
of the phenomena in physical and chemical sciences are of a deterministic
nature. However, there exists a number of phenomena where we cannot
make predictions with certainty or complete reliability and are known
as unpredictable or probabilistic phenomenon. Such phenomena are
frequently observed in business, economics and social sciences or even
in our day-to-day life. For example:
(i) In toss of a uniform coin we are not sure of getting the head
or tail.
(ii) A manufacturer cannot ascertain the future demand of his
product with certainty.
(iii) A sales manager cannot predict with certainty about the sales
target next year.
(iv) If an electric tube has lasted for one year, nothing can be
predicted about its future life.
Probability is also used informally in day-to-day life. We daily
come across the sentences like
(i) Possibly, it will rain to-night.
(ii) There is a high chance of your getting the job in October.
(iii) This year's demand for the, product is likely to exceed that of
the last year's.
(iv) The odds are 2 : I in favour of getting the contract applied for.
All the above sentences, with words like 'possibly', 'high chance'.
'likely' and 'odds' are expressions indicating a degree of uncertainty
about the happening of the event. A numerical measure of uncertainty is
provided by a very important branch of statistics called the "Theory
of Probability " Broadly. there are three possible states of expectation--
cerlainty', irnpasibilicy' and uncertainty' The probability theory des-
cribes 'certainty' by 1, impossibility by 0 and the various grades of un-
certainties by coefficients ranging between 0 and 1.
PROBABILITY

According to Prof Ya-Lin-chou "Stwis:icr is the science of decision-


piakinç wit/i calculated risks in the face of uncertainty''
MEASUREMENT OF PRO1L&BILITy
The following is the broad classification of the different schools of
thought in probability
PROBABILITY

--.4,
Objective Subjective

Classical Empirical
Approach Approach

Modern Approach
Brief description of these concepts is given below.
OBJECTIVE PROBABILITY
The objective probability is based on certain Jaws of nature, which
are undisputed, or on some experiments conducted for the purpose.
This is not based on the impressions of the individuals as is the case
with subjective probability. These theories, therefore, are free from personal
bias and ensure objectivity. The two approaches to objective probability
are (a) classical approach, (b) empirical approach.
Fundamental Concepts
1. Random Experiment. An operation which can produce any
result or outcome is called an experiment. An experiment is called a
random experiment if, when conducted repeatedly under essentially
homogeneous conditions, the result is not unique but may be any one of
the various possible outcomes (The word random may be taken as one
depending on chance' without any bias). For example
(1) Tossing a fair coin is an experiment. (A coin is a circular metal
disc, the two faces of which are somehow distinguishable and are called
'head' and 'tail'.) Whether the coin will throw up head or tail is
unpredictable.
(ii) Rolling an unbiased die is an experiment. (A die Is a solid cube,
Ibc'. faces of which are marked with 1, 2, 3, 4, 5 and 6 dots or actual
11ges 1, 2, 3. 4, 5. 6 respectively.) How many dots it will actually throw
up is unpredictable and is subject to chance.
(iii) Drawing a card from a well-shuffled pack of playing cards is
an experiment and as there are 52 cards in the pack and any of these may
be drawn in a s p ecific trial, which card it will turn out is unpredictable.
(h') Drawing two balls at random from a box containing, say. 8
white, 9 red and 7 green balls, all well-mixed is an experiment. Which
particular ball will be drawn is unpredictable
P-3 BUSINESS MATHEMATICS

(v) When a coin is tossed 100 times or 100 coins are tossed together,
there are hundred experiments.
(vi) Experiments in business world can be in regard to the obser-
vation of the number of defective items produced by a machine, or
recording the number of customers visiting a sale Counter. In an
advertising campaign for new product launched, the number of items
sold may be observed.
2. Elementary Event. Each one of the possible outcome in a
single experiment is called an elementary event.
(i) In an experiment of tossing a coin there are 2 possible elementary
events, the head and the tail.
Ui) In an experiment which consists of throwing a six-faced die, the
possible elementary events are 1, 2, 3, 4, 5 and 6.
, iii) in an experiment of drawing a card of a given designation
from a pack of cards, there are 4 possible outcomes corresponding to 4
suits with designations of heart, diamond, spade and club.
(iv) In a trial of drawing a card from a suit of spade alone, there are
13 elementary events, viz., 1 to 13 cards.
(v) In a trial amongst 12 face cards, there are 4 elementary events,
viz., king, queen and jack.
3. Exhaustive Cases or Outcomes. The total number of possible
outcomes of a random experiment is called the exhaustive cases for the
CXperinent. Thus in toss of a single coin, we can get head (II) or tail
j. Hence cxuausrive number of cases is 2, vii, (H, T). If two coins
are tossed, the various possibilities are HI!, HT, TH, Ti', where Hi'
means head on tue first coin and tail on second coin and TH means tail
on the first, coin and head on the second coin and so on. Thus in case of
toss of Iwo coins, exhaustive number of cases is 4, I.e., 2. Similarly, in a
toss of three coins the possible number of outcomes is
(ii, x (H, T) x (II, T)
- (HH, Hr. TH, TT) x (H, T)
=(1fH H, 11TH, TIll!, TTH, Hill', HTT, THT, TTT)
4. Favourable Cases. The number of outcomes of a random
experiment which entail kor result in) the happening of an event are termed
as the cases favourable to the event. For example
(1) In a toss of two coins, the number of cases favourable to the
event "exactly one head" is 2, HT, TH and for getting two heads' is
one, viz., HI!.
(ii) In drawing a card from a pack of cards, the cases favourable to
'getting a club' are 13 and to 'getting an ace of club' is only 1.
5. Mutually Exclusive Events or Cases. Two or more events
are said to be mutually exclusive if the happening of any one of them
precludes the happening of all others in the same experiment. For
example, in tossing of a coin the events 'head' and 'tail' are mutually exclu-
PROf3fl(UfY p-4

sive because if head comes, we can't get tail and if tail comes we can't get
head. Sitmiarly, i ll throw of a die, the six faces numbered 1, 2. 3, 4,
5 and 6 are mutually exclusive. Thus events are said to be mutually
exclusive if no two or more of them can happen simultaneously.
6. Equally Likely Cases. The outcomes are said to be equally
likely or equally probable if none of them is expected to occur in pre-
(die), all the outcomes, viz.,
ference to other. Thus, in tossing of a coin (die),
H. 1 (the faces I, 2, 3, 4, 5, 6) are equally likely if the coin (die) is
unbiased.
Independent Events. Events are said to be independent if
the occurrence of one event in no way affects the occurrence of the other.
For example
(t) In tossing of a coin, the event of getting 'head' in first throw is
independent of getting 'head' in second, third or subsequent throws.
(ii) In drawing cards from a pack of cards, the result of the second
draw will depend upon the card drawn in the first draw. However, if
the card drasn in the first draw is replaced before drawing the second
card, then the result of second draw will be independent of the 1st draw.
Similarly, drawing of halls from an urn gives independent events if
the draws are made with replacement. If the ball drawn in the earlier
draw is not replaced, the resulting draws will not he independent.
Mathematical or Classical or 'a Priori' Probability
Definition. If a random experiment results in N exhaustive, mutually
exclusive and equally likely cases (outcomes), out of which ni are
favourable to the happening of an event A, then the probability of occur-
rence of A, usually, denoted by P(A) is given by
Number of outcomes favourable to the occurrence of A-
P(A) Exhaustive number of outcomes
11
n
This definition was introduced by James Bernoulli.
Remarks. 1. Probability that event A will not occur, denoted by
P(A) j
Number of outcomes not favourable to occurrence of A
- Exhaustive number of outcomes
tv—rn

2 Probability of any event is a number lying between 0 and


1, i.e.,
0P(A)l,
for any event A. If P(A)==0, then A is called an impossible or nz'tl event.
If F(.4) I. then A is called a certain or sure event.
3. The probability of happening of the event A, le, P(A) is
also known as the probability of success and is usually written as p and
BUSINESS MATHEMATICS
the probability of the non-happening, i.e., P(A)
is known as the probabi-
lity of failure, which is usually denoted by q. Thus,
q=1—p
4. Limitations. The classical probability has its shortcomings
and fails in the following situations
(I) If N, the exhaustive number of outcomes of the random experi-
ment, is infinite.
(Ii) If the various outcomes of the random experiment are not
equally likely. For example, if a person jumps From the running train,
then the probability of his survival will not be 50, since in this case the
two mutually exclusive and exhaustive outcomes, viz . , survival and death,
are not equally likely.
St atistical or Enipirical Probability
Defitj0 (Von Mises). If an experiment is performed repeatedly
under essentially homogeneous and identical conditionr, then the limiting
value of the ratio of rie number of times the event occurs to the number of
trials, as the number of trials becomes indefinitely large, is called the
probability of happening of the event, if being assumed that the limit is
inite and unique.
f
Suppose that an event A occurs in times in N repetitions of a
random experiment Then the ratio in/N, gives the relative frequency of
the event A and it will not vary appreciably from one trial to another. In
the limiting case when N becomes sufficiently large, it more or less
co rresponds to a number which is called the probability of A.
cally, Symboli-

P(A)= urn
N-" oo N
Ileznarks. 1. Since in the relative frequency approach, the
Pro bability is obtained objectively by repetitive empirical observations,
it is also known as Empirical Probability.
2. The empirical probability provides validity to the classical theory
of p robability. If an unbiased coin is tossed at random, then the classical
Probability gives the probability of a head as J. Thus, if we toss an
u nbiased coin 10 times, then classical probability suggests we should have
5 heads. However, in practice, this will not generally he true. In fact
In 10 throws of a coin, we may get no head at all or I or 2 heads. J.E.
Kerrich conducted coin tossing experiment with 10 Sets of I ,000 tosses
each during his confinement in World War Ir. The number of heads
found by him were:
502, 511. 497, 529, 504, 476, 507, 520, 504, 529
This shows th.i: the probability of getting a head in a toss is nearly
. Thus, the empirical probability approaches the classical probability as
the number of trials becomes indefinitely large.
P-6
PROBABILITY

3, Limitations. (1) The experimental conditions may not remain


essentially homogeneous and identical in a large number of repetitions of
the experiment.
(ii) 'The relative frequency rn/N, may not attain a unique value no
matter however large N may be
Axiomatic Probability
The modern theory of probability is based o i l axiomatic
approach introduced by the Russian mathematician A.N. Kolmogoroy
in 1934. Kolmogorov axionilsed the theory of probability and his small
book Foundations of Probability' published in 1933, introduces probability
as a set function and is considered as a classic. In axiomatic approach.
to start with some concepts are laid down and certain properties
or 1,ostulate.c commonly known as atOrns are defined and from these
axioms alone the entire iheory is developed by logic of deduction. The
axiomatic definit i ons of probability includes both the classical and em-
pirical definitions of probability and at the same time is free from their
drawbacks. Before giving axiomatic definition of probability, we shall
explain certain concepts, used therein.
1. SrnpIe Space. A set whose elements represent the possible out-
comes of an experiment is called a sample space, which is a universal set
and is denoted by S. Each possible outcome given in the sample space
is called a sample point. The number of sample points in S may be
denoted as N(S). For example
(i) If one item is drawn from a manufactured product, the item
selected may either be defective D, or non-defective N. Then the sample
space is
S=-{D, N)
(ii) When a coin and a die are tossed together, there are twelve
sample points in the sample space:
S= {(7 1), (T. 2), (7', 3), (T, 4), (T, 5), (7', 6), (H, I), (H, 2), (11, 3),
(H. 4), (H, 5), (II, 6)).
- (iii) If a pair of dice is to be cast once, the 36 possible outcomes of
this experiment will be

Outcome of IOutcome of Second Die


First Die
1 2 3 4 5 6 The
I (1, 1) (1,2) (i, 3) (1,4) (1,5) (1,6) sample
2 (2, i) (2, 2) (2, 3) (2, 4) (2, 5) (2. 6) I space
3 (3, 1) (3, 2) (3, 3) (3, ';) (3, 5) (3, 6) " of the
4 (4, 1) (4, 2) (4, 3) (4, 4) (4, 5) (4. 6) experi-
5 (5. 1) (5, 2) (5, 3) (5, 4) (5, 5) (5, 6) ment.
6 (6. 1) (6,2) (6. 3) (6. 4) (6. 5) (6, 6) J
2 Evert Sets An event set' is a subset of the sample space. Thus
on a sample space there can he two or more event sets consisting of a
group of elementary events (sample point).
P- 7 BUSINESS MA1HEMATlC

For example, in the experiment of picking two items, one at a


time, at random, from a box containing defective and non-defective
items, "both items are defective" is one event, "both items are non
defective" is another event.
The event sets are denoted by capital letter A, B. C or E, £2,
etc. The sample points in each set may be denoted by small letters,
say a, b, c or a 1 , a2 , a 3 , or by any other suitable description. The number
of sample points in an event set A may he denoted by n(A).
Remarks I. An event E defined over a sample space S is said
to be a sample, or elementary, or fundamental event if it contains exactly
one sample point in S. An event F defined over a sample space S is
called a composite, or compound event or simply an event, if it Contains
more than one sample point in S. Thus, when 'a die is toss,'d, each of
the elements in the sample space S I. 2, 3. 4. 5 . 6 is a simple event
but the events F, -={l, 3, 5 and £=12. 4, 6 are conpsite.
2. A set of events defined over the same sample spice is said to
be tnutuall e xc lu sive, or disjoint, if no sample point is contained in more
than one of these events, i.e , a set of events E. is mutually
exclusive if no two sets have any sample points in COIflUlOfl.
3. Two or more events defined over the same sample space are
said to be collectivclj' exhaustive if their uuiou is equal to the sample
Space.
Azioruatic probability (DefinLtioi). Given a sample space or
a random cx perimcnt. the probability of the occurrence of any event
A is defined as a proIability function P('l) satisfying ihe fol!owing
axioms.
Axiom 1. The probability of an event exists, is real and non
negative, i.e
P(A)Q
Axiom 2. The probability of the entire sample space is I, i.e.,
P(S)= I
Axiom 3. If A 1 , A, 4 3,...be a finite or infinite sequence of disjoint
events of S, then
P(A1+A2+...)=P(A1)+P(A.,)+ P(4.).l
Remark. The above axioms are also krown as axioms of
positiveness , certainty and complete additivity respecTively.
Example 1. (a) Find the probahilitt' of getting head ( e throw of
a coin.
(b) If two coins are tossed once, what the 'rohabilitu' of getting
(I) both heads, (ii) at least one head ?
So'ution. (a) When a coin is tossed, there are two possible
outconies--head or tail.
n=2
PROBABILITY P-S
The outcome head' is the favourable casc.

Hence P(Head)=

(b) When two coins are tossed there are four possible cases, viz.,
Jill: Head on the first coin and head on the second coin
HT: Head on the first coin and tail on the second.

TH; Tail on the first coin and head on the second.

TT: Tail on the first and tail on the second.


n-4

(1) Out of these 4 cases, we need heads on both, i.e., the HH


tnr=l

Hence P(hoth hcads)=P(HH)=k


(ii) In three cases HI!, HT and TI!, we get at least one head.
P(at least one head)= or 1 -1
Eim pie 2. What is the chance that a leap lear selected at random
will contain 53 Sundays?

In a leap year (which consists of 366 days) there are 52


complete weeks and 2 more days. The folowing are the possible combi-
nations for these two 'over' days
(i) Sunday and Monday, (ii) Monday and Tuesday, ((ii) Tuesday
and Wednesday. (iv) Wednesday and Thursday, (v) Thursday and Friday,
(vi) Friday and Saturday, and (vu) Saturday and Sunday.

In order that a leap year selected at random should contain 53


Sundays, one of the two 'over' days must be a Sunda y . Since out of
the above 7 possibilities, 2, viz., (i) and (vii) are favourable to this event,
the required probitability=-

Exawple 3. Three unbiased coins are toss/. What is the pro-


bability of obtaining (i) all heads, (ii) two heads , (Iii) one head, (iv)
at
least one head, (v) at least two heads , (vi) all tails ?
Solution. There are 2 3 or 8 possible cases, viz, NH!!, T-IHT, 117W,
TI!!!, I!TT, THT. 7 T11 and TTT (the three letters in each case denoting
the results on the 1st, 2nd and 3rd coins respectively). These are mutually
exclusive, exhaustive and equally likely cases.

P-9 BUSINESS MATHEMATICS

The cases favourable to the events are as follows


Event Favourable cases Number of
favourable cases
A : All heads uHf! 1
B : Two heads uHF, 11TH, Till! 3
C : One head [ITT, TilT, TTH 3
D At least one JuTE, THT, TTII, If HT,
head If Til, Till!, If HI! 7
E : At least two heads HHT, 11T11, THH, HHH 4
F All tails TTT 1
Applying the classical definition of probability, we have
(0 l'(A)P(a]l heads) = 118
(ii) P(B)==P(two hcads)=3/8
F(C)=P(on head)=3/8
(iv) P(D)=P(at least one head)=7/8
(v) P(E)=P(at least two heads)-4/8=12
(vi) P(F)=P(alI tails)= 1/8
Example 4. Two unbiased dice are thrown. Find the probability
at
(a) both the dice show the same number,
(b) the first (lice shows 6.
(c) the total of the numbers on the dice is greater than 8
Solution. Each of the six faces of one die can be associated with
each of the six faces of the other die, so that the total number of equally
likely cases which can arise would be 6x6, i.e., 36. These can be
denoted as
(1, 1) (2, 1) (3, 1) (4, 1) (5, 1) (6, 1)
(1,2) (2,2) (3,2) (4,2) (5,2) (6,2)
(1, 3) (2.3) (3, 3) (4, 3) (5,3) (6,3)
(1,4) (2,4) (3,4) (4,4) (5,4) (6,4)
(1, 5) 2, 5) (3, 5) (4, 5) (5, 5) (6,5)
(1,6) (2,6) (3, 6) (4,6) (5, 6) (6,6)
The expression say (3, 4) means the first die shows 3 and the second
die 4. The total of all possible events is
N=36
(a) The favourable cases are (1, 1), (2, 2), (3, 3), (4, 4), (5, 5) and
(6, 6). Therefore ,n=6
Probability that the two dice show the same number
6 1
36 -W

PROBABILITY P-10
(b) For out of 36 cases, the first die shows 6 in the following
cases : (6, 1), (6, 2), (6, 3), (6, 4), (6, 5) and (6, 6). m=6.
• ,, 6 1
• • Probability that the first die shows 6
36 =
(c) The cases which give a total of more than 8 re (3, 6), (4, 5),
(4, 6), (5, 4), (5, 5), (5, 6), (6, 3), (6. 4) (6, 5) and (6, 6).
m= 10
5
Probability that the total is greater than 8= 10 -
361 -18
Example 5. A bag contains 5 green and 7 red balls. Two balls
are drawn. JV/zat is the probability that one is green and the other red ?
Solution. Total number ofballs=r5 + 7=12
Now, out of 12 balls, 2 can be drawn in ISC ways.
12x11
Exhaustive number of cases= " C ,
2 - = 66
Out of 5 green balls. I green ball can be drawn in C 1 ways and
out of 7 red balls, one red ball can be drawn in C 1 ways. Since each of
the former cases call associated with each of the latter cases, the
total number of favourable cases is 5 C2 x C,=-5 x 7=35.

Required probability= 35
-

Example 6. Five men in a company of 20 are graduates. If 3 men


are picked out of the 20 at random, what is the probability that they are
all graduates ? What is the probability of at least one graduate ?
Solution. There are °C3 possible ways of selecting groups of 3
men out of 20, and these groups are mutually exclusive, exhaustive and
equally likely.
However, a group of 3 men (all graduates) out of 5 can be obtained
in 5 C3 ways. Similarly, a group of no graduate out of remaining 15
can be obtained in 1 C0 ways. Therefore, We number of cases favourable
to the event is C3 x
Hence, the probability that all are graduates
6 C3 x' 5 C0 •
= 2 C
1140 114
In order to find the probability of at Icasi one graduate, it will he
easier to find the probability of the complementary event, viz., that none
is a graduate'.
I5CX5C 455 91
Probability that there is no graduate=
= Fl
the probability that there is at least one graduate
91 137
—1
228 228
-1 1 BUSINESS MATHEMATICS

ADDITION RULE OF PROBABILITY


Statennt. The probability of occurrence of at least one of The
two events A and B is given by
P(A(J 13) =r P(A)+P(J3)—P(AflB)

Proof. Let us suppose that a random experiment results in a


sample space S with N sample points (exhaustive number of outcomes).
Then by definition
n(Aufl)
n(S) - N
where n(ALJB) is the number of outcomes (sample points) favourable to
the event (AUB).

From the above diagram, we get


(n(A)—n(AflB)l-- n(AflB)+ (n(B)—n(4 flB)
N
n('4)+n(B)—n(AflB)

n(A) n(B) n(AflA)


- N ±7
=P(A)+P(B)_P(A (B).
Remarks I.. If the events A and B are mutually exclusive, i.e., if
(A(B)oS, then

P (Afl 8 n(AflB) n()


N N
Thus the probability of happening of any one of the two mutually
disjoint events is equal to the sum of their individual probabilities.
Symbolically,

2. For three events A, B and C, the probability of


of at least one of them is given by occurrence
P (AUBUC)Z. P(A)+P(B)+P(C)_pLAnn)p(flfc)
--P(AflC)fP(4flnc)
PROI3AI3ILITY P-12
Example 7. A card is drawn from a well siru/Jied pack of playing
cards. Find the prohubility that it is either a king or a spade.

Solution. Let A denote the event of drawing a king and B denote


the event of drawing a spade from a pack of cards, Then we have

and P()=2. _

There is only one outcome favourable to the event AflB, viz., king of
spade, Hence F(AflB)-.

or -j- _ =.
Example B. The probability that a student passes an Accountancy
rest is
4- and the probability that he passes both an Accountancy
and Law test is . The probability that he passes at least one test

Is . What is the probability that he passes in the Law lest?

Solution. Let us define the following events


A : The student passes an Accountancy test.
B : The student passes a Law test.
We are given

P(A)=, P (AflB)- and P(AUB)=4.



45

Now P(AUB)=P(A)+p(B)_p(,1flB)

4 14 2 4

Example 9. The probability that a contractor will get a plumbing


contract is 213, and the probability that he will not gel an electric contract
is 519. If the probability of getting at least one contract is 4/5, what is
the probability that he will get both the contracts ?
Solution. Let A and B denote the events that the contractor will
get a 'plumbing'- contract and 'electric' contract respectively. Then we
are given:
P-I 3 SWINESS MATHEMATICS

P(A)_- P()_

P(l3)=1_-P(i)
and P(AUB)Prob. that contractor gets at least one contract
= 4/5
P(A)-P(13)_P(Afl/?)-± [By addition rule of probability]

--+- —P(,I()B)-

Hence the probability that the contractor will get both the contracts
is 14/45.
Example 10. A question paper contains 6 questions of equal value
divided into two sections of three questions each, If each question poses
the same amour( of a'ftIcuhty to Mr. A', an exatninee and he has only 50%
chance of solving it correctly , find the answer to any one of the following
(I) If Mr. X is required to answer only three questions from any
one of the sections, find the proba/,iliry that he will solve all the three
questions.
(ii) If Mr. X is given the option to answer the three questions by
selecting one question out of the two standing at serial number one in the
two sections, one question out of the two standing at serial number two
in the two sections, and one question out of the two standing at serial
number three in the two sections, find the probability that he will solve all
the three questions correctly, [Del/ti Univ., B. Corn. (lions ), 19921
Solution. (i) Mr. X will solve all the three questions correctly',
if he is able to solve
(I) all the questions of the first section and not all the questions
of the second section
(2) all the questions of the second section and not all the questions
of the first section ; or
(3) all the questions of both the sections.
Hence required probability
1 1 'f 1 1 1
T')+( '- ( T )(
7 7 1 15
+f'
(iii) Mr. X will solve a question correctly, if he is able to solve at
least one of the questions standing at the particular serial number in the

PROBABI LITY
P-14

two sections, the probability of which is 1— -

hence required probability


/ 3 27
= 4 I 64

MULTtPLICATION RULE OF PROBABILITY


Statement, The probability of siinulianeoits occurrenc of two events
A and B is given by
P(AflB) P (A). P(B I A); P(A)^()
Or
P(j3flA)7 P(/3) . P(A I B) ; P(13)-^60
where P(B I A) is the conditional probability of happening of B under the
condition that A has already happened and P(A I B) is the conditional
probability of happening of .4 under the condition that fi has already
happened.
Proof. Let A and B be the events associated with the sample space
S of a random experiment with exhaustive number of outcomes (sample
points) N, i.e., n(S) = N Then by definition

P(AflB)=-

For the conditional event A I B (i.e., the happening of A under the


condition that B has happened), the favourable outcomes (sample points)
must be out of the sample points of B In other words, for the event
A I B, the sample space is n(B) and hence

P(A I
---
Similarly, we have
n(BflA)
P(I3 I
A)— n(A)
Rewriting (), we get
n(A) n(A fl B)
P(AflB)=—x-
n(A)
=P(A) . P(B I A) [From ..()]
Also
n(B) n(Aflfi)
>
n(S) n(B)
=P(fl). P(A I B)
Remarks. 1. Multiplication Rule for Independent Events. If
A and B are independent so that the probability of occurrence or flOfl•
P-15 BUSINESS MATHEMATICS
occurrence of A is not affected by the occurrence or non-occurrence of B,
we have
P(A I B)=P(A) and P(B I A)=P(B)
Hence substituting in ('), we get
P(AflB)=P(A) PB)
Hence the probability of simultaneous happening of two independeni
events is equal to the product of their individual probabilities,
2. The multiplication rule of probability can be extended to more
than two events. Thus, for three events A, B and C, we have
P(AflBflC)-P(A) P(B I A) F (C I AflB)
3. If events A and B are independent then the complementary events
A and fl are also independent
Proof. We know
P(AU/3)+P(AUB)—1
P(AuB)+ P(A)=1 (By Dc-Morgan's Law)
P(AflB)1P(AUB)
I —[P(4)- P(B)—P(A flJJ)
= I —P(A)--- P(B)+ P(A)P(B)
(. A and B are independent events)

=1l—F(A)j[1 —P(B)J-pP()
A and B are independent events,
4.
P (happening of at least one of the events A, B and C)
I —P(none of the events A, B. C happens)
or equivalently,
P(AUBU C)1 —P(A fl Bn5
=1-.-- P(A). P(. P(C)
(If A, B and C are independent events).
Example if. ,4 bag containts 8 red and 5 white balls. Two
successive drawings of 3 balls are made such that (i) balls are replaced
before the second trial, (ii) the balls are not replaced before the second
trial. Find the probability that the first drawing will give 3 white and the
second 3 red balls.
Solution Let A denote the event of drawing 3 white balls in thç
first draw and B denote the event of drawing 3 red balls in the second
draw. Then we have to find the probability P(AflB).
(i) Draws with replacement If
the balls drawn in the first draw are
replaced back in the hag before the 2nd draw then the event A and
B
are independent and the required probability is given (by the multi-
plication rule of probability) by the expression
PROIIAnILTT y p16

P(,1n8)-P(A) P(B) ...y)


X 13C

(u) Draws without replacement . If the balls drawn are not replaced
back before the second draw, then the cvents A and B are not indepen-
dent and the required probability is given by
P(Aflfl)=P(4) . P(B I A)
As discussed hi part (i),

Now, if the 3 white halls which werc drawn in the first draw are not
replaced hack, there are 13 -3 10 balls left in the hag and P(B A) is
the conditional probabilit y of drawing 3 red balls from the bag containing
10 balls out of which 2 are wh lie and 8 are red.
c3
Hence /'(B i A) =ri
Substituting in ()7 we get
ic 8c
P(A fl13 <

Example 12, i-ct A and B he the two possible outcomes of an


experiment and suppose
P(A)=O 4, J'(/tuJJ)=_-0*7 and
(i) For what choice of p are A and B muival/y exclusive )
(ii) For isliai choice of p are A wid B independent ?
Solutjn = (1) We have
I(-i UB)r P(4) -f- P(13)—P(AflB)
I'(AflB)=P(A)+P(B)_p(Au/I)

P-03
If A and B are mutually exclusive, then
P(AflJi):O 41 p-03=o PO3
(ii) A and B are independent if
F( A CtB)=PtA) P(B)
P—O3=O4>p
06p=(y3
03
P06-05
Example 13. The probability that a management ir(ij, y
ee will re,,ioj,j
With a company is 0 60. The probainlay that on employee earns more 111an
Rs. 10,000 per year is 0 50. The probability that an employee is a
management trainee who reniaiiied nih the company or who earns more
than Rs. 10,000 per .vear is 070. What is the probability that an employee
p-17 BUSINESS MATHEMATICS

earn more than Rs. 10000 per year given that lie is a management trainee
who stayed with the company ?
Solution. Let US (kline the events
A : A management trainee will remain with the company.
B : Au employee who earns more than Rs. 10,000/-

Then we are given


P(A) '=ft60 and P(R)= 050
Also we are given
P(A managmen1 trainee remains with the company or earns more
than Rs. 10,000 per year) 010
P(AUB) = 010
PtA-P(8—P(AflB)=070
l'(AnB)=P(4)+P(8)-07
=-06-t 05-07— 04
Required probability is
PB I A P(AflfJ) 04 2
- P(A) 06 3

Extnp)e 14. The oddc against student X slrilig a Business


Statistics problem are 8 : 0, and odds in Ji'our of student Y solving the
some problem are 14 16.
(I) What Is the prob a bilit y that neither solves the problem, if they
bout try, independently of each oilier ?
(ii) What is the chance that the problem will be salted.
Solution. Let A denote the event that student K solves the pro-
blem and B denote the event that the student Y so ves the problem.
Then we are given

14 7 16 8

(I) The probability that neither K nor Y solves the problem is


given by
P(A fl J3)=P(A)XP(B)
[Since A and B are independent A and are independent]
4 8 32

PRO BADILIIY -l8


(Ii) The problem will be solved if at least one of the students X and
Y solves the problem. Hence the required probability is given by
I'(A U B)Probability that at least one of X and Y solves the
problem
= I '--Probability that none solves the problem
32 73
1 - P( A () B) I - -
105 105
Ex tvpe 15. It is 8 5 against a husii,qrd who is
55-year-olc1 living
fill he is 75 and 4 3 against his wf i/ia is non' 4$, living till she is 68,
Find the probability that (i the couple w ill be alive 20 years hc,ire,
an0f
(ii) at least one of thciii will he alive 20 years hence

So1utiji. Let A denote the event that husband will he alive 20


years hence and I? denote the event that wife will be alive 20 years hence.
Then we are given that

J'(A)=i
13 13
3 3 4
> P('=l

(i) The event that catiple is alive 20 years h


A fl B, ence is given by

Required prohability-P'(AflB)
=P(A) x P(B)
(By multiplication rule of probability, since A and 8
are independent
and consequently A and B are independent).
5 3 15
uj- x-=-
(ii) The event that at least one of the persons
.1 and B is alive 20
years hence is given by AUB.
Required probability=P(AUB)
= I —P (None of and B is alive 20 years
hence)
I —P(A)fljj)
r 1_P(A) P(B)

8 4 59
X

Exiiple 16. A candidate is selected for interview for three posts.


For the flFit post there are 3 candidates for the second there are 4 and for
the third there are 2. What are the chances of his getting at least one post 7
p-19 BUSINESS MATHEMATICS

Solution. Let iL B and C denote tiic events that the candidate is


selected for the first, second and third l)OSt respectively. Since the selection
of each candidate is equally likely, we have

P(A)=-

The probability that the candidate is selected for at least one post is
given Oy
P(AUBUC)= i-P(Anhihd)
= l—P(A). P(B). P(C)
[Since the events A, B and C are independent]
2 3 1 3
-l- X X24
Example 12 A piece of equipment will function only when all the
three components A, 13 and C are working. The probability of A failing
during one _vear is 015, that of B failing is 1) 0.5 and that of C failing is
010. What is the probability that the equipment will fail before the end
of the year 2
Solution. Let us define the events
A1 Component A fails
A 2 : Component B fails
A 3 : Component C cans
We are given
P(A 1 )=015, P(A 2)005, P(A3)=010
Probability that equipment will fail before the end of the year is
given by
P(A1UA2U143)=l —P( Zn2n3
= I —/'()1'(73)P(5)
=l--(l-015)x(1--005)x(1_0'10)
= I —0'72675=027325
Example 18. A bag contains 5 white and 3 black balls and four
are successively drawn out and not replaced What is the probability that
they are alternatively of swne colours 2
Solution The required event can materialise in the f6owing
mutually exclusive ways
PR)8ABtLi IN'
p-20

(i) The balls are white, black, white and black in the first, second,
third and fourth draw respectively.
(ii) The halls are black, white, black and white in the first, second,
third and fourth draw respectively.
Hence by addition rule, the required probability 'p ' is given by
p=P(i)+P(ii) .
Let A, B, C and 1) denote the event of drawing a white, black, white and
black in the first, second, third and fourth draw respectively. Since the
balls drawn arc not replaced before the next draw, the constitution of the
bag ill the four draws is respectively

pj
1st draw 2nd draw
W3i 2B

3rd draw
HH 4th draw
P(f)=P(AflflflCflD)
=P(A). l'(li I ..l). P(C I 'lflB). P(D 1fl8flC)
5 3 4 2 1
_X T X 6 X5-l4

3 5 2 4
Si p((i) A
< 6x
Substituting in ('), the required probability is
I I I
14 7
Example 19. A hag contains 5 red and 3 black balls and the second
one 4 red and 5 black balls. One of these Is selected at random and a
draw of two la/is is made from it. What Is the probability that one of
them is re(l and the oilier black 7
Solution. Two balls (one red and one black) can be obtained in the
following mutually exclusive ways
A : when bag I is selected and two balls are drawn from it.
B: when bag 11 is selected and two balls are drawn from it.
Hence by the addition rule, the required probability is given by
pP(A)+P(R)
But A is itself a compound event consisting of (1) the selection 01
bag I, with probability t, and (ii) the drawing of two balls, one red and
C1 x C-
other black from it, with probability .
Hence by the multiplication rule, we have

P-21 BUSINESS MATHEMATICS

P(A)=(Probability of selection of hag I) x (Probability of drawing


one red and one black ball assuming that bag I is selected)
I 5C1xC1 1 15 15
TX
8C2
1
Similarly, P(B)—_- x 4C,xC 1 20 5
6
T
Hence the required probability is
15 5 275

Example 20. The odds that a look on Business Mathematics will


be favourably reviewed by 3 indepen/ent critics' are 3 to 2, 4 to 3 and 2
o 3 respectively. What is the probability that , of the three reviews
(a) all will be favourable
(b) majority of the reviews will be favouraHe
(c) exactly one review will be favourable and
(d) exactly two reviews will be favourable,
(e) at least one of the reviews will be favourable
Solution. Let A, B and C denote respectively the events that
the book is favourably reviewed by first, second and third critic respec-
t i vely. Then we are given that

P(A) = ---, P(B)=_ , and

E(A) --, P(B) and P(C)


(1) P(all the three reviews will be favourable
- P(,nBnC)
= P(A).P(B).P(C)
[: A, B and C are independent]
3 4 2 24

(ii) P(majoriiy, i.e., at least 2 reviews will be favout able)


=P(AflBfl )+ P(A nn c )+ p(nBnc)+ P(AflBfl1C)
P(4)P(/3)P(+p(4)pj)p(C)+p()p(B)j)(C)
-- P(A)P(8)P(C)
('. A, B and C are independent)
3 4 3 3 3 2 2 4 2
=Tx
3 4 2 94
+-- xTx

PROBARILITY P-22
(iii) The probability that exactly one review will he favourable is
given by
I'(Afl B7 C)± P(Afl13fl C)+ P(AflBPflC)
= P(A)P(B)P(C) f J'(A)P( 13)P(C) + P(A)P(B)P(C)
3 3 32 4 3 2 3 2 63
= 5 X 7 I X7K X7X5
+
(h) Similarly, the probability that exactly two reviews will be
favourable is given by
p(AflBflC)+p(AflflflC)+ P(AflJ3flC)
- P(A)P(B)P(C) + P(A)P(B)P((.') 4- P(A)P(B)P(C)
3 4 3 3 3 2 2 4 2 105
+

(lv) The probability that at least one of the reviews will be favour-
able is given by
P(AUBU C) = l—l'(AflBfl C)
= I __P(A)l'(B)P(C)
2 3 3 157
=1- 7 175

BAYES' RULE
One of the important applications of the conditional probability
is in the computation of unknown probabilities, on the basis of the infor-
mation supplied by the experiment or past records. For example, suppose
we have two boxes containing defective and non-defective items. One item
is picked at random from either one of the boxes and is found defective,
and now we might like to know the probability that it came from Box I or
Box 2. These probabilities are computed by Bayes' Rule, named so after
the British Mathematician Thomas Bayes who propounded it in 1763.
Quite often the businessman has the extra information in a partt-
cular event, either through a personal belief or from the past history of
the event Probabilities assigned oil basis of personal experience,
before observing the outcomes of the experiment, are called prior pro-
babilities. For example, probabilities assigned to past sales records, to
past number of defectives produced by a machine. are examples of prior
probabilities When the probabilities are revised with the use of Bayes'
rule, they are called posterior probabilities. Bayes' rule is very useful
in solving practical business problems in the light of additional infor-
mation to arrive at valid decisions in the face of uncertainties,
Statement. If an event B can only occur in conjunction with one one
of the n mutually exclusive and exhaustive events A 1 , A,.,4,, A. and if B
actually happens, then the probability that it was preceded by the parti-

P-23
BUSINESS MATJJIMAT1C
cu/ar event A, (1= 1, 2, n) Is give,, by
P(BflA,) P(A,) P(B I A,)
-
P(B 1 A,) P(A, P(B I A,)

Proof. Since the event ii


the mutually exclusive andexhatistjve canevents
occur in combination with any of
A 1 , A t ,..., A, we have
B(BflA,) U (Bn4 2)
U ... U(BflA)
where B fl,, BflA ,.., Bfl4 ,
are all disjoint (mutually exclusive)
events. Hence, by addition rule of probability, we have
I>(B) P (.IJfl,1 1 ) +P(Bfl A
2 ) .. . - /'( BflA)
P(A,) P(B I A)+ P(A) J(B
I 4) + ... -- P(A,) P(/i I A,)

/'(A,) P(B I A,)

For any particular event A,, the conditional probability


givcji by P(A, 13) is
P(A(flfl)p(B) P(A, I B)
' P(A, J fl)Ll).

I A,)
tl

P(A) P(B 1 A,)

Which iS the Bay' rule For obtaining the


co nditional Probabilities.
Remark. The probabilities 1'(A1),
P(A 2 ), ..., J-'(A,,) which are
already given or known before conductin
priori or prior probabliti
probabi g the experiment are termed as a
P(A 2 B), •, P(A, B) lities. The conditional probabilities P(A, B),
I which are computed after conduct'
ment, viz., occurrence of A, are called a ng the experi-
probabilities Posteriori or posterior
Example 21. Two sets of candidates are co
on the Board of D mpeting for the positions
and second sets w irectors of a company. The p robabilities that the first
ill win are 06 and 04 respectively . If the fi
the probability of introducing a new product is 08 rst set wins,
probablfi,, , and th e corresponding
pro duc, will if
hethe second set wins is 03. Whnt j' th probability that the
introduce d?
Let
sets of ca ndidates A , A denote the events that the first and second
product' is introduced. W in respectively. 1_ct B
denote the event that 'new

PROBABILITY P-24
We are given
P(A 1 )-O'6, P(,4,) ==
P(Ti I A,) =0 , 8 = Probability that 'new product' will be introduced
given that first set wins.
P(B I A2)-03
The event B can materialise in the following mutually exclusive
ways:
(1) First set wins and the new product is introduced, i.e., A 1 fl ii
happens
(ii) Second set wins and the new product is introduced, i.e., A fl B
happens. Thus
B=(APB) U (Afli3),
where A 1 flB and A 2 fl13 are disjoint.
Hence using addition rule of probability, we have
P(B)= P(A1fl13)+ P(A2fl/3)
=P(A 1 ) I'(B I A)+P(A 2) .P(B
=0r6x 0'8+04x03

Example 22, Suppose that a product is produceJ in three factories,


A, B and C. 11 Is known that factory A produces twice as many items aS
factory B, and that factories B and C produce the same number of items.
Assume that it is known that 2 per cent of the items produced by each of
the factories A and C are defective while 4 per cent of those manufactured
by factory B are defective . All the items produced in the three factories
are stocked, and an item of product Is selected at random. What is the
probability that this Item is defective ?
Solution. Let the number of item-, produced by each of factories
B and C be n. Then the number of items produced by the factory A is
2n. Let A, A 2 and A 3 denote the events that the item is produced by
factory A, B and C respectively and let £ be the event of the iteni being
defective. Then we have
2n 2n
PA' 05
2n+n+n4n
=025

P(A2) = = -- = 025
p(E I A1 )P(E I A 3)=002 and P(E I A,)=0-04 (Given)
The probability that an item selected at random from the stock 19
defective is given by
P(E)==Pf(EflA 1 ) U (EflA) U (EflA3)J

P-25 BUSINESS MATHEMATICS

=F(EflA 1 ) -I- P(Efl /1 2 ) ± P(EflA2)


[By addition rule of probability]
: P(A 1 ) P(E I A 1 )-f-P( A 2 ) P(E 1 As) -lF(A 0) P(E I A)
05 x 002+ 0 • 25 x 004 +025 x 002
=0025.
Example 23. A company has two plants to manufacture scooters
Plant I manufactures 70% oJ (lie scooters and Plant ii manufac tu r e s 30%.
At plant 1, 8001 of scooters are rated standard quality and at plant 1190%
of scooters are rated standard quality. A scooter Is picked up at random and
isjbund to he of standard quality. What is the chance that It has come
from plant I?
Solution. Let us define the following events
Scooter is manufactured by plant I
A2 : Scooter is mmufactured by plant II
13 : Scooter is rated as standard quality.
Then we are given
P(A 1 )=070, F(A2)=030,
I
P(B A 1 )=080, P(B I 42)=090
Using Bayes' rule, required probability is
P(A1) P(B I A)
P(B I A 1 ) P (4) P(B I A2)
070x080 056 56
070X080-I030x090 313
Example 24. In an automobile factory, certain parts are to be fixed
to the chasis in a section before it moves into another section. on a given
day, one of the three persons A, B and C carries out this task. A has 45%,
B has 3501 and C has 20% chance of doing Ii. The probabilities that A, B
and C will take more than the elicited time are 1116, 1/10 and 1120 respec-
tively, If It is found that none of them has taken more time, what is the
probability that A has taken more time?
[Delhi Uni BCovn. (lions.) 19921
Solution. Let E1 , E2 E 3 denote the events of carrying out the task by
A, B and C respectively. Let 11 denote the event of taking more time.
Then we have
P(E1 )=-045, P(E2 )=0'35, F(E)= 020
P(H I E)rr 16'
-_ P(H E)=-.L
2
, P(H I Es) — 1
10
The required probability
P(EI). P(H I E1)
P(E1 ) . P(H I E3+P(E 2 ) PU! I E2) -4 P k E,) . p(H I E)


PROBABILITY P-26

-
045x 1-- -l-035x . +020x
10 20
5
=11
Example 25. In a bolt factory, mac/lines A, B and C manufacture
respectively 25% 35% and 40% of the total, of their output 5, 4, 2 per
cents are defective bolts, A bolt is drawn at random from the product and
is found to be defcctie. What are the probabilities that it was manufactured
by mac/lines A, B and C ?
Solution. Let us define the events
/1 1 Bolt is manufactured by machine A.
A 2 =Bolt is manufactured by machine B.
A 3 = Bolt is manufactured by machine C.
The data of the problem give the following probabilities
P(A 1 )=0'25, P(A 2 )=035, P(A,)= 040
P(B I A 1 )=0'OS, P(B I A 2 ) = 004, P(B I A S ) = O02
P(BflA 1 )=J'(A 1 ) PB I A 1 )=025 x 0'05=00125
P(BnA,) = 0'35 x 004= 00140
P(Bfl1 3 )= 040 x 002=00080
Hence the probability that a defective bolt chosen at random is
manufactured by factory A is given by Bayes' rule as
P'A fl \_ P(A))P(B_A1)
I
' P(A 1 )P(B I A 1 )+I'(A 2 ) P(BA 2 )f-P(A 2 ) P(BpA)

00125 0'0125 25
0'0125-I-00140+00080 0034569
Similarly, we get
00140 28
P(A 2 1 B )-5=-
00080 16
P(A

The above information concerning various probabilities may be


summarized in the following table
Event Prior Conditional Joint Posterior
Probability Probability Probability Probability

A 1 025 25
0'05 00125
69
28
035 004 00140
69
16
Al 040 002 00080
69
Total l'OO 00345 1'OO
P-27 BUSINESS MATHItMA'I'JCS
Important Remark. P(A 3 ) is greatest, on the basis of 'a prior'
probabilities alone we are likely to conclude that a defective bolt drawn at
random from the product is manufactured by machine C. After using the
additional information we obtain the 'posterior' probabilities which give
P(A2 B) as maximum. Thus, we shall now say that it is probable that
the defective bolt has been manufactured by machine B, a result which is
different from the earlier conclusion. However, latter conclusion is a
much valid conclusion as it is based on the entire, information at our
disposal. Thus, Bayes rule provides a very powerful tool in improving the
quality of probability and this helps the management executive in arriving
at valid decisions in the face of uncertainty. Thus, the additional informa-
tion reduces the importance of the prior probabilities. The only requirement
for the use of Bayesian Rule is that all the hypotheses under consideration
must be valid and that none is assigned 'a prior' probability 0 or 1.

EXERCISES
I. (a) Define random experiment, trial and event.
(b) What do you understand b y (i) equally likely, (ii) mutually
exclusive and (iii) independent events.
(c) Define independent and mutually exclusive events.. Can two
events he mutually exclusive and indc3endcnt simultaneously ? Support
your answer with an example.
2. Discuss the different schools of thought on the interpretation
of probability flow does each school define probability ?
3. Explain tile meaning and illustrate byau example how probabi-
lity can be calculated in the following cases
(i) Mutually exclusive events, (Ii) Dependent events.
(iii) Independent events.
4. Differentiate the following pairs of concepts
(i) Mutually exclusive events and overlapping events.
(ii) Simple events and composite events.
(iii) Mutually exclusive events and independent events.
5. Define independent and mutually exclusive events. Can the
two events be mutually exclusive and independent simultaneously.
Support your answer with examples.
6. Explain with examples the rules of Addition and Multiplication
in theory of probability.
7. A card is drawn from a pack of cards. Find the probability
that it is
(i) queen, (ii) queen of diamond or heart,
(Iii) not a diamond,
() a ten, a jack, a queen or a king.
[An s. (1) 1/13, (ii) 1/25, (iii) 3/4, (iv) 4/131

PROBABILITY

(a) Given the following data


0-10 10--20 20-30 30- 4() 40-50
J: 2 8 13 7 5
What is the probability that an item chosen at random from the
data falls between 30 and 40? (Ans. 1/5)
(b) Given the following probabilities concerning the number of
accounting personnel that will be needed in a company during the next
two years.
No of
Accountants: <100 100-199 200--299 300-399 400-499 50599
Probability : 010 015 030 030 0 10 005
(1) What is the probability that the company will need 400 or
more additional accountants during the next two years.
(ii) What is the probability that the company wili need at least
200 but not more than 399 additional Accountants ?
[Atis. (I) 0 - 1040 - 05, (ii) 0 - 304-0 - 301
9. The following data shows the length of life of wholesale grocers
in a particular city
Length of Life Percentage of
(years) wholesalers
0-- 5 65
5—JO 16
10-15 9
15-25 5
23 and over 5
Total [00

(i) During the period studied, what is the probability that an


entrant to this business will fail within five years ?
(ii) That he will survive at least 25 years ?
[Ans. (i) 065, (ii) 095)
10. From 30 tickets marked with the first 30 numerals, one is drawn
at random. Find the chance that,
(i) it is a multiple of 5 or of 7, (ii) it is a multiple of 3 or of 7.

[
A, (ii)

11. A number is chosen from each of the two sets


1 ,2,3,4,5,6,7,8,9; 1, 2, 3, 4, 5, 6, 7, 8, 9.

P-29
BUSINESS MATHEMATICS
If p1 is the probability that the sum of the two numbers be 10 and
P 2 the probability that their sum be 8, find p1 {-p2 ,
[Ans. 16/181
V. From a pack of 52 cards, 2 are drawn at random. Find the
chance that one is a king and the other a queen.
r ____
Ans. -_____
L
13. A bag Contains 3 red, 4 white and 5 black balls. Three balls
are taken From the bag. Find the probability that
(/) all are black,
(ii) all are of different colours.

[Ans. (I) ' (Ii)


- 12V3-
14. Two cubical dice are tossed.
following events Find the probabilities of the

The sum of numbers


(I) Divisible by three, (II) Less than 7,
(iii) Not less than 7 (or at least 7 or more than 6).
[Ans. (1) 1/3, (Ii) 15136, (Iii) 21/361
15. An urn contains 5 white, 3 black and 6 red balls, 3 balls are
drawn at random. Find the probability that
(i)
two of the balls drawn are white, (ii) one of each colour,
(iii) none is black, and (iv)
at least one is white.
Ans. (i) - ,. 5x3x6 .., "C5 .
--_--- . (u) •HC8 (xii) , (w) I -
C5
L
16 There are 3 economists, 4 engineers. 2 statisticians and I
doctor. A committee of 4 from among them is to be formed. Find the
probability that
the committee
(1) consists of one of each kind ; (ii)
has at least one economist
(ii) has the doctor as a member and three others.
r
L '.24..
210'
84
210
35 .
(Iii)
17.
fivc . paise A bag contains 12 rupee coins, 7 fifty paise coins and 4 twenty-
coins. Find the probability of drawing
(1) a rupee coin ; (ii) three rupee coins, and
(iii) three coins, one of each type.

18. The Federal Match Company has forty female employees and
sixty male employees If two employees are selected at random, what is
the probability that
PROBABILITY p.30

(1) both will be males ? (ii) both will be females '1


(iii) there will be one of each sex ?
Since the three events are collectively exhaustive and mutually
exclusive, what is the sum of the three probabilities 1 [Ans. One]

19. In a box there are 4 granite stones, S sand stones and 6 bricks
of identical size and shape. Out of them 3 are chosen at random. Find
the chance that
(1) They all belong to different varieties.
(ii) They all belong to the same variety.
(iii) They are all granite stones.
20. If the probability is 0 30 that a Management Accountant's
job applicant has a post-graduate degree, 070 that he has had some
work experience as a Chief Financial Accountant, and 0-20 that he has
both. Out of 300 applicants, approximately what number would have
either a post graduate degree or some professional work experience ?
[Ans. 240]
21. Find the probability of getting 6 at least once in two tosses
of a die.
(hint. Using Addition rule, the required probability is
J'(A or B)=P(A)-j-j'(B)—p(A and B)=
--_-_-
22. (a) A chartered Accountant applies for a job iii two firms X and
Y. 1-Ic estimates that the probability of his being selected in hrni X is
07, and being rejected at Y is 05 and the probability of at least one of
his applications being rejected is 06. What is the probability that he
will be selected in one of the firms ?
[Hint. Let A and B denote the events of his being selected in firms
X and Y respectively.
P(A)=07, P(B) nr 05, P(Aor B)=0 6
The required probability that he will be selected in one of the
firms is obtained by using addition rule as follows
P(A or B)=P(jl)±p(B)—.P(A and B)
Also we know
P(A and B)=l—P(A or
Hence P(A or B) r 07+0 5-0'4=0-8j
23. Two vacancies exist at the junior executive level of a certain com-
pany. Twenty people, fourteen men and six women, are eligible and
equally qualified. The company has decided to draw two names at
random from the list of eligibles. What is the probability that
(a) both positions will be filled by women ?
P-31 BUSINESS MATHtMATICS

(b) at least one of the position will be lifled by women ?


(c) neither of the position will be filled by women ?I'C

[ Ans. (a) (b) 1 (c)

24. Sixty per cent of the employees of the ABC Corporation are
college graduates. Of these, ten per cent are in sales. Of the employees
who did not graduate from college, eighty per cent are in sales.
(i) What is the probability that an employee selected at random is
in sales ?
(ii) What is the probability that all selected at random is
neither in sales nor a college graduate ?
(Ans. (a) 033, (b) 0'081
2. A small insurance company has written theft insurance for
two different businesses In any one year, the probability that business
A IS burglarized is 001. In any one year, the probability that business
B is burglarized is 015, (Assume these are independent events.) Find
the probability that
(a) both will be burglarized this year.
(h) neither will be burglarized this year.
(c) exactly one will be burglarized this year.

26. The probability that a person stopping at a gas station Will ask
to have his tyrcs checked is 0'12, the probability that he will ask to have
his oil checked is 029 and the probability that he will ask to have them
both checked is 007.
(i) What is the P,obabilfly that a person stopping at th gas station
will have either his tyres or his oil checked ?
(ii) What is the probability that a person who has his tyres checked
will also have his oil checked ?
(iii)
What is the probability that a person who has his oil checked
will also have his tyres checked ?
[Ans. (1) 034, (ii) 0 58, (iii) 0241
27. A card is drawii from a full pack of cards. What is the pro-
bability of drawing a "black" king (either spade or club) given that the
card drawn was "face" card (jack, queen or king) ?
28. A bag contains 6 white and 9 black balls. Two dra¼ ings of 4
balls (in each draw) are made in such a way that
(i) the balls are replaced before the second trial.
(ii) the balls are not replaced before the second trial.
Find the probability that first drawings will give 4 white and the
second 4 black balls in each case.

[Ans. (ii)
15 C4 15C4 15C4 IC41
PROBARILITY
P-32
29. If the probability that A'
project will have an economic life ot'
20 years is 0'7 and the probability that 13' project will have anonoj
cc
life of 20 years is 05. What is the probability that both will have art
ec onomic life of 20 years ?
(Axis. 07 x 05J
30 ,\ salesman has a ] 0 per cent chance of making a sale to each
Customer. The behaviour of successive customers is assumed to he
independent. If two customers A and B
enter, what is the probability
that the salesman will make a sale to A or 13?
(Axis. 0' 9.]
31. It is known that bolts produced by a certain process arc too
large 10 per cent of the time and are too small 5 per cent of the time.
If a prospective buyer selects a bolt at random from a lot of 500 such
bolts, what is the probability that it will be neither too tong nor too
short ?

32, A Problem in Statistics is given to three students


A, 1] and C
whose chances of solving it are . and - respectively. Firal the pro-
bab,liy that the problem will he solved by at least one of them.
[An, 3/5]
33. The probabilities that three drivers will be able to drive home
safely after drinking are, and respectively. If they set out to drive
home after it party, what is the probability that all three drivers will have
accident ? What is the probability that at least one driver wnll drive
home safely ?

34. (a) Find the probability of throwing 6 at least once in six


throws, with a single die.
[Aus. I --(5/6)11
(b)
Suppose tso six-faced dice are thrown I() times. What is the
probability of getting a double six ill least one of the throws "
[Aim. I _(35i36)boj
35. In the milk section of a self-service market there are 150 quarts,
100 of which are frcsli and 50 are a day old.

(1) If two quarts are selected, hat is the probability that both will
be fresh ?

(ii) Suppose two quarts are selected after 50 quarts have been
removed from the selection. What is the probability that both will he
Fresh ?.

(iii) What is the conditional probability that both will be fresh, gi veil
that at least one of thein is fresh,

36. An urn .4 Contains 2 white and 4 black halls, Another


L11-1) /1
contains 5 white and 7 black halls. A ball is transferred from urn
the urn B. Then it ball is drawn ficin the urn .1 to
13. Find the probability
that it will be white.

37. A hag contains 5 hre uid 3


black balls. Another hag con-
tains 4 white and 5 black halis. From any one of these bags single draw
P-33 BUSINESS MATHEMA'tICS
of two halls is made. Find the probability that one of them would be
white and another black ball.
38. An urn contains 10 white and 3 red balls. Another urn con-
tains 3 white and 5 red balls. Two balls are transferred from the first
urn and placed in the second, and then one ball is taken from the latter.
What is the probability that it is a white ball ?
39. There are two groups of subjects, one of which consists of 5
science subjects and 3 engineering subjects and the other consists of 3
science subjects and 5 engineering subjects. An unbiased die is cast. If
number 3 or number 5 turns up, a subject is selected at randout
from the first group. Otherwise, a subject is selected at random
from the second group. Find the probability that an engineering subiect
is selected ultimately.
F 325
LAn5.
RI )X
44), An urn contains 7 red marbles and 3 white marbles. Three
marbles are drawn from the urn, one after the other, without replacement.
Find the probability that the first two are red and the third is white.
41. One shot is fired from each of the three guns. E1 , E2 , E3 denote
the events that the target is hit by the first, second and third gun respec-
tively. If I'(E,) =0'S, P(E 2)= 06 and P(E3 )= 08 and E1 , E2 , E 3 are
indcpcndent events, find the probability that ( a ) exactly one hit is regis-
tered. (b) at least two hits are registered. [Ans. ( a ) 026 (b) 0701
42. A certain part can he defective because it has one or more out
of three possible defects insufficient tensile strength, a burr, or a dia-
meter outside tolerance limits. In a lot of 1000 pieces it is known that
120 have a tensile strength defect.
80 have a burr.
60 have an unacceptable diameter.
22 have tensile strength and burr defects.
16 have tensile strength and diameter defects.
20 have burr and diameter defects.
8 have all three defects.
If a piece is drawn at random from the lot, what is the probability
that the piece
(a) is not defective ?
(b) has at least one defect, and
(C) has exactly two defects
43. An investment firm purchases 3 stocks for one-week trading
purposes. It assesses the probability that the stocks will increase in value
over the week as 0'8, 07 and 0'6 respectively. What is the chance that
(1) all three stocks will increase, and (ii) at least 2 stocks will increase ?
PROBABILITY p34
[Assume that the movements of these stocks are independent
Also find the probability that : (iii) Exactly one stock will increase
in value, (iv) Exactly two stocks will increase in value and (v) At least one
of the stocks will increase in value.
{Hint. Let A, B and C denote respectively the events that 1st, 2nd
and 3rd stocks increase in value. Then we are given that
P (A)= 08, P(B)=07 and P(C)06
P(A)=02, P(U)=03 and F(0=-04
(1) The probability that all the three stocks will increase in value is
P(Afl8flC) P(A)P(I?)p(C)
[. Movements of the stocks are independent]
(ii) The event that at least two of the stocks increase in value can
materialise in the following mutually exclusive ways
(a) Afl B flC happens, (h) A flflC happens,
(c) A flBfl C happens, and (d) Afl 13flC happens.
Hence by the addition rule the required probability is given by
P( A fl B fl C )+ p (AflflC) P(4fl13flC)+P(,lfl131C)
P(A)P(B)F(C) + P(A)P(B)P(C) P(A)J>(B)p(C) H- P(A)P( B)P(C)
('.• A, ii, C are ide penden t]
(iii) Arguing as in case (ii) the probability that exactly one stock
will increase in value isgiven by
P(A)I'(B)p(C) +P(A)P(B)P(C)+p(A)p(B)p(C)
(: Movements of stocks are independent]
(iv) Similarly, the probability that exactly two stocks will increase in
value is given by
P(AflBfl C )+ P ( A f
l Bf
l C)+ P(Aflhj'flC)
= P(A)P(B)P() + P(A)P)P(C)+ P(A)P(B)P(C)
(v) The probability that at least one of the stocks will increase in
value is given by
P(AUBUC) l—P(Afl B fl I
44. Ina multiple choice examination there are 20 questions. Each
question has 4 alternative answers following it and the student must
select one correct answer. 4 marks are given for the correct answer
and I mark is deducted for every wron g answer. A student must secure
at least 50% of maximum possible marks to pass the examination.
Suppose a student has not studied at all so that he decides to select the
answers tothe question oil random basis. What is the probability that
he will pass in the examination ?
P-35 BUSINESS MA1tIEMAT1CS

45. A speaks truth 4 out of 5 times. He throws a die and reports


that there was a six. What is the chance that actually there was a six ?

[hint. P(AflB) -
5 30 30
4/30 4
Required probability -

46, ( a ) Iii 1992 there will be three candidates for the position of
principal Dr. Sioghal, Mr. Mehra and Dr. Chatierji whose chances of
getting appointment are in the proportion 4 2 3 respectively. The
prohabdity that Dr. Singhal if selected will abolish co-education in the
college is 0 3. The probability of Mr. Mehra and Dr. Chattcrji doing
the same are repectivclv 0 5 and 08. What is the probability that co-
education will be abolislied froin the college in 1992 7 [Ans. 23/451
(b) Suppose that one of three men, a politician, a businessman,
and an educator will be appointed as the vice-chancellor of a university.
The respective probabilities of their appointments are 050, 030, 020.
The probahitties that research activities will be promoted by these People if
they are appointed are 0 30. 0'7() and 0'80 respectively. What is the
probability that research will be promoted by the new vice-chancellor 7
[An. 052)
47. Electric light bulbs are manufactured at two plants. The first
plant furnished 70% and second 30% of all required production of
bulbs. At the lust plant among every 100 bulbs, 83 are on the average
standar(j , whereas only 63 per hundred are standard at the second plant.
What is the probability that a bulb chosen at random is manufactured at
the second plant, given that the bull) is standard. (Ans. 0 245
4. Suppose that there is a chance fora newly constructed house
to collapse vhcther the design is faulty or not. The chance that the
design is faulty is 20 % . The chance that the house collapses if the design
is faulty is 9S 11/0 and otherwise it is 25. It is seen that the house collapcd.
What is the probability that it is due to faulty design 7
[l'4uit. We are given
f)(4 ) 0'2 and I '( 4 2 )-08 ; P(B I Aa)0'98 and l'(B I A2) =0-25.
Using Bayes' rule, we have
P(A1) . P(B
P(A 1 I P(A ). P(/3 I A)-i-P(A 2 )P(B I A2)
(0r2) (098)
(0-2) (0'98)+(0'8) (05)]
IV The president of a company must decide which of two actions
to take, say whether to rent or buy expensive machinery. His vice-
president is likely to make a faulty analysis and thus recommend the
wrong decision with probability 005. The president hires two consultants,
who separately study the problem and make their recommendations.
PROIJAB)LIIY
p-36

After watching them at work, the president estimates that one consultant
is likely to recommend the wrong decision with probability 005, the other
with probability 0A0. He decides to take the action recommended by a
majority of the three reports be receives. What is the probability that
lie :'ill make a wrong decision ? Does the assumption of independence.
you ha made seem reasonable for this problem ?
[Ans. 0 012.J
54'. A factory produces a certain type of output by three types of
machines. The respective daily production figures are
i'iIac/:we I 3,000 units
AJGChiOL' If 2,500 units
Mahjue III 4,500 units
Past experience shows that I . per cent of the output produced by
Machine I is defective The corresponding fraction of defectives for
the other two machines are respectively 1 , 2 per cent and 2 per cent.
An item is drawn at random from the day's production run and is
found to be defective. What is the probability that it comes from the
output of ((i) Machine I, (b) Machine II, and (C) Machine [11 7
[Ans. (a) 1/5, (b) 1/5, (c) 3/5]
MATHEMATICAL EXPECTATION
If X is a random variable which can assume any one of the vues
Y . with respective probabilities p 1 , p.,,... then the
niathematical expectation of X usually called the expected value of X and
denoted by E(X) is defined as
E(X)-'x 1 i' 1 -f •.. +,, I'.

Properiies of Expected Value


(i) The expected value of a constant is the constant itself, i.e.
E(k)k, for every constant k.

(ii) The expected value of the product of a constant and a random


variable is equal to the product of the constant with expected value of
the random variable, i e.,
E(J X)=k E(X)

(iii) The expected value of the sum or difference of two random


variables is equal to the sum or difference of the expi' ,'od values of the
individual random variables, i.e
E(K± Y)=E(X)±E(Y)
(iv) The expected value of the product of two independent random
variables is equal to the product of their individual expected values, I e..
E(XY)=-E(X). E(Y)

P-37
flUS1Nr5 MATHEMATICS
(v) EfX—E(X)Jo
Illustration A dealer in radio sets estimates from his past
experience the probabilities of his selling radio sets in a day. These are
given below
No. of radio---
q --
sold in a day 0 1 2 3 4 5 6
Probability 02 l02l 32 72 O 09 06
W e observe now that the number of radio sets sold in a day is a
random variable which can assume values 0, 1. 2, 3, 4, 5, 6 with the
respective probabilities given in the table. We may also note that the
dealer has estimated the probability zero of selling seven or more radio
sets in a day.
Now
Mean number of radio sets sold in a day
O)(•02+1x10+2x21±3<.32+4)<.2015x.09 -6x'06
l0+42+'96-l- 0-{-4S+36- 309
Example 26. A bakery has the following sch edule
Cl daily demand
for cakes. Find the expected number of cakes demanded per day.
/V(}. of cokes
demanded iii o
I I 4 5 I I
hundreds 6 / 7
J j_y
Ptob0bIft,
002 007 1009 012 020 020 018 010 001 001

Sol ution. We observe that number of cakes demanded per day


is a random variable (X) which can assume the values 0, 1, 2,...,
respective probabilities given in the table. 9 with
Now
E(X)0XO.02+1 X007+2x0'09+3x0.12
+4x020+5 xO2O+6xo.]8+7xo.lo

436
Exup),. 27. And & Company estimates the net profit on a new
product it is launching to be Rs. 3,000,000 during the first year
if it is
'successful' ; Rs 1,000,000 f it is 'mo.derately successful' and a loss of
Rs. 1,000,000 if it is 'unsuccessful' The firm assigns the follow g
probabilities to if r st year prospects for the product in
m oderately successfzjl 025 What is the expected value Success /i d
of f - 0-15,
net profit for this product ? ir st year
Sohitior,. Taking loss as negative profit, the probability distrihu-
tluii of net profit (.) on the new product in the first year is

P-38
' RO BA B! L 11'!

3 1 —1
P.oJit
(in million Rs
---

Probability 015 025 1-015 —025


p(x) =060

Expected value of first year net profit is


E(X)=x p(x)
3x0154-1 x(Y25-l(—UXO6O
010 million Rs. =Rs. 1,00,000
Ern pIe 28 .4 lottery sells 10,000 tickets at Re. I per ticket, a
prize of Rs. 5,000 will be given to the winner of the first draw. Suppose
you have bought a ticket, how much should you expect to win ?
Solution. Here, the random variable 'win', W, has two possible
values : Re I and Rs. 4,999. Their respective probabilities are

d
10000 1000
9999
Thus E(W)=(-- l)x +4999 x - 0 Rc 050
1
Hence a minus 50 paisa is the amount we expect to will on the
avet age if we play this game over and over again.
Exmp1e 29. A box contains 6 tickets. Two of the tickets carry
a prize of Rs. 5 each, the other four a prize of Re. I, (a) If one ticket is
drawn, what is the expected value of the prize ? (b) If two tickets are drawn
what is the expected value of the game ?
Solution. (a) The sample space consists of C 1
6 6 sample points.
Let X be the random variable associated with the experiment and let it
denote the amount of prize associated with the sample point. Here X
assumes values Rs. 5 and Re. 1 respectively for 2 and 4 sample points.
2 . 1 4 2
Also p(5)=' -i-- -- and p( l ) = --

E(X)=Expctcd value of the prize


x 1 p(x))+x2,p(X2)
1 2 5 2 7
=5'' x T.T+1 =---=Rs. 213

The expected amount of prize is Rs, 213.


(b) The sample space consists of C 2
=-l5 sample points. Let X be
random variable associated with the experiment and let it denote the
amount of prize associated with sample points. Then X assumes follow-
ing values
P-39
BUSINESS MATHr14T(S
(I) Rs. 10 (when both the tickets carry prize Rs. 5 each
Number of sample points 2C21) Le.,
(ii) Rs. 6 (when one ticket carries prize Rs. 5 and the other Re. 1
i.e., Number of sample points =G1 >< 1C18)
(iii) Rs. 2 (when both the tickets carry prize Re. 1 each, i.'., No, of
sample pointsr-- l C2 = 6)
Also p(10),
p(6) -j-, p(2)=-
E(X)v1p(x1).fxp(x) -f X3.J)(x3)

10x-+6x ' +2x


F
2 16 4 10448-1 12 70 14 -
355 ------- I5 =467
Hence expected amount of prize is Rs. 467.
Example 30. A player pays Re. I to play a game. The game consists
of repeatedly tossing a coin and recording the number
h eads. The game ends as soon OS of ürnes it falls
the coinfails
f tails or when it has Jo//eu
3 heads in succession The player is paLl R . 2 for each head which
appears, calculate (a) his expectation in each game, (b) the amount won
or lost, on the average, in 20 games.
Solution. According to the rules, the game ends
the following Outcomes with either of
1' Tail in 1st throw
HT
head in 1st throw and Tail in 2nd (i.e., I head)
HHT
Head in 1st, Head in 2nd and 'rail in 3rd throw, (ie., 2
heads)
Il/f/f
1-lead in 1st, 2nd and 3rd throws (i.e., 3 heads)

shownThe probabilities of these events and the amounts received are


below
Outci,rnes Probability .4inounz Received (Rs)
(Rs. 2 for each head)
T 1/2 1)
HT 1/4 2
HHT 18 4
HI/H 1/8 6
(a)
Mathematical expectationLxo+ -- x2+ -_ >4-f-

PROBABILITY P -40

(b) Average loss in one game I - =

Loss in 20 games -- x20=—Rs. 15

Example 31 The manager of a machine shop has a choice of com-


peting for one of the' two contracts shown it) the table below

Contract /1 Contract B
Event
probabilities Consequences probabilities Consequences

Contract awarded 050 ±Rs. 60,000 040 +Rs, 80,000


Contract not
awarded 050 —Rs. 10,000 060 —Rs. 14,000

Which contract should be preferred if the expected monetary value


is considered as an oppropriate measure.
Solution. For contract A : Let X he the random variable which
assumes the values 60,000 and —10,000 with probabilities 050 and 050
respectively.
Then
Mean of X= 60,000 x 0-50 - 10,000 X Q5O
25,000
Similarly, for contract B, we can define a random variable Y, and
we find that
Mean of Y=80,000x040-14,000X060
=23,600
Thus, if the expected monetary value is considered as an appropriate
measure, then contract A should be preferred.
Example 32. There are four different choices available to a customer
who wants to buy a transistor set. The first type costs Rs 800, the second
type Rs. 650, the third type Rs 880 and the fourth type Rs. 760. The
probabilities that the customer will buy these types are 113, 116, 114 and
114 resp ectively. The retailer of these sets gets a commission @ 20%, 12%,
250 and 15 01, for iliece sets respectively. What is the expected commission
of the retailer 7 [Delhi Univ., B. Corn. (Hon.), 19921

P-41 BUSINBSS MATHEMATICS


Solution. We have

Type Price (Rs.) Commission


Probability Expectation

(1) (2) (3) (4) (2)x(3)x(4)==5


Frist 800 20% 53,33
Second 680 12% 1360
Third 880 25 5500
T
Fourth /60 15% 2850

Total 15043

Hence the retailer's expactation is Rs. 15043.

EXERCISES
1. (a) What do you understand by 'the expectation of a random
variable' ? Explain as clearly as you can ?
(b) A balanced coin is tossed 4 times. Find probability distri-
bution of the number of heads and its expectation.
(c) In a business venture a man can make a profit of Rs. 2,000 with
a probability of 0 4 or have a loss of Rs. 1,000 with a probability of 06.
What is his expected profit [Ans. Rs. 2001
2. A random variable K has the following probability distribu-
lion:
X —1 0 1 2
Probability : 113 1/6 1/6 1/3
Compute the expectation of X. [Ans. 1121
3. Calculate the expected value of K, the sum of the scores when
two dice are rolled. [Ans. 71
4. A box contains 8 items of which 2 are defective. A man
selects 3 items at random. Find the expected number of defective
items he has drawn. [Ann. 3141
5. A player tosses two fair coins. He wins Rs. 5 if 2 heads
appear, Rs. 2 if I head appears and Re. 1 if no head occurs. Find
his
expected amount of winning [An. Rs. 2'51
6. A player tosses 3 fair coins. He wins Rs. 5 if 3 heads appear.
Rs. 3 if 2 heads appear. Re. 1 if 1 head occurs. On the other hand, he
losses Rs. 15 if 3 tails occur. Find expected gain of the player.
(Ans. Rs 0251

P-42
PROBABILITY

7. An urr, contains 7 white and 3 red balls. Two balls are kh


together, at random, from this urn. Compute the probability th
neither of them is white. Find also the probability of getting one white
and one red ball. Hence compute the expected number of white halls
drawn.
3C 1 C xC 63
IHint. E(X)=Oxj±1x- Cc,
OC2 45
8. The monthly demand for transistors is known to have the
following probability distribution
Demand 1 2 3 4 5 5
Probability 010 015 020 025 018 012
Determine the expected demand for transistors. Also obtain the
variance. Suppose that the cost (C) of producing (n) transistors is given
by the rule, C 10,000 +- 500 ii. Determine the expected cost.
[flint. E(C)=E[ [0,000±500 nJ
10,000f 500 E(n)
=l0,000±500[( fl XP) }=l0,00t)+-500X362]
9. The probability that there is at least one error in accounts state-
ment prepared by A is 02 and for B and C they are 025 and 04 respec-
tively. A, 13 and C prepared 10, 16 aud 20 statements respectively. Find
the expected number of correct statements in all.
[Hint. Expected number of correct statements is
(1— 0'2)x 10 4-(1--025)x 16±(1 —04)x 20
x 10±075x 16-l-06x20
=32]
10 (a) Suppose an insurance company offers a 45 year old man
a Rs. 1,000 one year term insurance policy for an annual premium of
Rs. 12. Assume that the number of deaths per one thousand is five for
persons in this age group What is the expected gain for the insurance
company on a policy of this type.
[flint. Expected gain — 12 x (1-0005)—(1000--12)X 0005]
(b) The probability that a house of a .ertain type will b burned
by fire in any twelve month period is 0005. An insurance company
offers to sell the owner of such a house Rs. 29,000 one year term fire
Insurance policy for a premium of Rs 150. What is the company ex-
pected to gain ? [Any. Rs. 51
U. A firm plans to bid Rs. 300 per tonne for a contract to supply
1,000 tonnes of a metal. It has two competitors A and B and it
assumes that the probability that A will bid less than Rs. 300 per tonne
is 03 and that B will bid less than Rs. 300 per tonne is 07. If the lowest
bidder gets all the business and the firms bid independently, what is the
pxee'ed value of the contract to the firm 7
P-43
BUSINESS MATHEMATICS
[Hint 300x 1000 [J'(botb bid less than 300)
+P(A bids less than 300 but B bids more 300)
+ I'(A bids more than 300 but 13
bids less than 300)
= 300000(03 0'7+0'3>< O'34-0'7x 07)=R, 2,37,01)0.]
12. A gamester has a disc with a freely revolving needle. The
isc is divided into 20 equal Sectors by thin lines and the sectors are
marked 0, 1, 2 , 19. The gamester treats 5 or any multiple of 5 as
lucky numbers and zero as a special lucky number. He allows a player
to whirl the needle on a charge of 10 paise. When the needle stops at
the lucky number the gamester pays back the player twice the sum charged
and at the special lucky number the gamester pays to the player 5
times of the sum charged. Is the game fair ? What is the expectation of
the player ?
(Hint.

Event Favourable cases p(.) Gain (x)


f
Lucky number 5, 10, 153/20
Special Lucky No. 0
20--10= 10 P
Otbers 1/20 S0—I040 P
1,2.3.4.6.7.8,9. II. 16/20
12, 13. 14, 16, 17, 18, 19
---iO p

E(X)<lo+j_x4l6
20

13. In a college fete a stall is run where on buying a ticket a person


is allowed one throw of two dice. 11 this gives a double six, 10 times the
money is refunded, if only one six turns up, double the ticket
money is refunded and in other cases nothing is refunded. Will it be
profitable to run such a stall ? Yhat is the expectation of a player ?
State clearly the assumptions, if any, for your answer.
SECTION D

Some Additional Topics


DE-MOIVRE'S THEOREM
Statement. For all ratiof101 values of n (positive negative or frac-
tion) cos n O-F- i sin nO is the value or one of the values of (cos OF i sin 0)".
Proof. Case I. When n is a positive integer.
By actual multiplication, we have
(cos 01+ 1 sin 0 1 )(cos 0 2 11 sin 02)
=(cos 0 1 cos 02 —sin 0 sin 02)+I(sin 0 1 cos 0-}- cos O sin 0)
=cos (0 1 -1-0 2 )-1- i sin (01+02)
Again (cos 0-1-i sin 0) (cos 0 4-i sin 0 2) ( cos 03 +1 sin 0)
=(Cos (0 +0,)+i sin (0,+0,)] (cos 0,+i sin Oa)
C0s (01+ 02 +03)-I- i Sin (0 1 +0 2 +0 0 ), as before.
Proceeding as above, the product of n factors
(cos O+i sin o) (cos 0 3 4_I sin O) ... (cos O-j-i sin 0)
=cos (01+02+...+0.)+isin (03+03+...4-0)
Putting 0 I = 0 2 ... =0=0 on both sides of(), we have
(cos 0-4-i sin O)tt= cos nO4 1 sin nO
Miter. The proof can be obtained by the method of mathematical
induction also.
For n L the result is obviously true.
For n=2, consider
(cos 0+ 1 sin 0)2=cos2 j2 sin 2 O -1-2 1 sin 0 cos 0
r=(cos 2 0—sin2 0)+ i (2 sin 0 cos 0)
=cos 20+isin 20
hence the result is also true for n=-2.
Let the result be true for n—rn, i.e.,
(cos 0-1-i sin 0)-= Cos rn O+ i sin mO
Now (cos O+i sin 0)MfI=(cos 0-4-i sin 0)' (cos 0-F-i sin 0)
==(cos rnO+ I sin mU) (cos 0 ± i sin 0)
— cos (m+l) 0+1 sin (rn4-1) 0.
Hence the result is true for n=rn4- I also.
Thus we conclude that if the result is true for n2, then it should
be true for 11= 2-; I, Ic., n= 3. Therefore, proceeding in this manner we
find that the theorem is true for all positive integral values of n
Case 11. When n is a negative integer.
Let us suppose n= - in, where In is a positive integer.
BUSINESS MATHE4ATICS
A-2
(cos 0-1 .1 sin 0)=(cos 0-f-i sin 0)'
1 1
cos 0-1- I sin o)-(eos ,nO +- i sin mO)
- 1 (cos mO—i sin mO)
- (cos 'nO I -i sin tno) >( (cos 'nO - I sin mU)
cos 'nO—i sin ,nO
- cos 'nO -. I sin ,nQ
-

(Cos" n(J -t- sial mU)


0

= COS (- M O) .-j-i sin (—'nO)


=cos nO+ i sin no n= —mJ
Case in. When it afracc'ion, positive or negative.
Let n= , where q is a positive integer and p an integer positive

or negative.
By case I, we have
0 0
.. 0' r=cos (qx__)F:srn(qx 0
(cos -i- - i -__)

cos 84-i sin 0


Taking the qth root of both sides, we get
(cos .- -i-i sin -9—) is one of the values of (cos 0-j-i sin O)'

Raising both sides to the power p, we get


0
- 4- i sin -is) one of the values of (cos 0-1-i
(cos 0
q
(cos -- o -Fi sin -? e )is one of the values of (cos 0-I- i sin 0)''
(cos nO -F - i sin no) is one of the values of (cos 0 + i sin O)n
Remarks. I. (i) (cos 0+1 sin 0)"—cos (—no)--l-i sin (—no)
=cos nO—i sin no
sin (-0)=—sin 0; cos (-0)=cos 0]
(ii) (cos 0 —i sin 0) -={cos (-0)-I-i sin (---0)}
=cos (—no)+i sin (—no)
—cos nO — isinne
(iii) (cos 0—i sin 0)={cos (-8)+i sin (-0)}
= cos nO + i sin no
2. Students often wrongly apply De-Moivre's theorem in the
following way
(sin 0-f-i cos O)-=Sin no -i--i cos no
It should be noted that the real part must be cos 0 and imaginary
part should be sin e, but 0 must be the same with cos and sin both.
SOME ADDITIONAL ioics A-3
(sin 9 -+ I cos 0)106in no -}- i COS no
3. (cos 0-I I sin0)"-Acos no -f- i sin n
4. -.-----------L--.----.---
ios O) sin nO
=cos 0 f-i sin fl
5. Every complex quantity can be put in the form r (cos 0-f-i sin 0),
where r and 0 are both real.
Let a given complex quantity be x+fy.
Also let xi-iy=r (cos 0-I- i sin 0)
or x-j-iy=r cos O-f- Ir sin Q
Equating the real and timagiriary parts oil sides, we get
x=r cog 0 (*)
and y=r sin 0
Squaring and adding () and ('), we have
or rr=

Dividing () by ('), we get tan O r- - .. 0=tan1


Here r is always positive and is called the Modulus of the complex
number. 0 is called the aniplliude of the given complex quantity. That
value of 0 which satisfies equations (*) and () also lying between n and
-n is called the principal value of arnpIiiuJe . We shall always take
principal value of the amplitude expressing any complex quantity in the
form
r(coS 0+ 1 sin 0)
Example 1. Simplify
(cos 30 4- 1 sill (co g 0—i sin O)
(cos 50+1 sill (cos 20—i sill
Solution. Expression
(cos ISO f-i sin 150) [cos (-0) -I-i sin (_0)19
-
(cos 350 1 z sill [cos (-20) - sin (-2O)]
(cos 150+1 sin 150) [cos (-30)+i sin (-30)J
(cos 350--f sin 350) [cos (- 100)-f--i sin -(-100))
cos (150-30)+i sin (15o3e)
cos (350—. 100)—i sin (350 -. 100)
— cos 120+isill 120
cos 50+i sin250
=(cos 120+1 sin 120) (cos 250 +i sin 250)'
(cos 120+ 1 sin 120) [cos (-250)-i--i sin (-250)]
=cos (12-25) 0+ 1 sin (12-25)0
=cos l3O—isin 130
Example 2. Show that
r+ sin 0 +i cos 0 1' cos (-. -
L 1+iln, —I cos \ 2 (--_n
J / \2


A-4 BUSINESS MATHEMATICS

_(sin2 b -- cos 2 ) '- sin -1-i cos ) (


Solution.
- L.H.S.
[ 1sin
-4- —icos
[_(sin '-li cos ) (sin -- i cos )+ (sin 0 I -i cos )
- (1+sill —icos ) T
cos ) 1-4-sin --COS (
-L-
--
=(sin -l i cos
(I-j-sin—i Cos ) T
=[coS (_ )+i sin (--_i)]"
(fl z t i —n
' Tt
- - ----n )+i )-=R.H.S
sin (---
Ecample 3, Prove that
[(cos O+cos ) , I-i (sin Oi sin )}"-f [(cos o+cos )—i (sin 0 +sin
2n+1 COS, (0—) eo .s n (0+0)
2 2
Solution. L.HS.
640 6-0 0— - i
2cos
==[----f --j
cos — i2sin - cos —

2 cos ----
- — i 2 sill ----cos
--J
2" cos" --- - i sin
0+)1
4cos ---------1sifl ------
J
=r2cos coSn 2
____
(O (0+)1
-1cos n -_jfl

(8—) (0-f')
=2's cos 2 cos
2 2
=2 1 1 Cosii
$+
2 2
Example 4. Ifx= cos o-I-isin o, y=cos )+1 sin 0 and m and n
are integers, prove that
xv') yn
=2 cos (in 0—n)
Y 11
X
xm cos mO-F i sin mO
y" cos n+I sin no
Solution. —, +,,—
i sin nO Cos7n0 -1- ( sin ino
=(cos ,nO-- i sin mO) (cos n' 4-i sin ti)
-F (cos 'I ± i sin ") (cos mO 4-i sin m0)'

SOME ADDITIONAL TOPICS A-5


(cos ,nO t I sin mO) [cos (--iz) +- i sin (—n)]
-I- (cos nO i-i sin "t) [cos (---mO) f-i sin (--'nO)j
=cos (mnO mt) +1 sin ('ad - I cos (maO -

i sin ('nO ._iiq)


=2 cos (mnO—m/.)
Example 5, If(U +-1b 1 )(11 2 b 2 )(a -f-ib 3 ). ..(a,, f- ib,,) =4 011,
prove huh

() ( 0 2 1 b2)(a21b2)(a1b2)......(a,,2 f h,,2) =A-4-B2


bb 2 h b A
(1)) %afl -1 1. -f- ..... . -+-Iafl —i- tan-'
a, U, a8 a,,
Solution. (a) Let a 1 +lb,=r 1 (cos 0, -+- i sin 9,)
a2 +ib, r., (cos 0 2 -f-i sin 02)
and similar other expressions.
\/a12-1-b,, r 2 -. \/a 2 2 4b 2 ,..-, etc.

and 0, Lw' L , . tarr'- . ..,etc. (*)

Now it is given that


(a -{- ib )@ 1 2 -F ib)(a 3 + ib,) . . . (a,, + ib,) =A f-- 113
or [r, (cos 0, f- i sin (),)]()-,(Cos 0 2 I-i sin 0 2 ))[r 2(cos 024- 1 Sill 08)1
o f-/ sin 0)J=-A+iB
or r1r2r3.... . ,Icos (0 1 +02 1- .. . +0,,) f-i sin (e 1 +0 2 -t- ... +9)]:__A.f 1/3

Equating real and imaginary parts on both sides, we get


A=r,r2r3 ..... .r, cos (01+02-i-... ±0,,) (S*)
B=r ,r2r3......r,, sin (0,+ r2 -f- ... -- 0,,)
Squaring and adding () and (**, we get
A 2 +li-_=r 1 2 r.4 2 r 82 •.. r,,2
= (a,' f..b(a8+b2 ). ..(a,,' 4 . b,,')
(b) Dividing (4**) by (*1) we get
4.rtan(0i-{-.02-f....+0)

or
b b
l -4- ... +tan'—b
tan - ' ._L +tan --
a2 a.
Example 6. Show tItct

- -+Iir
(1 i)+(1—i)'=2 cos

A-6 BUSINESS MATLTBMTLGS

Solution. Let 1 + i =r (cos U +i sill


Equating real and imaginary parts of both sides, we have
r cos O=J and r sin 0=l
Squaring and adding we have
r 2 =H. 1=2 or r-/2

Dividing, we have tan ti -- I - O-,-r'4

l + i=r V 2' cos- i

(1 - i)v%_=2IL'[ cos- -- +1 sin


-
rfliT
==2I2J cos - - -f i sin--J

Similarly (I - i)'
2i/t - -- sin

fl1ri FliT
(1 +f)-+(I_i)"2[ 2 cos 2 cos

Example 7. Prove that


(a-f- ib) "+(a—iby Jn 2(a ! ±& 2 ) Tn co,. /

Solutjo,h The cartesian coordinates can bc transformed into


polar coordinates by means of the following relation
a=r cos 0, b:- sill

where rI=a+bt and Q r tan L- .

Putting this value in L.H.S., we get


F?i

(a4ib) nl +(a_(b)i=(r cos 0+ ir sin 0)


F??

4-(r cos 0-jr sin 0)

r[(cos 0 + 1 sin 0)-f (cosO--i sill

r ri
[
cos—
n
r sin
s - 04-cos 0-i sin

0
J
1
(Fly using Dc-Moivre's Theorem]

SO4[ ADDITIONAL TOPICS

In
hJ
2 COS 02 (a2 1 h') 2 "cos [.tan_1

r=
L
fn

Thus (a + ib ) +(a -ib) 2(a b2) cos [—tail- 1


'rIIEoREM. Find q roots of (cos. U ii sin 0)" N , where p and
q a re inteore piline to each other,
[flint. We know that
cos (2n {-O) r cos 8 and sin (2'i+ 0) --sill &
where n is any integer.
(cos 6-f-1 sin O) .Pi =c 2n 1-0)4 i sin (2nr-4•0}r'
I p(2nr E0). . ( P(2'1r+0)
ens
q ----- I- sill ( q
Now giving the successive values 0, I, 2.....(q - I), we obtain the
n
q values as
p p
. sin --0 when n:-0

cos sin2 1-0) when ii

cos--------- --I sin p(4+e)


----- ---, when n-=-2

cosL2(q-I)+0}±f
{2(q- 1)

when n-=q----1
When n=q, we obtain the values as
(2q Tc - .
+t sIn -------
cos q

=cosp( 2T+---)+isinp( 2n+--)

'CO5 (2pr+ 2 ) +i sin( 2p7T-t-_)

COS -+1 sin—


q
which is the same value as obtained by putting q=0.

A-8 BUSINESS MA1}1IMATICS

F;&ample 8. Find (Ill the values of (16)11


Solution, 1602(1)h1
=-2 (cos 04 I sin 0)hti==2 (cos 2ni + i sill
/ flT . fliT
2 --- +
-r)
Giving fl the values 0, 1, 2 and 3 we get the required values as

2, 2 ( cos -- i sin--), 2 (cos -r 1 I sin i)

/ 3ir 3,i
and 2 ens -i-- +'sjn—- )
i.e., 2. 2i, —2 and —21,

Example 9. Find the all values of (8f)k

Solution. Since ens - =0 and sin -- 1

8i=8 [

1! .
(Si) =8 [ COS—t-: sin - ]

=2 [ cos( 211+--).1-i s(

-1- -I .
=2 cos ----- -z sin -- -.
[
Giving n the values 0, 1 and 2, the required values are
r . . " E
JL
2
L COS4l
SIn_.,2 cos+I
-'- sinJ

arid r 3rr
2 [ cos---j--z sin--
.

2 [.Fi --
j, 2[9
-
2 fi. L ] and 2 + 1 (-1)]
10

i.e., 2 tt), 2(.±±) and —21


i.e., 3 -f-i, - 3+ i and —21.
Example 10. imud till the values of (—J + 13iI
Solution. Let - I + /3i=r (cos 9+ 1 sin 0), then
r cos- O =— I and rsji 9= . / 3 r=-2
I 2ir
and ens P== ---- and sin 6 = - i-- 0=


SOME ADDITIONAL TOPICS A-9

T 4-i
2
(-1 + /3i)=2 7 [co
g Si.
J
141v 147c
=2 ----1-f sin
14 \
=2 [005 (2n+__)+i sill ( +)]

or (—I - ! % 1'3i)'1'-- 2/ [cos ( 6n4-14 —) - i sin ,-_6n-j-l4 )]

Giving n the values 0, 1 and 2, the required values are


l4it l4r Orr 2Ot 1
2' sill --- 2' ---_+ i
sin---- i
and
26t
2' [co g _- i sin 26
1
Example ii 11/Id the continued product of the four values of
It - It 3j4
(cos --- -1 1 sin

-
-'
I 3 1 114
Solutioxi. -- -I-i Sin -- ) co, -- +1 sin '-M-

= (cos r -I-- i sin)' I'
[cos (21irt f- r-0 f-i sill +xt)1' (4
2iiIt i It 2'ln i
Cos 4sin(
4 )
Required continued product
( It - t 3 31t / 5-t 5i
=COS -- 4 1 Sill )(cos sin —-)Cos--- -f-i sin

f' 77
xcos —+iSifl -
4
( I 3 5 7n f 3, 5 7
=cos (-- +---+-- ±T)-•l Sin
-- f---;-

-=Cos 4It4i sin 41+jX0:--I


Example 12. Expand cos 8e and siO 88.
Solat ion. By De-Moivrc's Theorem
(cos 0-1 1 sin 0) , — Cos 8 0+i Sin 80
Also by Binomial Theorem, we have
(cos 04-i sin 8)8 ='cos8 0 + 1 c, cos 7 8 (i sin O )+ c. cos" 0 (1 sill
+ 8 c 3 cos 0 (1 sin Ø)!_4_ 8 c cos1 0 (1 sin 0)

- 8c5 cos 0 (1 sin 0)5 cos 2 0 (i sin 6)r


H 8 c_ cos 0 (1 Sill + CB . ( i sin 0)

A-10
BUSINESS MATT1EMAflCS
= cos' 9+81 cos 7 8 sin 0-48 cosd 9 sin' O--56 I cos 8 0 sin' 0
+70 cos 4 9 sin 4 0+561 cosa 8 sin s 0-48 cos' 0 sin e 8
cos 0 sin 0+sin' 0
or (cos 8+1 sin 0) 8 =(co3' 0-48 cos' o siii 2 0-1-70 cos' 8 si l l ' 0 ,
—48 cos' 0 sin 6 O-l-sin 9 9)
4-1(8 cos 1 0 sin 0-56 cos 5 9 sin 0 4- 56 C0s 3
0 Si11 0e
--8 cos 0 sin' 0) (**)
From ($) rind (* ), we have
=
cos 89+1 sin 89 (cos' 0-48 cos 6 0 sin 2 9-4-70 cos' 9 sin' 8
—48 cos 2 0 sin C -- sin 8 + i(8 cos 7 0 sin 9 ---56
COS- C sin a 0
s
f-56 cos' C sin 0---8 cos 9 sin 9
Equating imaginary and real parts on both sides, we have
sill 8 cos 7 0 sin 6-56 cos-6 9 sill' 01-56 c0s 3 0 sin s
0
-- 8 cos 0 sin 7 0
and =
co, 0 cos' 0-43 cos' 9 sill' 1-70 cos 1 0 sin 4 0 0
—48 cos 2 6 sin' &1-sin' 0
Example 13. E.vpress:
(a) COS 7 in a series of cosines of multiples of Q.
(b) Sin" 8 in a series of cosines of multiples of 0.

So1ut 0 Let X-=Cos 0 + 1 sill so that -f- = cos 0—i sin 0


X"=Cos n8-{-i sin nO and
4- =cos n O — isin no
x+ ± =2 Cos 9

and
x+ --=2
X11 Cos n0

(a) (2 cos 8)7=(x+

• . . . '-i- + 7 c . x'
3

7c 3 7C
-+7c7 -

=x7 +7x5 +21x3 4-35x+35 -i-- +21 . - +7. - +-

(x7+)+7(x5+4-)+2l(3+I)+3S( J)
2 cos7 8=2 COS 70+7. 2 cos 59-1-21 . 2 cos 30+ 35. 2 cos 6
SOML ADDITIONAL TOPICS A-li

Hence cos' 0 = ( j [cos 70+7 cos 50-1-21 cos 39-4-35 cos 0]


(b) We have

(2 i sin 0)'°=(v _ 1 )1O_1O+1oCx (--- )+o 2 (_ _!_ )2

_l
+ 10c3 _L) 3 +ioc 4 (_
x7 (-

-f
0 5 5 (

1 )-°c

107 --'°c8 x2(_.)


-. )7
Yo
( --)°-i-'°c10 (

=x°— 10x 8 -4- 45— 120x 4 +210X 2 — 252

-210 _---120 45 —10

=r(Xi0+ xs+4)_45 ( °±-)


)—to(

_120( 0+)-l-2I0 ( x2+)_252.

or 210 110 sin' 0 0=2 - cos 10 0-10 . 2 cos 80+45 2 cos 60


—1202 . cos 40-1-210. 2 cos 20-252.
Hence sin° 0 - (— [cos 108-10 cos 80-f 45 cos 60
-----120 cos 40+210 cos 20-1261
i •• fI0(j)5=(_1)5ll

Example 14. prove that

sin e 0 cos' 0=- [-_-cos 8 0+ 4 cos 66-4 cos 4o — 4 cos 20 +51

Solution. Put
X : COS 04- I sin O-=C+iS
cos 8—i sin O=C—S

2 cos O=x+-1 -, 2iS=x—

and X= cos FIG + i sin nO, 4 =cos no —I sin nO.


2 cos nO -- x4- 2 i sin n0=x- xn
_\fl

A-12 BUSINESS MATHEMATICS

(21 sin 0) (2 cos 0)2

:=

x 4 -4x'. _L+ 6*. ----4x.

(x2+)

(
xs.+.-)-_4.( x+_-) + 4( x4+—)

-1-4 ( x 2 +4.-')+ 10

28 sin" 0 cos' 0
= 2 (cos 80-4 cos 60+4 cos 40-1-4 cos 20- 10)
Hence sin e 0 0
=r- (—cos 80+4 cos 60-4 cos 40 -4 cos 20±5)

SOME IMPORTANT THEOREMS ON MATRICES


Theorem 1. Matrix multiplication is associwive, i.e., if A, 8 are
conJormal for the product AB. and C are conformal for the product BC,
then, (AB) C•=A (BC)
Proof. Let A, B, C be the ?flXn, nxp and p q matrices and
A=[a,1], B=[b 11 ], C=-[c,,]
Here A, B and C are conformal for the product AB and BC.

AB=(a, 1 ) x [b,,]== [ bt, /)k1 J= [u,,J, say


(U,1] is an m x p matrix. = 1, 2.......,
j=1,2 ........ p.
(AB) and C are conformal for the product (AB) C.

(AB) C=(u11]x[c,j1=[ U,1 C,]


1=3
P iT

Z ( u, b,,,) c,1]
1=1 k=I
P ir
: 7, 0 tk bk, C,,] ;

: 1, 2........
1 = 1 k=
jJ,2 ........ p.

A-13
SOMS ADDITIONAL TOPICS

It is an nix q matrix.
P
BC [b 11 J X {c, [ , b,, c]= N,1, (say) 1=1, 2, ...,
s I
j1,2,...,q
[v,1] is an ax q matrix.
Therefore, A and (BC) are conformal for the product A(BC).

A(BC)=(a,1) X [v 1 ]= [ Y. a v,,]

P A

- [
a,, ( > b C 11 ] — I y. a, b, Cl)

t — sI s=I t==I

i=I,2.....
j=1,2,...,q.
Here (AB) C=A (BC)
Theorem 2. Matrix multiplication is distributive with respect to
addition of matrices, i.e., if A and B are conformal for the product AB, B
and C are conformal for addition, then
A(B+C)=AB± AC
Proof. Let A= [a,,] be m x n matrix
and B= [hill and C=[c 1 ] be each n x p matrices, so that
(B+C) is also nxp matrix.
Thus A (B+ C) is of order mxp.
Also AB-f AC is of order m xp.
Therefore, the matrices A(B+C) and AB+AC are conformable.
Further.
(i,j) th element of A (B+ C)
=-Sum of the product of the corresponding elements
of ith row of A and jtb column of B+C.
a
-= ) a1 (b1±ck)
k= I
n N
= a b,1+ y. 0k Ctc1
k'I
(Since in case of real number, multiplication
is distributive w.r.t. addition
=(i,j) th element of AB-4-(i,J) th element of AC
=.i, I) lb element of (AB +AC).
Hence A (B+C)=AB+AC

A-14
BUSINESS MATHEMATICS

Theorem 3. The transpose of the product of two matrices is equal


to the product of transposes taken in reverse order, i.e., if the matrices A
and B are conformable for the product
AB. (lien the matrices B', A' are
conformable jor the product B' A' and
(AB)'=B' A'
Proof. Let A=[a,1 ] be an lnxp matrix and
B=-[b, 1 ] be a pi nx in matrix. Then
A'[a1,] is an n x m matrix.
and B'=[b,] is a pn matrix
'p
AB . [ a,1 ] x[hk]= [ a ,, b,,,]
/=1
Jt is an rnxp matrix.

(AB)'=[ y, b,k a,1]; k =i, 2, .., p


j=I
i=l,2.... . in
The elements in the kth row of B' are the elements of the kth
column of B.
They are b lk , b 2 ......., b . Similarly the elements of the ith column
of A' are a, 1 , b,,......., a,,,.

The scalar product of these two sets of elements= bjk a,1


./ =I
'p
B t Al. [ 7 b1i a,J; k_..1,2........p
1=1
1=1.2,,.... m
Ileizice (AB)'==B'A'
Theorem 4. Every square matrix can be uniquely expressed as the
sum of a symmetric and a skew-symmetric matrix.
Proof. Let A be a square matrix and A' its transpose.
Then we have
A= (A+A')+3 (A—A')
(say)
where P= (A+A') and Q = (A—A')
Now P'= (A+A')'== [ A1 +(A 1 )8 ]1 (A'+A)=p
and Q'= (A—A')'= [A'—(A')'J=-_ (A—A')=—Q
Thus P is a symmetric matrix and Q is a skew-symmetric matrix.
Hence from (1), we conclude that a square matrix can be expressed as
the sum of a symmerio and skew-symmetric matrix.
To prove the uniqueness of representation (1), let, if possible
A=B+C

S13 ADDITIONAL TOPICS A-iS


where B is symmetric and C is skew-symmetric matrix so that
13'=B and C'.=—C
Then A(B+C)'=B'+C'=B-C
On adding and subtracting (3) and (4), we get respectively
B= (A+A=p
C= (A—A')---Q
This establishes the uniqueness of (l).
Theorem 5. If A=[a, 1 ] is a square matrix of order ,:, prove that
A(adj A)=(a/ A) A I A jI.
Proof. We have
r a1 a, ... ar,, /1 11 A .. 4 'u )
A (adj A)= a21 0 22 0 2,,
1 X
1 Al2
... A 21 ... A
I I
L a, a,., ... a,,, J I A t ,, ,L,, ... A,,, J
IIAi 0 ... 0

OH!
9 I

L ô ó ...IA
since from the property of determinants, we have
A I, ifi=k
: a, 1 Akj=I
j-I 1 0 if iLk
Hence A (dj A)= I A I L..
Similarly it can be proved that
(adjA). A IA IL
Theoren3 6. The necessary and sufficient condition for the existence
of the inverse of sqiare matrix A is that A is non-singular.
Proof. The necessary condition Let B be the inverse of A.
AB=BA=l
AR I = IA I x I RI = III
A I 7O. Thus A is non-singular.
Tile sufficient condition If I A I ;610.
adfA ) çadfA
fAT)
adjA
and it exists.
IA I

A-16 BUSINESS MATHS MATICS

Theorem 7. A and B are non-singular matrices of the same order,


then AB Is also non-singular and
(AB)- 1 =B-'A-'
i.e., the Inverse of the product of two non-singular matrices A and B is
equal to the product of the inverses A'
and B- 1 in the reverse order,
Proof. If A and B are non-singular matrices of order n, then
A F :7--0, I B 1 y4_0.
Also IABL- IAIxIBI -#O
AB is also non-singular and hence has an inverse (AB)-'
We have
(W i A- 1 ) (AB)=B- 1 (A-' A) B=A-'L,, B==B' B=I,,
and (AB) (B-'AJ)=A(BB-l) A-'-=A I, A-'=AA-'=T,
Hence B-'A- 1 is an inverse of AB. In other words,
(AB)-'=BA'
EXAMPLES ON DETERMINANTS
Example 15. Show that

r+cc 13 1 =0
y 1
Solution. Operating C1 -1. C,+C2 , we have
a I 1 cc I
= 13 1 = ( a413-l- y ) 1 13 1 z+13+i)x0=0

y 1 1 y 1
1 a a
Example 16. Show that a' I a

a a' 1
Solution. Operating C1 -+C3 -f- C, C,, we get
l-}-a+a' a az I

L = I -j-a+a' 1 a

1+ a -4- a2 a' a
1 a a'

==(1+a+a') 1 1 a

I a?1

SOME ADDITIONAL TOPICS A-17


I a
OperatingR2-R2- R(
=-(1+ a + at) 0 1-a a-a2 and R2-R,-R3

() a1 -a 1--a2

1-a a(1-a)
(1 + (3 + a2)
a(a-1) 1-a2
1 a
=(1 -f- a4-a 2)(j- a)2
-a 1-I-a
=(1-a)
Example 17. Show that

I a a2-bc

I b b2 ---ca =0
I c c2_ab

1 a a2 1 a bc

solution. I h b2 - 1 b ca

I C ('2 1 c ab

1 a a2 a a2 abc

= I b bt - b b2 abc

I c ct c c2 abc
I a a2 a a2 1
= 1 b li' - b b' I
I C C1 C Ct 1
1 a a1 a I a2

=1 b b1+b I b2
1 C cz c 1 Ct

I a at i a a2

= 1 b b2 - 1 b b2 o

1 C C2 I 1 C C1
1-18 BUSINESS MATHEMATICS

Example 18. Show that


(b 4 c)z at 02

b2 (c+a) b2 —2abc(a+b-1-c)3
C2 c2 (a+b)2
Solution. Operating C1 - C1 —C and C2 -*C2 —C we get
(bc)2 & 0
0 (c+a)_b2bz
Cl— (a+ b)2 (a+ b)'

(bfc—a) (h-f c + a ) 0 a2

= 0 (c-j-a--- b)c+a+b) bt

(c—a—b)(c.4 a+b) (c.--a.--b)(c+a+b) (a±b)2


b-c—a () a2

0 c+a---b b8

c-- a — b c--a - b (a-t-b)'


b4-c---a 0 &
Operating
=(a-+-b4-c)2 0 c+a—h b

—2b —2a 2ab

Operating C,-4C,+ C and C3-,C2+-- C3, we get

at
b+c Ta2

(a -j-b+c)' c-i-a

o 0 2ab
a2
b+c

• 2ab

b
c+a

(expanding the determinant along its third column)


=2ab (a+b1c)2 Rb + C) (c+a)—abj
r=2abc ( a + b + c) -

SOME ADDITIONAL TOPICS


A-I9
Example 19 Show that

x - -a b c d

• x-f-b c d
==x' (x-l.af-b+c+6/)
• b x ± c (I
• b c X+j

Solution. Denoting the given determinant by


C1 -'1 4 C -!- C3
t , and operating
we get
x + a -l- b +c+d b c d
x+a+/,+c-f-d x+b c
X + a f b ± c + d 1) xfc d
x + a + h -f- c ±d b c xf-d

I h c d
1 x--b c d
= (x + a + b + c + (1)
b x+C d
b c x+d
(x -f- a -- h I- c Fd) is common in C1]
I b c d
o 0 0 Operating
= (x± a 1-b+c+d) R2-R2R1
o 0 x 0 R8 -R8 —R 1 and
R4-^R4-_R1
o o o
X

(x+a+b+c+d) 0 x 0 expanding the above


determinant along C1
o o
x 0
=r(x+ajbc4 d). x
expanding the above
0 x 1 1 determinant along C1
x x'
=x (x+a+b+c+d)


A-20 BUS1NSS MATHEMATICS

Example 20. Prove that


32 3a 1'

a' 02+20 2a + 1 I
=(a— 1)6
a 2a+I a+2 1

1 3 3 1

Solution. Denoting the given determinant by t and operating


we get

30—i 3a' 3a 1

0 a6 +2a 2a+1 1

0 2a-4-1 0+2 1

0 3 3 1

a2 +2a 20+1 1
=(a-3a'+3a-1) 2af1 0+2 1

3 3 1

(expanding the above determinant along C)

a—i 0
2(a-1) a—i 0
3 3 1

(operating R 1 -+R2 —R, and R,-+R,—R,)



0+1 1 0

.=(-1) 2 1 0 , (a—i) is common to R1
1 and R.
3 3

a+1 I
(a— 0 expanding the above determinant
2 1 1 along C,

=(a_I)1[(a+1). 1_I.21=(a_1)6(a__1)=(a_I)6

SO?,fl3 ADDIrIONAL TOPICS


A-21
Example 21. Prove that
i-f-a2 ab ac ad
ab I+b2 bc bd
ac bc l + c Cd
ad bd cd i+d2
Solution, Multiplying C 1 , C2 , C3 and
tively and dividing by aocd, we get
C4 by a, b, c and d respcc-

a(1 +a 2) abt ac2 ad2


1 a l b b(l +b 2) bc s bd'
abed
ac b2c c({4.c2) cr/a
a2 d b2 c2d d(1+c/2)
i+a 2 b c2 d2
(2bCd a2 I ±b 2 C2 d2
02 b2 I +c2 d

b2 c2
(Taking a, b, C and dcommon from R, R2 ,
R 3 and R4 respectively).
Now operating C l* C L +Ccs±c
we get
1 b2 c2 d2

a
ê —(1 + a2 + b2 .f ct+d2) I 1+b2 c
I b2 1+c2 d

I 1'2 C2 l+d2
1 b2 c2 d2 Operating,

o i o o R2-+R2_R1
= (l + &+bI+c!+dt)
o 0 1 0 R3--*R3_R1
o 0 0 1 I
1 0 0
lja2+b2+c2+(1) 0 I 0

o 0 I
=(1-4- a2 F b*4C2-ld2) [Expanding along the first column)

A-22 BUSINESS MATHMA11CS


Example 22. Prove that
l-Fa 1 1 1

1 I.+1' 1 1
=abcd( J.3.±+L+L+L)
/ 1 1 1-i-il

Solution. Dividing C 1 , C2 , C3 and C4 by a, b, c and d respec-


tively, we get
(1/aH- I 1/b 1/c l/d

I/a (1/b) -1 11c lid


j -rabcd
I/a 1/b (l/c)+l i'd
1/a i/b 1/c (l/d)--i
Operating C 2 - C1 4- C2 -j- C+ C4 , we get

=abcd (1- .i H , +±)

1/h 1/c lid

I (1/b)+1 1/c 11'd


X
I i/b 0/0 +1 id

1 1/b I/c (l/d)+l

1 11b 1/c ifd

1 1 1 1 0 0
=abcd (i + - ± T+ +

'0 0 0 1
[Operating R1 -R,-- R, ; 2=2, 3, 41

=abcd (i----+--+--+-)

(Expanding by first column)


PRODUCT OF TWO DETERMINANTS
The product of two determinants of third order is a determinant of
third order. More precisely if t I and 2 are two determinants each of
order 3 as given below

SOME ADWrJONAL TOPICS


A-23
a 1 C1 Ii
A- (J2 b2 C3 ac1 /"r:01 2 P 2 Yl
a 3 b 3 C3

then their product L\ 1 . 2 is the determinant of order 3 given by


0,--hp1--c1 a3-j.b2-l-c..., a1b133-4-e.1
a21 -f b3 1_f- c2- a,,,, -I b, p 2 +c 2 '., a,c3 -f b2B-l-c,y3

a31+ b 3 1 + c 3 y 1 a3i+b332+c3y2 a33 + b3--c2y


Now can be split up into 3 x 3 x 3, i.e.. 27 determinants, since
there are three constituents in each column. These 27 determinants
are
a11 a1r.z2 a1B 02 (1y

21 O2x2 a2cz 3 ac1 O137 a 2Y3

a 3 a3 32
h1 3 1 a12
b 2 , a22 21

b31 a8cx2 C3y5

The first of these determinants


a l a1 a2
=0, since the columns in the determinant
a, a2 a2 are identical.
0 7 05 03

The second of these determinants


a b1 c1

=a 13 'y 3 a b3 c 2 =y

a b7 C3
The third of these determinants
b1 a c1 a

b2 a 2 c. p l a lya 02 b2 (3

b3 03 (3 h8 C3
a3
= 2I1Y3 X

BUSINESS MATHEMATICS
A-24
In this way we can easily find out that 21 out of these 27 deter-
minants will vanish and the remaining six determinants will be
t 1aA i iflzL\i ± 2aY1 A 1 —°131yA1
-F 3l y 3 L\ I — 2y1Ai
cc aP2r) Al

=(c y@3y2Y2
(3 1 y 3 —f3y 1 )+cc3 I
( 3 1 y 9 — c2 y 1)] Al

Gb

==l

TI •t'2 13

Remark. Formula gives the so-called row by row rule of the


multiplication of two determinants and may he described in words as
follows
"The three elements of the first row of the product determinant
are given by the sum of the products of tire elements of the first
row of L'j with the corresponding elements of the first, second and third
row of rcspeciir'ely. Similarly the elements of the second row of A
given by the sum of the products of the constituents of 2nd row of A1 with
the constituents of the first, 2nd and 3rd row of A , respective !)' ; and so
on for the third row of f".
by column or
Similarly, we can obtain the value of A by COhlIflPl
column by row rule of multiplication of two ,lcterrninatltS A, and A2.
Thus the value of A can be obtained by any of the four rules of multi-
plication, viz., row by column rule, row by row rule, column by column
rule or column by row rule. In other words, we call multiply two deter-
minants of the same order by any of tile four r'41Cs of ,nuitiplicaiiofl.
Example 22. Express the product

a 0 1110 a

/ b Olxlb 0 I

0 1 c I' c U
as a determinant and find the value of it
Solution.
a U 1 0 l a

1 b oxb 0 1

o I c I c 0
axO+Oxl±lxa axb-l-0x04-1 xi oxl-i-0xc+lx0
Ix -- b x c 4 Ox 0
1lx04-lxl+Oxa Ixb-i-bx0+0I
OxO+lx11cxa Oxb+IXO+CXI 0<1+! xc-l-cXO

SOME ADDITIONAt roPcs


(1 ab-1 a

= b b i±bc

1ca c c
Also
a o I a
a () IJ
b o b I I) .-- h 0 1
I
e I U c I c U
1) 1
(by interchanging the coimns)
=(abc-4- I) [by ex1ntiding1
ProJuct = (abc
Example. 23. /'rove that
C1 a1 b1 C1
A B

a.4 b2 c

C3 a, b3 c3
A 3 84
where capital letters denote the cofactors of the corresponding small letters
i,r the de;erininwjl on I/IC rig/it hand side, provided it is not zero,
Solution. Let us write
a c1 A B1 C1
/'

an(l .... C,
I\= b (3 43 j

a 3 b 3 C3 43 J C
Now
a 1 134 ;1 J C,
cl
a.2 b, c. X ,, B C0

a 3 b3 c3 A I ('a

a 4 A 4 +h 1 B i + C 1 C 1 a.,4, , --:'), B24 c1C3 a4A3+6iB3--ciC3

= (la/I j + '8L-1- c 2 C 1 (12 ,f 2 - b3B2 + 2'32 a 3 A 3 + h 2 B 3 -I- c3C3

aA -- b 3 8 1 + c 3 C 1 a 3 A,1 -f h 3 !? 2 4 3( 3 arl 8 -1- b 3 n -I- c3C3

(By row-by-row rule of rnuitiplicatton)


0 0
= 0 Lx 0
0 0
.2
hence if t, :A () ; then ,

A.26
UUSi5S MATHa.MATICS

Example 24. Express


(a-x)' (b-x)' (c-x)'
(a-y)' (b-y)' (C-y)'
(a-z)' (b_z)2 (c-z)I
as a product of Iwo determinants and prove that the value
minant Is of the deter-

2 (b -- c) (c— a)(a-b) (y- z) (z- x) (x -y)


Solution. Given determinant
aI.2axx b'.-2bxfx' c'-2cx-4x'
= a'-2ay+y s b'244-y' c1 •- 2cy+y2
a'-2az+z' b2 -2bz+' c' -2cz+z'
a' -2a 1 1 x x'
= b' -2b I > I y y'
J c' -2c 1 1 z
(by inspection and trial)
I a a' I x X

=21 b b' xi y y
1 c 1 z z
(Interchanginp, the first and third column)
=2 (a_b) (b-c) (c-a) (x-y) (y-z) (z-x)
(On simplification)
Example 26. Solve she following equations by Cramer's rule:
x-2y+3z5
4X+3y+4z7
x + y - z = -4
Solution. We have
= Determinant of coefficients of x, y, z
1 -2 3
3 4
1 1 -1
(-3--4)-+(3-2)+I (-8-9)=-20=Ao

SOME ADDrflONAL TOPICS

Since p^0, the unique solution of the system is given by

Y = z—_L!
A' A' A
5 —2 3

where A1-= 7 3 4 =40 (on simplification)

--4 1 --1
1 5 3
1
4 7 4 20 (on simplification)

1 --4 —1
1 —2 S

4 3 7 =-60 (on simplification)

1 1 —4
Substituting in (s), we get
y=1
x=---2, and zr3.
Example 26. Use determinants io solve the folio wing equations:
ax-I- hy+cz=k
02x+b?y+ClZ_kI
a'x+ by -1- 8=k
Solution. The determinant of the system
a b c 1 1 1

A= ' b' c2 =abc a b c

a s b' c' at b' c'


=abc(a—b)(b - c)(c —a)
Let us suppose that a, b, c are three distinct numbers and they are
different from zero.

k h c 1 1 1

k' b' ct khc k b c

A' bs CS k' b' c


A
kbc(k_b)(b—c)(c--k) - k(k—b)(c—k)
abc(a_.-b)(b—c)(C—a) - a(a — b)c — ä

A-28 BUSINESS MATII1MTIS

ej k c I I I
a 2 k CI kac a k c

I a3 k c3 a2 k' c2
Y ---

kac(a_k)(k_c)(c(A) k(a--
k(k --)
abc(a-- b)(b . - c)(c— a) -- a(a- b)(h C)
Similarly, we shall get
k(b k)(Jc--a)
- c(Ii -- (1 C .- a)
CHARACTERISTIC EQUATION AND ROOTS OF A MATRIX
Let A [a 1 1 be a n x it square matrix. Then matrix A-- Al is called
the characteristic matrix of A. The determinant
A -?I — N), say I (
which on expansion gives a polynomial of degree n in is called the
characteristic polynomial or characteristic determinant of characteristic
function of A. The equation
A—At I =0
is known as characteristic equation of A and its roots,
are called the characteristic roots or lateral roots of A.
Example 27. Find the characteristic equation and roots of
6 —2 2

A=r —2 3 —1

2 —I 3
Solution. The characteristic equation is
I A—?l
6--N —2 2

—2 3—N —1 =0

2 —1 3—)
I 3— -- I --2 --1 1-2 3—N
(6 - N) + 2 -1- 2 0
2 3-N 2 —1
(6—N) f(3 —N)— 1J4-2[ -6-t-2N+2]-f-2[2-6+2N]= 0
-12N2 +44N-48=0 (on simplification)
(N-2)(N-4)(N-6)O
Hence characteristic roots tire 2, 4 and 6

SOMR ADDIT[Q/L TOPICS A-29


Cayley-Ilamihoii T1ioreiii
Every square matrix satisfies its characteristic equation. Thus if
A—Al F =(—I),$
is the characteristic ecluation of the matrix A then
q5(A) (- fl" [A n -I- p 1 A' - p2 A" 2 + ... -p1) = 0......(')

Remark. Cayley-Hamilton theorem may also he used to obtain


the inverse of a non-singular matrix A. If A is non-singular ( A
then premultiplying () by A 1 and transposing, we get

A -= [A- 1 +p, A 14 I-..- +p-1 II
P.
Example 28. Verify tAat the matrix
r 2 —1 •/i
A= J 2 —1
1 --1 2
satisfies its characteristic equation. Hence compute A'.
Solution. ihe characteristic equation of A is
A--1 -0
E 2 --1 l ri 0 01
—1 2 —1 -- 0 1 0 1=0
I l 2J Lo 0 lJ
12--A —1

—1 2—i --1 I-=0

I —1 2—A
2--h --1 --1 -1 r1 2--
(2•—?) +1 -1- 1
---I 2—Ai I 2—i I
(2—a) ((2— A)— lJ+(-2+?+ 1)+[I—(2—A)]-0
. 6A I- 9A-4=0 (On simplification)
By Cayley-Hamilton theorem, we get
A-6A2 -l- 9A-410 (.)
Verification of (*)
2-1 l 2 —1 1
A2 =A . —1 2 —1 lx —1 2 -
1 —1 2J 1 --1 2
6 —5 5
—5 6 —5
5= —5 6
22 —21 21
A3 =A2 . A[ —21 22 —21
21 —21 22


A-30 BUSINESS MATHEMATICS

We have A3-6A'-l-9A-41
[ 22 —21 211 [ 6 —5 5
r
—21 22 —21 I —6 I —5 6 —5
L 21 —21 22J L 5 —5 6
r 2 —' ii rio °]=[00001
+9 I —1 2 —1 I —41 0 1 0 0 0 0 =O
1 —1 2J [oo ' 0 OJ
Premultiplying (P) by A', we get
A1 -6A-- 91-4A-1=O
A 1 -=-, [A'-6A+91)
r 6 —5 5 1 r 2 —1 1 1 r 1 0 0
=l —5 6 —5 1-61 —1 2 —1 1+91 0 1 0
L 5 —5 6J L 1 —i 2J LO 0 1
r
I 3
t
1
A
1•1
A
I
r
j
3

1 1
I
4
1 —.1
—1 r t
L
t
1 i I
L
i
T
1
I
3
t
Example 29. Obtain the characteristic equation of the matrix
r' 0 2
A=l 0 2 1
[2 0 3
Hence or otherwise calculate its Inverse.
(Delhi Univ, B.A. (lions.) Eec,. 1992]
Solution. The characteristic equation of A is
A—?fl=0
11 0 21 [1 0 0=O
01
* I 0 2 1 I—Al 0 1
L2 0 3J [o 0 1J
0 2
0 2-1 1=0
2 0 3-?
*
—A+6)'-7A-2=0
By Cayley. Hamilton Theorem, A satisfies its characteristic equation.
herefore, we have
—A'+6A'-7A--210.
Premuitiplying by A 1 we have
- A' + 6A— 71— 2A-' = 0
A_11=4[_A246A-7I)

fOmB AOUirIONL TOPICS A31

ri
A2 =I 0
0
211
1 II 0
2
0
2
2] = ^5
12
0
4
8
5
L2 3JL 2 0
0 3 8 0 13
5 0 8]+6.[ 102]-7.^ 100
—A'+6A-71=—I 2 4 5 021 010
L8 0 13 203 OOl
1-5+ 6-7 0-1- 0-0 -8+12- 0 1-6 0 4
—2+ 0-0 —4+12-7 - 5+ 6-0 1=1 —2 1 1
L-8+12--O Of 0--O —13-i-18--7J L 4 0 —2 -
r
—6 0 4 ] . [ _
A-'=! —2 1 1
-3I 0 2
4 4
4 0 —2 2 0 —1
EXERCISES
1. Determine the characteristic roots of each of the following
matrices
8 —6 2\ /2 2 1\ /2 1 0
(I) ( —6 7 —4 1. (ii) ( 1 3 1 . (iii) 1 0 2
\ 2-4 3) \1 2 2/ \0 02
2. Prove that each of the matrices
/0 hg\ (0 fh\ /ogf
A=Ih of ), B=( f o g, C=( g oh
\gfo) \h go!
has the same characteristic roots.
3. Prove that the following matrices have the same characteristic
equation
/abc \ C a\ 1cab
A 1 = ( b c a ) , A2 =( c a b ), A3 =1 a b C
\cab/ \b cal \bca
4. Find the characteristic equation of the matrix
11 0 —1-)
I I
A== 3 4 5I

10 —6 —7J
Verify Cayley-Hamilton theorem. Hence or otherwise compute A_L
[Hint. Characteristic equation of A is
' + 2 N - —20=0
By Cayley-Hamilton theorem, we have
A+2A*_A —201=0

This gives A' [A'+2A—I]=51


r
2
21
6
—7 —8
4
—I8 6 4

A32 BUSINESS MATHEMATICS


5. Show that the matrix
c
A== r co
- 0 a
b --a 0
satisfy Cayley-Ilamiltori theorem.
[Hint. Characteristic equation of A is
A 3 4 1(a2 + b+ c2)=0
To satisfy Caylcy's Theorem, we have to verify that
A3 f(a2 lb2 j&) A-OJ
0 0
6. If A= "1I () I
'0 I 0
show that for every integer 11>3,

A ll = A- 2 + A2-!
Hence determine A 5° and A'°°
[Hint. Cayley-Hamilton theorem gives
A3--A2-A--J==O (s)

Premultiplying () by k>3, we get


Ak_ (A2 __ l) (A2-1) (**)
Putting k=h, n-I, •.., 3 in succession and multiplying the resulting
equations, we shall get
A 1 (A2-t)=At----I ; n3
An -A"2-=A---1 ; n>3
as desired.
To obtain A, put n =50, 48, ..., 4 successively in (*) and add
the resulting equations. Similarly for A'°°)
SUCCESSIVE DIFFERENTIATION
Example 30. If x- sin t, y= sin pt; prore that
(I-x') Y2-y1+py==O
dx dy
Solution.
-a--- =cos /, -ar-- p cos P
dy p Cos pt
dx cos I
OS I . Yi'P cospi
cos2 t . Y12=p cos 2 p1


SOME ADDITIONAL TOPICS A-33

- (I —sin 2 t) ; ' 1 2p2 ( I - sin pi)


(I x2)y12.p2( I --y)
Differentiating again, N y e get
(I---x 2 ) 2y 1 y 0 y 2 (-2x) -p2 . 2yy1
(I

ExmpIe 29. If y -- i e- cos (pi .-- -f e), 5/lOW that

+ 2k -- n'y = 0, where n' =p -f k2.


TI 2

Solution. -- k4e' cos (p1 -j e) -- pAe sin (pt -f e)

--ky—pAc -II sin (pt-fe)

or pAe sin (p1 e)= k -


Ift

Differentiating (*), we get


d2y d
ke' sin (pt+e) L e p cos (pt+ e)J
dly (it

pAe' sin (p(-e) - p2y

—k .k( ky )p2v (Using )


_

--2k p2+k2)y
tit
dy
= 2k - --- ny. Transposing c get the result.
(It

Example 20. If p 2 r=- 0 2 Cos4. b 2 • i°'e, prove that

el2p a2b2
p4 d0 2
p

We have
2:0? COS T e- b 2 S i 112 0
=a 2 (1 — sin 2 0)-f b 2 sin' 0
0 2 _ (a'-h2) sin 2 9 _(2)
Again pt=-.a2 cos2 0 b2 (l--cos 2 0)
(a 2 —b 2 ) cos 2 0-t-h2
(a 2 -1) 2 ) cos? 0=p2—b2
A-34 IiUSR435 MATH14ATICS
Differentiating (1), we get
dp
2p -=--2a t sin cos 0+2b i sin cosO

dp
p - a
=— ( 2 —b') sin O cog ü (4)
Differentiating again, we get
dlp((1,0
p +-) =_(a2....b2) (cos' 0—sin' 0) (5)
dp (a'—b) sin 0 cos 0
From (4), we get --=----
do p
Substituting in (5), we have
d2p ( a 2 — b t )2 sin' 0 cos' 0
Pji+
=_(a2 _b*) (cos 0—sin? 0)
d 2p (u T — b') sin' 0 . (a 2—b 1 ) cos2 0
- Pd62+
_(a 2_ bt) cos, 0+(a2 — b2) sin 0
dip (ai---p2 ) (p2_hi)
P do 2

[Using (2) and (3)]


a2b'
d02 P2

dp &b2
p2-1-p1=--—.
Dividing by p, we get the result.
E.&mp1e 31. Ifx'+2xy4-3yl, show that

(x+3y)' -- +2=0.
Solution. Differentiating the given relation, we get
dy
2x+2

dy
dx x+3y

[X +Y dy 1
dly 1+—(x+y)1+3
dx
dx2 (x+30'

SOME ADD1flOAL TOPICS A- 35

[(x43y) -.(x±y)+(x+3y_3X-3Y)
dv
(x+3y)'
2y_2x]-:(x+3Y)
=,_[
-- (-tY—)]--' (X+3y)2
Y+ -x^
2[
= —2 (xy-13y1+x2+xy)—(x+3y)'
=-2 (x'+2xy4-3y')±(x-F-3y)'
x2+2xy±3y=1)

(x+3y)'

LEIBN1TVS THEOREM
Statement. If fix) and g(x) be two functions differentiable up to
order n, then
'I
(fg )1 == "C, f_,g,

f
where the suffixes in and g denote the order of differentiation wYl .x.
Proof The theorem can he established using the 'Prinipl of
Mathematical
(cal Induction'.
Step I. By actual differentiation, we have
(fg) =jjg +fg1 == 'C0 f1g 'C1fg1
(fg)2=(f,g+f1g1)+(f,g1+fg2)
— 'cc f2g -I-'C, fa, -l-'C2fg1
Thus the theorem is true lot n==1 and n=2.
Step II. Let us assume that the theorem is true for n—m, so that

(fg) mC,f,g,
F=O
Differentiating both sides, we have
,n
g, +f,,_, g,41}
rO "C, {f,,,,.,
C0{f+,g+fgj± C,{f,,g,4-f_,g,}
+-c, ( f-1g1+f 2 g ±... + "C_{ f1 g +fg,,,1}
rC.f1
Step Ill. We know
mC. lIic0, _C_ +1 C.. +,, "C, + "C,_ 1 zr4'C,
A-36 BUSINESS MATHEMATICS

'a + J
= > g
F 0
Step IV. Thus if the theorem is true for n = 'ii, it is certainly true
for ?= rn 4- 1 . It is already verified for a = I and 2, hence the theorem is
true for all positive integral values of a.
Remark. We choose g for a function whose nth derivative is
known, and f should be such function that vanishes after a few
differentiations.
Example 32. Find i/ic nt/i derieaiive of
y:X 3 sin ax
Solut iou. I Icre- we take sill ax as .1 and x 3 as g.
Now 91=3x7, 92r 3. 2x, g3==3. 2, g40
flr
Also f,,=a Sin ( a.v-- , etc.
Hence by Lcihnitz's theorem ; we have
- . n—i
in ( ax-f . .--- , a 3x 2 . a sin ( ax4.
)
a -1) . /
-- ----,- . 3 . 2x u'- s ill ---yr

n(n - 1) (11-2)
a —3
- ----------- 3 .2. 1 a - Sill
sin t lJ X-f--------7r

Remark. If one of the factors be a power of x it will be advisable


to take that factor as g
Example 33. Let y=x 4 . e ; /indy5.
Solution. Here g—x,f=e"
so that 91 4x 3 92=12x 2 , g 3 =24x, g4==24
and 95 etc. all vanish.
Also f,, ac ; etc,
whence y5=u e°' x-f-.50 e 4x3 -j- 10 . a3 ex . 12x2
+ l0a2 e . 24x+5a e ll 24
a e' {114x4 4- 200x 3 f- 1 20a2x2 +240 a x-f- 1201
Example 34. Di/Jerentiate n times the equation
dy (ly
X,

(I'
Solution. ---- (x )2 )= x 2y,, + .±n . 2x . .Y+ 1+ n(n-1) . 2y.
(I,,
d (xy1) =X 4) -f- fl,.


SOM13 ADDITIONAL TOPICS A - 37

d"y
(IX'
therefore, by addition,
I 1) XV f1 -f (n
I (2'? 1) v ()
(jl) (/Y
Of -1--(2n 1 1) X j t +(' I' 1)--- :-()

Example 35. If J r--a (OS (log .v) f) sin (10: x), shov that
xv -I- (2n --f- 1) .vy., i (a 2 1) p -. 0
Soliti on. Differentiating, we have

—a sin (log x). -f b Co s (lo.) .

xY --a Sin (log x) 1-" cos (log .v)


Differentiating again, we get
0 cos (log ) /1 •-iii (log v)
-
. X2)1.XV1 _(a cos (log x) +b Sill (log x)J
X?y +X)1 *Y -

Differentiating !his cqnatioo 11 times using I ibnitz's theorem,


2v y rC -(-y.41-l-'-C1 . vJ fyI)
2. y .,
-F 2':xy^, -f- n( -- 1) y,, --f n ) !, f -,, 0,
1) XY, j 1?:(,:'-- l )+': i-u j )r)
Remark. It may be noted that nth derivative of j' -= y ., ;
derivative of y 1 ; nth derivative of j'

Example 36. If) r A (x± Yx? -- 1 ) -1- l3(x Ir


prove that
(a) (x2 —1) y 2 -f- vy1
(h) (x2 -- I) y-- (21:

Solution. Y: (x ± Y x2 -j ) -
174
t I - F 2Y-2._1 2x
+ fl B(x — x I—- 2x
( 2v x2—l)

- - n.-1 X-\X—t)'-
X-I

- 'I / (x - ' x--i)"

(\/ x—I) y 1 - ,lA (x-- \/ x—I)--nh1 (x- ' x-1)


BUSINESS MATHEMATICS
A-38
Differentiating again, we get

(\/'1)y'+ 2x . y1

.=rflA 1+ 2x }
{ 2V 32i .

n28 (x i_
.{ }

:n 2 A (x4- - ____
i/ x a — I

n2J3(/xI_I)-t
•1 x2-1
(x t — 1) y +x y 1 =n2 [A(x-1 V'x--1)-l-B(x— V x_1i5'1
= n?y (1
Differentiating equation (4) n tunes by Leibnitz's theorem,
(x2— 1) y+ 2 + ' C 1 . 2xy, + "C2
. 2 . y,,4 xy,. 1 +C1 . I . y-= n'y

* (x'— 1)y 1 -(2n+1)XY+1=O


Example 37. Ify 1 ty l/ 4 2x . prove that
(x t — 1)y 2 -j (2n + ] ) xy41+ (nt-- m2)y.=0
Solution. y1 l+y -'I" = 2x gives (yI')2._2X(Yl 1n)4 1=0
yI={2x±V'(4x2.4)}/2=X±s/ ( X 2 _ 0

Thus y[xf/ (x'-1))'

If y==(x+\/X-1Y, then
[i+

= ,nx + \/ )m/.\/(x2_ 1)=nly/VX'—1

If y(.i_ / then

y_m(x - 1-

- zn(X -- V xt_1) M /VX 2 I = - my/V xt —1

Thus in either case


m ty 2 . (x2_)y1?=FflY2

- (x' - 1)
Differentiating, WC get
(x2— I)2y1y2Y1221YYl
r. (x2— 1 b'2 • xy - 1?1 2Y= 0

SOME ADDITIONAL TOPICS


A-39
Using Leibiitz's theorem for "•tirncs differentiation, we get
r(x- l)y + "C1(2x)y41 + ' C2(2)y.J+[xy,+1 ftC1 y. ]_
rn'y=Ø
£
* (x—l)y42(2n+
Example 38. If c03-1 (y/b)_ log (x/n)', prove that
(a) x 1y3 -xy1 n'y=O
(b) x2y.+2f(2n+ l)xyf1_32n1y,r
Solution. cos'(y/b)=n log (x/n)=n flog x— log nJ

Differentiating, -v1==

n

,1(52
_y2)
Differentiating again, we get
(-2yy)
Dividing by 2y 1 , we get
x2y 2 +xy 1 +n2y=o
Differentiating n times using Leibnitz theorem, we get
f x2y 24- c1 2x.y+1f-C2 .2 .yJ
[xy 1 -f-"C1 . I . yj- ny=O
c3. X 2y,,+ 2
n xy+ +fl(fl—l)y+xy+i+ny+n1y0
* xy2.+2-f(2n4
Ezmple 39. If y -(x-3- /(x27? prove that
(I+X2) y,-4-xy1-_p'yO
Hence find the value of y when x=O, n
being an even integer.
Also find y (0) when n Is an odd Integer.
Solution. We have yc=(x--./x -f-I)'
Differentiating, )1P(x+Yj)'-1 . ( 1+—)

• ±±
VXZf 1
=p (x+
=py/Vf
\i%/fl
or
,
i.e., (r2 + I )y2

BU3INf3SS MATIflMAT(CS
A-4t)

Differentiating, we get
(.v 2 }- 1) 2 yy.2 4-. 2x.y 1 21)2yy1

Dividing by 2 y 1 , we get
(x 2 + I )y, ­ x =p3J' S. (3)
which wits to be proved.
Differentiating ( 2) /' tunes h Lebnitz's theorem, sc get
"C ) . 2x y, 4 r C ! . 2
Simp!if) ing. we get
(.0- 1) y—, 1) k Yf ) -l( fl •p2)y=0.
Putting .V

(pa-- n1)y(0)
From (1). putting .v =0, .v(1)) -= I.
From (2), py! /iv2 I
i() py(})/l =p.
From (3), putting A =fl.
y(0)=p2 y(0)= 11
In (4), put ?Z 2. 4, 6, ... successively ; then
= (P*-'— 2) y .,tO) =(p -.-22),p2
42) (p-22)p2
v(0) =(p 2 —4 2 ).y 1 (U) =(p.
yU) .(p? 6) (p 42) (j, 2 -2 1 ) etc.

Hence, when I t is an even integer,


y(0) [ p in —2)2][p -
4), put = 1, 3. 5, . successively then
it
(0) = (1,2 -- 12) p
y3 (0) = ( p2 - I ) y 1
(p? - 1 2 ) p , etc.
Y (0) 3) Y 3(0) = (p . )
Hence. slien /i is an odd integer.
y(0) = [ 1 ,2 - (n -- 2)?] [i - 4)2 1.. V 1 2)p.

EXERCISES

I. If y , prove that
\ A.

(1 — x 2 ) y — ( 2 j 1) x - y, -- 11 2 y, 0

[ I-lint.
,.,/l\! . .v sin -1
Differentiating w. r.t ..v, we have

V 1 x7. Y 1 +Y . (I --x" 2 (-2x)==


—x-

(1—x)—Y. —l0 .. (•)


We now apply Leihoit rule for 17-times differentiation.
II X 2 ) v -1.C 1 (-- 2.v)y -I- 1 C2 (_2)y1}-1XY1 --C1yJ=O

SOME ADDtT1CNiL TOPICS

(I —x 2 ) y.+i—(211+ 1)xY, —fl2Y-i 01


2. If Y- sin (m sin x), prove that
0
(I - x 1 )y 42 -- (2,1-4- i)xy --(nt— "")Y,
3. If y = cos (m log x), show that
x 2y. - ( 2n - I )xy. - (r n 2 +fl ! ) y Ø•

(1-lint =
- - sill (m log x) mJx
2 y 2 =-ni sine ( III log x)
=rn 2 l — cos 7 (m tog x)}
=-'n 2 (t—y).
Differentiating, we get
X2 .

Divide by 2y, and differentiate n times by Leibiiitz's theorem.]


Slfl show that
4. if y= ecx X ,

(t) (1
(ii) (•.)y.—(2'

[hint. y 1 re'' jr . ct/v'l.Y/V'1


Y13 (I __x)=='y2.

Differentiatilig, we get
([_xt)2y1y_2XyI212cL.YY1.
n times]
Divide by 2 y and transpose. Then differentiate
X2
5. If y -= -( --- l). prove that
(x'-- l)y.+1+2xY+i+l)YO.
d,
Bence if 1'. = - (x'—l), show that

0.

[Hint. y 1 =fl(x 2 l) n . 2x
Mul t iplying by x 1 —1, we get
2x—ny . 2x.
Differentiate ('1 ± I) times to get the first result.
Now
Hence the second result required is
dr d
(yb) ]+ nn +n Y.
,TIC A
d ((l_X 2 )y+ i J4fl(fl .+
l)y—O,

A-42 BUSINESS Mi11iIMA11cS

Multiplying by --1, we get


(x1 — l)y.+2+2xy+1—n(n--l- 1) y- 0,
which is the same as the first result already proved].
6. Iff(x)=tan X, prove that
f"(0)_ " C2 f"-'(0)+"C4(0)--.-... =Sill (nw /2)
[Hint. Jx)sin X /COS x
or cos x . f(x)=sin x.
Differentiating ii times, we get
cos x f(x)+"C 1 . (—sin x) . J 1 (x)+C2 . (—cos x)
=sin (x+nrr/2),
Putting n =-O, we get the required result. f"(o) means the value of
f(x), when x= 0)
PARTIAL DIFFERENTIATION

Example 40. Find the partial derivatives with respect to x and y if


Z= 3xy —y -1- (y2
Solution. 3xy_..y+(y2_2x)3l2
=3y+ T(y22x)l2 (-2)== 3[y— (
y2 --- 2x)' ']

-i- (y 2 -2x)" (2y)

= 3[x_y2 -}y (y2--- 2x)' 01

Example 4 If u=si,r -- -ftaPr 1'- , show that


all au

Solution. u=-sin` ---+tan :!


Y
Ou
ax
)
Y X,
_L
= \/y2 _x2 XI+Y1
au
X )2 -(—)+2.
Y ) Y, ^(Yx
+

V2
X


SOME ADDITIONAL TOPICS A-43

0.

Example 42 If u=f ( y- ), prove that

au u
av
Solution. Differentiating partially w.r.t. x, y ; we get
ç, 3LI (Y (Y\
5
)J J
( y /1
and
X

x_+Y=±)f' (--)+. L •j' (L)o.

Example 43. If f(x,y)=log ( x2 + y2 )±Ian -1 2_ , then prove that

± 4._Lrrr 0.
x2 ay,
Solution. f y )= log (x2-f-y2)+tan-1

f 2x I/))

2x—y
- x2 + y1
a 2 f (x2 y? ) 2—(2x—y). 2x
ax 2 (XL ty')2

2y 2 2.v 2 4- 2xy

f 2y
X
Y2
+ 1+--
2y t x

f (2-4-y2) 2 -(2) -x) . 2y


av2 (2+ yl )2

2x 2 2y2


A-44 BUSINESS MATHI3MATLC3

• a'f a'f_
BY

Example 44. If Usshow that


how
a'u a'u a'u
* +y2+)
Solution. We have
au --
- _(x2 +y i + zl)_ S I 2 . 2x-= —x(X 2 fy2+
Z2)'
--
a2u
3x1 (XI +y2+ z2) 2
ax!
Similarly, we get

(x'+y'+Z')'12+3y' (x2+yj Z2)_5l


ay 2
atU
and
aza = — (x2 +y'+z 2 ) - " + 3z2(x* - - y2 + z)-'/
a2uu 'u
Adding, we get -- +- +
axt

== .3(x2 +yt + z2) _ 31I + 3(x' + y + z 513 (x' +y'+ z')


—3(x'4-y + z')-' + 3(x*+yt+r)_3
=0
Example 45. Jfu=e-' cos (x — at), show that

ar2
Solution -!f_=ex_i cos (x—at)-f-e-' [—sin (x—at)]
—e- (x—ar)--sjn (x—at)]
a'u
-=e' [con (x—at ) — sin (x—al)]
ax'
+e" (—sin (x—at) —cos (x—at)]

=-2e'' sin (x---ai)


au cos (x _al)4_ex_aI [ — sin (x— at)].(—a)
at
r-=ae" 1 ' Icos (x—aI) —sin (x—at)]
atu
—ae-' (—a)[cos (x —at)—sin (x_-aI)}

—ae'- ' 1n (x — at) - -acos (x-- at)


=-2a'e'--' sin (x—at)

SOME ADDITIONAL TOPICS A-45
a:u a2
tz --
Example 46 If u(og r, where
show that
a 2u u u f

Solution. Differentiating partially, we get


U I a r -
(x—a)[.: 2r - . 2(x-- a) ]
X r ax V2

a2u I '\ ar I 2(x—a)1


--i+(x--a) -----,----
r2 -2(x - 0)2

V4

Similarly
2u r2 _2(y_b) 7 ' u r2-2(z---c)'
y2 '
Adding, we get
a2U 5 2u 3r1— 2
2u
[(x—a) 2 + (x—b)3 +(-'c—c)9
r4

Example 47. If u=f(r), where r2= xI +y2, prove that


a 2 1
_—=f'(r)+ --f'(r)

aT 1
2•(x$y ) l$ 2x=—
Solution. r= /x+y2 =
ax
u au ar x
ax ar ax -J •

alu r [ f'(r) + xf"(rl'—r ] — xf '(P). 'r


l ax ax
TI
I
=-- [rf'(r)+xlf'(,)-_.(x2/r) f'(r)]
—1
f'(r)+x If'(r) f'(r)1
r
Since r is a symmetric function, by interchanging x, y; we get
a2 u I f(r) f'(r) 1
Trf'(r)+y1rL (*)
j-y! r' - r J

BUSINEM MATHEMATICS
A-46
Adding (*) and (**) we get
3 2" 321i 2 fr)]
r 2 - r3
- 2 f(P)+T2rLjr) D-0
Lrrzi
2
r
ô2u i 2 u 1
Hence +y2 f'(r)+f'(r)
ã•.
Homogeneous function. A function f(x, y) is said to be a
homogeneous of degree n if on replacing x by kx and y by ky, the function
is multiplied by k r', i.e., if
f(kx, Icy) = k n f(x, y)
For example log x -log y is of zero degree since
log kx-log ky=log x-log y=k° (log x---lo y)
Again V x2-yt sin'-- is a homogeneous function of degree 1

since i/ (kx) (ky) sin' L =- k I .1 sin-, L.

Another way of defining the homogeneous function f(x, y ) of


degree n is that it can be expressible as

x'f()

Now log x-log y_x° tog -

and /.-sin' -is x' [,,/(Y)2 sin'-j

EULER'S THEOREM ON HOMOGENEOUS FUNCTIONS


Statement. If z=f(x, y) be a homogeneous function of x and y of
degree n and possesses continuous partial derivatives, then
3z Z
x--y =nz
ax ay
Proof. Step I. Since z -=. f(x, y ) is a homogeneous function of
degree ii in x and y,
z-=f(x. y)=x1 q5 (y/x) (1)
Step Ii. Differentiating (I) partially w.r.t. x, we have
8Z a [x(y/x)
ax
[(yfx) . i x 1 1- t 4 x (x/ y) ( -y[x2)1


SOME ADDITtONAI. TOPICS A-47

or x-- nx"(y/x) —x ' yb '(y/x) (2)

Step lu. Again, difIercritiating (1) partially w.r.t. y, we have

DY ay
.v' '(y/x)(l/x) -x'' '(yfx)

or x-1 0 ,(Y/X)
Step IV. Adding (I) and (2), we have
z az
Y -nx"
Oy ay
This proves the theorem
Deduction. if z f(x, y) be a homogeneous function of degree n,
then

- 2xy . - fy 1 _: fl(fl -- I )z.


ax 2 axaz ay!

Since z is a lioniogeneous function of degree .,, L and - are


ax
homogeneous functions of degree (rr
az
Appl y ing Eulei's theorem to the functions --and -, we have
ax
) Z z
) + --(---
ax) (n— I)---
ax
Z) a ,az'
and x ax
- ( - --y - (
ay
= (ii - 1 )-
a2z a'z az
le " x --'1-y —=(n—I)__ (1)
ax2 aya ' ax
az 'z
x
xy +Y a2=(fl1) (2)
azz a'z
Multiply (1) by x and (2) by y and add, taking
aX t -+2xy 1 : atz az azl
xy +Yt=(n_t)[ x— +Y_- J
ax2ay
But by Euler's theorem x az ) az
ax
az z z
x-1+2xy á
-----+Y'------n(n—zz.)

A-48 BUSINESS MAT1tEMATLc

- - i x -
Eaniple 48. If f(x, y) = y2 •-2

show that Y _ f (X , y)-= 0


ax ±Y ay

Solution. f(x, y ) = \/ T' sin


k2x2 /<2 2
f (kx, ky) =- /k1y2_k22

=k[

f (x, y) is a homogeneous function of degree 1.

Hence X-i[-y..L-rf(x,y)
ay
or XLY_f(X,Y) -O
ax dy

Example 49. If u-=COS (_) ), prove that
L" au u
X —+y------fz -=0
x ay az
1Xy--y2•-zx \
Solution. u=f(x, y, z)=cos 2212 )

k2 xy+ k . yz-.k2
f(kx ky, kz)=cos
(- )

=COS(4)=kof(x,yz)
X2+y7+z2

u is a homogeneous function of zero degree.


au
Hence x±yi+z_=oxu=o

Example 50. If u_s1n , show that


au au
—=tan
ax + y y
X U.

Solution. Here Li is not a homogeneous function but if


x2
z=sin u= --Y2,then z is a homogeneous function of x, y of degree 1.

By Euler's theorem, x --+y -=Z


ax ay

SOME ADDiTIONAL TOPICS

Since Zrsin it is a function of


z dz Oil all
- - ---- =cos U
x du ax ax

aZ (IZ U
and -- =cos u.
y u a)'
Substituting these values in ('), we get
Dil all
X COS U -y----- cos U sin U

all all
or x --4- y - = tan U.
ex ay

Example 51. If zsin (-:. show that


\ •/ x + y I
au Y au
--- - x ay
i-—Vy\
Solution. sill ` i
Z= - ._- I X Sifl I
\V x / 1—yIx

z is a homogeneous function of degree zero.

Using Euler's theorem, we have


F.
x----- 4- Y - - = 0. z= 0

ax x •y

Example 52. If it
+Y3')
(X!+2),
2 ,Prove that x -i-a --
M

3-fy3
Solution. u=log (xX1+y2) e

[1'(jx')
-
Li -I- ( y2/x2)
Here e' is a homogeneous function of degree one.
By Euler's theorem, we have
(e l ) + y —p-- (e s ) = I . e
DY

X+YWl

A-50 9USiNFM MATHEMATICS

EXERCISES .
1. Find the partial derivatives with respect to each variable of
(i) f(x, y, z, i)=x2 e2YF31 cos 4

Z) r(r - cos 20)


(it) f(r, 0,
r2-I-zt
2. If?(x-1 y)—x-4 y 2 , show that
Ia z az
ax ay
3. If u= /x.fy2.;z2 show that
( au_ )2.i_/ \ 2
/

4. If x 1 y2 -j- z J,j prove that


a?u a 2u u -
— j +--
ax ay

5. If u=- -2 tan- 1 Y —y2 tan 1 prove that

x2 -y2
ay x+y2

xy
6. If u=taii-' --j-----, show that

- (i-1-x2+y)3

7. If (i) z= log (ii) z=(x—y)V2,:-i-'


XY
tZ a2z
verify the relation

8. If z =r tan (y+ax)—(y_--ax)312, show that


- -as

9. if Z :3xy-y-3'(y 2 -2x)3/2 verify that


a ? z a1Z 2z '
a ( 2z
and ----
axt =j

10, If u=)og (x t 1.z._.3xyz), show that


au au a u 3
(a)
.,.5 I
SOMI! ADDITIONAL TOPICS

'jU a 2 u a10
(h)

11. If U=Iog (X2y2 f z Z), show that

- --= z
ay az zx ax—
ay
12. If U=(y-ax) 4- (y --ax), show that
a'u
-
0x 2 ay1

[Hint. f-' (y -+ ax) a +/ (y - ax) (—a)

alu -'
() , H- ax) a 2 4-4' (y---(Ix). a'

all ='(y- ax). I + ' ( y ax). I


ay

(),+x)+ (Y- . ax)]

x y
11. If U=rSlfl 1 tan- ' -, prove that

all au
x --- y
ax A
14. if z =xyf(y/x) prove that
az
v -- -
ox ay

U. if z= sin - ' prove that

z az
X --- f- y - ---=tari
ax ay
16. z=tan' ( prove that
),

4_Y___ = sin 2x
cX cy
A1 1
17. If u= sin-' (r77s)' prove that

au au I
X +Y tan


A- 52
BUSINESS MATHEMATICS

18. If U=COs'
xf-y
-•-------•--------, show that
\/X+ /y
u au I
x- +Y - + --- cot u=0
ay
EXAMPLES ON INTEGRATION
Example 53. Evaluate' __________
J Sifl x CO5 1 X
r dXr --
Solntioij.
inXc0s2x
=f sec2 x dx+f cosec 2 x dx=- tan x----cot X.

cos x
Example 54. Evaluate dx.

Solution. Let I=- dx.


J cos (x±a)
Put x+a=t (IX =(It

1-_-f cos(1—a) dt _r acost-4-s,itsina (1':


- I cost -- J cost
=cos of dt .4-sin a f tan t dl
=rt cos a+sin a log sect
cos a+sin a log sec (x+a).
r dx
Example 55. Evaluate

dx (dx
Solution,
J,V: +SIfl X j
—=1
/ COS2- + Sjflh --+2 sin 4-cos -.-

I dx 1 (dx
X X\4/2j x n
cos -- -f sin cos cos -- + sin -.1 sin 2x

I dx
---=— J see (f+)dx
cos (--)
(-2) X \
log tan +
i - T)
Example 56. Evaluate dx
J sin (x— a) sin (x—b)

SOMF ADDITIONAL TOPICS A-53


dx
Solution.
sin - a) sin ( - b)
I f sin (x-a)
sin(a --b)
sin (x-b) (Note this step)

1 Sin ((x-b)--(x-a)}
f
sin (a3 J sin (x-a) sin (x.-b) "
1__________________
sin (a-b) f sin (x-b) cos (x--a)--- cos (x -- b) sin(x— a)
- dx
sin (x--a) sin (x--h)

cot (x—a) dx_J cot (x-b) d ]


= sin (a -- b) L
=cosec (a-h) [log sin (x—a)--log sin (x-/,)]
I
cosec (a -b) log sin (xx--a)

sin 2x dx
Example 57. Evaluate
J
sin 2x A C 2 sin x cos
Solution. Let r=rIsin'
XCOSX j SIfl.V+C&X dx
f (2 sin x cos x)/coS'
A
= J (sinx+cosx)'cos
f 2 tan x se V
) 1tanx
f- -
Put tan2 x=t 2 tan x sec' x dx=di
di
• • l+( =tan' t= tan' (tan 5 x)
Example 58. Evaluate Irafr \/ _!_— x dx.
I +x
solution. Put X=-COS 0

L-x
tan-' • i = tan' I_!_. =tan' ( tan __ )
J I+x 1+cosO
O 1
cos-1 X

4 I
dx== - J (cos-' x). I d
X
_f- ' 4- -.-, dx ]
J V 1-x3

A54 BUSINESS MATHEMATICS

rr[Xcos - X— V1

Example 59. Evaluate


I sin (j-;--) d.

r5_1
Solution. Let 1=
\J
Put X---il tan 0 64-=21j tan 0 sec2 0 dQ.
I f / a tan() '\
I - I sin' ----) 2a tan see2 e do
J \j a-i- a tan2O '
tan O
= 2a I. l ( ---------- tan 0 sec' 0 (16
J sec0 I
r -2a $0 (tan 0 sec' 0) d
-2a [0 tan 0-f I tan' 0 dO)
rO [0 tan' 0-- J(sec2 0-I) tie]
-a [0 tan" 0--tan 0 UJ

a[tan'J} -Jtan' ']

Vax

A
Example 60. Evaluate
X(X+ 1)
dx
Solution Let 1- ¶
j x (x'--I)

Put x=_t .-. nxn- 1 dx=jt


I in
1

di
(1i l)LtJt1] dl cdi

-- - [log f—log ( t + log


(if )
I Xn
_____
log(
-- fl x+1 )•
Example 61. Evaluate Jlog (x+ \/&--x) dx.

Solution. S I . log (x+ \/ a 2 -t-x2 )dx=x log (x+\/){a2)

('+-- ) dX

SOMe ADDITIONAL TOPICS A-55

- x log (x v /rx7 - dx

X log (x I- V'a2 l-x2) —31 2x(a 2 +x 2 ) clx

x lo g (x 1- a -t x ) -- V a - x

Lvaluatc - - 3x
-
Example 62. E valuate
J cos v
Solution, sin 3x=j (x+ 2x)5jit x cos 2x cos x sin 2x
=sin x (2 COS 2 -- 1) j - c os x sin 2x
f sin 3x 1 2 sin 'c cos' x --sin x-f- cos x sill
/x -- ---- -- dx
J cosx J cosx
J(2 sin x cos 'c -tan x -f sin 2x) dx
1(2 sin 2x -- tan x) dx
- –cos 2x -1 log CnS x.
Example 63. Evaluate J cos 2x log (I+ taii x) dx
Solution. Integrating by parts, we get

I sin 2x
cos 2-v log 0 1 tan -c) dx - —i-- Tog (1 -I-tan x)

I sin 2x sec x
- dx
J 2 1 ltanx

• sit)
-'3 sill (H tan ---(IX.
J sin x+ Cos X

sin2 xlog (I-I-tan 'c)-- II
cr ii (cos x - sin x1
A- I---------- I dx.
sin X-fcosX j

= sin rr 1 1 cos x—sin xl
2x log (14-tan x)----- II -- ---- I dx.
J L 2 2 sin x+ cos xJ

=- sin 2x log (I - tan x)—-


-- -- log (sin X+cOS x).

1
3
Example 64, Evaluate dx.
V
^ _ 0 XI)

Solution. Here - – (sin' x)-


c ( •c

Substitution is sin- 1 x .1 so tlat;- dx di


A-56 BU3 1NM .lATIIMVFtC5

Also when x=O, (=0 and when x =1. s ince sin - =1


2 2
t 42 t/2
x3 sin-i x
dx= I
I
sin 3 t dt= - (3 g ia I—sin 31) /t.
I /(l—_x)
o 1) 0

t/2
- - -- C sin I dt - - I sin 3t I(
o 0
,tJ2 m /2 t/2 r/2
3 I - -
. cos idi [-_L.] I Cos 3' di
t Cos C ] 3 4 3
o 0 0 0

42 42
3 [ sin t ^O — L
n 31 1
o 0

3 1 28 7

dx
Integrals of the type -_
where X and Y are linear or
J Xi/y
quadratic expressions iu x.
The following substitutions will render the above type to the
integrable forms:
Case I. A' and Y are both linear.
The substitution is Y=t'.
Case It. A' Is quadratic and Y is linear.
The substitution of Y=t3.
Case III. A' is linear and Y is quadratic.

The substitution i s X= I
quadratic.
Case W. A' and Y are both
I
The substitution is -i-
dx
Example 65. Evaluate (i) f
X2 -1-1
(II) dx.
(3x+2) /(x—l)

SOM13 ADDITIONAL TOPICS A-5')

Solution. (1) Put I ---x1 2 dx= —21 dr and x 1 -ti


(-2'_di ç di 1 1-4-1
h)(1_1z)12 --fog-j—j-

1+1 -
=—log

(ii) Put x—I =t so that /x=21 i(.

di
1(3x f-2) -V( x IT J (3i{ 3--2)
11-1 1 13 2
2 di2JL 3+9+9(313f5)J di
21 2!
2 26
-f

2 26
-- xV (x — Tj-j.— tan
vI(3)
3

Example 66 Evaluate --- -- -- --= - A
I (x 3 -f2)t/(x.j)
Solution. Put x-{- 1 SO that dx==21 (/t
• I Xy2 (t3+1).2zdi
dx—
J( x f3x+3) .iT45 J(12_L)1+3(2_1)±3j

- -2'U'I( I
= 2j dt.

r du
where

2
1 14
tanr ç- =-;- tin-' /(3x-j-3)
'i--I x 2

(jx
Lxm p1e 67. Lvaluae /
^ (x-3)

Solution, Put x-3=rJ_ so that dx= - --- arid x=

—di

-- ___ ______

ift
--j V:6i

A-58 us1r,n3s3 MATHEMATICS


di
-Sifl t -siü-1
X-3

mple 68, Evaluate


Example x ____

Solution. Put XI+ so that

Taking logarithmic differentiation, we get


2x' 2
—2---2--+1 - )d=--- di
22x-l-2x--2
(I—X2)(Xj.l) -- di
t
_ 4x(fx 2
(1—x2)(x'+l) -1d1
dx 1 v'(j- x2)

- 1i2)2./ji

ritdt 1
_ 2f I
dt-2 sin-' I

=— ,'2 Sjfl1 (
VI
it/4
Example 69. Evaluate i/ian xdx
J
0
Solution. Put /=: so that tan x= to and sec' x dx=21 dl
21
d
l+tanx 1+1'
Also when x=O, i'JtanO:=()

when x=-'_, t=\/tan_4.


Hence the given integral becomes

fl.21d1 to
j
0 0

50MJ3 ADDIflONAL TOPICS A59

I
12__1

=2[?log 4±i./2 tan-' J—

0
2—/2 V2
2[- tan-'
--
(V 2 log 1_2tan_1co}]

in
—2r1o/2
- L8 v'2 •2J
g--2--

V2
log 2— V 2 /2 I 2—/2 1
= -;r 27 log2 +

r
Example 70. Evaluate smnx - dx and hence find the
ii XCos
+ X
0
a
_ ilx
value of
Ix___

sin x
Solution. Let 1= dx, then
sin x+cos X
0

sin Ix \
d'
o sin (_x)+cos(_x5
n12
r COSX
dx
= cosx sin _
x
0
7t12
Sin dXfc dx
J Sin X+COSX J SiflX --COg
o 0

= dxr-.
... l=1
0
a
AToevalut put x=a cos 1, then
x+i/(a_x')
dx== —a sin t di
JiUSINCSS MATI1UMAflC

Also when x=O, t=—- and when x=a, t--)

a 0 rc/2
dx a sin I dg f sin t di
x/äiTTi yJ a cos t+a TIJ os t -+sin t
0 :t/2 0

Eamp1e 71. Prove that


f log (I+ tan 0) di - - log 2
a'id hence find the value
I
log (l+x) dx
J I± x1
I)
44
Solution. Let 1= log (I +tan 0) dO, then
0
44

'=1 log [ l± tan (..!..o )]iio


0

44
1• 1±Etan 2
log og —1----------do
L l± tan Oi =
o 0
44

=1 log
0

J4

21_1 log 2do=-- log

I!og2
8
For the second integral put x=tan 0, then dxscc' 0 do

Also when x-O =0 aud when X ---1, fj=


-Tc

OMB ADDrrLONAL 'lopics

[log (I+tan 0)
sect Q dO
•j 1
j - X2
j sec20
0 0
,c /4

log (I +tan 0) de=-1- log 2


o

()
dx
Example 72. Evaluate log ( x+ .
0
Co

Solution. Let I log (x+_-_)

v — tan
Put 0

(1vSCC5 0 (1O
(..)

Also, when x-- c, () gives 0 - Tc

and when .i=O, O=O


/2
I ' sec2O
1^- ^
log ( tan
o
0

= [log
j
(L_Q_) tan
dx
0

12i

=log 20)dO
sin
0

(log 2--log sin 2e) dO


0

71
rr log 2— log sin 20 dO
0

A-62 BUSIN6SS MATHEMATICS

it/2 itl2
log 2 (log 2) do log sin 0 dO

- log cos 0 d

=- log 2--T- log 2+-- log 2+-- log 2

= i log 2.

X tan
Example 73, Evaluate iecx - dx.
f- Iax

Solution. Let I=x, dx, then


i Sec x -F tan x

1f(lr—x)tan(n.-._x) -
sec (1r—x)jn (71---.x) d
0
it
f (ir - x)tanx
dx
J sec x±tan x
0
it It
t lT tan x
J sec x+ tan x see j x+tanx
o 0
IT

= f1 -dx--!
j Sec X--tan x
0
IT

r tanxdx
21=
j secx-ItanX
0

I
=,, ' (sec x —tan x) tan x dx
0
it
S&x—tarjX

X tan x-(sec2 x- I)] x



SOME ADOCI1ONAI. TOPICS 63

= [sccx - tan .v - -x IT

REDUCTION FORMULAE
'r/2

Example 74. Prove that sin" x dx cos" x (IX


0 0

I
n(n-2)(n-4) ... 7T
when is is even
i(n—l)(n-3)(n - 5)...4.2 .1,
I n(n-2)(n-4) ..5
when is odd
n!2
Solution. sin" x dx= sin" cos a x dx
—x )=

[.: AX) dx=Jf(a_x)dx]

Now J sin" x dx-rJ Sinn-1 x . sin x dx


x ( — cos x) — f Q?—I) sin" •2 x cos x (—cos x) dx
- (Integrating by parts)
x cos x-j-(n—l) j sin"' x (1—sin' x) dx
x cos x +(n -1) f sin"' x dx—(n-1) j sin x dx

R12

Let 1= sin x dx. Then by transposition, we get

—1) x cos x ]4 (n — i) f_,

• r
is

fl—I n-3
1i4 (Changing is to n-2)

A-4
BUSINESS MA1*1PMAUC
flj-i fl-3n-5
n n— n-4
(Changing n to n--4)

and so on.

• I - (n— I)(n-3)(r__5) ... 3 1


_ ____________
- n(_n— 2)(n--4)...4. 2 10, When n is even

(n I)(n-3)(n_5)4 2
- — n(n-2)(n-4),..5.3 when n is odd
7!2
Now !Osin- • dr dx=[
o
J
0 0

and sin x d
1 r_ cos x

l2
Example 75. Prove that J in x cos" L- J cos x sine x
0 ()
{J35 (,iz_ !) {i.3.5. ..(n_J)} ir
I 2.46,.. T
I
-
when both m and a are even integers.
- (rn— i)
(±i)(n+3)...(n+rn) when one of the two indices, say
I
n is an odd integer.
S110 S Sin" x cos" x dx==j cos"' x (sinm X cos x) dx.
I ntegrating by parts, we get
Sifl
J sin- x cos x d cos ' x_J (n -1))cos 2 x (— Sin x)

x1'"1 ' dr
" rn+1
1xcos"-'x n—I
sin M+2 x cosx-3 x dx
sin'xcOs"-'x n—I
+ f Sill" X
cos" x(J —cos' x) dx,
5jm cospt-1n — I
,n-- + f sin .r cos"-' x A

— ---j- , siI n" x cos" x A.


n—I

SOME ADDITIONAL TOF'cs

By transposition, V' C Fet

( - h-- f sine' X COS" -' X


1------------ 11 Sin-" xeos' - (IX -
in I- it] -I--I

x cos' x 'Ix.

Ir Sifl .1 COS" xd-=j -


L
sill" , x cos . -- -
rn-I--n
0 0

-1- 1- i-
-
U 12

I
0
sill' x cos" 2 x dx.

nI
,fln Jf
-+-
.sinxcos2dx
0

I
n12

or = X COS X (JX.
ffl-fflj
0 0
It can he similarly proved that

sin x cos 11 x dxJ sin 1 x (cos' X sin x)

rc12
in—IC
sin' 2 x cos . dx (proceeding as before) (2)
=--- ,)
0
1-Jere, with the notation,
n/2
- x cos x

We get, from (I) and (2)


fl—I ," tn—i
""

Thus i_. -±
m4 rr

(m—l)(n-3)
-


A-66 BUSINESS MATHEMATICS

00-_3)(n._5)
(ii— - _____ -
(,n -- n)(m - n —2)(m ± n —4) -6 and so on.

(n-1)(n---3)(n-5)..J

where n is an even integers


C (n—l)(n--3)(n-5),I . (in-1)Qn-3)...1
I (rn--n) rn- i )4 . 2 2
when in and n are both even integers
(n--1)(n-3)(n-5)-.,3 . 1(in—l)(m-3 4 .2
I (n+m)(n±rn-2)...(n-_3)(n_1) ... 3 . 1
when a is even and in is odd
C_ ( m— Win — 3)...4 . 2
(in -l-n)(Fn . n-2) ... (n1T)
- I when n is even and rn is odd
I(n---!)(n-3)...4 .2
('n+n)(rnt-n-2)...(in-_1)
L when a is odd and III ever)
When both in and a are odd
I (fl--1)(n---3 ... 4 .2
(m -f-n)(m + a —2) . (in -+1)
(rn—l)(,n —3)...4 .2
L (in + nJ(rn -I- n —2). . . (a + 1)
IT/2

Also sin' x cos x dx


0

sinm (-I—x ) cos*x x dx


o 0

Here I,.. = __ - -
2.4.6 ... (in+n-2)(rn+n) 2
when both m and a are even integers
2.4.6 ... (m-1)
= (n +l)(n+3) ..(n+'n)'
when any of the indices is odd.
Exampb 76. If I_, =1 cos"' x sin nx dx; show that
— CO5"X Cos flx m
•mn I__I, U-I.

SOME ADDITIONAL TOPICS A-67

Solution. Taking cos- X as the first function and Sin ,tx as the
second function, we have oil by parts,
I COSUX
I =cos'xç_

-J In cos' x (sin x)( cs flX )


dx
Cos n x cos nx in r
x sin cos nxdx ,,(4)
Now sin (n-- 1) X=sjn ( nX -x) = sj ri nx cos x-cos fix sin x
sin x cos nx = s jn FiX cos x-j ('i—I) x
Substituting this value in (), we get

I.. ,,-- Cos' X Cos flX


f1


ti I cos' x (sin n. cos X-sin (n—I) x} d
-- cos" x cos lix in

+'- J cos x sin (n — i) x dx


in cos" x cos fix
fl -

In I
Cos-
+7j -' X sin(n — I)xdx

* cos" x cos flx fn


M +n --
Remark. There is another form of the reduction formula for
f cos - X Sin nx dx

cos"xsrnnxdx= COS'XCOSflX mJ
J rn—a —+--- m n iX sill (n+1)xdx.
This is left as an exercise for the students.
Example 77. 1j1., ,=f cos" x cos flx, show that
fix
I = x sin rn
COS
m--n rnn
Solution. Taking cos" x as the first function
and cos nx as the
5000nd function, we have on integrating by parts,
innx J
Im p '=cos"x. - rncos'x(_sjn
'5 )

A-68 BUSINESS MAIHLMATJCS


COS" X sin ii in f
= ------------ -f- - j COs" x (sill sin x) A (')
But cos (n-- 1) x=COS (nx--X) - coS , x COS x - sin nX sin x
sin rjx sin x--'cos (n-- I) x— Cos fl y cos x
cos' x sin 11-V
:- -
() gives 1,,,,

III
±- Cos --I x (ens (ii I) xcos nX cos x)) /x

COS" X Sin 7x m f
- -----f---- cos"' xcos(n— l)xdx

=cos ri x
sin-
--' '--
in
-_

in
n
M I COS M x cos flX cL'

P1 P1 "'' fl

( cos" X sin fIX ill


I-f- -) l, ,------------- ± -,-

cos t" x sin fix in


I =-----+—I-1 -i
III + ii

which is the required reduction formula.


Remark. There are two other forms of the reduction formulae for
I cos' x cos nx (IA

(1) cos'. sin nx


COS X cos fl.\ CIX
rn -- ri
In
cos1 X Cos (n-f I) x dx

(ix) f n cos X sin 'ix in cos' X Sin X COS 71X


cos' x cos fix dx = - 2 - t +
11.11_ ill
rn(rn—l)
cos"' x cos nx dx.

i2
Example 78. Prove that cos" x cos n dx= 1 , n being a

positive integer.

Solution Let I. cosft x cos flx dx


SOME ADL)JTIONAF, TOPIC, A-O9


,/2 ,c'2
[ Cos" i
COS' t COS (a- 1) x
I) ()

Writing (n —I)for n iii ( s ) we have

() gives

Proceeding in this way and applying h reduction triai_,t


we have t

to '1 factors) I
'I2 ,I2
(cos aY' COS Ox 1v.•
2.

RU
II-
x
()
Example 79. Find a redr4L'iion formula /;ir sin ax (IX
i X,
Solution, Let I.. f x' sin rix dv
Taking as the first function and sin ax as
the SCCQfld fUflCtjoij
we have oil by parts.

()SflX) Cos
I rx' m-i ( _i' (IX
n j II J
X'" Cos fl y in f
= -
- --J
n x"
in cos Fax dx
,, ()
X" cos flv,fl[ (sin v'
n 11

(m — ) x-z (A)
xrn cos ax in
Pa Sill nX
m(m -
— -- .Ixm_! S i ll FIX (I
n2 j
I COS flx + mX -' Sit a
Hence x sin fl.r dx =
j ns
I)
Sin nv
the required :cducticui formula,

A-70 BUSINESS MATHEMATICS

Remark. A reduction formula for J x' cos nx dx is given by


f flX' sin nx4mX' cos nx
I X COS x ux---------------------.
j nl
m(m — I) (
2 COS Xdx
z j

This is left as an exercise for the reader.


n/2

Example 80. If U,= x" sin x dx and n>!, prove that

1
U-j-n(n—J) U.-,=n

(2

Solution. We have X" sin x dx

,
7 12 rr !2
cos x J-fn x' cos x dx

x' cos x dx

" 1242
X' sin x -2 sin X dx]
[
lr '-1

Hence U-l-n(n— 1) U_2n


(T)
Example 81. (a) Find a reductiqn formula for f tann x dx.

(b) If 'A Ian1 x dx, prove that


0
1
(i)
(ii) 1(IN_1+I4j)='.
Solution. (a) J tan x dx= tan' - ' X taii2 x d)
I
=-J tan , ` x (sec' x— 1) dx
= tae 2 x sec2 x dx—S tan' I x dx

SOME ADDITIONAL TOPICS A-71 I


tan' x
Hence tan" x dx=-1----__j tan X dx,
n
which is the required reduction formula.
Remark. The reduction formula for J cor x dx, viz
I cot-' x f
cot x dx== - cot-' x (IX
- J
is left as an exercise for the reader.
71(4 12/4

(b) (I) I, tan x dx tan"-' X tan 2 x dx

1214

tan'-2 y (sec 2 x— 1) dx

i/4
= tan x sec 2 X clx I tan" 2 x dx
1
rr 14

,-2
n—i
0

71/4 12/4

(II) n
(1._7+II)=flr J tan"-' 0 dOF tan k ' 0 dO ]

=n 1( t an— t 0 ftan' 0) do

71)4

;7= n tan 0 (14-tan' 0) dO


f

dn tanG
12)4
O I
=n tan' 0 sect 0 I.
0 0
1fl •1

• . . • •

I.-

I.
STATISTICAL TABLES

LOGA R ITHMS

F 1 011111*14151617 1 81 9 1123f5818,1
10 . 0.20010043 0086 0123 0I70 59131.72126 303433
02I2 0253 224 0334 0374 48 12116 2o 24 2)236
11 0 4 1 41 0 453 0492 0531 0 5 691 4812 162(223 273135

ir 03i0828 0864 C.399 09341


I
i°7 21645 0632 0719 075 4711 15 18
37 11 141821 252832
I299 ! L !2Y? LV71 iLL
1$ ij]i173i2oô 1239 137r( 3• 10 13 6 19 1321529
I )1303 1 3 ' 14 !9 1225 29
11 523 155) iTf 36 9 l:.3 222528
6l4 144 16 7.3 1 79 1732 3 b9 121417 20232
15 I 7901 1818 1547 ; 3 9 II 1q 17 22326

--
P2u62ogc 2122 214S
1)3 1931 1959 1957 2914 36
3
2175 22012 227 2253 22 79J5 8 Irl 3 16
P.
11(411 ui)222S
4 l6 £92224
17 j3o ss 1315 820 23
2430 24 743 c;5'4 U .1 3 2' '11 5 172022
ï i777 -..- 4 17192!
J2672 2505 27i1 242 16L8217'.y;
19 28S 2450 .l5I 24 7 91!
:00 2015 2045 207 :('S. 71 0 17 19
201 1C r lT3> I ' 1 3 311 I 7 ( 1 3 I 17 ic
21 3222 34,Z.,, !'4 3 . 2 124 3j'%. 3404 :4 1:1 1:':: 1.%1.i)
21 I l 34 1,- I., 4! 15 17
23
24
31
So
3I ;l)r, J4 •'. II 5;.'' 37 . 1; 37614 374 , 1 7 17 II 131$ 17
320 33S 2.58 5: 3o1 59Y) 397 92451 002 24 Si 7 . I 12 24 121
26 3C)9 307 4014 4031 44 ,)( çC.32 4 ) . 6' 3' 9 1 0 22 14 15
28 so 41J 41cr ) 4:11. 4232 4249 4i63 478! 42 23 51 7 810 II 13 15
i 4314 4330 .49 4302 .1'7.) 493 4400 .5425 444'2 4456 27 5 19 1! 1314
28 4472 4437 4507 4518 4U 4543 4564 .;579 454 469 23 81 6 8 9 111224
29 .1024 41532 4 65 5 .:t1 25 4698 4714 4728 4742 4757 1 4 6 7 9 101213
30 47;! 4780 4800 4 8 11, 439 4843 4S5 7 771 .4553 45(X) l / 6 7 9 10!! 13
81 4914 492.S 494 2 4955 4959 4933 4997 5011 5024 5034 23 6 6 7 S 10 1] 23
8 2 501 5)125 5079 5097510i 5111) 5132 5145 5159 5172 13 4 5 7 91! II
38 5 1 s 519 5211 5224 5227 525(7 5261 5775 5289 5302 1 4 5 6 10 12
84 531) 5425 534(; 535 3 5266 5 .378 5391 i 5403 5416 5428 13 .7 5 3 91')!!
36 5441 5,153 540c c:8 5.90 5102 5l4 5527 5539 5551 S 7 910!!
36 Y.63 5575 5587 5509 5101 5'..23 5035 5647 5053 56 7 0 1. 5 5 7 510 II
37 5082 5004 505 3717 5729 5740 5752 573 5775 57 31 3 ó 7 8 9 to
38 598 5509 532! (632 58;I s•55 5566 5377 5888 5 Sço 5 6 S
39 5912 52 5933 5944J •: cc 36 5977 53S 5999 6-:- 1 L, 2 3! 4 3 7 8 910
40 6021 5031 6042 6053 606 4 6075 6085 6095 6107 6,17, 12 3 4 5 £ 8 910
41 6128 6135 6( 4 9 616) (.171. 61&) 619! 6221! 6212 6222 12 3 4 6 8
2 6232 6 4 3 6353 6263 6374 628 4 6204 604 6314 6325 12 3 455 6 7 8 9
7
43 6335 6335 6355 6365 635 63.45 6395 64215 ((415 5425 I 2 3 4 5 6 7 8 9
44 0435 6444 3454 6164 1 6-1 74 614 f.; 6503 6y3 6522 I 2 3 4 5 6 7 S 9
45 6 z 6542 655t 661 0371 Oç8o 65'' 65 0509 (J61 3 12 3 4 5 6 7 3
48 6o:S 6637 6645 606 666 6675 US 4 6693 6702 0717 1 2 3 4 5 6
7 / 8
47 6721 573r)6739 5ç 575 67157 I676 6715 6704 803 12 . 6 / 8
II 6E2 68226630 6839 68486857 616'. 6815 6884 6893 12 3 4 4 6 S
4 a 6ç,Il&92o 6928 693769'.69s 6964 697216981
2 3 4' 5 7
T-2 01INS8 MATHI MATIC3

LOGARITHMS

2 8 4 6 6 89 123 454
50 6990 5998 7007 7016 7024 7033 7042 7050 7059 7067 123 345
7076 7084 !093 7101 7110 7218 7126 7135 7143 7152 123 3456781
52 7160 7 i6 177 7185 7193 7202 7210 7218 7226 7235 122 345 6'
63 7243 725! 7259 7267 7275 7284 7292 732)) 7308 7316 122 345667
64 7324 7332 7340 7348 7356 7364 7372 7380 738.8 7396 122 345 667
66 74)3 7412 7419 7427 7435 7443 7451 7459 7466 7474 122 345 567
66 7482 7490 7497 7505 7513 7520 7528 7536 7543 7551 '22 345
7559 7566 7574 7582 7589 7597 7604 7612 719 7627 £22 345 567
68 I 7634 7642 7649 7657 7664 7672 7679 7686 7694 7701 112 344 567
69 7709 7716 7723 7731 7738 7745 7753 7760 7767 7774 112 344 567
1
7789 7796 7803 7820 7818 7825 7832 7839 7846 lIZ 544 566
31 7853 7860 7863 7875 7882 7889 7896 7903 7910 7917 112 344 566
62 7924 7931 7938 7945 7952 7959 7966 7973 7980 7987 112 334 566
83 7993 S000 3007 8014 2021 4028 8035 8041 8 048 80 112 334 556
84 8o6 2 8069 3075 8032 8089 8096 8102 8199 8116 8122 112 334
65 8i9 8136 3142 8149 8156 8162 8,69 8176 8182 8189 112 334 556
66 5195 830: 8209 8215 8222 8228 8235 8241 8248 8254 '12 334 5 56
67 8261 3267 3274 8280 8287 8293 8299 8306 8312 8319 112 334
68 .3325 833! 8338 8344 8351 3357 8363 8370 8376 8382 112 334 456
69 33S3 8395 8401 8407 8414 8420 8436 8432 8439 112 234456
70 18411 8457 8463 8470 8476 8482 8488 8494 800 8506 112 234 456
71 51513 8519 8525 853! 8537 8543 8549 856, 8567 112 234 455
72 S573 8579I 9585 8591 8597 8603 Sóoq 561 8621 8627 112 234455
73: S ", 3J 863 8645 8651 8657 8663 8669 8675 8681 8686 112 234455
74 2092 3698 8704 8710 8716 8722 8727 8733 3739 8745 I I 2 23 4 4 55
7t 8751 8756 8762 8763 3774 8779 8785 8791 8797 8802 112 233 455
78 '4o8 1814 8Sao 8825 8831 8837 8842 8848 8854 8859 112 233455
77 SS-,l 8876 8882 8887 8893 8899 8904 89,0 8915 113 233445
78 59:i 3927 8932 8933 8943 8949 8954 896o 8965 8971 112 233 445
79 3982 2987 8993 8 99 8 9004 9009 9015 9020 9025 112 233445
80 9031 9046 9042 9047 9053 908 9063 9069 9074 9079 112 233 445
B! 9090 9096 0101 9106 9112 9117 9122 9128 9133 112 233 445
82 91p 9143 9149 9154 9159 9165 9170 9175 9180 9186 112 733 445
83 OI)1 9I91 920! 9206 9212 9217 9222 9227 9232 9238 112 233445
84 9243 9253 9258 9263 9269 9274 9279 9284 9289 lIZ 233 445
85 9294 0:99 9304 9309 9315 9320 9325 9330 9335 9340 112 233 445
86 0345 (fl55 9360 9365 9370 9375 9380 9385 9390 112 233 445
87 9S 94) 9405 9410 9415 9420 9425 9430 9435 9440 OIl 223 344
88 9450 9455 9460 9465 9469 19474 9479 9484 9489 01 1 223 544
89 .1499 9504 9509 9513 9528 9523 9528 9533 9538 Oh 3 2 3134 4
90 Q5 954? 9552 9557 9562 9566 9571 9576 9581 9586 011 223344
9595 9(x)0 9605 9609 9614 9619 9624 9628 9633 0! I 223 344
9'S 9)43 9647 9652 9657 9661 9666 9671 9615 9680 0!! 223 344
I 93 95 90S9 9694 9099 9703 9708 9713 9717 9722 9727 01 1• 223 344
94 9731 9736 9742 9745 9750 9754 9759 9763 9763 9773 Oil 223 344
°I9777 9782 9786 979' 9795 9800 9805 9809 9814 9818 oil 223 344
go; jS23 9827 9832 9836 9841 9845 9850 9854 9 8 59 9863 01 1 323 344
97 119-968 9572 9877 9821 9886 9890 9894 9899 9903 9908 011 223 344
98 9912 99,7 992! 9936 9930 9934 9939 9943 9948 9952 011 223 344
99 9956 9961 9965 9969 9974 9978 9983 993799919996 0il 2131134

)*U3IZIICAL TAILU T-3
ANTMOGARrMUS
1 S 3 4 5 8 7 7 9 1234567l$
00 1000 *002 1005 *007 1009 *0*2 1014 1016 1019 *021 001 1* I 232
-01 1023 1026 ioaS * 00 133 p035 2038 1040 *042 *045 00* 1* 1 211
-02 *047 1050 1032 1054 *057*059 IO6i £064 *067 *069 00* 1 I I 2* 1
-03 1072 *074 1076 1079 2081 1084 1086 1089 *091 1094 001 IlL 23*
04 l lso96 fO99 1102 1204 1*07 *109 11*2 1214 *117 11*9 of *13 31*
06 1122 1125 *127 1130 1132 *135i*j8 1140 1143 1146 91 112 *23
08 1148 1151 1153 1156 1159 ii6i ;164 1167 1169 1*71 0*1 III 332
-07 1175 ii8 ii8o *183 ii86 2189 1191 3 I 1197 1139 012 III 31*
-08 1209 1205 *208 1211 1213 1316 1219 1222 *225 1227 0*1 112 233
-09 1230 *233 *236 1239 *242 1245 *247 *350 *253 1256 0* I 213 133
10 *259 1262 1763 1268 1271 *274 1276 1279 1282 1285 01* I *2 123
-11 i38-8 1291 *294 2297 1 3 00 7303 1306 1309 *312 1315 01 1 1 32 233
-12 13*8 132* 2324 1327 *330 1334 7337 *340 1343 1346 011 122 223
-18 *349 1352 1355 7358 136* 7365 *368 *371 1374 1377 Of 122 233
• 14 1380 1384 1387 *390 *393 *396 1 4 00 1403 1406 1409 of *32 333
15 1413 *4*6 14*9 *422 1426 1429 7432 1435 *439 14.43 011 *22 233
18 1445 *449 1452 1455 1459 1462 *466 1469 *472 1476 of 133 233
'11 1479 *483 *486 1489 '493 1496 1 5001
503 1307 15*0 01* 122 23
.18 15*4 1517 *53* *524 1528 *531 1 535 1538 1542 1545 01 * 123 2333
• 19 1549 1553 1556 *560 *563 7567 1570 1574 1578 1581 01 1 132
333
20 158 189 1592 1596 1600 1603 *607 1611 *614 *6*8 011 *32 333
31 1622 1626 029 1693 1637 *641 1644 *648 1652 *656 01* 232 333
-12 1660 *663 1667 167* 1675 *679 1683 *687 1690 *694 01 7 222 33 3
23 *698 1702 1706 *710 1714 1718 1722 *726 1730 1734 011 222 334
44 1738 1742 1746 io 175. *738 *762 1766 1770 1774 01 223 334
26 *778 7782 1786 *79! *795 1799 1803 1807 181, *8*6 011 222 334
18 1820 1824 1828 1832 1837 *841 1845 18. 18S4 1838 011 223 334
-*7 *862 *866 1871 1875 *879 1884 1888 1892 1897 1901 011 223 334
28 1905 *910 19*4 911 1933 *928 1932 *936 *941 *945 01 223 344
*9 1950 *954 1959 *963 *9 68 97 2 1977 *982 *986 *991 01 223 344
30 19435 2000 2004 2009 20*4 2018 2023 20282C32 2037 01 1 22 3 344
31 2042 2046 2051 2056 2061 2065 2070 2075 2080 2084 01 233 344
32 2059 2094 2099 2*04 2*09 2113 21*8 2123 2*28 2*33 01* 223 344
*3 2138 2*43 2148 2*53 2*58 2*6316 2 2173 2178 2*83 011 223 344
44 2188 2*93 2*98 2203 2208 22*3 2218 2223 2228 2234 112 233 445
36 2239 2244 2249 2254 2259 2265 2270 2275 2280 2286 II 2 2 33 443
*6 229* 3296 2301 2307 2312 2317 -2323 2328 2333 2339 *72 233 445
37 244 2350 2355 2360 2366 237* 2377 2382 2388 2393 II 2 333 445
88 3399 2404 241012415 2421 2427 2432 2438 *443 2449 112 333 443
-39 2455 2460 2466 2472 2477 2483 2489 2495 2500 2506 113 233 4S5
40 25*2 2518 2523 259 2535 2541 2547 2553 2559 2564 11 2 234 455
41 1570 2576 2582 2588 2594 2600 2606 6*i 26182624 113 234 435
42 2630 2636 2642 3649 2655 3645* 2667 2673 2679 268 I 3 2 234 456
-43 2691 2698 2704 27*0 27*6 2733 2729 2735 2742 2748 *22 334 456
'44 2754 2761 2767 2773 2780 2786 2793 2799 280 2812 113 334 4
45 2818 2825 283* 2838 2844 2851 2858 2864 387* 2S77 I 1 2 334 556
•44 2884 2891 3897 2904 2911 2917 2924 2931 2938 2944 II 2 334 SS6
-47 2951 2958 2965 2973 2979 3985 2992 2999 3006 3013 II 2 334 6
4$ 3020 3037 304' 30.8 3055 306* 3069 30163083 II a 34 64
'43 3090 3097 3*05 3112 3*19 3124j3133 3141 3143 3255 122 344 66
L


T. 4 BUSINESS MATHDMATICS

ANTtLOG R ITHS

• I 1 1 4 1 ,as
33 41)14 1 • 4A
ADD
3563 $130 3577 3484 3*91 3494 p06 3414 3111 3.11*
316J 4 I 3 3 4 4 1 4 4.
11 33$ p43 3355 PSI 3166 3)7) jaSt 1214 j$4 3304 ' It * 3 3 4 5. 6 6
1' 3)5) ))1 3317 33)1 3341 3)50 3)33 374333733311 3 4 5 4 6 7
-51 3394 3404 14') 340 34183434 1443 5435 3459 *3 3 4 3 6 6 i
.34 3467 3473 3481 449 1 34 3504 3515 34_4 3ir 35.40 )443a 5 t 4
.54 3546 3356 )5.43 )7) 3344 3519 1597 1" )6t 0U 4 .i * 3 4 3 6 $
.3' 3674 )691'2.4 l 3656 )664 3471 3631 369036313b' 61 A f 4 345 66
-r 3153 377, 37)) 5741 3730 5758 3347 3751 371) 9 4 7 3 .4 4 5 6 7 a
-$ 3*03 .I' 3119 3621 3437 3*46 3*53 3664 387) 3642 ') 4 4 4 3 6 7 8
-4, 3499 jol YO 3936 3936 3945 3954 394) 777* 9 I 2 4 3 5 6 7 S
-4. ))*I 3990 3999 4009 4013 4017 4036 4 646 3035 .4)64 9 I I 3 4 5 5 6 7 8
4074 4083 4093 402 4111 4121 4130 4140 4550 4159 40 I I ; 4 J 6 7 I
4. 4169 4171 4118 4195 4207 4311 412.7 4.14 4146 41$6 to I I 3 4 1 6 78 ,
4466 4376 4385 4295 4303 43 1 5 411 I 1 4315 4 143 4353 50 I 4 ) 1 4 5 6 1 I 0
!473 4335 4375 4385 4395 4406 4414 4426 4436 4.146 4457 10 S3 4 4 $ 6 7 6
14s 446 7 4477 487 4498 4508 4559 4511 4 q 39 4530 4560 40 I .1 . 6 7S9
1-4' 457' 4311 4392 4603 4653 4634 4634 4045 4656 4667 II 5 3 3 1 3 7 2 9 10
41 967 4881 4699 47 5 0 4735 4731 474 3 4753 4764 4715 II 1 3 3 4 3 3 9 14
4734 4197 4408 4149 4835 4841 4833 4864 4873 4887 SI I 3) 4 6 789 ID
43 410 49094910 4932 4943 4335 4966 4977 4989 5000 IS S 3 3 4 6 ' J 9 40
-10 3011 5011 $035 5047 5054 5070 5082 3093 $105 3557 43 I 34 3 4 7 8 50 II
5519 55403153 5164 513651855200 3ZI3 5 224 ;33 6 *3 114 3 4 7 kio,*
•1 5048 5260 5173 5144 5'97 5309 5)11 5333 5346 5358 12 I 3 4 6 7 8 40 II
13 5370 5313 3395 5401 5410 5433 5445 541 '470 5483 II 534 j 6 1
¶473 330155 1 15.534 5546 3559 5573 3575 550 5610 53 1)4 5 6 6 94051,
2 5633 3636 5649 566* I 5675 5689 3703 375 3734 5741 I) I 34 3 7 8 9 40 14
$154 3761 5741 5194 SW $831 38.3 3748 s861 5575 53 I 3 4 3 7 4 9 40 II
.73 5*84 5902 5946 5929 5943 5957 5970 5964 5ç93 6011 44 i 1 4 6 7 6 40 II 13
15 6.n6 6039 6053 6660 6o8i 6ogs 6409 612.4 6138 2u, 54 I 34 6 7 S 10 II 53
6166 6s3o 6194 6wo 6,133 6*37 6251 6166 611, 6.5 1 4 I 34 .6 7 U to II 53
40 6310 6324 6319 6353 661 6313 6397 6413 6437 6441 55 I 34 6 7 9 50 II I)
.8; 647 6.1 6486 6501 6516 63, 66 661 657? 659: ij * ) 1 6 39 It 53 54
•13 66o 662 663 6653 6664 6683 6699 601467506745 13 2 3 S 6 I 9 II 13 14
$3 6761 6776 6791 68o8 6823 6139 6*55 6174 68*7 6903 33 * j 5 4. I 10 II 1314
•54 69i8 693469306966 6982699*30 4 5 7031 7047 704. 16 1)3 6 1 6 111)14
43 7079 7096 7113 7119 7143 7161 '1 7494 7311 1231 ,6 i 33 6 $ 10 II I) 94
16 714.4 7361 7278 7393 7335 7331 7343 7363 7339 7396 57 1 3 3 7 9 10 I) II IS
7443 700 7447 7464 7433 7499 7546 7334 7351 7361 47 1 35 7 9 40 12 54 53
•8* 7586 7603 3631 7638 76s6 7674 701 130571*7 7745 II 3 1 5 7 9 It 5)54 14.
•Ie 7761 771073917116 7194744_I 7170 111979077915 It 345 1 95* 1 ) 1 4 16
'S 7943 7961 7910 7998 8017 9035 8054 $072 809.4 115* 18 2 4 3 7 4 II (354 II
1 11 1$ 8147 1566 MIS 5304 5223 $141 8*60 5379 1*99 19 1 4 6 8 5* II 13 1 5 17
-91 8351 17774356 8373 $395 444.4 44)) 5453 8471 5493 49 3 4 8 S so It 1315 57
1) $jij $5)! 5354 8o 999oUlo $6,o 165004104690 10 146. 55053 14161$
14 Silo $730 $730 lp, 4790 5$io $131 $151 8*73 8493 30 3 4 6 $ 10 II 14 16 If
'S $ 9 1 5933 $95 5974 9995 9048 9036 9037 9374 9099 35 * 4 6 I jo 13 43 I) 59
-0 9110 9545 9161 9(5) 9)01 93369147 9361 9390 9311 31 1 4 6 6 ii I) 53 I) I
7 p3)) 9554 9376 9397 9449 944; 9443 9454 9506 9535 34 3 4 7 9 II I) 53 1530
I' 9330 1 9572 O M 9616 9634 9661 9643 9705 9737 9750 31 1 4 7 9 II 4) 55S
19 9773 9795 94 1 79440 9363 98*6 990* 9931 995.4 9977 13 3 5 7 9 Ii 14 16 10 It

NUMIRICAL TABLES T-
NATURAL SINES

oi 12 1&4 30 1 85 t42I' I 64' 3


11 0•o IG•-e O3 0' 3 0* 4
4 6
0000 0017 0035 0052 0070 0105 0222 0140j0157 369
0175 0192 0209 0227 0244
I? 15
0202 0279 0297 0314 0332 3 6 12 15
0349 0366 0384 0401 0419 0 1,6 0454 0471 0488 0506 369 II 13
0523 0 5* 1 08 0576 0593 oôuo 0628 0645 066 3 0680 3 6 g
0698 0715 0737 07! 1 0767 I2 45
078 5 0802 082 9 08 37 08 54 3 6 9 I? 45
087 2 0889 0906 0924 0941 0958 0976 0993 loll 1028 3 6 9
2015 1063 1 080 2097 1115
12 l
IT 1210 2236 1253 1133 1149 1167 1184 1201 3 69 '12 14
'27' 38 2305 2323 1340 1357 1374 3 6 12 24
1392 1 4 09 1426 1444 I41 2 478'495 1513 1530 1547 3 6 Il 14
IS64 2581 2599 1616 1A3 l'5O 1668 03 2702 1 7 1 9 3 6 q *2 14
10 2736 2754 1?71 3738 180 18,72 18 4 0 18c7 18 1891 6 13 14
18 1933 1975 1941 1959 1'fl7 1994 1011 2028 2045 2062 3 36 g n IA
11 '49 2096 211 3 7130 2147 2264 2181 2193 2215 2332 3 6 9 II 14
1$ 250 1167 2284 2300 2317 2334 2351 2368 1385 2402 j 6 8 II 14
14 2419 2436 2453 2470 2437 2504 2521 2538 255 4 757! 3 6 8 II 14
16 2588 2605 2642 2639 2656 2672 3689 3706 2723 2740 , 6 8 II 4
16 2756 2 773 2 790 3807 282 2840 2857 2874 2890 2907 3 6 8 II 14
17 2924 2940 1757 2974 2990 3007 3024 3040 3057 3074 3 6 8 II *4
14 '3090 3107 3123 3140 3156 3173 3199 3,206 3223 3239 3 6 8 II 14
19 '3156 3272 3289 3305 3322 3338 3355 3372 3381 3404 3 5 8 II 14
20 3420 3437 3453 3469 3486 3502 3528 3 5 35 3551 J S6 7 3 5 8 II 14
21 354 3600 3626 3633 3649 3665 3632 3697 3724 373 0 3 3 8 II 14
£2 3746 3762 3778 3795 3R,1 3 82 4843 3859 3875 3391 3 5 S II 14
23 3007 3923 3939 3955 3972 3987 4003 40194035 4052 3 5 8 II 14
24 4067 4031 4999 4215 413 4147 4263 4179 4195 4210 3, 5 8 II 13
25 4226 4742 44258 4274 4289 4305 4321 4337 4352 4363 j 5 8 II .3
4384 4300 4415 4431 4446 4462 4478 4493 4509 4524 3 5 8 20 13
11 4540 4555 4572 4586 4602 4617 4(23) 4048 4664 467(7 3 5 S 10 lJ
£8 '4695 4710 4726 4741 4750 4773 4737 4'02 4813 4833 3 5 8 10 I)
'4848 4363 4379 4894 4909 4924 4939 4955 4970 4985 3 5 8 (0 I,,
80 5000 5025 503 3015 5060 5075 5090 5205 5220 51;5 3 5 8 *0 13
$1 5150 5165 5195 5210 5225 5240 5255 5270 5254 2 5 7 10 SI
82 5299 5314 3329 5344 5358 5373 $333 5402 5417 5432 2 5 7 10 II
$1 5446 5461 5476 5490 5505 53 1 9 5534 5548 5563 5577 2 5 7 so
'4 5592 5606 5621 5635 5650 5664 5 678 5693 5707 5722 2 5 7 10 .3
35 5736 5750. 5764 5779 5793 5" 5821 583S c30 155(24 10 .
$4 5878 5892 5906 5920 5934 5948 5962 5976 6004 z 9 Ii
'7 6o,8 6032 6o6 6o6o 6074 088 óioI 611 (229 6243 z 5 7 9 22
I! 6157 6270 6184 6198 6111 5225 6139 6252 6e06 (.730 3 5 7 9 II
1$ 6293 6307 6320 6334 63.. 5361 6374 6358 6.o. 6414 2 4 7 9 II
40 '68 61 645$ 6468 6431 5494 0508 63216534 6547 3 1 7 9 41
41 '6561 6574 (2587 6600 6613 1616 6639 6652 6665 6673 2 4 7 9 II
43 6691 67046717 673 0 6743 1.756 6769 6782 6794 6807 2 4 6 9 ii
6Szo 6833 68 4 5 68c8 6871 1.884 6896 6909 6931 6934 24 6 S
4 '6947 6959 6973 6984 67 P009 7021 7034 7046 7059 1 4 6 8 to

T6 3UIINII$ UATEMMATICS

NATURAL SINES

• S 11' iS It W $8' 4W 0' 54 Doss


O'.o O'I 02 Q13 04 05 o6 07 o-8 0'9 5
-7071 708 7096 7108 7*20 7133 3145 7157 7169 718* 2 4 6 8 *0
4$ 1193 Yus 72*8 7230 7242 7254 7266 7278 7290 7302 1 4 6 8 10
47 1314 732S 7337 7349 7361 7373 7385 7396 7408 7420 2 4- 6 8 to
143' 7443 7455 7466 7478 7490 7501 75*3 7524 7536 2 46 8 to
4$ 1547 7553 7570 738 1 7593 7604 76*5 7627 7638 7649 3 46 8 9
5. 1660 7672 7683 7694 7705 7716 7727 7738 7749 7760 2 4 6 7
11 -7771 77 82 7793 7 804 7 83 S7 826 7837 7848 7859 7869 245 7 9
8$ i 0 7891 7903 7912 7923 7934 7944 7955 7965 7976 2 4 5 7
8$ 7986 700' 8007 8018 8028 8039 8049 8059 8070 Solo 2 3 5 7
.4 logo Moo 8m 8121 811 8141 811 8161 8171 8181
5' 192 Sioi 8211 22 Z23t 8241 8251 8261 8272 828* 3 3 5 18.
.4 -8290 3300 8310 9 320 8329 8339 8348 8358 8368 8577 2 3 5 6 S
$7 5387 8396 8406 8415 *425 8434 84.43 8453 8.462 847! 2 3 6$
8$ •$ 46o 8490 8499 3508 8 5 17 8526 8536 85 8554 8563 2 3 6 1
I. 4573 81z 85go 8599 86o7 8616 8625 86348643165213467
so •866o 8669 8678 8686 8695 8704 3712 8721 8729 3738 1 34 6 7
11 3746 3755 8763 8771 860 8781 896 8.$o 8813 8831 * 3 4 6 7
0 $839 U38 88.0 8854 8862837088 7 888868394 89021 3 4 5 7
* - Soso 4928 8926 8934 8942 8949 8957 8963 3973 89*0 3 3 4 5 6
4$ 391$ 8996 9 00 3 9011 90 18 9026 9033 904* 9048 9056 I 3 4 s 6
1063 ,00 9078 9085 9092 9100 9207 9 114 9221 9128 I 2 4 5 6
4$ ;-9!35 9143 9150 9257 9264 9172 9178 9184 9291 9j98 I 23 5 6
95 9312 9219 9225 9232 9339 9345 9252 9259 9365 .1 i 3 6
4$ .9*72 iy8 92.85 929* 9298 9304 9311 937 9323 9330 I 3 3 4 5
4$ -9336 934.2 9348 9354 9361 9367 9373 p379 9315 9391 1 2 3 4 5
70 937 9403 9409 9415 9422 9436 9432 9438 9444 9449 1 2 3 4
71 .9455 9462 9466 9472 9478 9483 9489 9494 9500 9505 I 2 3 4 5
7. 452* 9526 9521 9527 953.3 9537 9542 9548 9553 9558 I 3 3 3 4
7. 95689573957895$3958895939598^039608 132 3 4
it ,613 9617 9622 9627 9632 9636 9641 9646 960 9655 1 2 2 3 4
75 1659 9664 9668 9673 9677 9681 9616 9690 9694 9699 11 2 3 4
7, 1703 9707 9711 9725 9720 9714 9728 9732 9736 9740 1 1 2 3 3
'7 9744 9748 9751 9755 9759 9763 9767 9770 9774 9778 1 1 3 3 3
7$ 9711 9785 9789 9092 9796 9799 9803 9806 9810 9813 I I 2 3 3
7. 9816 98 9826 9829 9833 9836 9839 9842 9845 1 I 2 2 3
SO .9841 7351 9854 9857 9860 9863 9866 9869 9871 9874 0! 1 2 2
1877 9880 9883 9885 9888 9890 9893 9895 9898 9900 0 I I 2 2
$1 -9903 9905 9907 9910 9912 9914 9927 9919 9921 9923 0 I 1 3 3
.4 -9925 9928 9930 9932 9934 9936 9938 9940 9942 9943 0 I I I 3
$4 .9945 9947 9949 9951 9952 9954 9956 9957 9959 9960 0 1 1 1 3
as 9962 9963 9965 9966 9968 9969 9971 9972 9973 9974 00 I I S
1$ -9976 199r, 7 997* 9979 9980 998* 9982 9983 9984 9985 00
I I 1
17 -9986 c.987 9988 9989 9990 99il0 999* 9992 9993 9993 000 I I
U -9994 995 9995 9996 9996 9947 9997 9997 9998 9998 4 00 0 o
N -999* 9999 9999 9999 9999 Z000 *000*000 l 1000 1'O00 000 0 0
I. IC
Index
A B

kbclian group 119 Base of indices 143-148


'b5cjssa 555 Biconditional statement 13
Absolute value 101 13ijectiv mapping 62
Absorption law 77 B i-quad nil IC
Addition —equations 289
—natural numbers 93 —surds 170
--rational numbers 98 Binary
—real numbers 102 —codes
Additive inverse 99, 803 —compositions 114
Adjoint of a square matrix 837 —relation 59
Algebra Binomial
—Boolean 1,73-91 —coefficients 340
-matrix 791-856 —terms 339
—propositions 8 —t heoi ems 332-362
—sets 45 Boolean
Allied angles 507 —algebra 3, 74
Alternating series 470 —addition 87
AND operation —Circuits 87
—statCfllentS 2 —functions 84
—Boolean 75 —multiplication 86
Angle 475, 584 —properties 75
Annuities 230 Briggsian 194
Anti-symmetry 103
Arbitrary constants 629 C
Are 478
Area Calculus
—quadrilateral 568 —differential 647
—triangle 564 —integral 722
Arguments 16 Cancellation law 96-121,803
Associative law Canonical form
—Boolean algebra 79 —Boolean 85
—groups, field, etc. 114-137 Cartesian product 54
—real number system 94-105 Centesitnal system 476
—matrices 802,806 Cent rol d 561
—vectors 776, 778 Circle 597-600
Auxiliary series 445-454 —equation 597
Axis 615 —forms 597
1-2 BU31NM MATHEMATICS
—general equation 600 —of power function 650
Circumccntre 559 —of the product of functions 653
Circuit 87 —of quotient of two functions 655
Circular —of trigonometric functions 660
-functions 475 Dc Morgan's law
—measure 478 —sets 38-45
Closure property 118-135 —statements 10
Co-initial vectors 772 —Boolean 77-79
Cotlinearity 567 Density Property 99-103
Collinear vectors 772 Depreciation 229
Combinations 318-331 Determinants 824-8%
—restricted 326 Diameter 482-483
Conditional statements II Difference sets 39
Coordinates 554-561 Differentiation
Commutative property —of implicit functions 682
—number systems 93-107 —logarithmicic 677
—group and field 114-137 —method of substitution 673
—matrices 802 Directed line 553
—vectors 776 Direct rix 614
Complement Disjoint sets 47
- of a set 37 Disjunction 4
—of statement 46 Distance between points 557
Complementation 78 Distributive law
Complementary theorems 326 —Boolean algebra 79
Completing squares 256 —compositions 118
Complex number 109 —field 136
Composite functions 632 —group 131
Composition —sets 35
—tables 117 —statements 10
—types 115 —real numbers 94, 99
Compound interest 225 —matrices 803
Compounding statements 3 —Vectors 778
Concurrent lines 583 Divergence 436-473
Conjunction :3 Dot product 779
Conjugate 109 Duality
Continuity —Boolean algebra 79
—function 638 —sets 47
—in an interval 643
Continuous functions 632 E
Convergence 436-473
Conversed sine 486 Electrical network
Cosecant, cosine, cotangent 485,486 —switching circuits 1, 5
Counting rule 300 —switching systems 86
Cross product 781 Elements
Cubic equation 289 —sets 24
---matrix 794
ii: Ellipse 610
Empty set 26
Derivative Equality with zero 99
—of a oonsant with function 651 Equations
—of a function of one —biquadrat Ic 289
variable 6-49 —cubic 289
—of logaritnmic function 663 —kgrees ac


INMX 1-3

—format ion 270 Homogeneous linear equations 249

—quadratic 252
—reciprocal 264

—nature of roots 268

Olutioti 241 Identities 240

—normal 606 —derived 241

—normal in slope form 622 —trigonometric 537

—tangent 605. 620 Identity law

—tangent in slope form 606 —Boolean 76-78

Equivalence relation 59 —binary composition 114

Euclidian geometry 553 —sets 46

Expressions syninietrica! 269 —statements It)

Expansion of determinants 835 Idempotent law

Explicit functions 630 —Boolean 76-78

External division 561 —sets 46

—statements 9

1' Image of a set 61

Imaginary numbers 108
Factorial notations
302 Implication in statements 11
Fallacies
7 Inclination 771
Fields 135 Inclusion property

Finite set 25 —vectors 776

—elements in a finite set 49 indices
Focal —positive

143

615 —negative 144

—distance 615 —zero and unity 144

—chord 615 —fractional 145

Functions Induction

—algebraic 630 —mathematical 365

—Boolean 84 Inequality 242

—composite 632 Infinite

—continuous and disconiinuo us 632 —set 26
—even and odd
632 —series 437

—increasing and decreasing 700 Inflexion 701

-mapping 60-63, 628 Injcctive mapping 61

—one valued 630 Integers 96

—periodic 632 Intercept

—transcendental 630 —axis 575

—types 630 —slope 575

-two 516
G Integral


—calculus 723
Gauss elimination
850 —definite 751

Graphic solutions 276, 289 —indefinile 725

Groups 114-141 —reducible to standard form 743

—Abelian 119 —standard form 738

—modulo 126 Integral domain 133

—properties 120 Intersection

Grouping symbols 245 —of sets 30

—of lines 583
H Integration

—by parts 746

Half angle formulae 544 —rules 725
1-4 BUSINSSS MATHBMAIIC3
—partial fractions 753 —orthogonal 823
—by substitution 730 —rank of matrix 854
flVCiSe —Sarrus diagram 828
—matrix 840 —scalar 798
—relation 59 —skew-symmetric 798
—inversed sine 486 —submatrix 797
Involution 77 —square 795
Irrational equation 262 —transpose 821
Irrational numbers 100 —triangular 797
—zero or null 797
J Maxima and minima 703
Mean
Joint denial 18 —arithmetic 396
—geometric 425
K Modulo 97, 126
Modulus 107
Kramp's factorial notation 302 Multiple angles 521
L
N
Latus rectum 615
Line
578 Napier (Napierian) 194
Linear
Natural numbers 93-96
—combination
772 Natural sine 504
—dependence
773 Negation 2-16
Limit
—concept 437
—evaluation 438, 633 0
—function 633
Locus of a point
572 One-to-one mapping 62
Logical statements 2 ON-OFF
Logarithms
191-238 On-to mapping 62
—change of base
195 Order relations 94, 103
—operations
216 Oscillatory series 442
Ordered pair 53

M Origin—slope form 575

Ordinate 555

Maclaui in's series 695 Orthogonal bases 779

Mathematical induction 364-383 Orthogonal vector 779

Matrix Orthocentre 589
—algebra 791
—addition and subtraction 801 P
—adjoint 837
—complex conjugate 798 Parabola

—Cramer's rule 825 —forms 615

—definition 794 —parametric presentation 617

—determinants 824, 835 —standard equation 615

—diagonal 796 Parallelism 585

—equality 799 Partition of a set 47

—inverse 840 Peano postulates 93
-minors 836 Permutations

—multiplication 804 —n different things 303

—notations 794 —circular 305

—operations 801 —not all different 301
1-5
INDEX
—restricted 309 —empty 26
26
Perpendicular from a point 590 —equal
Perpendicular c(ors 779-788 —equivalent 27
—family 29
Perpend icu ar sm 585
Positive integers 96 —finite 25
—infinite 26
Position vector 773
Power function 650 -power 29
28
Prime 96 —proper subset
Progressions —singleton 26
—arithmetic 384 —subset 27
29
—geometric 411 —universal
370
Propositions 8 Series
—alternating 470
Sexagesimat system 476
LSI
Sigma 371
485
Quadrants 476, 493, 554 Sine of an angle
Quadrilateral 568 Slope—intercept form 575
576
Quadratic equations 252 Slope—point form
487
Quotient relations 487 Square relations
Statements
R —conditional 11
-logical

Radian 478-483 Straight line
585
Rank of a matrix 854 —angle
573
Rational numbers 97 —gradient
574-78
Real number system lOt —equations
577
Reciprocal equations 264 —normal or perpendicular form
577
Reciprocal relations 487 —parametric form
577
Rectangular coordinates 554 —two point form
133
Reflexive property 58 Sub-ring
686
Regrouping of sets 48 Successive differentiation
Relations Surds
165
—binary 59 —operations
168
—Sets 58 -rationalisation
180
—numbers 103 —mixed
167
Rhombus 788 —simplification
62
Rings 131-135 Subjective mapping
269
- commut alive 133 Symmetric expiessiOns
58
—subrings 133 Symmetric relation

—with unity 133

—zero divisor 133 T
S Tabular method 25
Tangent and normal
596
Sarrus diagram 828 —equations
Scalar Tautologies 7

—matrix 798 Tests of convergence and divergence
463
—vectors 779 —Cauchy's
448
Secsion formula 560 —Comparison
Selector method 25 —d'Alembert's 456
470
Sequence of partial sums 442 —Leibnitz
Set —Raabe's 466
—elements 24 Total differentiation 720
1.6 BUSINES1 MATHEMATICS
Transitive relation 59 —inverse 121
Transitivity 94, 103 Unit vector 771
Transpose matrix 821
Triangle V
—properties 54'
-solutions 548 Variables
—Trichotomy Law 94 —in functions 630
Transposition 256 —continuous and real 632
Trapezium 564 Vectors
Trigonometry —algebra 770
—functions 484 —addition 775
—identities 537 —co-initial 172
--ratios 485 —coplanar 772
—relations 487 —free and equal 77'
—signs 493 —linear 773
—standard angles 499 —multiplication 771
—tables 504 —null 771
t-ratios of allied angles 501 —position 773
—t-ratios of sum and difference —product 778, 781
of angle 513 —reciprocal 771
—t-ratios of multiple angles 521 —subtract ion 777
—transformations of products and —unit 771
sums 530 Versed sine 486
Venn diagrams 29-41

Lei Z

Union of sets 32 Zero


Uniqueness —divisor 133
—identity 120 —clement 777
Statistical Methods
Dr. S.P. GUPTA
Dean, Faculty of Management Studies, University of Delhi. Delhi
Twenty-Ninth Revised Edn. 2001' Pp. xxiv+ 1428, 152 Diagrams 1000 Solved Illustrations
1300 Exercises with Answers ISBN 81-7014-772-7 As 27c

Special Features
• The book has been written in a very simple and lucid sytle The text material is self-explanatory and even
the students of correspondence courses can follow it without much stress and strain
• The treatment is nor)-malbemalcal in character and the readers can easily understand the text material
without much taxing their brains.
• Besides good theory the book contains a Largr iumber of solved illustrations (to be exact 1000). These
illustrations have been very carefully selected mainly from the latest examination question papers of various
Universities all over India and neighbouring Countries
• This book is primarily meant to cater to the needs of undergraduars and postgrarhiale stjdents Of commerce
and economics.

Elementary Statistical Methods


Dr. S.F. GUPTA
12th Edn. Reprint 2000, Pp. viii + 702 22 x 14 CM - ISBN 81 . 7014 . 402-7 As. 125.00

Special Features
• This book is especially designed for B Corn. and B.A. Economics Courses of indian universities. It has
devoted maximum space to explain in simple and clear language tire topics and at the ievel recmired or
them,
• There is art of solved problems (497 to be precise) of all varieties as illustrations to help quickly
understand the app l ication of various staristicat tools.

Statistics
D.C. SANCHETI V.K. KAPOOR
Ex-Principal, Shri Ram College of Commerce, Shri Ram College of Commerce,
University of Delhi, Delhi University of Delhi, Delhi
Formerly, Joint Director, Board of Studies,
Institute of Chartered Accountants, New Delhi
Seventh Thoroughly Revised and Enlarged Edition, 2000
Pp. xxxii+ 1447 566 Solved Illustrations 1510 Problems with Answers
22 x 14 cm. 152 Diagrams ISBN 817014•276-8 As. 210.00

Special Features
• A consc entious effort has been made all through to keep the treatment simple, lucid and non .matflematical.
• The theoretical description of each sub-section has been followed by simple illustrations. Complex problems
have been solved thereafter.
• Exercises at the end of each chapter are substantially reorganised. Hints and answers have been provided
forl most of the exercises.
• So ved examples and unsolved problems have been drawn from examination papers of hi Corn. B Corn, M.A.
(Eco.), B. A. (Eco.). CA. t.C.W.A. etc.
• 'Objective Type' questions are also there at the end of each chapter.

Operations Research
Dr KANTI SWARUP
Former Professor, Indian Institute of Public Administration, New Delhi

Dr P.K. GUPTA, Ph.D. Dr MAN MOHAN, M.Sc., Ph.D.


J. V Jain College, Saharanpur Ramjas College. University of Do/hi. Delhi
Eighth Revised Edn. 2000 Pages xiv+ 815 22x 14 cm. As 150.00
300 Solved Examples 150 Diagrams Over 900 Unsolved Problems with Answers
ISBN 81-7014-216-4
Special Features

N It is designed to satisfy the long-felt need of students of OR., Business Systems Analysis,
Management, Engineering. Mathematics, Statistics, Commerce and executives.
U It is rigorous in its treatment of the theory, comprehensive and lucid in its explanation of techniques.
U All important Algorithms have .been summarised in a step-wise manner, followed by their
Flow Charts.
U Great emphasis on mathematical formulation of O.R. problems through sample problems
arising in the fields of Management. Economics, Defence, Manpower Planning. Agriculture, etc.
U Addition of two new chapters on Decision Analysis and Resource Analysis in Network Scheduling.
N A chapter on case studies for the students of management.
An appendix presents answers to problems contained in the book.

Problems in Operations Research


Dr P.K. GUPTA Dr MAN MOHAN
J. V. Jain College. Saharanpur Ramjas College, University of Delhi, Delhi
Eighth Edition 2000 Pages xvi+ 972 ISBN 81-7014-3594 As 170.00
760 Typical Problems Fully Solved 350 Unsolved Prob/ems with Answers
Special Features
• Contains sufficiently large number of solved problems on each topic.
• Problems have been framed so as to include ticklish points.
Contents
Operations Research—An Over-view • Linear Progamming Problem—Mathematical
Formulation • Linear Programming—Graphical Solution • Linear Programming—
Representation in Standard Form & Basic Solution • Simplex Methpd • Degeneracy in
Linear Programming • Duality in Linear Programming • Dual Simplex Method
Revised Simplex Method • Bounded Variable Problem • Integer Programming
Post-Optimal Analysis • Parametric Linear Problems • Transportation Problem
Assignment Problem • Sequencing Problem • Dynamic Programming • Decision
Analysis • Competitive Games • Markov Analysis Problems • Queuing Problems
Inventory Problems • Replacement Problems • Non-Linear Programming • Quadratic
Programming • Project Scheduling by PERT/CPM • Cost Considerations in PERT.'CPM
Simulation • Information Theory • Statistical Tables.

Sultan Chand & Sons


Helping students to achieve their aspirations
23, Daryaganj, New Delhi-1 10002
Phones: 3266105, 3277843, 3281876, 3286788; Fax 011-326-6357
Operations Research
for Managerial Decision-making
V. K. KAPOOR
Co-author of Fundamentals of Mathematical Statistics
Fifth Revised Edition 200O Pages X1+8BB ISBN 81-7014-130-3 Rs 225-00

This well-organised and profusely illustrated book presents updated account of Uie
Operations Research Techniques.
Special Features
• It is lucid and practical in approach.
• Wide variety of carefully selected, adapted and specially designed problems with complete
solutions and detailed workings.
S 221 Worked examples are expertly woven into the text.
• Useful sets of 740 problems as exercises are given.
• The book completely covers the syllabi of M.B A MM.S. and M Corn courses of all Indian
Universities.
Contents
aning & Scope Linear Programming: Graphic Method • Linear Programming Simplex
d • Linear Programming : Duality • Transportation Problems
n ment Problems • Sequencing Problems • Replacement Decisions
C ng Theory • Decision Theory • Game Theory • Inventory Management
CPFvt • Forecastirg
3latistical Quality Control • Investment • PERT &
1chniques • Work Study • Simulation

Problems and Solutions in


Operations Research
V.K. KAPOOR
Fourth Rev. Edition 2003 Pages xri+ 835 ISBN 81-7014-605-4 Rs 235.00

Salient Features
• The book fully meets the course requirements of management and commerce students. It
would also be extremely useful for students of professiora' courses like ICA, ICWA
• Working rules, aid to memory, short-cuts, atternative methods are special attractions of the book
• Ideal book for the students involved in independent study
Contents
Meaning & Scope • Linear Programming Graphic Method • L i near Programming : Simplex
Method • Linear Programming: Duality • Transportation Problems • Assignment Problems
Replacement Decisions • Queuing Theory • Decision Theory • InventorL Management
Sequencing Problems • Pert & CPM • Cost Consideration in Pert • Game Thory • Statistical
Qualit*Control • Investment Decision Analysis • Simulation

Sultan Chand & Sons


Providing books - the never failing friends
23. Dai-yaganj, New Delhi- i 10 002
Phones: 3266105.3277843,3281876,3286788; Fax: 011-326-6357
Fundamentals of Mathematical Statistics
S.C. GUPTA V.K. KAPOOR
Hindu College, Shri Rem College of Commerce
University of Delhi. Delhi University of Delhi. Delhi
Ninth Edition 2000 Pages xx + 1284 22 x 14 cm ISBN 81-7014-120-6 Rs 210.00
Special Features
• Comprehensive and analytical treatment is given of all the topics
• Difficu l t mathematical deductions have beentreated logically and in a very simple manner
• It conforms to the latest syllab of the Degree and Post graduate examinations in Mathematics
and Economics Stabs t-, 5
Contents
Introduction • F requncy Distribution and Measures 04 Central Tendency •
• Theory of t'roOabilii / sparsn Skewness sri <urtics
Fianrjom Variables—Dsirbuson Function • Mathematical Eupeclairon, Generating unctions and
Law of Large Numbers • Theorei,cai Discrete Dlstr,b(rOrls • Theoie:cat Conlnuoi.s Dstibr-is • CureF Ftrng arid
Principle 01 Least Squares • Correlation Flegiesson, B'varale Normal Dslrbjt
Theory of Atnibutas -Sampling ano Large Sample Tests at Mean and Proponror-t • Samp:rrg
and Parrisi & Multiple Cor,et'
DisIrbutlon Exact (Cit-square
Drsilbui(.on) Enact Sarrtphn5 DSlrbuons I
Squeria Anirlyss and NOi-pvameoC Methods i , F and Z Distributions) • Theory 01 Estimation • Tesxg or Hypothesa,

Elements of Business Statistics &


Operations Research
- Dr S.P. Gupta • Dr. P.K. Gupta • Dr. Man Mohar,
Solh Edn 2000 Pp. xiv .e 896 IS13N 81-7014-401-9
- Rs
Special F Jtures
• 'he la 5 age is SlImpIe and the text-material self-expianatory in character
• Busdes good theory, the book Contains a large number ol itlustralons
of varied types

Contents
Pan l—Trars'.cs lPries 710,
Siaisrc—Wyra, & Wt'y Condcng a tuals:rcal Enquiry • Pr'ma'y a irs Secanoary Da:a au Soutce y • Sampling
TriChitidUOS • Diagrammatic anti Gr y
q:ui-c Preseniraliort • iririasures 01 Central Value • Measures Cl larsron-i
Suewnesm. Moments & KuOirs ••COrreiuicn AnSlySis Regression Analysis • Index Numbers
Sores • Ir.:erpoiaficn e E rpoialrbn • ?obaoiry T'rersry • Statistical Quaily Control Analysis of Time
Pin ll--Ceratons Piysearcr' (Pages 180 Iotypretato.s of Data
Lines Pr Cgranimrrnq . F-la. od • Goror iro Merrusi • Snrrp.sr Melnrcxt • Tr.ry.-.t.--, Poiio
. rt r Assnrr.n: Pobrtm

Fundamentals of Applied Statistics


S.C. GUPTA VK. KAPOOR
Third Edition 2000 Pages xx + 628 ISBN 81-7014-151-6 Rs 110.00
Special Features
• The book provides ccmpretlensve and exhaustive theoretical diSCuSSIOn.
• All basic concepts have been explained in an easy and understandable manner
• 125 simulating problems Selected from various university examinations have been Solved
• It conforms to the latest syt!abi of 8Sc. (Hone) and postgraduate examination in Statistics, Agriculture and
Economics
Contents
Slalintica; Ovary Control • Antalyyrs 01 Time Series (Mamormalical Trea t ment)
and Income EIas:icity • Anatysis of Vir.ance Index Numt'x,ir • Dernans Analysis , Price
Design 01 Experiments. Conrrpetety Randomsed Design • Randonriseri Bocu Design ,
Designs and Confounding Lain Square Design • Facto ria l
Design 04 Sample Surveys tMalnermuancal Treatmenil • Sample Random Sampling. Stratified Sampling, Systematic
Sampling. Multi-stage Sampling • Educational and PscboiogCa: Statistics • Vital Srascticai Methods

You might also like